GI XMASTER

Ace your homework & exams now with Quizwiz!

Case A 49-year-old woman presents with dysphagia. She is having difficulty swallowing both liquids and solids; she also notes regurgitation of undigested food. X-ray reveals a bird's beak appearance of the esophagus. Question What is the likely diagnosis?

orrect answer: Achalasia Explanation Achalasia is the correct response. Achalasia is the absence of peristalsis in the lower esophagus. As a result, the lower esophageal sphincter remains closed during swallowing, causing dysphagia and regurgitation. Achalasia causes a characteristic 'bird's beak' appearance of the esophagus; there is distal narrowing and proximal dilation. Gastroesophageal reflux disease (GERD) is an incorrect response. GERD may cause dysphagia, but it would not lead to the narrowing on X-ray. Scleroderma is an incorrect response. Scleroderma may cause weakening of the lower esophageal reflux with dysphagia and regurgitation, but it is not associated with a 'bird's beak' appearance on X-ray. Zenker diverticulum is an incorrect response. Zenker diverticulum is an outpouching in the esophagus that may cause regurgitation and dysphagia; however, it would not cause a 'bird's beak' appearance on X-ray. Esophagitis is an incorrect response. Esophagitis is associated with dysphagia, but it would not typically be seen on X-ray.

Case A 65-year-old man with a past medical history of hypertension, chronic arthritis, and herniated lumbar discs presents with a 1-week history of severe, tearing pain to his rectal area that occurs while he defecates. Following his bowel movement, he notes relief of the tearing pain, but he feels a throbbing sensation in the area. He has noted small drops of bright red blood on the toilet paper. He presently takes amlodipine and hydrochlorothiazide for hypertension as well as hydrocodone-acetaminophen for pain. He has noticed a reduced frequency of bowel movements of late due to pain. The physical examination is notable for a 5-mm midline crack-like lesion at the anus in the 6 o'clock position. The remainder of the physical exam is unremarkable. Question What is the most likely diagnosis?

orrect answer: Anal fissure Explanation This patient's most likely diagnosis is an anal fissure. These lesions are linear or rocket-shaped ulcers that are usually less than 5 mm in length and arise from trauma due to defecation. Underlying contributory conditions may include straining, constipation, or high internal sphincter tone. They occur most commonly in the posterior midline and appear as cracks in the epithelium; those that do not occur midline should raise suspicion for underlying Crohn's disease, HIV/AIDS, tuberculosis, syphilis, or anal carcinoma. Commonly, patients present with a severe, tearing pain during defecation followed by a throbbing discomfort. Often, constipation occurs due to fears of recurrent pain. There may also be hematochezia. Pain is unusual with internal hemorrhoids. Gonococcal proctitis may cause itching, burning, tenesmus, and a mucopurulent discharge. Perianal fistulas are typically preceded by abscesses, in which there is constant perianal pain and throbbing. Fistulas cause purulent discharge that may lead to itching, tenderness, and pain. A pilonidal cyst is a fairly common, usually asymptomatic, opening of a sinus tract located in the midline superficial to the coccyx or the lower sacrum. This opening may exhibit a small tuft of hair surrounded by a halo of erythema. The cyst may occasionally have slight drainage and may be associated with abscess formation and secondary sinus tracts.

Case You are evaluating a 15-year-old Caucasian male high school wrestler. He wants to improve his overall performance as much as possible and has been taking creatine, 5-10g after workouts, 3 times per week. While personnel at his local health food store have assured him that creatine is perfectly natural and safe, he asked for your opinion. Past medical history is unremarkable and there is no history of tobacco, alcohol, or street drug use. Growth and development have been normal and immunizations are current. Vital signs are normal. The examination is unremarkable, including average-appearing muscle mass for age. Question How would you advise him?

Correct answer: "Creatine's safety has not been well-studied in your age group; I do not recommend it." Explanation Creatine is a nitrogenous organic acid that is found in dietary fish and meat. It is not an essential nutrient since it can be synthesized in humans from glycine and arginine. It is touted as a performance-enhancing nutritional supplement that can delay the onset of muscle fatigue by promoting ATP production and increasing overall performance. However, no benefit has been demonstrated in aerobic activities or actual on-field athletic performance, with only small benefits in short duration, maximum intensity resistance training. While this nutritional supplement may be legally used in competition, its safety in adolescents has not been established and its use should be discouraged. Concerns exist regarding potential nephrotoxicity, especially in those at risk for renal dysfunction, but proven serious medical risks have not been identified at usual doses.

Case You are evaluating a 14-year-old African American girl. She is a high school sophomore and a standout on her school's tennis team. She is going to attend a 2-week tennis camp during the summer break. She states that she always becomes more easily fatigued during the summer months, despite adequate hydration. Although she would never consider using any drugs that would increase her energy level, she has begun consuming a variety of energy drinks on the advice of her older teammates. Her favorite contains caffeine (110 mg/8 oz) and guarana (30 mg/8 oz), among other ingredients. She consumes approximately three 16 oz cans per day on heavy practice days and during competition. While her energy levels have increased, she occasionally notices some jitteriness, a rapid heart rate continuing well after cessation of exercise, and a feeling of tiredness about 4 hours after consuming the drink. She wants to know if she should continue consuming the energy drink during summer camp. Past medical history is unremarkable and there is no history of tobacco, alcohol, or street drug use. Growth and development have been normal and immunizations are current. Vital signs are normal except for a resting pulse of 110. The examination is otherwise unremarkable, consistent with Tanner stage IV. Question Ico-delete Highlights How do you respond?

Correct answer: "You should discontinue the use of all energy drinks." Explanation Energy drinks should be completely eliminated from her regimen. Energy drinks are touted as giving a quick burst of energy, increased mental alertness, and decreased fatigue. They are transiently effective in producing these results. Their components are largely a variety of stimulants, especially caffeine, but also guarana, a plant from South America with a caffeine-containing bean. Energy drinks are to be distinguished from sports drinks, which contain carbohydrates, electrolytes, minerals, and sometimes vitamins. While sports drinks do have a role in rehydration of athletes in the pediatric age group, energy drinks, while effective, should never be employed by children or teenagers. The actual ingredients and amounts are sometimes dubiously listed in the product labeling, but the amount of caffeine contained in a 16 oz can is such that consuming several drinks in a short period can cause tachycardia, hypertension, insomnia, and polyuria. Drinks containing guarana contain additional caffeine, frequently not included in the total caffeine content in the product labeling. Energy drinks are especially dangerous in hot weather, in part due to the diuretic effect of caffeine and increase in core temperature. While guarana contains caffeine, it contributes only a small percentage of total caffeine in energy drinks and switching to a drink without guarana is not sufficient advice.

Question A 15-year-old female adolescent is brought into the emergency department by her mother with a history of ingesting an unknown number of acetaminophen tablets within the past hour. Neither the mother nor the daughter had any idea how many pills had previously been used or were ingested. The daughter stated she took a lot of pills. The mother stated her daughter had no other medical problems. Both the mother and daughter said that no other medications were being taken. What is the most appropriate time to evaluate an acetaminophen level after the reported ingestion in this patient to determine if therapy is indicated?

Correct answer: 4 hours Explanation In an acute ingestion, the peak concentration may not be achieved until 4 hoursafter ingestion. Absorption may be affected by coingestants that affect gastric motility. Concretions of multiple tablets ingested at the same time may form bezoars in the stomach and alter absorption, or provide a continuing source of supply. A single acetaminophen level drawn at least 4 hours after ingestion is sufficient to determine patient management. If time of ingestion is unknown, an immediate level and at least 2 additional levels drawn 4 hours apart may be useful. Clinical toxicity may not be evident soon after overdose, and the risk of morbidity increases when the initiation of therapy is delayed. At therapeutic doses, peak concentration is generally achieved after 1 hour, with half-life of 2 to 4 hours. The toxic dose of acetaminophen is approximately 150 mg/kg or 7 grams in adults. Children may tolerate doses up to 200 mg/kg. Acetaminophen is rapidly absorbed from the stomach and small intestine and metabolized by conjugation in the liver to nontoxic agents. These water-soluble conjugates are then eliminated in the urine. Ninety percent are metabolized to either metabolites of sulfate (primarily in children) or glucuronide (primarily in adults). Approximately 4% is metabolized by the cytochrome p450 system to an active metabolite, NAPQI (n-acetyl-p-benzoquinoneimine). Normally, NAPQI is conjugated with glutathione resulting in detoxification and excreted as mercapturic acid and cysteine conjugates. In acute ingestion, glutathione is depleted, so NAPQI covalently binds to vital proteins and the lipid bilayer of hepatocyte membranes. The result is hepatocellular death and centrilobular liver necrosis. Other organs may be affected. The acetaminophen level should be plotted on the Rumack-Matthew nomogram, a semi-logarithmic plot of the acetaminophen level over time (4-24 hours post ingestion) based on adult data. Approximately 60% of patients with levels above the line will develop serum transaminases >1000 IU/L. A second line 25% below the first line was added. Levels between the first and second line are considered possibly toxic and take into account such factors as lab error and error in time of ingestion. There is no clinical evidence suggesting that patients in this range need to be treated. Levels below the second line are low risk. The nomogram is not valid for ingestions less than 4 hours, chronic ingestions, or for patients who have ingested toxic doses of extended release preparations. Because the absorption of extended release preparations is delayed, a measurement at 4 hours will not accurately reflect absorption. The nomogram is also not valid for patients with chronic alcohol consumption.

Case A 48-year-old woman with a past medical history of obesity presents with a 4-hour history of moderate severity epigastric and right upper quadrant pain. The pain is intermittent and occurs in "waves." She also notes nausea, vomiting, and radiation of pain to the right shoulder. Her physical exam reveals normal vital signs, but tenderness is noted in the right upper quadrant. There is no guarding or rebound. A bedside ultrasonography is obtained; it reveals the following imaging. Refer to the image. Question What is correct regarding this patient's health maintenance?

Correct answer: A low-fat, low-cholesterol diet is recommended. Explanation This patient is presenting with signs and symptoms consistent with acute cholecystitis. Cholecystitis is related to conditions associated with biliary stasis, including debilitation, major surgery, severe trauma, sepsis, long-term total parenteral nutrition (TPN), and prolonged fasting. A low-fat, low-cholesterol diet can prevent gallbladder-related symptoms. Outpatient treatment may be appropriate for cases of uncomplicated cholecystitis. If a patient can be treated as an outpatient, discharge with antibiotics (such as levofloxacin and metronidazole), appropriate analgesics, antiemetics and definitive follow-up care. Criteria for outpatient treatment include afebrile with stable vital signs, no evidence of obstruction based on laboratory values, no evidence of common bile duct obstruction on ultrasonography, no underlying medical problems, advanced age, pregnancy, or immunocompromised condition, adequate analgesia, reliable patient with transportation, easy access to a medical facility, and prompt follow-up care. Gallstones are most common among overweight, middle-aged women, but the elderly and men are more likely to experience more serious complications from gallstones. Patients with gallstones should only consume clear liquids to promote gallbladder rest; fatty or greasy meals should be avoided.

Case An obese 38-year-old woman presents with episodic, upper right-sided abdominal pain. She reports that the pain comes on 'out of the blue' and lasts for a couple of hours at a time. It seems to happen shortly after she eats, and often radiates to her back and her right shoulder. She admits to nausea with the pain, but denies vomiting or diarrhea. Question Keeping the most likely diagnosis in mind, what might be found on this patient's physical exam?

Correct answer: A positive Murphy's sign Explanation This patient most likely has some kind of gallbladder pathology, either cholelithiasis or cholecystitis. A patient who has pain when the right upper quadrant is palpated is said to have a positive Murphy's sign, which is often seen in patients with gallbladder problems. In imaging reports, an ultrasonographer will often mention a 'sonographic Murphy's sign' if the patient has pain while they are performing a diagnostic ultrasound. A negative Murphy's sign on physical exam does not necessarily rule out a gallbladder problem, but its presence is a strong indicator of gallbladder as a source of discomfort. A positive Cullen sign is ecchymosis of the umbilicus and a positive Grey Turner sign is ecchymosis of the flank. Either of these signs is usually representative of retroperitoneal bleeding, often from the pancreas; however, both these signs are quite rare (present in approximately 3 percent of patients with pancreatitis). A patient with gallbladder disease would not have a positive Cullen sign or Grey Turner sign. Hyperactive bowel sounds are usually seen in patients with diarrhea or other bowel pathology; these would not be an expected physical finding in a patient with a gallbladder problem. Rebound tenderness in the right lower quadrant is most suggestive of appendicitis or other acute abdominal pathology. It is not an expected physical exam finding in a patient with a gallbladder problem.

Case A 13-year-old boy presents with a long history of intermittent jaundice without other signs and symptoms. Yesterday he took several paracetamol tablets, and the headache and jaundice appeared again. His parents have a consanguineous marriage, and nobody in the family has similar symptoms. His physical examination today is within normal limits, except for mild scleral icterus. The initial laboratory examinations show: RBC 4.5mill/mm3; WBC 6000/mm3; total bilirubin 2.2 mg/dl; indirect bilirubin 2.0 mg/dl; direct bili 0.2 mg/dl; liver enzymes, serum copper, and all other parameters are normal; hepatitis B (-); and the full set of his autoimmune markers (-) and Coombs test (-). Blood smear is normal. Ultrasonography of liver, gallbladder, pancreas, spleen, and for both kidneys are normal. His urine appears very yellow. Question What is the most likely the diagnosis?

Correct answer: Gilbert's syndrome Explanation Your patient most probably has Gilbert's syndrome (GS), also known as Gilbert-Meulengracht syndrome. It is a relatively common genetic disease found in up to 5% - 10% of the population and generally does not need special treatment. Inherited non-haemolytic hyperbilirubinemic conditions include Dubin-Johnson, Rotor, and GB syndromes, and all are important differential diagnoses indicating benign disease that requires no immediate treatment. GB can be diagnosed by clinical presentation, biochemistry, and genotyping, and is significant because of the presence of the disposition towards drug-associated toxicity. A major characteristic is jaundice, caused by elevated levels of unconjugated bilirubin in the bloodstream. The cause of this hyperbilirubinemia is the reduced activity of the enzyme glucuronyltransferase, which conjugates both bilirubin and some lipophilic molecules, including drugs. Intravascular hemolysis, with resulting hemoglobinemia, hemoglobinuria, and bilirubinemia, will show fragments of the red blood cells ("schistocytes") and sometimes spherocytes in peripheral blood smear, reticulocytosis, elevated unconjugated bilirubin that may lead to jaundice, elevated lactate dehydrogenase (LDH) in the blood, and decreased haptoglobin levels. If the direct Coombs test is positive, hemolysis is caused by an immune process. Hemosiderin in the urine indicates chronic intravascular hemolysis. There is also urobilinogen in the urine. Viral hepatitis with jaundice will have elevated liver function tests (AST and ALT elevated out of proportion to alkaline phosphatase, usually with hyperbilirubinemia), and viral serologic testing will be positive. Crigler-Najjar syndrome is a rare inherited form of non-hemolytic jaundice, which results in high levels of unconjugated bilirubin and often leads to brain damage in infancy. Budd-Chiari syndrome is caused by occlusion of the hepatic veins. It presents with the classical triad of abdominal pain, ascites, and hepatomegaly. The syndrome can be fulminant, acute, chronic, or asymptomatic.

Case An obese 38-year-old woman presents with episodic, upper right-sided abdominal pain. She reports that the pain comes on 'out of the blue' and lasts for a couple of hours at a time. It seems to happen shortly after she eats, and often radiates to her back and her right shoulder. She admits to nausea with the pain, but denies vomiting or diarrhea. Question Ico-delete Highlights Keeping the most likely diagnosis in mind, what might be found on this patient's physical exam?

Correct answer: A positive Murphy's sign Explanation This patient most likely has some kind of gallbladder pathology, either cholelithiasis or cholecystitis. A patient who has pain when the right upper quadrant is palpated is said to have a positive Murphy's sign, which is often seen in patients with gallbladder problems. In imaging reports, an ultrasonographer will often mention a 'sonographic Murphy's sign' if the patient has pain while they are performing a diagnostic ultrasound. A negative Murphy's sign on physical exam does not necessarily rule out a gallbladder problem, but its presence is a strong indicator of gallbladder as a source of discomfort. A positive Cullen sign is ecchymosis of the umbilicus and a positive Grey Turner sign is ecchymosis of the flank. Either of these signs is usually representative of retroperitoneal bleeding, often from the pancreas; however, both these signs are quite rare (present in approximately 3 percent of patients with pancreatitis). A patient with gallbladder disease would not have a positive Cullen sign or Grey Turner sign. Hyperactive bowel sounds are usually seen in patients with diarrhea or other bowel pathology; these would not be an expected physical finding in a patient with a gallbladder problem. Rebound tenderness in the right lower quadrant is most suggestive of appendicitis or other acute abdominal pathology. It is not an expected physical exam finding in a patient with a gallbladder problem.

Case A 15-year-old boy presents with abdominal pain and rectal bleeding. His family history is significant for the premature deaths of his mother and maternal grandmother from metastatic colon cancer. Both died before the age of 35 years; in both, autopsy findings included hundreds of colon polyps along with multiple primary colon cancers. Endoscopy of the boy also demonstrates extensive colonic polyp disease. Question A mutation in what gene is causing this patient's symptoms?

Correct answer: APC Explanation The presentation of hundreds of colonic polyps often "carpeting" the large bowel, in multiple family members, is typical for familial adenomatosis polyposis syndrome. The typical course is for polyps to develop in late childhood and the early teenage years, with transformation of 1 or more polyps to colon cancer in the 3rd or 4th decade. Mutations in the APC gene, a known tumor suppressor gene, cause this syndrome. Mutations in BRCA1 and BRCA2 lead to breast cancer and ovarian cancer; the highest risk is seen in women, although some men with BRCA2 mutations may develop breast cancer. BRCA1 and BRCA2 are not associated with colonic polyposis. MSH2 mutations are a cause of hereditary non-polyposis colon cancer syndrome (HNPCC); this leads to colon cancer without extensive polyposis, although a few polyps may be present in some cases. Mutations in the RET gene are a cause of multiple endocrine neoplasia syndrome type II (MEN-II), which is typically associated with medullary thyroid cancer, parathyroid adenomas, and pheochromocytomas. Gastrointestinal ganglioneuromas are sometimes also found.

Case A 55-year-old woman presents with poor appetite and nausea. She has vomited 2 times over the past week and lost 4 pounds in the past month. Past medical history is significant for 20 years of alcoholism, 5 years with diabetes, and hypertension. She takes no medications; she is not involved in any therapy for her alcoholism. She has been drinking 2 pints of beer almost every day for the past 20 years, consuming greater quantities on weekends. Her vitals include a heart rate of 102 BPM, blood pressure of 140/100 mm Hg, respiratory rate of 20/min, and a temperature of 99.8°F. Physical exam reveals hepatomegaly. A liver biopsy reveals ballooning degeneration, spotty necrosis, and polymorphonuclear infiltration. Question Ico-delete Highlights What lab finding would be characteristic of this patient's condition?

Correct answer: AST/ALT >2 Explanation The patient's history of alcoholism, her physical findings, and the liver biopsy results point toward a diagnosis of alcoholic liver disease (ALD), where typically the AST/ALT >2 (the ratio of serum aspartate aminotransferase to serum alanine aminotransferase). Patients with increased alcohol consumption may not have a ratio >2, and it may be more of an indication of advanced alcoholic liver disease. ALD is a result of the consumption of large quantities of alcohol over a long duration of time. The threshold for men is 60-80 g/d for >10 years. Initially, there is a fatty infiltration of the liver, which gradually progresses to alcoholic hepatitis. Alcoholic hepatitis is characterized by hepatocyte injury, as revealed by the biopsy findings in this patient. Lab findings include a moderately elevated AST, ALT, γ-Glutamyl transpeptidase, and alkaline phosphatase. Both AST and ALT are elevated by two- to sevenfold, and AST is greater than ALT level (AST/ALT >2). Bilirubin is also elevated. Prothrombin time is >5 seconds (normal is 11-15 seconds), and macrocytic anemia may also be observed.

Question A 63-year-old woman presents with acute onset of abdominal pain. She describes it as a steady deep discomfort in the left lower quadrant. Initially she was constipated, but is now experiencing diarrhea. On physical examination, she has a temperature of 38°C. The abdomen is tender in the LLQ with guarding and rebound tenderness. She has positive fecal occult blood. What is the best test to determine her most likely diagnosis?

Correct answer: Abdominal CT Explanation Diverticulitis is a common cause of left lower quadrant abdominal pain that is associated with fecal occult blood. It is best diagnosed with abdominal CT. Findings of sigmoid diverticula, a thickened colonic wall and inflammation within the pericolonic fat suggest the diagnosis. While an ultrasound is less expensive and non-invasive, it is less specific and operator dependent. Barium enema and colonoscopy will both demonstrate diverticula, but there is risk of perforation with these modalities. An upper GI is not indicated in this patient with lower abdominal pain.

Case A 60-year-old man presents with severe abdominal pain that started 10 hours ago. It is increasing in severity and is colicky in nature. The patient has not had a bowel movement for 3 days. In the last 2 days, he vomited 4 times. He looks tired and dehydrated. On examination, increased bowel sounds were noted. There is also a mass in the right inguinal area. The patient said this mass has been present for 10 years, but until now, it has always disappeared when he lay on his back. Question What is the study of choice to confirm the diagnosis?

Correct answer: Abdominal X-Ray Erect film Explanation Intestinal obstruction occurs when bowel movement is encountering an obstacle in the passage through the bowel. Actually this can be caused by any mass, adhesion, or any other means of mechanical obstruction. The symptoms of obstruction vary according to the site of obstruction, but generally there is increased bowel movement proximal to obstruction with the reflected increase in bowel sounds. There may be vomiting. There may be no bowel movement, from absolute constipation to feces and flatus. Here the patient has inguinal hernia which is recently irreducible and obstructed. The best diagnostic procedure is erect X-ray film which shows multiple fluid levels and gives confirmation of the diagnosis. The treatment is exploratory surgery, and there is no place for any conservative treatment without surgery. Paralytic ileus is a type of intestinal obstruction that usually follows general anesthesia and is characterized by silent abdomen with no bowel sounds, usually treated by bowel rest and proper hydration. Colonoscopy here is contraindicated. Upper endoscopy is generally used to detect pathology in upper GIT, not in the case of intestinal obstruction. Abdominal ultrasound is of value in diagnosis of gall bladder disease and also urinary bladder pathology. CBC is generally used to detect leukocytosis in acute appendicitis or inflammatory reactions.

Case An 11-year-old boy presents with a chronic history of mild hemolytic anemia, intermittent jaundice, and right upper quadrant pain. He denies any shortness of breath, muscle aches, or joint pain. Question Supplementation with which of the following should be given to the patient as part of the treatment plan?

Correct answer: Folic Acid Explanation Folic acid should be given to patients who present with hemolytic anemias, such as those with hereditary spherocytosis as suggested in the above case. Hemolytic anemias cause reticulocytosis, which leads to an increased demand for folic acid in the formation of red blood cells. The treatment of choice for hereditary spherocytosis is splenectomy, as the spleen is responsible for sequestering and destroying the spherocytes, leading to hemolytic anemia. Thiamine (or B1), Cyanocobalamin (or B12), Niacin (or B3), and Riboflavin (or B2) are all B vitamins; increased amounts are not required in the setting of a hemolytic anemia with reticulocytosis.

Case A 35-year-old man presents after several episodes of vomiting in the last 24 hours; there is loose stool and strong pain that is localized in the upper-middle region of the abdomen. Physical examination indicates a temperature of 101°F and a tender epigastrium. Lab tests reveal an initial WBC count of 18x109/L. C-reactive protein level is 325 mg/L, and amylase is 130 U/L. There is a lactate dehydrogenase level of 816 U/L. The patient has no history of pancreatic disease, and declares himself a non-drinker. He is overweight. He has a history of diabetes type-2 and hypertension. He takes medicine to control his high blood pressure and obesity. Question Ico-delete Highlights What is the most appropriate next step in establishing diagnosis for this patient?

Correct answer: Abdominal computed tomography scanning Explanation The correct answer is abdominal computer tomography scanning. The symptoms and lab results are consistent with an acute episode of pancreatitis, but extra tests are necessary to confirm the diagnosis. The CT scan test is most appropriate because it provides the most accurate way to see the pancreas. A CT study is useful when other diagnostic studies are inconclusive, when the patient has severe symptoms, or if fever or secondary infection is present. The answer abdominal radiography is not correct. This test is mainlyperformed to detect free air in the abdomen, and it does not provide a good image of the pancreas. The answer abdominal ultrasonography is incorrect. Ultrasound imaging is often used as an initial diagnostic test for people suffering from mild pancreatitis, but when severe, acute pancreatitis is suspected a CT scan test is recommended. The answer endoscopic retrograde cholangiopancreatography is incorrect. This method combines the use of the endoscope and X-ray to determine the cause of pancreatitis. It requires anesthesia of the esophagus and stomach and sedation of the patient. It is considered too invasive to be recommended for patients with severe acute pancreatitis. Endoscopic ultrasonography is not the correct response. This test is usually performed to detect microlithiasis and periampullary lesions that are not easily revealed by other methods.

ase A 20-year-old female college student presents due to a 7-day history of daily heartburn. She has never had this as bad as she does currently. The patient denies any other significant past medical history and is currently taking only a multivitamin daily. She admits to recently having increased episodes of headaches that she believes are due to stress. For this reason, she has been taking ibuprofen 600 mg every 8 hours. She states that she has been taking this consistently every 8 hours for the last 10 days. She hopes that after finals are over her headaches will subside. She also states she was given a 10-day course of amoxicillin 2 weeks ago for a middle ear infection, which resolved without any further intervention. She denies any difficulty swallowing, weight loss, night sweats, chest pain, black tarry stools, use of tobacco/alcohol, or coughing up blood. Physical examination is unremarkable for any abnormalities. Question What medication would you suggest for the patient to begin for her headaches at this time?

Correct answer: Acetaminophen Explanation Patients classified as having mild or intermittent symptoms of esophagitis and/or gastroesophageal reflux disease (GERD) typically are seen as not adversely affecting the patient's quality of life. Initial action is taken in terms of modification of behaviors, such as eating smaller meals and eliminating acidic foods or foods known to precipitate reflux (fatty foods, alcohol, chocolate, or peppermint). The next step in terms of treatment would be to discontinue any medications that may be increasing the symptoms of GERD. Medications that potentially irritate the esophagus and have a high incidence of causing pill-induced esophagitis include tetracycline, bisphosphonates, iron supplements, Non-Steroidal Anti-inflammatories (NSAIDs), and potassium supplements. Medications that can increase acid reflux and worsen the condition of GERD include anticholinergics, calcium channel blockers, narcotics, progesterone, quinidine, benzodiazepines, or even theophylline. A proton pump inhibitor would be initiated in a patient who has moderate to severe esophagitis or gastroesophageal reflux symptoms; this patient does not fit this picture. Since this patient is currently choosing ibuprofen chronically for a recent onset of headaches, the appropriate instruction would be to discontinue the ibuprofen regimen and begin an acetaminophen regimen in its place. Antibiotics would be an incorrect answer. H1 receptor antagonists, or H1-antihistamines, are an inappropriate choice because these are used to treat the symptoms of allergies (seasonal and perennial). The mechanism of action of anticholinergics does not make it an indication in this type of medical complaint.

Case A 38-year-old woman is going through a divorce and simultaneously filing bankruptcy. She is very stressed about her financial situation and failed marriage. One day, after a particularly long crying spell, she notices sudden onset of extreme difficulty swallowing at dinnertime. She has difficulty swallowing both solids and liquids. She feels that the food is sticking in her throat. She ignores it. She has numerous bouts of these episodes of difficulty with swallowing. She notices that when she lies down, undigested food comes up. Finally, she sees her doctor. On questioning by her doctor, she insists she has no heartburn. She has not seen any blood when she regurgitates nor has she vomited blood. Manometry is performed which shows an absence of normal peristalsis and an elevated LES pressure. Question What is the most likely diagnosis?

Correct answer: Achalasia Explanation Achalasia is characterized by dysphagia. There is poor peristalsis in achalasia. This patient has a characteristic history for achalasia. The onset is during a period of stress. The symptoms consist of dysphagia, complaints of food sticking in the throat, and regurgitation. Scleroderma can present with dysphagia. However, manometry would show a decreased LES pressure, not an elevated LES pressure as the case here. In addition, scleroderma can be associated with heartburn. Esophageal varices usually present with hematemesis. Gastroesophageal reflux disease would present with heartburn. If anything, the LES pressure would be lower than normal, rather than elevated. Mallory-Weiss syndrome is an esophageal tear. This would present with hematemesis.

Case A 35-year-old man presents with a 4-week history of difficulty swallowing food. On detailed history, he says that the regurgitation contains undigested fragmented food material; it is not associated with a foul smell. He has lost 3 kg over the past month. On examination, his pulse is 94 bpm and blood pressure is 130/86 mm Hg. Mild pallor is noted, but there is no icterus. Epigastric tenderness is present. CBC shows a hemoglobin of 9.5 g/dL (total count 7400 cells/mm3 and ESR 12 mm/hr). An X-ray of the abdomen shows the absence of a fundic air shadow. The patient undergoes barium studies that show a dilated esophagus; the lower end appears beak-shaped. Question What is the most likely diagnosis?

Correct answer: Achalasia Explanation Gradual onset of difficulty in swallowing food, the regurgitation of undigested food, the absence of fundic air on a plain X-ray of the abdomen, and a dilated esophagus with a beak-shaped lower end are diagnostic of achalasia. It occurs due to the degeneration of inhibitory neurons in the lower end of the esophagus. The most common cause of achalasia is primary/idiopathic. There is a deficiency of inhibitory neurons containing vasoactive intestinal peptide and nitric oxide synthase in this condition. Barrett's esophagus is the metaplasia of the lower esophageal junction; it is usually a consequence of gastroesophageal reflux disorder. Hiatal hernia is usually a sliding type and is not associated with the esophageal narrowing. Plummer-Vinson syndrome is associated with web formation in the upper esophagus; it is associated with iron deficiency anemia in menopausal women. Zenker's diverticulum is a posterior pharyngeal wall abnormality due to dehiscence between the 2 layers of inferior constrictors. Undigested food usually comes back with a foul smell a long time after the ingestion.

Case A 4-day-old infant is seen for a routine newborn check; he has yellow skin from his face down to his chest. He is a full-term baby, had a normal vaginal delivery, weighed 3.180 kg at birth, and was 3.100 kg at discharge. He is breastfeeding well and stooling and voiding regularly. He is well-hydrated and active. Laboratory results reveal an indirect bilirubin of 12.2 mg/dL and a direct bilirubin of 1.0 mg/dL. Question What is the physiologic mechanism behind this jaundice?

Correct answer: An elevation of indirect bilirubin due to increased bilirubin production Explanation Unconjugated indirect bilirubin enters the liver and is excreted from the hepatic cells in a water-soluble form (conjugated and direct) into the biliary system. The direct bilirubin leaves the body in the form of stool. The most common cause of indirect hyperbilirubinemia is physiologic jaundice, which peaks at 2-4 days of life. Physiologic jaundice is due to increased bilirubin production after the breakdown of red blood cells, in conjunction with limited conjugation of bilirubin by the immature liver. The increased breakdown of red blood cells occurs because fetal red blood cells have a shorter lifespan than adult red blood cells. Physiologic jaundice can be worse in babies that are breastfed due to a higher level of dehydration as the mother waits for her milk to come in.

Case A 30-year-old woman presents with a 2-week history of difficulty swallowing food. She is not on any medications; she drinks alcohol over the weekends. The systemic examination is essentially normal. A barium swallow reveals bird beaking. Manometry shows increased lower esophageal sphincter pressures. Question What is the most likely diagnosis?

Correct answer: Achalasia Explanation Patients with dysphagia (or difficulty in swallowing) have a sensation of food getting stuck in the esophagus. Achalasia is a motility disorder of the esophagus characterized by impaired esophageal body peristalsis and failure of the lower esophageal sphincter (LES) to relax during deglutition. Patients usually present with dysphagia for both solids and liquids. They may also complain of a nocturnal cough and regurgitation of foodstuffs. Barium swallow reveals dilatation of the proximal esophagus with narrowing of the distal esophageal area and the bird-beak appearance. Esophageal manometry shows aperistalsis and elevated LES pressures. Diffuse esophageal spasm is another esophageal motility disorder. It is characterized by repetitive, nonpropulsive contractions of the esophageal smooth muscle without coordinated peristalsis. Patients usually present with dysphagia for both liquids and solids, and chest pain which may be precipitated by drinking cold liquids. Barium swallow reveals diffuse disordered esophageal spasms with poor bolus progression. Manometry shows uncoordinated esophageal contractions and normal LES pressures. There may be some normal peristaltic activity. Globus hystericus is a sensation of having a lump in the throat. Barium swallow and manometry are normal, as it is primarily emotional in origin. Zenker's diverticulum, also known as pharyngoesophageal diverticulum, is a herniation of the esophageal mucosa through the cricopharyngeal muscle. Patients usually present with regurgitation of undigested food when they bend over or lie down. Other symptoms include halitosis and dysphagia. Barium swallow reveals an outpouching from the posterior wall of the distal pharynx near the pharyngoesophageal junction that retains barium. Patients with scleroderma, who develop dysphagia as a result of impaired esophageal motility, complain of dysphagia for both solids and liquids. Barium swallow shows dilatation of the lower esophagus with poor sphincter tone. Manometry shows aperistalsis and low LES pressures. Acid reflux and development of peptic strictures are relatively common.

Case A 64-year-old man presents with a history of progressive dysphagia to solids; recently, he has been having trouble with liquids. He also experiences occasional regurgitation of undigested food. His physical exam is unremarkable. A barium esophagram reveals a distinctive bird's beak appearance of the distal esophagus. Question What is the most likely diagnosis?

Correct answer: Achalasia Explanation The correct answer is achalasia; it often presents with progressive dysphagia to liquids and solids as well as complaints of regurgitation of undigested food. Classic radiographic findings include a bird's beak appearance in the distal esophagus, demonstrating narrowing of the esophageal lumen. Gastroparesis is delayed gastric emptying, not an esophageal disorder that would cause the progressive dysphagia noted by the patient. Gastroparesis is often a feature of patient's with longstanding diabetes mellitus. Gastroesophageal reflux disease can cause dysphagia, but it is not typical; this condition causes regurgitation of undigested food and does not have the classic bird's beak appearance on esophagram seen in this case. Mallory-Weiss syndrome presents typically with hematemesis due to forceful vomiting or retching, which causes tears in the esophagus. It is associated with alcoholism. Candidal esophagitis is a fungal infection of the esophagus often seen in immunocompromised patients, such as those who are HIV infected. It presents with progressive dysphagia, but oral thrush is often present in these patients, and an esophagram will not show the classic bird's beak sign.

Case A 23-year-old woman presents with abdominal pain that began last night. It started as a dull ache in the lower abdomen, but has localized to the right lower quadrant over the past several hours. She has experienced anorexia and 2 episodes of vomiting. She recently entered into a sexual relationship with a new boyfriend. Her last menstrual period was 1 week prior to presentation. On exam, her temperature is 37.8° C, pulse is 90, and blood pressure is 120/72 mm Hg. Abdominal exam reveals a soft abdomen with tenderness to moderate palpation in the right lower quadrant. There appears to be guarding and mild rebound tenderness. Pelvic exam reveals a normal-appearing cervix with clear cervical discharge; there is no cervical motion tenderness. CBC reveals a white blood cell count of 11,800/μL.; urinalysis reveals a few squamous cells, but no bacteria or blood. Question What is the most likely diagnosis?

Correct answer: Acute appendicitis Explanation Acute appendicitis can affect all ages and is a common cause of emergency surgery; there is a lifetime incidence of approximately 6%. While appendicitis has some very classic features, it still can be very difficult to diagnose. It is especially dfficult to diagnose in very young or old patients, in patients with concomitant pregnancy, or in patients with other diseases (e.g., diabetes or AIDS). This patient's presentation is classic for appendicitis. The classic presentation is one of anorexia, with abdominal pain that begins in the periumbilical area. The pain becomes steady and migrates to the right lower quadrant in a 24-hour period. Nausea, emesis, and/or diarrhea are common. Rebound tenderness can be present and reflects peritoneal inflammation; rebound is less apparent in elderly or obese patients. Fever may or may not be present, although over 80% of patients with acute appendicitis will have an elevated white blood cell count. The most definitive imaging study is CT scan of the abdomen and pelvis, and should be performed when available. Less definitive imaging studies include plain x-ray films of the abdomen (to look for a fecalith or gas in the appendix); barium enema (to look for non-filling of the appendix), and ultrasound imaging (especially in pregnancy, to look for a tender, immobile, non-compressible structure). However, surgical evaluation with laparoscopy or laparotomy is the definitive test for appendicitis, and is the treatment of choice if suspicion is high.

Case Early one afternoon, a 15-year-old boy presents with abdominal pain, nausea, and vomiting. The pain has been worsening since the onset of symptoms in the morning. There is no known gastrointestinal disease in the history; no one in the immediate environment has one either. Physical examination finds no abdominal tenderness, but Psoas sign and tenderness on rectal examination is detected. Temperature and pulse are slightly elevated. Skin turgor is reduced, and there is a 10 mm Hg drop in postural blood pressure. Laboratory studies find 18,000 white blood cells per microliter. Question What is the most likely diagnosis?

Correct answer: Acute appendicitis Explanation The course and signs of the disease indicate appendicitis. The lack of abdominal tenderness and positive rectal examination indicate inflammation of retrocecal or pelvic appendix. Salmonella gastroenteritis would likely affect other persons in the child's environment. Regional enteritis is associated with a prolonged history. Acute mesenteric lymphadenitis poses a differential diagnostic challenge more frequently among children than adults. The diagnosis is impossible clinically, although the temperature tends to be higher and the pain is more diffuse. Culture of mesenteric nodes and serologic titers confirms Yersinia infection in some patients. Meckel's diverticulitis is rare, but it may be difficult to distinguish from appendicitis. Considering the risk of perforation, it is prudent to err toward the diagnosis of acute appendicitis unless workup for Meckel's can be undertaken expeditiously. The workup is mainly related to imaging and includes high-quality CT and Nuclear Imaging, not available readily in some situations.

Case A 36-year-old woman presents with right upper quadrant pain radiating to the shoulder blades, nausea, and vomiting. The pain started a few hours ago, and it is constant and excruciating. She has no diarrhea and denies constipation. She has no significant past medical history and is not on any medications. Family history is unremarkable. Her temperature is 98.1°F, pulse 96/min, and BP 110/70 mm Hg. There is mild icterus and right upper quadrant is tender to palpation. Bowel sounds are active and no masses are felt. Labs show Hb 12 g/dL, WBC 7800/uL, and platelets 450,000/uL. AST is 54 U/L and ALT 63 U/L. Bilirubin is 3.2 mg/dL, BUN 24 mg/dL and creatinine 1.1 mg/dL. Question What is causing the patient's symptoms?

Correct answer: Acute cholecystitis Explanation Acute cholecystitis is typically characterized by right upper quadrant pain with radiation to the back, nausea and vomiting, fever, and right upper quadrant tenderness, which is also known as Murphy's sign. Jaundice may be present occasionally, with elevated liver enzymes and leukocytosis. It is caused by inflammation of the gallbladder wall, usually with one or multiple gallstones. Infection within the biliary system also plays a role in acute cholecystitis. Ultrasound reveals gallstones in most cases. Gallbladder wall thickening more than 4-5 mm, edema (causing double wall sign), and sonographic Murphy's sign (tenderness in the right upper quadrant while palpation with the ultrasound transducer) are also seen in the ultrasound. Patients need hospitalization for proper management. Treatment is conservative and involves hydration, pain control, bowel rest, and correction of any electrolyte disturbances. Antibiotics are not recommended as a rule, but they are generally given. Cholecystectomy is recommended for empyema, gangrene, rupture of the gallbladder, recurrent disease, or gallstones. Asymptomatic gallstones, however, are not treated surgically. Acute appendicitis is a common cause of acute abdominal pain. It can occur at any age and is caused by the obstruction of the lumen of the appendix by a foreign body, inflammation, fecalith, or tumor. Increased intraluminal pressure causes necrosis and perforation with signs of localized peritonitis. Pain starting in the umbilical area and later migrating to the right lower quadrant at McBurney's point is classic. Fever, vomiting, diarrhea, and leukocytosis support the diagnosis. CT scan in adults and abdominal ultrasound in children are diagnostic, and treatment is immediate surgical removal. In a female patient, endometriosis, ovarian cyst, and ovarian torsion are the other differential diagnoses. Acute gastroenteritis can mimic appendicitis, but watery diarrhea is usually the predominant symptom, with vomiting in some infections. Abdominal pain is diffuse and crampy, and it is not localized to McBurney's point. Fever and leukocytosis may be present. CT scan rules out localized pathology and is necessary only in limited cases. Treatment is supportive since it is a self-limiting condition. Acute diverticulitis is usually left-sided and rarely right-sided. It presents with left lower quadrant pain, constipation or diarrhea, and left lower tenderness. Fever and mild leukocytosis are common, but liver enzymes are normal. If right-sided, it is frequently mistaken for appendicitis. Diverticulitis can occur anywhere in the GI tract and may be mistaken for other common conditions. For example, diverticulitis of the transverse colon may be confused with cholecystitis, pancreatitis, or peptic ulcer disease. A tender mass may be present in a few cases, but it is very rare in patients under 30, and CT scan differentiates it from other conditions. Treatment is usually conservative in the first episode: bowel rest and antibiotics. Colonic resection is advised in recurrent episodes; when there are complications like perforation, fistula or abscess; when the symptoms are intractable; or when there is suspicion of carcinoma. Incarcerated hernia presents with severe pain in the groin area with a mass that may be purplish in color due to ischemia and gangrene. It should be suspected when a reducible hernia becomes painful and non-reducible. Treatment is surgical.

Case A 25-year-old man presents due to heartburn. It only occurs 1 or 2 times a month, but it is extremely noticeable when these episodes do occur. He states that eating large meals, eating late at night right before going to sleep, and excessive alcohol consumption seems to exacerbate and even initiate episodes. He has noticed that limiting the aforementioned behaviors has decreased the severity of the symptoms in recent months, but it is still occurring despite this modification. He denies any difficulty swallowing, weight loss, night sweats, chest pain, use of tobacco, or coughing up blood. Physical examination is unremarkable for any abnormalities. Question Since this patient's symptoms would be classified as mild, intermittent symptoms and modifications have been attempted and failed, what would be the next best option for this patient?

Correct answer: Antacids Explanation Mild or intermittent symptoms of gastroesophageal reflux disease (GERD) are not typically seen as adversely affecting patients' quality of life. Initially, action is taken in terms of modification of behaviors, such as eating smaller meals as well as eliminating acidic foods and foods known to precipitate the reflux (fatty foods, alcohol, chocolate, or peppermint). Weight loss has also been shown to help decrease issues, and so has advising patients to avoid lying down at least 3 hours after eating meals. Elevation of the head of the bed is also suggested to initiate relief. From a pharmacological standpoint, antacids are considered the mainstay for rapid relief of occasional heartburn. Antacids work by neutralizing stomach acid only; alone, they will not heal any inflammation that may be caused by stomach acid. Patients should be told that the duration of action of these medications is typically less than 2 hours. Antacids are also contraindicated in any patients with renal failure. Many types of antacids are available over the counter. The next step up in terms of treatment would be considered to be H2-receptor antagonists. These medications work by reducing stomach acid production. Again, these agents are available over the counter as 50% the prescription dose. Examples include cimetidine 200 mg, ranitidine 75 mg, and famotidine 10 mg. When these agents are taken for active reflux symptoms, they have a delay of onset of at least 30 minutes; once they do take effect, relief is supplied for up to 8 hours per dose. Proton pump inhibitors are initial pharmacological treatment reserved for patients with moderate to severe symptoms of reflux as well as known complications from GERD. Proton pump inhibitors work by actually blocking the production of acid. Examples of these include omeprazole 20 mg or lansoprazole 30 mg. There are proton pump inhibitors available over the counter, but some are only available with a prescription. Proton pump inhibitors help patients achieve adequate control of their heartburn symptoms, and they can potentially offer resolution of these symptoms; they even aid in the healing of erosive esophagitis if it is present. Antibiotics in relation to reflux or heartburn would not be indicated directly for the symptoms of this pathology; antibiotics could potentially be used as a component of the treatment of an H. pylori infection. Prokinetic agents are not indicated as treatment of mild, intermittent reflux symptoms.

Case A 44-year-old premenopausal Caucasian woman with a BMI of 36 presents with persistent upper right quadrant abdominal pain that radiates to the back. It has gotten so bad that she has difficulty eating any food and needs to force herself to eat. She has nausea with some episodes of vomiting. She denies bulimia but admits to anorexia. On exam, the patient has a positive Murphy's sign and tenderness to palpation in the epigastric and upper right quadrant area. Patient has a slightly elevated temperature. The physician assistant is awaiting labs and imaging. Question What is the most likely diagnosis?

Correct answer: Acute cholecystitis Explanation Patients with acute cholecystitis present with abdominal pain accompanied by nausea and vomiting. Usually the patient consumes a large fatty meal 1 hour before the onset of symptoms, but the pain is persistent. This usually occurs due to cholelithiasis blocking the cystic duct. Patients commonly affected by cholelithiasis are those who meet the four Fs: fat, female, forty, and fertile (which describes the patient above). Chronic cholecystitis is usually an intermittent type of pain and a mild inflammation of the gallbladder. This can turn into an acute cholecystitis. Appendicitis is usually right lower quadrant pain that presents as a diffuse abdominal pain that localizes to the right lower quadrant. The patient would not have a positive Murphy's sign but would have McBurney's point, obturator sign, and psoas sign. The patient would have a fever as well. Cholangitis is defined as jaundice, fever, and right upper quadrant abdominal pain, known as Charcot's triad. Gastritis is inflammation of the stomach, which is localized in the epigastric area. This can occur for numerous reasons, but it is most commonly secondary to infectious or autoimmune causes.

Case A 56-year-old man presents with a 24-hour history of abdominal pain, nausea and vomiting, fever, and clammy skin. Home treatment included acetaminophen and fluids; treatment offered little relief. His past medical history includes gallstones and congestive heart disease. His medications include thiazide diuretics. On clinical exam, his skin is cool and clammy, and he has abdominal tenderness in the upper quadrants; his heart rate is 100, and his blood pressure is 110/70 mm Hg. Question What is the most likely diagnosis?

Correct answer: Acute pancreatitis Explanation Pancreatitis is an inflammation or infection of the pancreas. The chief causes of acute pancreatitis in adults are gallstones, other biliary disease, and alcohol use. Viral infection (mumps, Coxsackie B, mycoplasma pneumonia, and Campylobacter), injury, pancreatic or common bile duct surgical procedures, and certain medications (especially estrogens, corticosteroids, thiazide diuretics, acetaminophen, tetracycline) are other causes. After the triggering event, the process continues with autodigestion that causes swelling, hemorrhage, and damage to the blood vessels. An attack may last for 48 hours. Symptoms include abdominal pain (mainly located in the upper abdomen) nausea, vomiting, weakness, sweating, anxiety, fever, clammy skin, and mild jaundice. General examination may show a low blood pressure and a heart rate above 90. Tests should include (those that demonstrate pancreatic enzyme release) elevated serum amylase, elevated urine amylase, and elevated serum lipase. Abdominal ultrasound and CT scan will demonstrate an enlarged pancreas. Blood tests may show an elevated white blood cell count, elevated blood glucose, and decreased serum calcium. Treatment is aimed at supportive measures, such as fluid replacement by intravenous (IV) infusion, pain relief by analgesics, and withholding food or fluid by mouth to restrict pancreatic activity that makes symptoms worse. Most cases resolve within a week; however, some cases can be life threatening. Chronic pancreatitis is caused by alcohol abuse, hemochromatosis (a condition of excess iron in the blood), and other unknown factors. Inflammation and fibrosis cause the destruction of functioning glandular tissue in the pancreas. This results in an inability to properly digest fat due to a lack of pancreatic enzymes. The production of insulin is also affected. Symptoms include abdominal pain (mainly in the upper abdomen), nausea, vomiting, weight loss, and fatty stools. Pancreatic abscess occurs in 5 to 10% of people with acute pancreatitis. An abscess may be caused by inadequate drainage of a pancreatic pseudocyst, which is a complication associated with pancreatitis. Symptoms include fever, chills, abdominal pain, and abdominal mass. Physical exam will show signs of pancreatitis. Pancreatic cancer is the 4th most common cancer causing death in the U.S. The disease is more common in men, especially those between 60 and 70 years. A high fat diet, history of smoking, and chemical exposures may increase the risk. Symptoms include weight loss, abdominal pain, loss of appetite, jaundice, nausea, weakness, fatigue, vomiting, diarrhea, indigestion, back pain, stools (clay colored), pallor, and depression. Only 20% of the tumors are operable at the time of diagnosis. Palliation is generally the treatment, along with chemotherapy and radiation. Insulinomas are generally benign tumors of the insulin-secreting cells of the pancreas, which secrete excess amounts of insulin. Risk factors include a prior history of multiple endocrine neoplasia Type I (MEN I). Symptoms include sweating, tremor, rapid heart rate, anxiety, hunger, dizziness, headache, clouding of vision, confusion, behavioral changes, convulsions, and loss of consciousness. Surgery is the treatment of choice to remove the tumor. If the tumor is not found during surgery, diazoxide may be given. A diuretic is always given with this medication in order to prevent the patient from retaining too much salt.

Case A 57-year-old woman, with a 40-pack-year history of smoking and a long history of type II diabetes, presents with fatigue, middle abdominal pain, and loss of appetite. She describes the pain as diffuse, radiating to the back, and 3/10 in scale. She indicates that these bouts have been coming and going for the past 12 months, and she finally decided to get them checked out. She indicates that she has also lost considerable weight. On physical examination, she appears mildly jaundiced. A CT scan reveals a mass in the pancreas. Question What is the most likely diagnosis?

Correct answer: Adenocarcinoma Explanation The clinical picture is suggestive of chronic pancreatitis that has developed into pancreatic cancer. 90% of all pancreatic cancers are adenocarcinomas. The rest are adenosquamous, anaplastic, and acinar cell carcinomas. Virtually all pancreatic carcinomas (99%) originate in duct cells. Pain is present in over 70% of cases and is often vague, diffuse, and located in the epigrastrum or upper left quadrant when the lesion is in the tail. Cigarette smoking has the strongest overall association and is thought to account for 1/4 of cases diagnosed. A gastrinoma is a tumor that secretes the hormone gastrin. It is commonly found in the duodenum and is associated with Zollinger-Ellison syndrome. Symptoms include diarrhea, epigastric pain, hematemesis, and difficulty eating. Teratoma is a neoplasm consisting of all 3 germ layers. Insulinoma is a tumor of the pancreas; it is associated with beta cells. Sarcomas are neoplasms of connective tissue. Pancreatic cancers are of epithelial cell origin.

Question A 54-year-old male has had long term GERD symptoms. He has been on proton pump inhibitors and has had fair control of his symptoms. Other past history is unremarkable. He is a nonsmoker and drinks socially. Family history is significant for hypercholesterolemia in his father. Physical examination is unexceptional. An endoscopy a few years ago, revealed Barrett's esophagus by biopsy of the esophageal mucosa. He was recommended to have follow-up endoscopy every 2-3 years with mucosal biopsy. This screening was recommended to him because he is in danger of developing

Correct answer: Adenocarcinoma of esophagus Explanation Barrett's esophagus is an abnormality of the distal esophagus characterized by the replacement of normal squamous epithelium by metaplastic columnar epithelium with goblet cells, as a result of continuous inflammation most commonly from acid reflux. This is a premalignant condition with potential to develop into adenocarcinoma over time. Surveillance with screening endoscopy and mucosal biopsy is indicated to detect early onset of high grade dysplasia, which is then treated with esophagectomy. Failure to be compliant with the screening regimen may lead to adenocarcinoma of the esophagus, which has a poor prognosis as it presents late. Achalasia of the cardia is a motility disorder resulting from degeneration of ganglion cells in the esophagus and lower esophageal sphincter. Barrett's esophagus does not predispose to this condition. Esophageal stricture is a distinct complication of long-standing GERD and may co-exist with Barrett's esophagus, since both have the same etiological factor, namely, reflux esophagitis. However, one does not necessarily predispose to the other, and in this patient absence of dysphagia makes the possibility of a stricture unlikely. Squamous cell carcinoma of esophagus is as frequent as adenocarcinoma, but risk factors are smoking and drinking alcohol, not Barrett's esophagus. Dysphagia lusoria occurs when an anomalous blood vessel crosses behind the esophagus causing extrinsic obstruction. The most common vessel is the right subclavian artery.

Question A 50-year-old man is sent for a screening colonoscopy for colon cancer by his primary care physician. The test shows no evidence of cancer but reveals 1 polyp in the right colon. It is removed and sent for tissue biopsy by the gastroenterologist, who asks him to return to his office in 2 weeks for the result and further advice. What biopsy result indicates an increased risk for cancer?

Correct answer: Adenomatous polyp 2 cm in size Explanation Adenomatous polyps are precursors of adenocarcinoma, and the risk is more if the size is more than 1 cm, if it is sessile rather than pedunculated, and if it is villous rather than tubular. Adenomatous polyps with high-grade dysplasia and invasive cancer are at a high risk of developing colon cancer and should be treated aggressively. Removal by colonoscopy is advocated. The relative risk decreases to 2.3 after removal, as compared to those who do not get it removed, for whom the risk is 8. Hyperplastic polyp is the most common type of colon polyp in the general population and rarely premalignant. Mucosal polyps are benign, small, and of unknown significance. Whether pedunculated or sessile, they are not premalignant. Inflammatory pseudopolyps occur in inflammatory bowel disease (IBD) and are themselves not premalignant. They are caused by boggy mucosa between ulcerations and are not dysplastic. IBD, however, is a premalignant condition by itself. Small tubular adenomatous polyps do not increase colon cancer risk significantly.

Case A 65-year-old man presents with crampy and intermittent abdominal pain, nausea, and vomiting. Physical exam demonstrates abdominal distention and hyperactive bowel sounds. The patient has a past surgical history of a prior appendectomy, open cholecystectomy, and 2 prior incisional hernia repairs. Vital signs are as follows: temperature is 99.2°F, pulse is 102 beats/minute, respirations are 15/minute, and blood pressure is 152/86 mm Hg. A plain film of the abdomen is ordered, and it is given below for your interpretation. Refer to the image. Question What is the most likely etiology of this patient's condition?

Correct answer: Adhesions Explanation The most common causes of small bowel obstructions are postoperative adhesions, followed by neoplasms, Crohn's disease, and hernias. Cecal volvulus, although rare, can also cause a small bowel obstruction. Obstruction of the small bowel leads to diffuse bowel dilatation with associated air-fluid levels as increased secretions and air accumulate within and dilate the intestinal lumen. In cases of small bowel obstruction, the resulting bowel dilatation stimulates intestinal cell secretory activity, resulting in more fluid accumulation. In cases of small bowel obstruction, plain films of the abdomen show dilated small-bowel loops centrally with air-fluid levels and absent or minimal colonic gas, as in the image. Treatment of small bowel obstructions rests on relieving the mechanical obstruction; one way of doing so is surgical lysis of adhesions.

Case A 69-year-old edentulous, alcoholic man who lives alone presents for evaluation of a shoulder wound that is not healing well. On physical examination, numerous ecchymoses are noted on the posterior aspect of his legs and thighs. Careful examination of the man's skin reveals minute hemorrhages around hair follicles, and splinter hemorrhages in the nail beds. Laboratory analysis is remarkable for a hemoglobin of 10 (normal 14 - 18 g/dL); no other hematologic abnormalities are noted. Question What treatment should be included in therapy?

Correct answer: Administration of vitamin C Explanation The patient suffers from scurvy due to a deficiency of dietary vitamin C. Absence of vitamin C leads to impaired hydroxylation of proline residues in the nascent procollagen chains leading to weakness of blood vessel walls. Clinically, the deficiency syndrome is characterized by perifollicular hemorrhages, fragmentation of hairs, purpura, ecchymoses, splinter hemorrhages, and hemorrhages into muscle. In patients with normal dentition, gum changes (e.g., swelling, bleeding, and loosening of teeth) are also noted. Without supplementation with vitamin C, death may eventually occur. Administration of factor VIII would be indicated for factor VIII deficiency. This deficiency would lead to a prolonged PTT (partial thromboplastin time), which was not noted in this patient. Administration of iron would be of benefit in iron-deficiency anemia, but there is no indication of a hypochromic, microcytic anemia in this patient. The anemia of scurvy is typically normochromic and normocytic due to bleeding. Administration of vitamin B12 would be indicated for a megaloblastic anemia. Although a macrocytic anemia may be observed in scurvy (due to concomitant dietary folate deficiency or perturbations in the folate pool), this patient did not show macrocytosis. Administration of vitamin K would be appropriate for vitamin K deficiency. This deficiency would produce prolongations of the prothrombin time (PT), followed by prolongation of the PTT as the vitamin K-dependent factors (II, VII, IX, X, protein C, and protein S) are depleted.

Question A 55-year-old man from Southeast Asia is visiting his son in the U.S. for several months. While in the United States, he loses weight and starts to develop abdominal pain, anorexia, and ascites. His son takes him to the family doctor. On physical examination, ascites is present and his liver is enlarged. Interestingly, he is not jaundiced. Additional laboratory results are as follows: TEST RESULTS REFERENCE RANGE SGOT 134 IU/L 5-40 IU/L SGPT 121 IU/L 5-35 IU/L bilirubin (total) 2.8 mg/dL 0.2-1.5 mg/dL alkaline phosphatase 113 U/L 20-70 U/L A liver biopsy is done, and he is found to have hepatocellular carcinoma. Which of the following is most often associated with the development of hepatocellular carcinoma?

Correct answer: Aflatoxins Explanation Consistent with hepatocellular carcinoma are the weight loss, abdominal pain, and anorexia, which this patient experienced. The physical findings of ascites and hepatomegaly are consistent as well. The lab results reflecting elevated transaminases, elevated bilirubin, and elevated alkaline phosphatase are consistent was well. There is an association of aflatoxins and the development of hepatocellular carcinoma. Aflatoxins are produced by Aspergillus flavus. This is a mold that can be seen on peanuts and stored grains. Clonorchiasis and Opisthorchis viverrini infection are associated with cholangiocarcinoma. Cholangiocarcinoma is a tumor that arises from the bile ducts. It is an adenocarcinoma. Macadamia nuts are not associated with the development of hepatocellular carcinoma. Arsenic is associated with the development of angiosarcomas.

Case Ico-delete Highlights A 48-year-old woman presents for follow up of her diarrhea-predominant irritable bowel syndrome. Her main complaints are bloating and cramping; she has been having 2 to 3 bowel movements a day intermittently over the past few weeks. She has been feeling more stressed lately due to a promotion at work. She is having problems sleeping as well, which she attributes to stress. Past medical history is positive for mild premenstrual dysphoric disorder for which she has tried sertraline and fluoxetine; she discontinued these agents due to headaches and worsening diarrhea. Abdominal exam is benign. She asks for something to help her symptoms. Question What medication would you recommend to alleviate the patient's symptoms?

Correct answer: Amitriptyline at bedtime Explanation The correct response is amitriptyline at bedtime. Irritable bowel syndrome (IBS) is a disorder of gastrointestinal motility; it causes abdominal pain, constipation, diarrhea, and/or bloating. The cause of irritable bowel syndrome is unknown; however, many things may aggravate the symptoms. Some patients have mood disorders, but the gastrointestinal and psychological symptoms are not necessarily synchronous in these patients. In others, emotional factors, dietary, drug, or hormonal changes may be associated with changes in gastrointestinal motility. There is an increased incidence of irritable bowel syndrome in individuals who were victims of physical or sexual abuse, either as children or adults; therefore, when evaluating patients with IBS, particularly those with refractory complaints, an investigation for a history of prior abuse is important. Two major clinical types of IBS exist: constipation-predominant and diarrhea-predominant varieties. In constipation-predominant IBS, episodes of constipation are common, and many patients complain of abdominal pain, which may be intermittent and colicky or a dull constant ache. The pain may be relieved by a bowel movement. In diarrhea-predominant IBS, diarrhea often occurs after eating or upon waking. Pain and bloating are common. Treatment of IBS is largely supportive. The chronic nature of IBS should be emphasized, and education about the disorder is important. Increasing dietary fiber can help many patients' constipation. Regular exercise is also helpful, and it can help with psychological stressors that may be exacerbating their bowel symptoms. Tricyclic antidepressants, such as amitriptyline, help many patients with IBS, helping with pain, bloating, and diarrhea. They must be taken chronically, however, to be of use. Thus, amitriptyline at bedtime is the only correct choice of the options given. Selective serotonin reuptake inhibitor (SSRI) agents have also been useful in the management of IBS, but given this patient's failure of two such agents, a trial of amitriptyline is a better choice than trying paroxetine, which is another SSRI. Anticholinergic agents may also be useful, as may diphenoxylate or loperamide for those patients with diarrhea. Narcotics, such as codeine, and anxiolytics, such as benzodiazepines (lorazepam, diazepam),are not good choices for therapy because they have the potential for dependency, and IBS is a chronic condition.

Question A 30-year-old woman presents with a 2-day history of severe pain on defecation; the patient has not had a problem with bowel movements in the past. Recently, she has noticed some increasing constipation; this may be due to starting vitamins that contain calcium. She has had 3 bowel movements in the past 2 days, which she describes as feeling like she was being cut with glass each time she passed them. The pain is excruciating and usually lets up in intensity when the bowel movement ends, but the anal area throbs for 1 - 2 hours afterwards. There is a small amount of blood noted when wiping, but none is noted in the toilet. The patient's past medical history is negative. She takes no prescription medications. There is no family history of colon cancer. On exam, what would you would expect to find?

Correct answer: An anal fissure in the posterior anal verge Explanation The correct response is an anal fissure in the posterior anal verge. This patient presents with a common anorectal problem, which is an anal fissure. Most anal fissures are the result of constipation and hard stools; occasionally they are due to diarrhea. The history presented is typical, and the vast majority of fissures lie posteriorly in the anoderm; occasionally they are found anteriorly. Laterally positioned fissures are highly unusual and should raise a red flag. Conditions that may be associated with abnormally located fissures include Crohn's disease, HIV infection, anal sepsis, and trauma. An anal fistula (fistula in ano) is a communicating tract between the anal canal and the perineal skin. Patients with this condition would complain of a perianal discharge which may be persistent or intermittent; there may also be soreness. Most fistulas are the result of a persistent internal opening in the anal canal from a previous anorectal abscess. A thrombosed external hemorrhoid also presents as acute anal pain; however, in this instance the pain is continuous for 2 - 3 days. In addition, it is usually uncomfortable for the patient to sit down. The patient may report the presence of a highly tender lump in the anal area.

Case A 20-year-old woman presents with a 2-week history of anorectal pain and streaks of blood on her stool and toilet paper. She tells you that "she has a tearing pain during each bowel movement". She dreads having a bowel movement and attempts to hold it as long as she can. Her history is also significant for breaking her leg in a skiing accident 4 weeks ago, for which she was prescribed acetaminophen and oxycodone (Percocet) for the first few days due to her pain. Question Before you examine the patient, what is your top differential diagnosis for this patient?

Correct answer: Anal fissure Explanation Anal fissure is the correct choice. The patient will complain of significant burning, tearing pain with defecation with bright red blood. Most anal fissures are caused by local trauma to this area. This patient was taking acetaminophen and oxycodone, which often has the common side effect of constipation. She most likely passed hard stool that caused a tear in the anodermis. Hemorrhoids are typically not painful unless they are thrombosed. They are usually very itchy; however, hemorrhoids and anal fissure typically present together. Anorectal fistula usually has purulent drainage around the anal area. This can be painful initially when the tract is forming until it bursts. Rectal ulcers are very uncommon and should not be on the top of your list. Anal cancer would be low on this differential due to absence of sexual history mentioned in this question. However, rectal bleeding is the most common initial symptom of anal cancer. There also is a sensation of a rectal mass in a many individuals.

Case A 35-year-old Hispanic man presents for an appointment but is too embarrassed to tell the nurse his chief complaint. You enter the room and coerce him to give you the reason he has come in to seek medical treatment. He admits to severe, intense itching around his anus that has been worsening the last several weeks. He further states that he has noticed increasingly severe and tearing pain in the anal area with each bowel movement. He would rank this pain as a 10/10 on a pain scale and it lasts hours afterward. This intense pain makes him not want to have any bowel movements. He admits to only 1 episode of a small amount of bright red blood on the toilet paper as well as on the stool itself. The patient denies fever, diarrhea, or ever being diagnosed with inflammatory bowel disease. Question What is this patient's most likely diagnosis?

Correct answer: Anal fissure Explanation This patient is most likely suffering from an anal fissure. Anal fissures most often affect infants and middle-aged people. The majority of fissures are considered primary and caused by local trauma, such as passage of hard stool, prolonged diarrhea, vaginal delivery, or anal sex. Presentation of anal fissures is a tearing pain accompanying bowel movements, as well as bright red rectal bleeding that is limited to a small amount noted on the toilet paper or surface of the stool. The patient described all of these components. Patients will also complain of perianal pruritus or irritation, which he also admits to experiencing. According to Bleday and Breen, anal fissure should be suspected based on a history of anal pain that is provoked by defecation and lasts for hours afterward, often with associated anal bleeding. The diagnosis can be confirmed on physical exam by either directly visualizing a fissure (usually in thinner patients) or reproducing the patient's presenting complaints (anal pain) by gentle digital palpation of the posterior (or anterior) midline anal verge. Patients who have a perianal abscess will also experience severe pain, but it's not necessarily associated with bowel movements. There is also visible perianal swelling and erythema. There may also be signs of a fever and malaise and, if the abscess has begun to drain, the presence of purulent rectal drainage. Patients with Crohn's disease often have extremely variable signs and symptoms and possibly will have these for many years prior to diagnosis. Fatigue, prolonged diarrhea and abdominal pain, weight loss, and fever with or without gross bleeding are considered hallmarks of Crohn's disease. Signs and symptoms of anal carcinoma include rectal bleeding (occurring in 45% of patients), anorectal pain or sensation of a rectal mass (30%), or no issues (20%). Issues are often confused with hemorrhoids, causing patients may delay evaluation. Over 80% of patients developing anal cancer have detected human papillomavirus. Internal hemorrhoids create principle issues of bleeding, prolapse, and mucoid discharge. They are usually painless unless they are thrombosed (less common); even when this occurs, internal thrombosed hemorrhoids are less painful than external thrombosed hemorrhoids, so 10/10 pain would be unlikely. Bleeding with hemorrhoids may range from bright red blood that is seen as streaks on the toilet paper to actual dripping of blood into the toilet bowl after a bowel movement. Over time, hemorrhoids potentially will develop into a prolapsed state, resulting in a point in time when patients will note a sense of fullness or discomfort along with mucoid drainage that results in further irritation and soiling of underclothes.

Case A 28-year-old man presents with rectal bleeding. The patient has noticed blood with bowel movements 3 times. The blood is described as bright red in color and small in amount. He also complains of rectal pain, especially with passing hard stools. He has tried some over-the-counter hemorrhoid creams without relief. The patient admits episodic constipation. He denies dark, tarry stools, easy bruising, and prior episodes of rectal bleeding. He has not noticed blood in his urine or with brushing his teeth. He denies nausea, vomiting, diarrhea, fevers, and weight loss. He has no known medical conditions. Family history is negative for gastrointestinal disorders. Social history reveals he is in a heterosexual relationship, and he denies anal intercourse. On physical exam, his abdomen is normal. The anus has no visible protrusions or rash, but there is a very small erythematous and tender area that appears like a "paper cut" or crack in the skin. The patient experiences pain with digital rectal exam (DRE). No masses are noted in the rectal vault. Question What is the most likely diagnosis?

Correct answer: Anal fissure Explanation This patient most likely has an anal fissure. Fissures are small, painful lesions in which the skin has a "cut" or "torn" appearance. The fissure most likely occurred secondary to anal sphincter stretching and tearing with passing of large, firm bowel movements. Fissures can be associated with a small amount of bright red rectal bleeding. Anal cancer could present with anal bleeding, but it is very rare. Fissure or hemorrhoids are much more likely explanations for the bleeding in this patient. External hemorrhoids are a common condition and can be associated with anal pain, constipation, and bright red rectal bleeding. The physical exam differentiates hemorrhoids from fissures. Hemorrhoids are dilated blood vessels and appear as protrusions, either singly or several clustered together, such as a bunch of grapes. Genital warts (condyloma acuminata) can cause irritated anal lesions. They are caused by the human papillomavirus (HPV) and are not typically associated with anal pain, rectal bleeding, or constipation. The appearance is quite different, with a bumpy, "cauliflower-type" lesion. Pilonidal disease is associated with a sinus or abscess formation in the intergluteal cleft. It can produce pain, but it is not associated with rectal bleeding. Pilonidal disease would be easily differentiated in this patient by location.

Case A 35-year-old Hispanic man presents due to a sore that will not heal around his rectum and anal area. This lesion has been draining pus consistently for the last week. He has had intermittent pain with this lesion (4/10 on a 0-10 pain scale) that is made slightly worse when he has a bowel movement (ranking it a 6/10). More recently, especially in the last few days, he has noted pain increases with just sitting. The patient also admits to intermittent periods of itching. He denies fever or diarrhea. No past medical history of inflammatory bowel disease. Physical examination of the anorectal area reveals excoriated and inflamed perianal skin with a palpated induration. Question What is this patient's most likely diagnosis?

Correct answer: Anal fistula Explanation The patient in this scenario is experiencing signs and symptoms from an anorectal fistula. Anorectal fistulas are commonly a chronic manifestation of an acute perirectal process that eventually comes from an anorectal abscess. Many fistulas originate from an infected anal crypt gland. Patients with anorectal fistulas usually will present with a "non-healing" area (an anorectal abscess that is draining) or may describe chronic purulent drainage and a pustule-like lesion on the perianal or buttock area. They will only have intermittent pruritus or rectal pain; pain is increased during defecation, sitting, or standing. Physical examination reveals excoriated and inflamed perianal skin. An external opening of the fistula may be visualized. If no opening is seen, palpation will reveal induration just below the skin. Anal fissure is not the most likely diagnosis in this case scenario. Anal fissures most often affect infants as well as middle-aged individuals. The majority of fissures are considered primary and caused by local trauma such as the passage of hard stool, prolonged diarrhea, vaginal delivery, or anal sex. Presentation of anal fissures is a tearing pain accompanying bowel movements as well as bright red rectal bleeding that is limited to a small amount noted on the toilet paper or surface of the stool. The patient described all of these components. Patients will also complain of perianal pruritus or irritation, which he also admits to experiencing. Patients with Crohn's disease often have extremely variable signs and symptoms and possibly will have these for many years prior to diagnosis. Fatigue, prolonged diarrhea and abdominal pain, weight loss, and fever with or without gross bleeding are considered hallmarks of Crohn's disease. Signs and symptoms of anal carcinoma include rectal bleeding (occurring in 45% of patients), anorectal pain or sensation of a rectal mass (30%), or even no issues (20%). Very often issues are confused with hemorrhoids, so patients may delay evaluation. Over 80% of patients developing anal cancer have detected human papillomavirus. Internal hemorrhoids create principle issues of bleeding, prolapse, and mucoid discharge. Bleeding may range from bright red blood that is seen as streaks on the toilet paper to actual dripping of it into the toilet bowl after a bowel movement. Over time, hemorrhoids potentially will develop into a prolapsed state achieve a point in time when patients will note a sense of fullness or discomfort along with mucoid drainage that results in further irritation and soiling of underclothes. References

Case A 27-year-old man is admitted to the emergency department with frank bleeding per rectum. Large bleeding polyps are noted in the colon, for which a subtotal colectomy is performed. On careful review, it comes to light that both his father and grandfather had problems with colonic polyps, resulting in surgery. Shown below is a picture of his colon. Question What measures should have been initiated in this patient?

Correct answer: Annual colonoscopy Explanation The picture shows an excised portion of colon with multiple polyps. With the given history, a diagnosis of Familial Adenomatious Polyposis (FAP) is likely. Annual colonoscopy or sigmoidoscopy should be initiated early in patients with a history of familial adenomatous polyposis. Being an autosomal dominant disease with a high degree of penetrance, screening is essential. Although not clearly defined, most authorities recommend yearly endoscopy, starting with the onset of symptoms or in early teenage years if asymptomatic. A colectomy is usually performed if dysplasia is present or based on clinical judgment. Annual fecal occult blood testing is incorrect. It is not as sensitive as colonoscopy in detecting polyps and does not allow a biopsy. Annual upper GI endoscopy is incorrect. Polyps are usually in the colon. Annual CEA levels is incorrect. Annual CEA levels are not used in screening at all. They are used in the follow up of colonic malignancy. Prophylactic colectomy at 1 year is incorrect. Colectomy is done when dysplasia is noted, or when polyps are numerous.

Case A 28-year-old man presents with rectal bleeding. The patient has noticed blood with bowel movements 3 times. The blood is described as bright red in color and small in amount. He also complains of rectal pain, especially with passing hard stools. He has tried some over-the-counter hemorrhoid creams without relief. The patient admits episodic constipation, which he attributes to a poor, low-fiber diet when traveling. He denies dark, tarry stools, easy bruising, and prior episodes of rectal bleeding. He has not noticed blood in his urine or with brushing his teeth. He denies nausea, vomiting, diarrhea, fevers, and weight loss. He has no known medical conditions. Family history is negative for gastrointestinal disorders. Social history reveals he is in a heterosexual relationship and denies anal intercourse. On physical exam, abdomen is normal. The anus has no visible protrusions or rash; however, there is a very small, erythematous and tender area that appears like a "paper cut" or crack in the skin. The patient experiences pain with digital rectal exam. No masses are noted in the rectal vault. Question If further diagnostic testing was desired and tolerated by the patient, which of the following tests would be the next most appropriate for this patient's suspected condition?

Correct answer: Anoscopy Explanation This patient most likely has a anal fissure. Fissures are small, painful lesions, in which the skin has a "cut" or "torn" appearance. The fissure most likely occurred secondary to anal sphincter stretching and tearing with the passing of large, firm bowel movements. Fissures can be associated with a small amount of bright red rectal bleeding. Fissures are usually diagnosed clinically, but if any testing is needed to confirm, anoscopy is the test of choice. Anoscopy uses a short metal or plastic tubular device to examine the distal rectum and anal canal. Anoscopy would likely be very painful for this patient. Abdominal CT with oral and intravenous contrast is a useful test for many abdominal complaints. However, this patient's history and exam do not support abdominal etiology, and an abdominal CT would not be helpful. Barium enema is a specialized x-ray of the abdomen used to visual the colon. It may be used in evaluation of polyps, masses, malignancy, and bowel problems. It would not be helpful for evaluation of a fissure and would, in fact, be very painful for this patient. Colonoscopy is the evaluation of the entire colon via a specialized fiber optic scope introduced via the anus. It is helpful for a large variety of colonic issues (polyps, malignancy, ulcerative colitis, Crohn's disease, diarrhea, constipation, etc.). Colonoscopy is not used for fissures. Sigmoidoscopy is similar to a colonscopy, except that only the lower one-third of the colon (the sigmoid) is evaluated via fiber optic scope. As with colonscopy, this test is overkill in a young individual with no "red flags" for gastrointestinal malignancy. If the initial workup and recommended treatments do not lead to resolution, further testing would then be indicated.

Case A 42-year-old woman who uses IV drugs presents with vague symptoms of fatigue, aches, pains, and nausea. Recently, she appeared jaundiced for a few days, but this has passed. Preliminary blood tests indicate the patient does not have hepatitis B, but the tests could be wrong, as she may be in the window period. Question If she were in the window period, what would be the only evidence of hepatitis B infection?

Correct answer: Anti-HBc Explanation Hepatitis B is transmitted by exposure to blood or blood products. If she shared needles when using IV drugs, she has put herself at risk of infection with hepatitis B. The window is the period in which HBsAg (HBV surface antigen) is not detectable and anti-HBs (antibody to HBV surface antigen) has yet to appear. During the window period of a hepatitis B infection, the only marker present would be IgM anti-HBc (antibody to HBV core antigen). Because of this, the window period is also referred to as the core window. This IgM antibody is present in the early convalescence period (2-16 weeks after infection). It is an expensive test and is ordered only when there is very high suspicion. Positivity of this antibody, along with HBsAg, indicates acute infection. PCR test for genome identification is the other test useful during the window period. HBsAg (HBV surface antigen) is the first serological marker to appear in hepatitis B infection. After HBsAg is cleared from the serum, the patient is in the window period. HBcAg is the HBV core antigen. Although it is present during infection, antibody develops quickly, so it is usually undetectable. HBeAg is seen with acute infection and is a marker of infectivity. Anti-HBe develops during convalescence. Anti-HBs (antibody to HBV surface antigen) is seen at the end of an infection. The appearance of anti-HBs marks the end of the window period. The presence of anti-HBs gives a patient immunity.

Case A 42-year-old female IV drug user experiences vague symptoms consisting of fatigue, aches and pains, and nausea. She has developed distaste for her cigarettes. She appeared jaundiced for a few days, but the condition spontaneously resolved. She goes to the free clinic and they run preliminary blood tests. The staff at the free clinic tells the patient she does not have hepatitis B; however, they are wrong. In reality, she is in the window period. Question What would be evidence of a hepatitis B infection during the window period?

Correct answer: Anti-HBc Explanation Hepatitis B is transmitted by exposure to blood or blood products. Since this patient has used IV drugs, if she shared needles, she has put herself at risk to infection with hepatitis B. The window is the period in which HBsAg (HBV surface antigen) is not detectable. Although anti-HBs (antibody to HBV surface antigen) are present, they are actively bound to HBSAg, and therefore not detectable. During the window period of a hepatitis B infection, the only marker present would be anti-HBc (antibody to HBV core antigen). Because of this, the window period is also referred to as the core window. HBsAg (HBV surface antigen) is the first serological marker to appear in hepatitis B infection. After HBsAg is cleared from the serum, the patient is in the window period. Anti-HBs (antibody to HBV surface antigen) is seen at the end of an infection. The appearance of anti-HBs marks the end of the window period. The presence of anti-HBs gives a patient immunity. HBeAg (HBV "e" antigen) can be seen with acute hepatitis. It is a marker for infectivity.

Case A 28-year-old man presents with recurrent anal discomfort. He has pain with every bowel movement, and the pain will often last for several hours after defecation. He notes blood on the toilet tissue after wiping his anal area. He said he had similar symptoms about 1 year ago, but they resolved without treatment; however, this time he has experienced the pain for several weeks without resolution. He has tried warm sitz baths (which help the pain), but the episodes continue. He denies any anal intercourse. Past medical history is negative. Physical exam reveals a 1 cm narrow ulcer in the posterior midline segment of the anal canal. No fistulae are seen. Question What would you advise?

Correct answer: Apply topical nitroglycerin 0.2% to the perianal area Explanation The patient should apply topical nitroglycerin 0.2% to the perianal area. This patient has an anal fissure. Anal fissures are thought to be caused by anal trauma from hard or large stools; often the fissures become secondarily infected. Fissures often heal with conservative therapy (e.g., warm sitz baths and stool softeners) to improve constipation and lessen trauma to the area during defecation. Glycerin suppositories can help with lubrication of the anal canal while the fissure is healing. For patients who fail conservative therapy, topical nitroglycerin has been shown to be a highly effective and well-tolerated treatment for anal fissure. Nitroglycerin is a smooth muscle relaxant, thereby decreasing anal sphincter pressure. Some patients complain of headaches with nitrate therapy. For those patients who fail a trial of topical nitroglycerin, surgery may be indicated. The other answers are not good choices. Sitz baths should always be warm, hot. Hydrocortisone suppositories can be useful for hemorrhoids, but they may interfere with the healing of a fissure and have not been shown to be beneficial in this condition. Sublingual nitroglycerin is not indicated, nor is loperamide; it would worsen this patient's constipation and anal trauma.

Case A 48-year-old Caucasian woman with a chronic history of inability to tolerate oral intake is admitted to the hospital for J-tube placement. The patient also reports a rash that has developed on her upper extremities. Examination reveals diffuse petechiae and perifollicular hemorrhage. Question What is the most likely etiology of the rash?

Correct answer: Ascorbic acid deficiency Explanation The correct answer is ascorbic acid deficiency. This patient has scurvy due to ascorbic acid or vitamin C deficiency. Patients with scurvy typically report a history of a poor diet (tea and toast), or in this case, malnutrition secondary to a chronic inability to tolerate oral intake of foods. Patients with scurvy also present with skin petechiae, ecchymosis, and characteristic perifollicular hemorrhage. Note that lysyl oxidase, an important enzyme involved in cross-linking collagen, utilizes vitamin C. Niacin deficiency usually presents as the 3 Ds: diarrhea, dermatitis, and dementia. Patients on corn diets or those with an inability to reabsorb tryptophan (Hartnup disease) present with this condition. The tongue develops bright red glossitis and a pigmented, scaling rash is common. Cobalamin deficiency presents with megaloblastic anemia and peripheral neuropathy. Patients may also present with neurologic disease (subacute combined degeneration). Biotin deficiency presents with dermatitis, alopecia, and lactic acidosis. A scaling or erythematous rash may also occur around the eyes, nose, mouth, and extremities. Choline deficiency is a risk in patients on parental nutrition. Manifestations are mainly elevated liver enzymes and fatty liver infiltration.

Case A 40-year-old woman presents with a 1-month history of having at least 3 pasty, malodorous bowel movements each day; the bowel movements are hard to flush. The patient is not taking any medications. On examination, she has mild pallor and pedal pitting edema. Her BP is 110/60 mm Hg. She has evidence of osteoporosis on DEXA scan. She states that her father had similar symptoms and died from some form of intestinal cancer. Question What is the most likely diagnosis?

Correct answer: Celiac disease Explanation Celiac disease is a hereditary disorder caused by gluten intolerance. Gluten is found in wheat, rye, and barley. Patients may be asymptomatic or may present with diarrhea, abdominal discomfort, distention, and steatorrhea (stools are pale, malodorous, and difficult to flush because they float on the toilet water). Patients may have anemia from iron and folate deficiencies; osteoporosis from calcium deficiency; and edema from hypoproteinemia. Diagnosis is confirmed by a small intestine mucosal biopsy which shows a flat mucosa due to villous atrophy, and by subsequent improvement on a gluten-free diet. There is a higher incidence of cancers of the GI tract such as lymphoma in patients with celiac sprue. Specific treatment includes a gluten-free diet. Whipple's disease mainly affects men; it is caused by the bacterium Tropheryma whippelii. It is a multisystemic disease that affects the small intestines, joints, brain, heart, and eyes. Patients can present with diarrhea, steatorrhea, abdominal pain, weight loss, and joint pains. On examination, patients may be pale with lymphadenopathy. Histological examination of a small bowel mucosal biopsy reveals PAS-positive, foamy macrophages. Treatment is with trimethoprim-sulfamethoxazole or chloramphenicol. Short bowel syndrome is usually the result of surgical resection of the intestines or a jejunoileal bypass. The malabsorption is a result of inadequate absorptive surface. Malabsorption of vitamin B12 results in paresthesias; malabsorption of calcium results in bone pain and carpopedal spasms. Tropical sprue is an acquired disease that affects both visitors and natives of tropical areas (e.g., the Caribbean and South India). Its etiology is unknown. Patients usually present with diarrhea and weight loss; they report that their stools are soft and bulky (steatorrhea). They may also develop deficiencies of folate and cobalamin. Stool microscopy should be done to look for cysts and trophozoites. Histological examination of a small bowel mucosal biopsy aids in making the diagnosis. Treatment is with tetracycline or oxytetracycline. Lactose intolerance results from a deficiency of lactose, which is a disaccharidase in the mucosal cells of the small intestine; it splits the disaccharide lactose into glucose and galactose. Patients complain of borborygmi, flatulence, nausea, abdominal cramps, pain, and diarrhea after ingesting lactose-containing food (e.g., milk). Treatment includes following a lactose-free diet. Osteomalacia is not a presenting feature.

Case A 7-year-old girl presents because her parents are concerned about her growth. Her prenatal history was normal; she sat up when she was 6 months old, walked without assistance at 10 months; all other milestones of the development were normal. Her immunizations are up to date, and she has had no serious illnesses. On examination, you find a cooperative and friendly child. Except for a short stature, physical findings are normal. Question What will your next diagnostic step be?

Correct answer: Ask about parents' growth Explanation Because the girl is short, all methods to confirm short stature and determine the cause may be implemented. However, you should first ask about the growth pattern of both parents, because they are already in your office and the most common cause is familiar short stature. Growth curve examination is your next step to evaluate growth. Short stature is defined as growth below the third percentile. Bone age, as determined by plain radiograph of left hand and wrist, will match bone age to chronological age. If there is a discrepancy, one would consider causes other than familiar or constitutional delay. Nutritional factors can be the cause of growth delay, and such an examination should be performed after you exclude more common factors. You should first ask about the growth pattern of both parents. Psychosocial issues can be a cause of growth delay. However, you should always ask about the growth pattern of both parents first.

Case A 73-year-old man presents to the emergency department after a suspected overdose. History is unobtainable; he is currently febrile; his blood pressure is 125/76 mmHg; and laboratory results are as follows: Na+ 141 mEq/dL K+ 4.1 mEq/dL Cl- 107 mEq/dL HCO3- 11 mEq/dL Creatinine 0.9 mg/dL Glucose 118 mg/dL Serum pH 7.11 Question Which of the following drugs was most likely taken?

Correct answer: Aspirin Explanation The correct answer is aspirin overdose, as it initially causes a respiratory alkalosis, eventually leading to a high anion gap acidosis as noted above. A high anion gap acidosis occurs when HCO3- is lost without a concomitant increase in Cl. Lithium, bromine, acetazolamide, and topiramate toxicities do not cause an increase in the anion gap as demonstrated above.

Question 4 hours after eating fried rice at a restaurant, a 30-year-old woman, her husband, and his sister developed nausea, vomiting, and diarrhea. What organism is likely to be involved in the patients' condition?

Correct answer: Bacillus cereus Explanation Bacillus cereus cause 2 syndromes: (1) one involves a short incubation period with nausea and vomiting and is similar to staphylococcal food poisoning; (2) the other involves a long incubation period (18 hours) with watery, non-bloody diarrhea and resembles clostridial gastroenteritis. Clostridium perfringens cause food poisoning with an 8- to 16-hour incubation period. It is characterized by watery diarrhea with cramps and little vomiting. It resolves in 24 hours. Staphylococcus aureus cause food poisoning characterized by vomiting being more prominent than diarrhea. The disease is due to ingestion of enterotoxin, which is performed in foods and hence has a short incubation period (1-8 hours). Vibrio parahaemolyticus are a marine organism transmitted by contaminated seafood, especially when raw fish is eaten. The clinical picture varies from mild to quite severe watery diarrhea, nausea and vomiting, abdominal cramps, and fever. The illness is self-limited, lasting about 3 days. Salmonella enteritidis cause enterocolitis after an incubation period of 6-48 hours. The disease begins with nausea and vomiting and then progresses to abdominal pain and diarrhea, which can vary from mild to severe and present with or without blood. Usually, the disease lasts a few days and is self-limited. Bacterial diarrheal diseases transmitted by foods Bacterium Typical food Disease Gram-positive cocci Staphylococcus aureus Custard-filled pastries; potato, egg, or tuna fish salad Food poisoning, especially vomiting Gram positive rods Bacillus cereus Reheated rice Diarrhea Clostridium perfringens Cooked meat, stew, gravy Diarrhea Gram negative rods Escherichia coli Various foods and water Diarrhea Escherichia coli O157:H7 strain Undercooked meat Hemorrhagic colitis Salmonella enteritidis Poultry, meats, and eggs Diarrhea Shigella species Various foods and water Diarrhea (dysentery) Vibrio cholerae Various foods, e.g. seafood, and water Diarrhea Vibrio parahaemolyticus Seafood Diarrhea Campylobacter jejuni Various foods Diarrhea Yersinia enterocolitica Various foods Diarrhea

Question A 30-year-old woman presents with diarrhea and abdominal cramps which started earlier in the day. She does not remember eating any meat, chicken, pudding, or ice cream the previous day. She lives alone and she says she warmed up leftover rice for supper last night. She looks sick, but not dehydrated. She has no fever and her blood pressure and pulse are within normal limits. What is the most likely cause of her symptoms?

Correct answer: Bacillus cereus Explanation The correct response is Bacillus cereus. This patient is suffering from acute food poisoning from reheated rice. The most common organism that might cause this condition is Bacillus cereus, a Gram-positive, spore-forming rod. Spores on grains (e.g., rice) survive steaming and rapid frying; the spores germinate when rice is kept warm for many hours. B. cereus produces 2 enterotoxins. The mode of action of 1 of the enterotoxins is the same as that of cholera toxin (i.e., it ADP-ribosylates a G protein which stimulates adenylate cyclase and leads to an increased concentration of cyclic AMP within the enterocyte). The mode of the other enterotoxin is uncertain. Clinically, it is a self-limiting situation and requires supportive treatment only. The incubation period is about 18 hours. See the table below for differential diagnoses.

Case A 2-day-old male neonate is noticed to have some abdominal distension. The parents are concerned since he has not yet passed his first meconium. A few hours later, he starts vomiting and the abdominal distension increases. He continues to throw up all day and his vomitus is noted to be greenish-yellow. The baby is restless, irritable, and constantly crying. On exam, he has a temperature of 100.4°F and pulse 112/minute. His abdomen is distended and tympanic; rectal exam reveals no stool. The physician considers Hirschsprung's disease. Question What should be ordered first?

Correct answer: Barium enema Explanation The neonate probably has Hirschsprung's disease, a congenital motility disorder of the colon caused by lack of innervation in a short segment of the colon, usually the rectosigmoid. This results in failure of relaxation of that part and functional obstruction. Failure of passage of meconium in the first 48 hours is suggestive. If not treated quickly, sometimes enterocolitis can occur with fever, sepsis, and life-threatening toxic megacolon, as is the case in this patient. Barium enema shows a narrowed segment with a dilated proximal colon. Rectal biopsy is the standard and will reveal absence of ganglion cells in the affected segment. Treatment is surgical. The goal is to resect the affected segment, bring down normal ganglionic bowel up to the anus, and preserve sphincter function. It may be done as two-step procedure with an initial diverting colostomy followed by definitive repair or a single stage repair. It should be done soon after diagnosis, often within the first few days or months of diagnosis. Emergency surgery is indicated in Hirschsprung-associated enterocolitis (HAEC), which is the most lethal complication. Volume resuscitation, intravenous antibiotics, and rectal irrigation are the supportive measures. Ultrasound of the abdomen will be non-diagnostic and is not indicated. Anal manometry is a good screening test for this condition but is less accurate in newborns (less than 1 month old). It demonstrates lack of relaxation of the internal anal sphincter on rectal distension. Endoscopy is not indicated, as visualization is adequate with barium enema; rectal biopsy is done by a mucosal suction technique. Blood cultures may be done in a septic baby for antibiotics, but it is non-diagnostic for the cause of the obstruction.

Question What is the preferred treatment for infantile intestinal obstruction due to intussusception?

Correct answer: Barium or air enema Explanation In cases of childhood intussusception, the most common location is in the right colon. The treatment of choice is a low-pressure barium enema to reduce the intussusception. If this procedure is not successful, then laparotomy with manual reduction is indicated. Reduction of the intussusception is necessary in order to prevent ischemia and gangrene of that segment of intestine

Case A 65-year-old man presents because this morning a morsel of meat he had eaten 3 days ago reappeared on his pillow. About a year ago, he noticed difficulty swallowing, particularly solid foods, which seems to be worsening. His wife complains about his bad breath, and he noticed that people are avoiding being close to him. He does not drink, does not smoke, and was in a good health before. His physical examination is within normal range for his age, except that you notice that he is repeatedly clearing his throat, as if he is embarrassed. Question What is the next best step in the management of this patient?

Correct answer: Barium study Explanation The recollection of slow progressing difficulties in initiating swallowing and regurgitation in an older patient makes you suspect Zenker diverticulum. Your clinical diagnosis is supported by bad breath and throat clearing. Definitive diagnosis is made by barium studies. Endoscopy is actually contraindicated in this patient because the lesion is proximal and there is a risk of perforation of the pharynx. Endoscopic ultrasound is performed in patients with esophageal carcinoma for staging. Your patient has no typical signs of esophageal carcinoma (progressive dysphagia for solid foods first, followed by liquids, with weight loss in a smoker and/or alcoholic). That diagnosis is not excluded; however, the probability of Zenker diverticulum will prevent you from choosing endoscopy as the first step in management. 24 hours pH monitoring should be considered in a patient with symptoms of gastroesophageal reflux (GERD) when the diagnosis is not clear. GERD will present with epigastric or substernal pain, sore throat, metallic taste in mouth, hoarseness, cough, and wheezing. It is not likely that your patient has GERD. Manometry should be considered if you suspect a disorder of esophageal peristalsis, as is the case in achalasia, esophageal spasm, nutcracker esophagus, or lower esophageal sphincter diseases. These disorders can present with dysphagia for both solid foods and liquids, sometimes accompanied by pain and weight loss. Manometry includes the placement of a nasogastric tube, and the procedure will increase the risk of pharyngeal perforation. Manometry should be performed only after barium studies exclude Zenker diverticulum.

Case A 60-year-old man presents with a 6-month history of dysphagia to solids, regurgitation of undigested food, and halitosis. He denies a decrease in appetite, abdominal pain, weight loss, or change in bowel habits. His past medical history is significant for a total hip replacement. His lab work is as follows: Hemoglobin 14.0 g/dL Hematocrit 43% White cell count 6.0 x 109/L Platelets 300 x 109/L Sodium 135 mEq/L Potassium 4.0 mEq/L Urea 6 mg/dL Creatinine 0.6 mg/dL Glucose 98 mg/dL Chloride 100 mEq/L HCO3- 25 mEq/L His liver panel is normal. Question The diagnosis can be determined initially by what technique?

Correct answer: Barium swallow Explanation The patient may have a Zenker's diverticulum, which could predispose the patient to an aspiration pneumonia. Zenker's diverticulum is an outpouching of the posterior pharyngeal wall immediately above the upper esophageal sphincter. Collection of food particles in the pouch results in halitosis and the symptom of regurgitation of undigested food particles. The diagnosis is likely to be Zenker's diverticulum because the patient has dysphagia to solids, regurgitation of undigested food particles, and halitosis. Barium swallow is the diagnostic procedure of choice because it is the least invasive. Esophageal manometry is normally used in evaluating patients with noncardiac chest pain, motor disorders of the esophagus, achalasia, or gastroesophageal reflux disease. MRI of the head and neck is not routinely used for the diagnosis of Zenker's diverticulum. Radionuclide scanning would be used in evaluating the patient for a thyroid tumor, which is low on the differential list for this patient based on his symptoms (e.g., lack of weight loss or appetite loss). As part of the workup before surgery (to reduce the size of the diverticulum), an upper endoscopy is used to rule out esophageal cancer.

Case A 52-year-old woman with a 5-year history of heartburn presents with difficulty in swallowing food. She is able to drink fluids without any problem, but solids go down with difficulty. She denies any weight loss or weight gain. She has had dyspepsia-like symptoms for a long time, with belching and abdominal fullness after meals. Her medications include the intermittent use of over-the-counter ranitidine and acetaminophen for mild aches and pains. Her vitals are stable, and the physical examination is unremarkable. Question What is the next step in the evaluation of this patient?

Correct answer: Barium swallow Explanation This patient has had a long standing gastrointestinal reflux disease (GERD), which is obvious based on her symptoms. She has not used H2 blockers consistently as she should have. They would have likely reduced or eliminated her symptoms. Now, due to repeated acid-induced injury to the esophageal lining, she has developed symptoms suggestive of an esophageal stricture; this has caused dysphagia, which usually starts with solids and progresses to both solids and liquids. A barium swallow is indicated because it is the least invasive and will give information regarding the location, length, number of strictures, and size of the lumen. Relative to endoscopy, it is more sentitive at detecting small luminal narrowings or irregularities, especially in the upper esophagus. It will also impact the next step in the treatment, which often includes endoscopic dilatation of the stricture(s).Dysphagia due to other pathologies, like esophageal diverticulum, extra-luminal pathology, malignancy, and/or perforation, can be ruled out too. Empiric treatment with proton pump inhibitors is indicated in simple GERD without symptoms of stricture. Once the patient has dysphagia, however, it is important to evaluate them for complications, like stricture, especially if they have had long standing GERD. 24-hour esophageal pH monitoring is usually used to correlate pH to symptoms in patients with GERD who have failed typical treatments. In this patient's condition, the most concerning new finding is dysphagia, and pH monitoring would not be useful in determining a cause for this symptom. Swallow evaluation and video fluoroscopy permit evaluation of the cervical esophagus only. The rest of the esophagus cannot be evaluated; therefore, this option is inadequate. These tests are more important for pathology suspected in the pharynx, larynx, and upper esophageal sphincter. Esophageal manometry is indicated in motility disorders of the esophagus. Motility disorders present with dysphagia to both liquids and solids, as in achalasia, esophageal spasms, and scleroderma. Manometry is indicated when a barium swallow has ruled out the more common causes of dysphagia.

Case A 4-day-old male infant is seen for a routine newborn check; he has yellow skin from his face down to his chest. He is a full-term infant; vaginal delivery was normal. He weighed 3.180 kg at birth, and 3.100 kg at discharge. He is breastfeeding well; he is also stooling and voiding regularly. He is well-hydrated and active. Laboratory results reveal a total serum bilirubin of 15.7 mg/dL (normal <10 mg/dL), with elevation in both direct bilirubin and indirect bilirubin. Question Based on the presentation and labs, what is the most likely cause of this infant's jaundice?

Correct answer: Biliary atresia Explanation The correct response is biliary atresia. Breastfeeding contributing to dehydration and intensified physiologic jaundice, breast milk jaundice, and birth trauma causing cephalohematoma with increased hemolysis are all possible causes of indirect hyperbilirubinemia. Since the infant is 4 days old, his indirect bilirubin level is not so high as to require treatment or concern at this time. Any direct bilirubin >2.0 mg/dL is abnormal. If it is persistent or increasing it is considered pathologic; it must be evaluated immediately. TPN cholestasis and biliary atresia can both contribute to direct hyperbilirubinemia. TPN cholestasis typically occurs when an infant has received hyperalimentation for an extended period of time, so it would not occur in a 4-day-old infant. Cholestasis is impaired excretion of the conjugated bilirubin in bile. The most common hepatic cause of jaundice of direct hyperbilirubinemia in infancy is biliary atresia. Ultrasound should be performed to identify blockages or anatomical anomalies.

Question Which of the following is true regarding Mallory-Weiss syndrome?

Correct answer: Bleeding stops spontaneously in the majority of cases Explanation Mallory-Weiss syndrome involves a tear of the mucosa and submucosa of the lower esophagus or gastric cardia and accounts for up to 9% of cases of significant upper gastrointestinal bleeding. This syndrome results from forceful events that increase intra-abdominal pressure, including trauma, seizures, coughing, and, most commonly, vomiting, with retching preceding hematemesis in approximately 50% of patients. Mallory-Weiss syndrome is associated with alcoholism, hiatal hernias, esophagitis, or gastritis. The majority of episodes involve only mild to moderate bleeding that stops spontaneously, and surgery is rarely required. Only 3% of deaths from upper GI bleeding are due to Mallory-Weiss tears. Endoscopy is the diagnostic procedure of choice if performed within the first 12 to 24 hours. Esophagitis secondary to reflux and repeated vomiting is the most common cause of upper GI bleeding during the pregnancy.

Case An 18-year-old man presents for a screening physical exam prior to joining his college's freshman lacrosse team. He reports no medical complaints, and he does not take any medications. Physical exam is unremarkable. His immunizations are up-to-date, and he denies sexual activity or smoking. Review of routine labs reveals an elevation in unconjugated bilirubin. Liver enzymes, serum electrolytes, and complete blood count are within normal limits, as is conjugated bilirubin level. Question What is the most likely diagnosis?

Correct answer: Gilbert's syndrome Explanation The asymptomatic, isolated elevation of unconjugated bilirubin in this patient is most likely caused by Gilbert's syndrome. The underlying pathology in Gilbert's syndrome is underactivity of the conjugating enzyme bilirubin-uridine diphosphate glucuronyl transferase. The condition is relatively common; there is an estimated frequency of about 5% in the United States. Aside from mild intermittent jaundice, patients are usually asymptomatic. Several conditions may exacerbate the condition, including stress, menstruation, and dehydration. The condition is benign, and no treatment or physical limitations are required. The most important consideration in a patient presenting with an elevated unconjugated bilirubin level is exclusion of other more serious causes, including ongoing hemolysis and liver disease. The results of physical exam and other blood work make these diagnoses unlikely in this patient. Hepatitis, whether due to alcohol or infection, usually results in elevated liver enzymes and is associated with other constitutional symptoms, including nausea, fever, and liver enlargement. Treatment is generally supportive; further care is directed to the etiology of the hepatitis. Crigler-Najjar syndrome is a rare and far more serious autosomal recessive disorder of bilirubin metabolism; it is caused by the absence or deficiency of the enzyme uridine diphosphate glycosyltransferase. The condition presents at birth with persistent neonatal jaundice and significantly elevated unconjugated bilirubin levels in the presence of otherwise normal liver function tests. Recognition and intervention must be prompt in order to avoid kernicterus and permanent neurological impairment. Treatment involves plasma exchange and ongoing phototherapy to facilitate bilirubin conversion to soluble forms that can be excreted in the urine. Dubin-Johnson syndrome is a relatively rare cause of hyperbilirubinemia; it is mainly of the conjugated form. Patients are often asymptomatic or have mild jaundice and other nonspecific constitutional complaints. The characteristic finding in these patients is dark pigmentation concentrated in centrilobular hepatocytes, making the liver appear black. The pathophysiology of Dubin-Johnson syndrome is an abnormality in urinary excretion of coproporphyrin. No specific treatment is indicated.

Case A 65-year-old man presents with watery diarrhea and mild abdominal pain. He has had about 8 or 9 non-bloody bowel movements per day for the last 10 days; he currently feels a general abdominal discomfort. On examination, his pulse rate, blood pressure, and oral temperature are normal. There is no abdominal tenderness. The stool guaiac test is negative. He does not travel. He eats home-cooked meals most of the time, and he reports using water from his private well for all of his needs. Question What is the ideal recommendation in this case?

Correct answer: Boil water before using and ask the US Environmental Protection Agency for advice on well decontamination and maintenance. Explanation The water should be boiled before it is used, and the patient should ask the US Environmental Protection Agency for advice on well decontamination and safe maintenance. The symptoms are most likely caused by infection with an enterotoxigenic strain of E. coli, the bacteria commonly associated with gastroenteritis, which is also known as traveler's diarrhea. The patient did not travel, but he drinks water from a private well that is most likely contaminated. Coliforms are naturally present in the environment, and pathogenic strains can occasionally infect water sources if they are not well maintained. Boiling the water is sufficient to free it from such contaminants. The answer 'close the well and never use it again' is not correct. Such drastic measures are only required if contamination is recurring and the water does not become clean and safe to drink after following standard decontamination procedures. The answers 'filter the water before using and ask the US Environmental Protection Agency for advice on well decontamination and safe maintenance' and 'chlorinate water before using and ask the US Environmental Protection Agency for advise on well decontamination and safe maintenance' are not correct. Most in-home available filters do not remove all bacteria and viruses from water. This method is less effective than boiling. Chlorine, UV-radiation, and ozone are the 3 main water decontaminants, but strict procedures have to be followed. It is not advised to drink highly chlorinated water. The answer 'send samples of water for lab analysis to identify possible contaminants is incorrect. Such analysis of the well-water should be performed as part of a regular maintenance procedure to ensure the water is safe to drink. Under this circumstances, boiling water before consumption is the best solution until proper well decontamination is performed.

Case A 25-year-old woman who is 36 weeks pregnant presents for her regular obstetrics care examination. She was diagnosed with Hepatitis C a couple years prior. No viral RNA was detected during her pregnancy, which progressed smoothly and without major discomfort. She has no history of intravenous drug use or blood transfusions. She is HIV negative. She plans on breastfeeding her baby, but is concerned about transmitting the Hepatitis C virus to the newborn. Question What do you advise this patient regarding breastfeeding her infant?

Correct answer: Breastfeed, but temporarily stop if nipples become cracked or bleed. Explanation The answer breastfeed, but temporarily stop if nipples become cracked or bleed is correct, because transmission of the HVC virus through breast milk has not been observed. However, cracked and bloody nipples could facilitate the infection of the baby, as hepatitis C is a bloodborne illness. The answer breastfeed, because the virus will be transmitted during birth anyway is incorrect. Even though Hepatitis C is a bloodborne disease that can be transmitted from an infected mother to her newborn, when no viral RNA is detected in the mother, the baby is not infected. Moreover, this patient has no history of intravenous drug use or HIV. The risk of viral infection for her baby is low. The answers breastfeed, because there is no risk to the baby and breastfeed, but maintain good hygiene of the nipples at all times are incorrect, because even though it is generally correct that mother's milk does not contain HVC, contact with bloody nipples might facilitate viral transmission while breastfeeding. Maintaining good nipple hygiene does not remove the risk of viral transmission from cracked or bloody nipples or areolas. The answer breastfeed, but undergo treatment with pegylated IFNa is incorrect. Pegylated interferon-alpha (IFNa) is an IFNa molecule that has been covalently linked to polyethylene glycol for better absorption and longer half-life; it is commonly used for treatment of chronic Hepatitis C in adults. Treatment with interferon leads to a temporary reduction in white blood cells and platelets, and increased vulnerability to infection, bleeding, or bruising. Because the degree of IFN transmission to babies through human milk is unknown, it is advisable not to breastfeed while being treated with IFN.

Case A 60-year-old man with a past medical history of COPD, hypertension, peptic ulcer disease, and hyperlipidemia is being evaluated for a 1-hour history of severe pain in the mid-abdominal region. His history includes abdominal surgery 6 months ago for a small intestinal obstruction, and his pain emanates from his incisional site. He also states that he has shaking chills, nausea, and vomiting, but denies chest pain, shortness of breath, wheezing, or abnormal bowel habits. Upon physical exam, he is found to be hypotensive, tachycardic, diaphoretic, and in acute painful distress. The abdomen is obese, bowel sounds are hypoactive, and rebound tenderness is noted. There is an exquisitely tender 2.0 cm ventral hernia located inferior to the umbilicus that is indurated, tender to palpation, and is not reducible. Auscultation of the mass reveals the presence of bowel sounds. Question What is the most appropriate initial therapeutic intervention for this patient at this time?

Correct answer: Broad-spectrum antibiotics Explanation This patient's presentation is significant of a strangulated hernia. Manifestations of hernia strangulation, a medical emergency, include an exquisitely tender mass associated with systemic signs and symptoms, such as intestinal obstruction, toxic appearance, peritonitis, or meeting sepsis criteria. General surgery should be consulted immediately. Additional interventions include administering broad-spectrum IV antibiotics such as cefoxitin, providing fluid resuscitation and adequate narcotic analgesia, and obtaining preoperative laboratory studies. If there is any concern for strangulation, do not attempt hernia reduction. The reintroduction of ischemic, necrotic bowel back into the peritoneal cavity can result in subsequent perforation and sepsis. Bedside ED US, using a linear high-frequency probe with color or power Doppler of the hernia sac can be useful in these borderline cases to establish the presence or absence of blood flow.

Case A 62-year-old woman has an unexplained weight loss. She has a vague non-descriptive pain in her stomach accompanied by diarrhea. When her son sees that she has become yellow, he insists that she seek medical attention. Numerous tests are done, including endoscopic retrograde pancreatography, and she is told that she has pancreatic cancer. The tests reveal that the pancreatic cancer is located at the head of the pancreas. Question What tumor marker is most likely to be found?

Correct answer: CA 19-9 (carbohydrate antigen 19-9) Explanation Pancreatic carcinoma can present with abdominal pain, jaundice, and weight loss. Jaundice would be seen with pancreatic carcinoma located at the head of the pancreas because it obstructs the common bile duct. CA 19-9 is carbohydrate antigen (CA) 19-9. This marker is a sensitive marker in patients with pancreatic carcinoma. This marker can be elevated in other GI cancers as well as some other medical conditions. AFP is alpha-fetoprotein. Alpha-fetoprotein is a protein. It is produced by the human fetus. Alpha-fetoprotein is found with an assortment of neoplasms. Elevated levels of alpha-fetoprotein are seen with hepatocellular carcinoma. It is also seen with pancreatic carcinoma, teratocarcinomas, and also some gonadal tumors. CALLA is common acute lymphoblastic leukemia-associated antigen. CALLA can be seen on malignant lymphocytes. PSA is prostate-specific antigen. It is a protein that is made in the prostate gland. PSA is elevated with prostatic tumors. An elevated PSA may be the first sign of prostate cancer. Gastrin is a hormone. Gastrin is also produced by gastrinomas. Gastrinomas are a type of islet cell tumor. Because of the excess gastrin production, peptic ulcers are seen with gastrinomas

Case An 8-month-old is admitted to the hospital for the evaluation of possible middle ear infection and chronic diarrhea. Since birth, he has episodes of frequent loose stools. He has had 2 episodes of lower respiratory tract infection with bronchospasm, treated with intravenous antibiotics and bronchodilators. His parents are not related, and they report that their firstborn died of pneumonia at the age of 8 months. On examination, the patient appears irritable, screams, shakes his head, and tries to rub the right ear. Signs of dehydration are noted. His weight is < 5th percentile; his height is 5th percentile, and his head circumference is 50th percentile. His temperature is 38 C. Mucocutaneous changes suggestive of a fungal infection are noted. Right tympanic membrane is erythematous and bulging, with poor mobility on pneumatic otoscopy. Left tympanic membrane appears clear with good mobility. Throat is erythematous. Thyroid, heart, lungs, genitalia, and nervous system are within normal limits; his abdomen is tender. You are not able to palpate lymph nodes, and there are no signs of hepatosplenomegaly. Question What is the next step in the diagnosis of the primary disorder in this child?

Correct answer: CBC with differential Explanation Your patient experiences recurrent respiratory infections and diarrhea; he also has skin changes and a family history highly suggestive of an immunodeficiency; most likely, it is severe combined immunodeficiency (SCID). SCID is a group of congenital diseases caused by different genetic mutations, resulting in the severe deficiency of both T- and B-lymphocytes. X-linked is the most common type. The clinical picture is, however, similar in all of them: recurrent infections caused by bacteria, viruses, fungi, and opportunistic infections. The initial step in the evaluation of immune status should be complete blood count with the differential. You should evaluate blood cell counts and cell morphology for the presence of lymphopenia, which is the classic hallmark of SCID. KoH prep may help you to differentiate dermatophytes and Candida albicans symptoms from other skin disorders, but it will not be useful in the diagnosis of the primary condition. You may consider genetic testing to differentiate between various forms of SCID as well as the other combined immune deficiencies, but testing should only be done after you prove the existence of immune deficiency (lymphocytes, immune globulins). Testing stool for ova and parasites may be considered in cases of diarrhea, but the testing will not help you to establish the diagnosis of the primary condition. The patient probably has otitis media, and he may have the mastoiditis (peak incidence for the mastoiditis is at age 6 - 13 months; it is much more common in immunocompromised patients). In the case of mastoiditis, fluid extracted from the middle ear through either perforated drums or by tympanocentesis may be sent for Gram staining, culture, and acid-fast stain; however, this procedure will not contribute to the evaluation of immune deficiency.

Case An 8-year-old boy has abdominal pain and vomits twice. No fever is noted, and he has not had any diarrhea. On examination the next morning, he presents with diffuse lower abdominal pain with rebound tenderness on the lower right side. On rectal examination, there is no localization. Question What is the next recommendation?

Correct answer: CBC, UA, and abdominal ultrasound Explanation The correct response is CBC, UA, and abdominal ultrasound. This child's history and physical examination are not specific enough to go directly to surgery for presumed appendicitis; however, the child needs more of a work-up than just observation. While not all children with appendicitis have an elevated white count and a left shift, other findings on the CBC (e.g., hemolytic anemia with thrombocytopenia) may lead one to consider Henoch-Schönlein purpura. The same is true if there is marked hematuria and proteinuria. A periappendiceal abscess against the bladder can produce some hematuria and pyuria. A barium enema is sometimes used to assist in the diagnosis of appendicitis. If the appendix fills with barium, it is not swollen from inflammation; however, in this case, a CBC, UA, and ultrasound to look for a swollen appendix or evidence of a periappendiceal abscess would be the least invasive evaluative protocol with the lowest risk of side effects.

Question A 57-year-old man presents with a sudden onset of left lower quadrant pain, fever, and guarding. The patient appears septic and ill. His hemoglobin is 12.2 g/dL, and hematocrit is 37%; white blood cell count (WBC) is 15 x109/L, and platelets are 150 x109/L. You suspect diverticulitis, with a probable perforation. What study would best confirm the diagnosis?

Correct answer: CT scan of the abdomen Explanation The correct response is a CT scan of the abdomen. Patients with acute diverticulitis can be diagnosed based on the clinical presentation. Since other acute surgical conditions can present in a similar clinical pattern, confirmation of the diagnosis is important. Contrast radiography (barium enema) or colonoscopy should be undertaken with caution in patients with suspected acute diverticulitis. The increased luminal pressure from the injected contrast material, or from insufflation of air, may lead to free rupture of a previously well-localized peridiverticular abscess or phlegmon. Although studies have suggested that both contrast enemas and colonoscopies (when performed by experienced personnel) can be done safely with a high diagnostic yield, the introduction of computerized tomography has substantially reduced their use. CT scanning has the advantages of being non-invasive, avoiding increased luminal pressure, and being able to detect extraluminal disease; therefore, it is the test of choice to confirm the diagnosis of diverticulitis. Fiber-optic endoscopy offers little information in the evaluation of suspected diverticulitis and carries a significant risk of increasing the pericolic contamination. A sonogram is not as sensitive in picking up diverticulitis as a CT scan, and it would probably end up requiring a CT scan to confirm.

Question A 3-year-old girl presents with a 1-day history of irritability and weakness in her legs. Neurologic exam reveals an ascending symmetrical paralysis with cranial neuropathy. A lumbar puncture is performed and cerebrospinal fluid is found to have a normal glucose level, <10 leukocytes/mm3, and elevated protein. Medical history shows the child recently recovered from a mild diarrheal illness. What organism is triggering this syndrome?

Correct answer: Campylobacter jejuni Explanation Post-infectious polyneuropathy, Guillain-Barré syndrome (GBS), sometimes follows a gastrointestinal infection caused by Campylobacter jejuni, which is a Gram-negative rod. About 1-12 weeks after infection, weakness frequently begins in the lower extremities and ascends to the trunk; paralysis is usually symmetric. Guillain-Barré syndrome may also be preceded by a nonspecific viral infection or a respiratory infection caused by Mycoplasma pneumoniae. Myalgia is common and children are often irritable. Cerebral spinal fluid (CSF) examination is necessary for diagnosis. Typically, CSF protein is elevated to more than twice the normal limit, while glucose is normal and pleocytosis is absent. The infective agent implicated in causing GBS is Campylobacter jejuni. Clostridium botulinum, Escherichia coli, Salmonella enteritidis, and Shigella dysenteriae are not the organisms that are involved in causing GBS. Clostridium botulinum is an anaerobic, Gram-positive rod that causes 3 main types of botulism. These include infant, food-borne and wound contamination. Botulism can be confused with GBS; however, there is descending symmetric paralysis in botulism. Escherichia coli is a facultative, anaerobic, Gram-negative rod belonging to the family Enterobacteriaceae. It can cause urinary tract infections, neonatal meningitis, and intestinal infections (such as gastroenteritis). Salmonella enteritidis is a Gram-negative rod that causes egg-borne salmonellosis, which is characterized by fever, abdominal cramps, and diarrhea. Symptoms occur after consuming contaminated food. In infants, the disease is very severe. Shigella dysenteriae is a Gram-negative organism that causes dysentery. Neurological manifestations include headache, seizures, lethargy, and meningitis.

Case A 35-year-old female comes to your office complaining of frequent flushing and chronic diarrhea that started a few months ago. She also notices progressive edema of her lower limbs and enlargement of her abdomen. On examination, she looks cachexic with congested neck veins, lower limb edema, and a palpable tender liver with irregular surface. There is a pansystolic murmur heard inside the apex that increases in intensity with inspiration. Chest X-ray shows cardiomegaly with dilated right atrium and clear lung fields. ECG shows right atrial abnormality (P pulmonale). Echocardiography shows thickened, fibrotic and fixed tricuspid valve in a semi opened position along with right atrial and ventricular enlargement. 24-hour urinary excretion of 5-hydroxy-indole-acetic-acid (5HIAA) is grossly elevated. Liver ultrasonography shows multiple small nodules. Question What is the most probable diagnosis?

Correct answer: Carcinoid syndrome Explanation In Carcinoid Syndrome, 60-90% of tumors arise from the small bowel. It is caused by a metastasizing carcinoid tumor and is characterized by cutaneous flushing, diarrhea, bronchoconstriction, and endocardial plaques composed of a unique type of fibrous tissue. These effects are due to the circulating humoral substances as serotonin and bradykinin. Cardiac affection happens only when there is metastasis to the liver. The right side of the heart is mainly affected when the humoral substances are not inactivated by the liver due to hepatic metastasis. The tricuspid and pulmonary valves are affected with fibrous plaques (composed of smooth muscle cells in stroma rich in mucopolysaccharide) and as a result, they are fibrosed and fixed in a semi-opened position with both incompetence and stenosis. Fibrous plaques can invade the endocardium of the cardiac chambers, the intima of the inferior vena cava, pulmonary artery, and coronary sinus. Left side valvular affection occurs less commonly and is usually associated with right to left shunts, or when the tumor originates from the lung. The patients mainly complain of flushing, diarrhea with manifestations of systemic congestion as lower limbs edema, congested liver, congested neck veins, and ascites. Physical examination usually reveals a systolic murmur along the left sternal border produced by tricuspid regurgitation; there may be a mid-systolic murmur or early diastolic murmur due to pulmonary valve affection. There are 2 key diagnostic modalities for the diagnosis of carcinoid heart disease, which includes 24-hour urinary excretion of 5-hydroxy-indole-acetic-acid (5HIAA) and transthoracic echocardiography. ECG and chest X-ray are of limited value. The chest X-ray is normal in half the patients, but may reveal enlargement of the heart and pleural effusions or nodules. The pulmonary artery trunk is typically of normal size without evidence of post-stenotic dilatation as occurs in congenital pulmonic stenosis. ECG shows ST, T wave changes, right atrial abnormalities, and possibly sinus tachycardia. Echocardiography may reveal evidence of tricuspid and/or pulmonary valve thickening, along with right atrial and right ventricular dilatation; small pericardial effusions are present in a minority. The principles of management of patient with carcinoid heart disease involve treatment of heart failure, pharmacotherapy for the reduction of tumor products, and surgical/interventional management of valvular pathology. Treatment includes symptomatic digitalis and diuretics for treatment of heart failure, and somatostatin analogs such as (Octreotide). Octreotide has been found to improve symptoms as well as survival. GIT main side effects are loose stool, abdominal discomfort, and cholelithiasis. Balloon valvuloplasty of the tricuspid and pulmonic valves may improve survival and symptoms. Surgical treatment, like tricuspid valvotomy and valvectomy, may result in symptomatic improvement; mortality is high especially in the elderly. The long-term mortality remains high regardless of treatment modality, with half the patients dead within 1 to 2 years. Churg-Strauss syndrome is a granulomatous medium and small vessel vasculitis. The American College of Rheumatology (ACR) has proposed 6 criteria for the diagnosis of Churg-Strauss syndrome, which include the following: 1. Asthma 2. Eosinophilia of more than 10% in the peripheral blood 3. Pulmonary involvement such as infiltrates 4. Paranasal sinusitis 5. Mononeuritis multiplex or polyneuropathy 6. Histological proof of vasculitis with extravascular eosinophils. Cardiac manifestations include heart failure, myocarditis, pericarditis, constrictive pericarditis, and myocardial infarction. Chest X-ray may show mild infiltrates or nodules. Treatment is symptomatic and systemic anti-inflammatory may improve symptoms. Dilated cardiomyopathy is characterized by dilated hypokinetic left ventricle with impaired both systolic and diastolic functions and mitral incompetence. Patients complain of left sided heart failure manifestations such as dyspnea and orthopnea. The right ventricle may be affected as well with manifestations of congestive heart failure. Treatment is usually symptomatic with diuretics, vasodilators, and inotropic, cardiac transplantation may be the treatment of choice especially in end stage patients. Hypereosinophilic Syndrome typically presents in fourth decade males living in a temperate climate. The cause of hypereosinophilia (persistent eosinophilia with 1500 eosinophils/μL for at least 6 months) is unknown and is not associated with leukemia or reaction to parasitic infestations, allergic, hypersensitivity, or neoplastic disorders. There is organ involvement other than the heart, such as the lungs, skin, and bone marrow. Early in the disease (before fibrosis), medical therapy is the treatment of choice. It includes corticosteroids and cytotoxic drugs such as hydroxyurea. Routine cardiac therapy with digitalis, diuretics, afterload reduction, and anticoagulation as indicated are adjuncts in the management of these patients. Surgical therapy is indicated with advanced stage of the disease (fibrosis). Endocardial fibroelastosis is a condition commonly seen in infants and young children, which is characterized by endocardial thickening and myocardial dysfunction. This condition has a poor prognosis and terminates in death within weeks, especially in progressive heart failure. Viral etiology has been implicated in most cases. Infants present with feeding difficulty, failure to thrive, breathlessness, excessive sweating, and wheezing. Treatment is usually symptomatic with diuretics, vasodilators, and inotropic drugs. Cardiac transplantation may be the treatment of choice, especially in end stage patients. References:

Case A 65-year-old man presents with a history of progressive loss of appetite and weakness. On examination, he is cachectic, pale, and jaundiced. A walnut-sized, non-tender mass is palpable in the right hypochondrium. Question What is the most likely diagnosis?

Correct answer: Carcinoma of head of pancreas Explanation Carcinoma of the head of pancreas is a common 'duodenal C-loop cancer'. Seen mostly in men, it is usually a ductal adenocarcinoma. It causes obstructive jaundice, weight loss, itching, and sometimes upper abdominal pain. There may be diabetes, bleeding problems, malabsorption syndromes, and LFT showing obstructive jaundice. This carcinoma has a better prognosis than cancer of the body and tail of the pancreas when resection is possible. Courvoisier's law states that when the gall bladder is palpable (but not tender) in the presence of jaundice, the cause is usually cancer. Cholecystitis is a diagnosis of 'female, fat, fertile, forty, and flatulent'. It does occur in men. Jaundice may or may not be present. Vomiting usually brings relief to the upper abdominal pain. Muscle guarding and rebound tenderness is present. A fibrolipoma over the right hypochondrium may be a coincidental finding, but would not be the cause of the overall clinical features of a malignancy. A hydatid cyst results from an E. Granulosus infection, and cysts may form in the liver, lung and other organs. It may become palpable in the right hypochondrium and cause pressure symptoms. Migrant workers and immigrants from endemic areas may carry this infection. Primary biliary cirrhosis is an autoimmune disease that destroys the liver structure and system; it is mostly seen in women over the age of 40. There is itching, xanthomata, and hepatosplenomegaly.

Case A 48-year-old man presents with right upper quadrant pain. His pain developed after eating fried chicken at a cookout, and it is now continuous. He has experienced previous episodes of less severe right upper quadrant pain; they resolved spontaneously. The pain radiates to his right shoulder; it is accompanied by nausea, vomiting, and chills. On physical examination, the patient is noted to have scleral icterus and jaundice. Rigors are present. The abdomen is slightly distended, with positive Murphy's sign. Vital signs include a BP of 120/85 mm Hg, pulse 117, respirations 18, and temperature 102.5 degrees F. CBC reveals mild leukocytosis, elevations of serum aminotransferases and bilirubin, as well as normal lipase. Question What is the likely diagnosis in this patient?

Correct answer: Cholangitis Explanation Cholangitis is the correct response. The patient has right upper quadrant pain, jaundice and fever, which are all part of the Charcot triad associated with cholangitis. Cholangitis is an infection of the common bile duct that typically occurs when the duct becomes obstructed. The patient reports previous episodes of right upper quadrant pain; they were likely caused by gallstones (cholelithiasis). The patient probably has a stone obstructing the bile duct, causing cholangitis. Cholelithiasis is an incorrect response. The patient likely does have gallstones. However, gallstones do not always lead to cholangitis. It is only when a stone becomes obstructed that cholangitis may develop. Therefore, while cholelithiasis is likely contributing to the patient's diagnosis, cholangitis is a more appropriate diagnosis. Acute pancreatitis is an incorrect response. The pain associated with pancreatitis is usually epigastric, with radiation into the back. Pancreatitis may be associated with jaundice if the cause is gallstone pancreatitis. However, if the patient had pancreatitis, his lipase would be elevated. Cirrhosis and hepatocellular carcinoma are both incorrect responses. While these conditions may be associated with jaundice, the presence of Charcot's triad strongly suggests cholangitis.

Case Ico-delete Highlights A 40-year-old woman is seen in the emergency department because of right upper quadrant pain and fever. She has been experiencing episodic epigastric pain over the past few months, but this is the worst her symptoms have been. She also reports anorexia and vomiting. She is in obvious distress. On physical examination, her doctor notes the presence of jaundice. Her lab results are as follows: TEST RESULTS REFERENCE RANGE alkaline phosphatase 89 U/L 20-70 U/L bilirubin (total) 3.8 mg/dL 0.2-1.5 mg/dL AST 50 U/L 10-40 U/L ALT 70 U/L 10-60 U/L amylase 35 U/L 25-85 U/L lipase 80 U/L 0-160 U/L Question Ico-delete Highlights What diagnosis is most likely to be the cause of her symptoms?

Correct answer: Cholangitis Explanation The correct answer is cholangitis, an infection of the biliary tract. Patients classically present with Charcot's triad: jaundice, abdominal pain, and fever. It is seen with acute cholangitis. Elevated bilirubin and alkaline phosphatase are present due to biliary inflammation. Liver enzymes are usually only mildly elevated. Treatment is with antibiotics. Cholelithiasis is the presence of gallstones. Patients may have epigastric right upper quadrant or epigastric pain from periodic blockage of the bile duct. Bilirubin and alkaline phosphatase are usually only elevated if obstruction of the common bile duct is present; however, fever and jaundice are not typically present. Pancreatitis is an inflammation of the pancreas. Epigastric pain and anorexia are common symptoms. Elevated amylase and lipase are diagnostic. Hepatocellular carcinoma is primary liver cancer. Typically, hepatocellular carcinoma presents abdominal pain and weight loss. An elevation of liver enzymes is also common. Jaundice may occur if the bile duct is blocked. Fever is not a presenting symptom. Gilbert's syndrome is an inherited disorder affecting the metabolism of bilirubin. Unconjugated hyperbilirubinemia is typical. Jaundice may be present during periods of dehydration or illness, although the condition is largely asymptomatic.

Question What is the definitive treatment for acute cholecystitis?

Correct answer: Cholecystectomy Explanation The definitive treatment for acute cholecystitis is cholecystectomy, surgical removal of the gallbladder. Antibiotics and cholecystostomy may be temporizing maneuvers to calm down an acute situation or to temporize a patient who is otherwise too ill to undergo surgery, but the only treatment that will cure the pathology causing the problem is surgery, be it an open cholecystectomy or a laparoscopic cholecystectomy.

Case A 66-year-old man arrives at the ED with a history of upper right quadrant pain that has been getting worse over the past 2 days. He has a history of eating fatty foods, and when last visiting his PCP, he was told he was obese. Upon a physical exam, he has abdominal distention and appears ill or uncomfortable. He appears to have diaphoresis and is febrile. The patient also reports radiating pain to his right shoulder which started after dinner and reports no history of trauma. Laboratory findings show that the patient's liver enzymes are normal and his CA 19-9 assay is negative. Furthermore, ultrasound demonstrates that the gallbladder wall is greater than 3 mm and the technologist reports that shadowing is identified. Question What is the most likely diagnosis?

Correct answer: Cholecystitis Explanation The clinical picture is suggestive of cholecystitis due to the presenting symptoms: upper right quadrant pain, abdominal extension, swelling, and jaundice. Furthermore, AST and ALT are often elevated with cholecystitis. Lastly, a sludge-filled thick-walled gallbladder is also present on the ultrasound. Cholangitis is not correct because patients with cholangitis usually present with right upper quadrant pain, fever, and jaundice. Reynold's pentad (Charcot's triad with septic shock and altered mental status), which are reliable signs for cholangitis, is not present in this case. Choledocholithiasis is not correct because the ultrasound would have displayed 1 or more gallstones in the common bile duct. Furthermore, the symptoms present in the patient are not found with choledocholithiasis. Cholelithiasis is not correct because patients with cholelithiasis do not often present with acute symptoms as stated in this patient. GERD is not correct, as laboratory finding of CA 19-9 assay is negative.

Case A 32-year-old woman presents with decreased vibratory sensation. Her friend had insisted that she see a psychiatrist because she has been 'depressed' and 'slow' as of late. She has no prior or family history of depression, and she denies fever and weight loss; there is no history of a major life event. The pertinent findings on her physical examination are mild pallor, smooth shiny tongue, hypopigmented skin patches, and decreased vibration and joint position senses. Her laboratory values are: Leukocytes count 6,600/μL Segmented neutrophils 65% Hemoglobin 8.1g/dL Mean corpuscular volume 116 fl Platelet count 145,000 /μL Serum glucose 106 mg/dl Sodium 146 mmol/L Potassium 3.9 mmol/L Chloride 108 mmol/L Bicarbonate 23 mmol/L Total bilirubin 1.8 mg/dL Lactate dehydrogenase (LDH) 360 U/L Red cell folate 244 ng/mL Vitamin B12 87 pg/ml Question What finding may account for this patient's features?

Correct answer: Chronic atrophic gastritis Explanation The patient has vitamin B12 deficiency. The suggestive features are depressive symptoms, decreased vibratory sensation, vitiligo, smooth tongue, anemia with macrocytic RBC, evidence of hemolysis (high bilirubin and LDH), and a low vitamin B12 level. A normal red cell folate level and the presence of neurologic manifestations rule out folate deficiency as the cause of the macrocytic anemia. Of all the listed options, the most likely cause is chronic atrophic gastritis. This is seen in patients with pernicious anemia. They have deficiency of intrinsic factor, which is necessary for the absorption of vitamin B12 from the gut. The gastritis is of the 'A' type and affects the body and fundus, but it spares the antrum. They also have anti-IF antibody, and they some have anti-parietal cell antibody. Treatment is by parenteral vitamin B12 supplementation. With correction, reticulocytosis will be noted after a week, and hemoglobin will start to rise after about 6 weeks; however, there may not be a full recovery from the neurologic manifestations. Alcoholism may cause folate deficiency. This is unlikely in this patient with a normal folate level. Low serum ferritin, which is indicative of iron deficiency anemia, is associated with microcytosis and hypochromia, and it also lacks neurologic manifestations. Menorrhagia is a cause of iron deficiency anemia and not a deficiency of vitamin B12. Methotrexate use may cause folate deficiency by virtue of its mechanism of action, which is inhibition of dihydrofolate reductase. References

Case A 58-year-old man presents with a recent episode of rectal bleeding. A brief history reveals that his bowel patterns have been increasingly erratic over the past 6 weeks. He reports periods of 2 - 6 days without bowel movements, followed by copious production of thin coils of stool. He has lost 10 pounds over the last month. His family history is significant for the death of his cousin from colon cancer. Physical exam reveals the patient is pale and febrile (temperature 101°F); the rectal exam is heme-positive with scant stool, and his prostate is soft and moderately enlarged. Question What is the most appropriate evaluation method in the diagnosis of colorectal cancer?

Correct answer: Colonoscopy Explanation In the United States, colorectal cancer is the 3rd most common cancer occurring in both men and women. The key to a good prognosis is early detection of the cancer. This patient presents with signs and symptoms consistent with colorectal cancer. The criterion standard for examination of the colon and rectum is colonoscopy. To view a colonoscopic mass lesion, refer to the image. In symptomatic patients having a positive family history of colorectal cancer, colonoscopy is a useful imaging modality for diagnosis; it has a diagnostic role by evaluating and detecting the cause of symptomatology or excluding the presence of malignancy by direct visualization. Colonoscopy has the advantage of direct bowel visualization, biopsy, and excision of lesions (when present). In this case, abdominal plain films would probably not provide as much information as the other imaging modalities would. Barium enema is cheaper and has a lower complication rate than colonoscopy, but it is invasive and requires full bowel preparation. While therapeutic interventions (polyp removal and biopsy) are possible with colonoscopy, barium enema does not afford a means for these interventions. Anoscopic examination might be appropriate in an unexplained lower GI bleed; however, this patient requires a more substantial investigation given his history suggestive of colorectal cancer. Anoscopic examination would not be able to visualize lesions higher in the rectum and colon. Exploratory laparotomy is not indicated in this case. There is no mention of unstable vital signs, nor is there any evidence to suggest a surgical abdomen (e.g., guarding, rebound tenderness, rigidity, or absent bowel sounds). Abdominal computed tomography (CT) and virtual colonoscopy using CT are considered by some to be as, or more, valuable than colonoscopy.

Case A 52-year-old man presents with vomiting and epigastric distress for the past few hours. He has been drinking alcohol for over 20 years, and he has been a moderate-to-heavy drinker. 5 years ago, he was diagnosed with a "gastric/duodenal ulcer," for which he has been taking cimetidine and antacids. The pain now radiates towards the left along the costal margin. He has noticed his appetite has been reduced lately, and his stools are bulky and foul smelling. His friends have commented on his sickly look and weight loss. Question What is the most likely diagnosis?

Correct answer: Chronic pancreatitis Explanation Chronic pancreatitis is common in patients with alcoholism when pancreatic proteins become denatured and cause destruction of glandular and ductal tissue in the pancreas. There is recurrent epigastric or upper abdominal colicky pain; it radiates towards the left, usually along the left costal margin in the direction of the tail of the pancreas. There may be nausea and vomiting along with weight loss, appetite disturbance, and bowel disturbance. Due to fat malabsorption, the stools are bulky and foul smelling, which is called steatorrhea. There may be varying degrees of liver and gallbladder problems accompanying chronic pancreatitis. ERCP is the best diagnostic tool. Acute recurrent pancreatitis is usually a more acute event that is initiated by pathologic activity within the glandular/ductal system. Deep epigastric pain radiates to the back, and it is relieved by leaning forward. There may be an accompanying fever with a history of similar attacks in the past. An inferior MI can mimic this clinical picture, but pain direction, radiation, and EKG would be diagnostic. Acute cholecystitis is more common in women, and it can be excluded by direction and radiation of the pain leaning towards the right hypochondrium. In left ureteric colic, the pain would not present in the epigastrium; instead, it would be located more in the left lumbar region and flank, and it would radiate towards the scrotum on that side.

Case A well-known, 42-year-old male patient has come in today for a yearly checkup. The client has complaints of lassitude, anorexia, dyspepsia, flatulence, and maybe a slight weight loss (10 lb). He states these symptoms have been happening for the last year or so. In taking a social history the client notes he attends social functions 2-3 times a week. In asking how much alcohol he drinks, he states not that much, just 2 or 3 drinks at parties. He smokes 1½ packs of cigarettes a day. He is 50 pounds over his ideal weight. He does not exercise on a regular basis. Question What is your working diagnosis?

Correct answer: Cirrhosis Explanation For many years cirrhosis is latent, the pathologic changes progressing slowly until major symptoms induce awareness of the disease. Early symptoms are vague and nonspecific and include lassitude, anorexia, dyspepsia, flatulence, a change in bowel habits (either constipation or diarrhea), and slight weight loss. Nausea and vomiting, especially in the morning, are common. A dull ache or heavy feeling in the epigastrium or right upper quadrant is present in about half of the patients. In most cases, the liver is hard and palpable regardless of whether it is enlarged or atrophied. Gastrointestinal bleeding is the most serious result of cirrhosis. The patient presents with either melena or hematemesis. If or when the patient recovers from the bleeding, a portacaval shunt operation may be performed to reduce the portal pressure by anastomosing the portal vein (high pressure) to the inferior vena cava (low pressure). Gastrointestinal bleeding is one of the important precipitating causes of hepatic encephalopathy. The encephalopathy results when ammonia and other toxins enter the systemic circulation. The ammonia is the bacterial breakdown of protein in the gastrointestinal tract. Gastric aspiration, saline cathartics, and cleansing enemas are used to remove the ammonia. Two diseases of the biliary tree are cholelithiasis and associated chronic inflammation (cholecystitis). Cholelithiasis is the formation of stones in the bile duct or gallbladder. Gallstones are made up of cholesterol, bilirubin, bile salts, calcium, and protein. Patients with gallstones have symptoms of acute or chronic cholecystitis. The acute form is characterized by the sudden onset of agonizing pain in the upper abdomen, mostly in the mid-epigastrium; it can radiate to the back and right shoulder. Nausea and vomiting are common. The pain may last for several hours or may recur after a partial remission. The diagnosis of both the acute and chronic forms cholecystitis and cholelithiasis often rests on cholecystography or ultrasound to reveal the presence of stones or malfunctioning of the gallbladder. Treatment is often surgical removal of the gallbladder (cholecystectomy) or removal of stones from the common bile duct (choledocholithotomy), which will effect a cure in about 95% of the cases.

Case A 37-year-old woman takes a 10-day course of antibiotics for an infection. The infection clears up, but she develops watery diarrhea 2 weeks later. The diarrhea is non-bloody. She also has anorexia, nausea, and cramping. On physical examination, her temperature is 39°C. There is diffuse tenderness in her abdomen. A stool sample reveals pus in her stool. Her condition deteriorates, and based on a constellation of findings and variables, her doctor recommends endoscopy. Pseudomembranes are seen on the mucosal surface with endoscopy. Question What is the most likely causative agent of this patient's condition?

Correct answer: Clostridium difficile Explanation This patient has pseudomembranous colitis. The vast majority of cases of pseudomembranous colitis are caused by Clostridium difficile. Pseudomembranous colitis can follow many antibiotics with a broad antibacterial spectrum of effect. As such, pseudomembranous colitis is sometimes referred to as antibiotic-associated colitis. Entamoeba histolytica is a cause of amebic colitis and can cause diarrhea, but it is not associated with antibiotic usage. Escherichia coli is a cause of traveler's diarrhea, but it is not associated with antibiotic usage either. Pus would not be in the stool with Giardia. Salmonella can be associated with diarrhea but not with antibiotic usage.

Case A 40-year-old man presents with diarrhea. The diarrhea is not bloodstained. It began 12 hours after eating some meat at a cookout. Examination of his stool reveals no leukocytes or trophozoites. Question What is the causative organism?

Correct answer: Clostridium perfringens Explanation In Clostridium perfringens food poisoning, patients usually present with abdominal cramps and watery diarrhea 8-24 hours after ingestion of the contaminated food. Common culprits include inadequately cooked meat or legumes. Diagnosis is made by isolating the organisms from the patient's stool. No fecal leukocytes or trophozoites are present on stool examination. Treatment is supportive with rehydration, as it is usually a mild self-limiting illness. In Escherichia coli serotype O157:H7 food poisoning, patients present with watery diarrhea, which can become bloodstained. Sources of infection include undercooked beef. A stool examination reveals polymorphonuclear leukocytes. Giardia lamblia can be acquired by drinking contaminated water. Patients present with watery diarrhea. The incubation period is around 1 week. There are trophozoites in the stool but no leukocytes. Patients with Salmonella typhi food poisoning usually present with bloody diarrhea. Sources of infection include beef, poultry, eggs, and dairy products. Examination of the stool reveals mononuclear leukocytes. Patients with Staphylococcus aureus food poisoning may present with vomiting and watery diarrhea 1-6 hours after ingesting the contaminated food like processed meat. There are no fecal leukocytes or trophozoites.

Question A 16-year-old girl with a 2-year history of ulcerative colitis presents with signs of an acute exacerbation (i.e., abdominal pain and passing large, frequent quantities of blood and mucus from the rectum). It is treated with sulfasalazine, glucocorticoids, and intravenous alimentation. The diarrhea decreases markedly, but her status continues to deteriorate. Tachycardia, volume depletion, and electrolyte imbalance develops; her current temperature is 101.8° F. Physical examination finds abdominal tenderness, but no mass. Plain radiography shows extensive ulceration and the transverse colon is dilated up to 7 cm. What is the most appropriate next step in the management?

Correct answer: Colectomy Explanation The correct response is colectomy. This patient's symptoms and signs are characteristic of toxic megacolon, a dangerous complication of ulcerative colitis. It carries the life-threatening risk of perforation and septicemic shock. A barium enema and colonoscopy have to be avoided due to the danger of further precipitating toxic megacolon. The failure of medical treatment also indicates the need for surgery. Proctocolectomy, distal rectal mucosectomy, and ileal pouch-anal anastomosis is an increasingly popular surgical treatment of colitis ulcerosa; however, it cannot be performed in the midst of an acute exacerbation. Emergency colectomy remains the only viable option. Tapering of glucocorticoids is appropriate when healing progresses, but it does not represent an effective treatment of toxic megacolon.

Question What is the most common cause of hemodynamically significant bleeding from the lower gastrointestinal tract?

Correct answer: Colonic diverticula Explanation The most common cause of hemodynamically significant hemorrhage from the lower gastrointestinal tract is colonic diverticula, which accounts for about 25% of cases. The bleeding is typically painless and associated with voluminous hematochezia. Colonic cancer and polyps together make up about 20% of the cases of hemodynamically significant bleeding from the lower gastrointestinal tract; inflammatory bowel disease and vascular ectasia are considerably less frequent causes.

Question A 35-year-old man has a routine physical examination with no abnormal findings. His family history, however, is positive for familial adenomatous polyposis. What screening test would be best for him to obtain?

Correct answer: Colonoscopy Explanation The best test to recommend is a colonoscopy since it detects both polyps and cancer. Colorectal cancer is the 4th most common form of cancer in the United States; it arises from adenomatous polyps in the colon. Familial adenomatous polyposis (FAP) has different inheritance patterns and different genetic causes. In this patient, the pattern is probably autosomal dominant which puts him at a risk of nearly 100% for developing colorectal cancer by the age of 40. That means that screening tests for colorectal cancer must be performed earlier than suggested for the rest of the population (i.e., before the age of 50). A fecal occult blood test, fecal immunochemical test, and stool DNA test (sDNA) find cancer, not polyps. The AFP (alpha-fetoprotein) tumor marker is a useful marker for hepatocellular carcinoma and germ cell tumors, but not for colorectal cancer.

Case A 62-year-old woman with breast cancer is terminally ill and is receiving hospice care. She receives around-the-clock opioids for continuous pain and liquid morphine as rescue medication. She is currently experiencing abdominal discomfort and pain. Question What is the best initial treatment based on the most likely diagnosis?

Correct answer: Combination of senna and docusate Explanation The correct answer is a combination of senna and docusate (laxative and stool softener). The patient is suffering from opioid-induced constipation (OIC). Studies strongly recommend routine prescription of laxatives for the management or prophylaxis of opioid-induced constipation1. A combination of drugs with different modes of action is likely to be more effective in resistant constipation than a single agent1. Methylnaltrexone administered by subcutaneous injection should be considered in the treatment of opioid-related constipation when traditional laxatives are not effective1; therefore, it is not used for initial treatment of OIC. Methylnaltrexone is a drug that targets opioid induced constipation by blocking peripheral opioid receptors, but not central receptors for analgesia2. Neither adequate hydration nor dietary fiber intake alone is effective in treating OIC2. Similarly, physical activity has little role in the treatment of OIC. In addition, given that the patient is terminally ill, physical activity is likely to be limited by fatigue.

Case A 65-year-old man presents with a 2-day history of diffuse, colicky abdominal pain originating in the RUQ (right upper quadrant). It is not aggravated by food or activity. There has been nausea, but there has not been any vomiting. His skin and eyes have turned yellow in the last several days. His urine has been dark brown, and his stools have been white and chalky. He stopped drinking alcohol about 2 years ago and has been sober since. There is no history of drug abuse. He smokes 2 packs of cigarettes a day. He had gallstones diagnosed at the time he stopped drinking, but he refused surgery. There is a history of previously diagnosed but untreated hypertension. He denies temperature elevation, but he has felt "warm". There was an episode of "shaking chills" earlier in the day. When he walks, he gets dizzy. Vital signs are: T=103 F (oral); P=115/min; BP=100/65 mmHg; R=32/min (labored); Pulse Ox=90% (room air). He is mildly icteric. Abdomen is slightly protuberant, and there is tenderness and rebound tenderness restricted to the RUQ. No surgical scars are present. Chest examination is within normal limits except for scattered wheezes and rhonchi. Cardiac examination reveals mild cardiomegaly. On rectal examination, pale soft stool is obtained; it is guaiac negative. Good sphincter tone is present. Mild atrophy of the testicles is noted. The remainder of the physical examination, including a neurological exam, is within normal limits. Question What is the most likely cause of this man's primary problem?

Correct answer: Common bile duct stones Explanation The most common cause of cholangitis is common bile duct (CBD) stones with obstruction of the CBD. The subsequent development of edema together with the presence of stones produces biliary stasis. Regardless of the source or bacteria, when bacterial infection occurs, cholangitis occurs. Smoking is not known to cause biliary stasis, a precursor of cholangitis. Renal calculi could mimic this patient's abdominal pain, but they are unlikely to produce jaundice. Although malignant severe hypertension is associated with multiple end organ failure, the gallbladder has not been reported as one of the end organs. Liver failure is associated with jaundice and hypotension; it is usually a terminal event, which is a state this patient has yet to reach.

Case A 48-year-old woman presents with a 1-week history of diarrhea and fever. Stool is watery and associated with abdominal pain. She has no other medical conditions other than osteomyelitis, for which she was treated with clindamycin 3 weeks ago. The patient was started on oral metronidazole after a diagnosis of diarrhea secondary to Clostridium difficile was made 1 week ago. The patient's condition has not resolved after 7 days of metronidazole treatment. Question What is the next best step in her management?

Correct answer: Continue metronidazole. Explanation The patient has an uncomplicated case of Clostridium difficile colitis. Current guidelines call for her to continue metronidazole for a course of 10-14 days. C. difficile is a gram-positive, spore-forming rod; it accounts for 15-20% of antibiotic-related cases of diarrhea and almost all cases of pseudomembranous colitis. Disruption of the normal colonic microflora is usually the precipitating event for C. difficile colitis. This disruption is generally caused by the use of broad-spectrum antibiotics, such as clindamycin, broad-spectrum penicillin, and cephalosporins. Colonization of C. difficile occurs after the microfloral disruption, during which the heat-resistant spores are converted into vegetative spores. Asymptomatic carrier state or clinical manifestations of C. difficile colitis develop depending on the host immune factor. Manifestations range from mild diarrhea to life-threatening C. difficile pseudomembranous colitis. The use of clindamycin prior to the occurrence of diarrhea strongly suggests pseudomembranous colitis as the cause of the diarrhea. Pseudomembranous colitis is an inflammatory bowel disorder associated with antibiotic use. It is due to toxins of Clostridium difficile. The toxins bind to intestinal mucosal epithelial cells and cause watery, secretory diarrhea, abdominal pain, fever, and dehydration. The mucosa shows yellow plaques, focal ulceration, and exudates, which form the characteristic pseudomembrane. Stool samples are positive for fecal leukocytes, and the toxin and the organism may be cultured. Metronidazole is a first-line therapeutic agent in the treatment of C. difficile colitis given in the dose of 500 mg orally 3 times a day for 10-14 days. It is inexpensive and has an efficacy >90%. The response to treatment with metronidazole is usually rapid, with fever resolution occurring within 24 hours and diarrhea resolution within 4-5 days. Vancomycin is approximately as effective as metronidazole in the treatment of C. difficile colitis. Vancomycin should be initiated in patients with severe symptoms (e.g., leukocytosis >15,000 cells/uL or serum Cr 1.5 x normal). It is administered orally in the dose of 125 mg 3 times a day for 10-14 days. Initial treatment of severe cases of pseudomembranous colitis must be aggressive, with intravenous metronidazole and oral vancomycin given in combination. In cases where medical therapy fails, surgical intervention, such as colectomy and ileostomy, may be necessary. Ampicillin and clindamycin, being the agents that most commonly cause C. difficile colitis, should not be administered; they may cause the condition to worsen. Ciprofloxacin is not the drug of choice for treating unresolved cases of pseudomembranous colitis following metronidazole treatment.

Question A 15-year-old boy presents with bloody diarrhea and abdominal cramping. A double contrast barium enema shows fine serrations and narrowing of the rectum and sigmoid. Stool contains mucus, blood, and white blood cells, but no parasites or bacterial pathogens. Endoscopy shows inflamed mucosa and pseudopolyps. A biopsy finds an extensive inflammatory process in the mucosa and submucosa. The glands are filled with eosinophilic secretions; there is also mild involvement of the terminal ileum. Sulfasalazine treatment is attempted, but fails to bring about improvement. What is the most appropriate next step in the management?

Correct answer: Corticosteroids Explanation Fine serrations (ulcers) and pseudopolyps suggest colitis ulcerosa (ulcerative colitis). It is usually limited to the colon but occasionally, as in this case, the terminal ileum is also affected (backwash ileitis). In addition to sulfasalazine or 5-amino-salicylacid, more severe ulcerative colitis cases require corticosteroids. Corticosteroids are also helpful in Crohn's disease involving the small bowel. Metronidazole is helpful in treating the characteristic fistulas of Crohn's disease. Immunosuppressive agents (e.g., 6-mercaptopurine) are used in severe Crohn's disease cases. Diphenoxylate and loperamide provide only symptomatic relief. While surgical removal of the colon can give relief in ulcerative colitis, Crohn's disease tends to recur in approximately 1/2 of the cases involving removal of the diseased bowel.

Case A 15-year-old girl presents with a 1-year history of intermittent abdominal pain with nausea and occasional bloody diarrhea. She denies fever or weight loss, as well as any travel history. Past medical history is significant only for migraines. She takes a multivitamin. Her vital signs are within normal limits. She has mild diffuse abdominal tenderness to palpation and guaiac-positive stool. Her exam is otherwise normal. Her hemoglobin is 9.7, hematocrit is 28%, and her WBC is 12,000/uL. Question What finding would indicate a possible cure if a colectomy is performed?

Correct answer: Crypt abscesses, superficial mucosal involvement on colonoscopy Explanation Crypt abscesses and superficial mucosal involvement are colonoscopy findings associated with ulcerative colitis. A colectomy is performed for severe and intractable disease. A colectomy can be a relative cure since the disease is limited to the colon. Skip lesions, transmural involvement, fistulas, and fissures are all associated with colonoscopy findings in Crohn's disease. Crohn's disease can also affect any structure of the alimentary tract from the mouth to anus, causing ileal disease and mouth ulcers.

Case Ico-delete Highlights A 60-year-old African American man presents due to dysphagia. The dysphagia started 3-4 months ago and has progressively gotten worse. He has also lost weight; current weight and height are 170 lb and 6 feet, respectively. He appears older than his stated age. He wants something to help him in swallowing. He does not complain of heartburn. You note he does not eat on a regular basis and when he does eat, it is usually fast food. He has smoked for the last 40 years, 2 packs a day. He drinks 12 cans of beer on weekdays and approximately 48 cans of beer during the weekend. He uses recreational drugs occasionally. Question Based on the presentation, what is your most likely diagnosis?

Correct answer: Esophagus neoplasm Explanation Esophageal neoplasm is 60% squamous cell carcinomas arising from the surface epithelium, most commonly in the middle and lower esophagus. Adenocarcinomas, which constitute 35%, arise from the gastric fundus and develop in the lower third of the esophageal tract. This cancer is most common in older adults, with African Americans affected 3 times as often as Caucasians, and men 3 times as often as women. Risks factors: Smoking and tobacco use (chewing) Alcohol abuse Drug abuse (e.g., morphine, opium) Malnutrition Environmental carcinogens (e.g., nitrosamines, silica, fungi) History of cancer of the larynx or pharynx History of chronic inflammation of the esophagus, achalasia (failure of esophageal sphincter to relax), tylosis, or caustic burns to esophagus. Dysphagia is the most common presenting symptom. Regurgitation and weight loss may also occur. In gastroesophageal reflux disease the essential features are: Heartburn; may be exacerbated by meals, bending, or recumbency. Clinical diagnosis; typical uncomplicated cases do not require diagnostic studies. Endoscopy demonstrates abnormalities, such as erythema and friability of the squamocolumnar junction in <50% of patients. Barium esophagography is seldom helpful. The typical symptom is heartburn. This most often occurs 30-60 minutes after meals and upon reclining. Certain foods or beverages like coffee or alcohol, drugs like calcium channel blockers or beta blocker, and obesity are contributing factors. Patients often report relief from taking antacids or baking soda. When this symptom is dominant, the diagnosis is established with a high degree of reliability. Overall, a clinical diagnosis of gastroesophageal reflux has a sensitivity of 80% but a specificity of only 70%. Severity is not correlated with the degree of tissue damage. Clinical Manifestations: Early heartburn (particularly with spicy or fatty meals, exercise, and recumbent positions) that is relieved by antacids. Midcourse heartburn accompanied by high epigastric and substernal pain, regurgitation, dysphagia Late bleeding, dysphagia, disappearance of heartburn, 50% patients develop esophagitis. Esophageal stricture formation occurs in about 10% of clients with esophagitis. It is manifested by the gradual development of solid food dysphagia, progressive over months to years. Often there is a reduction in heartburn because the stricture acts as a barrier to reflux. Most strictures are located at the gastroesophageal junction. Esophageal varices can be caused by portal hypertension (elevated portal pressure beyond 10 mmHg) in association with cirrhosis, liver parenchymal disease, duodenal ulcer, or acute pancreatitis, as well as Budd Chiari syndrome and infections such as schistosomiasis. Bleeding esophageal varices eventually develop in about 50% of individuals with cirrhosis. Portal veins narrow and become obstructed as a result of the underlying disease process. As the lumen narrows, the venous blood returning to the right atrium from the intestine and spleen seeks new routes through collateral vessels. These collateral vessels enlarge and become tortuous, and the mucosa ulcerates. With acute bleeding, the mortality rate is about 50%. Approximately 60% of individuals die within a year of the first episode of bleeding. Weight loss and dysphagia are not typical. Mallory-Weiss syndrome is a non-penetrating mucosal tear at the gastroesophageal junction. The syndrome is brought on by events that suddenly raise trans-abdominal pressure, such as lifting, retching or vomiting, ulcers, malrotation, volvulus, or inflammatory conditions of the stomach and intestine. Any disorder that brings on vomiting may result in the development of a Mallory-Weiss tear. The disorder develops as a linear laceration at the gastroesophageal junction. The round shape makes longitudinal tears occur more easily. Alcoholism is a strong predisposing factor for Mallory-Weiss. Hiatus hernia is seen in 35-100% of the patients. Mallory-Weiss tears appear most often in the 50 to 60-year-old clients. The tears can occur also in children. Mallory-Weiss tears occur equally in men and women; however, in women of child bearing age, pregnancy should always be ruled out as the cause, due to hyperemesis gravidarum in the first trimester. Bleeding is the main sign of Mallory-Weiss syndrome; weight loss and dysphagia are not typical.

Case A family friend calls you for medical advice. She just gave birth to her third child 1 week ago. She is worried after a nurse from the hospital called her. The nurse said the baby tested high on a newborn screening for phenylalanine and that she would set up an appointment at a specialty center for the family. Your friend could not remember the name of the disorder, but she was told that it could prevent the infant from normal neurological development if left untreated. The friend reports that her pregnancy was uncomplicated and that the infant appears healthy so far. No one in the infant's immediate family has any health problems. Question What will most likely be recommended treatment for the newborn's condition?

Correct answer: Dietary control Explanation This newborn most likely has phenylketonuria (PKU), which is an autosomal recessive metabolic disorder that causes abnormal metabolism of phenylalanine and results in elevated serum phenylalanine. Dietary control, primarily by restriction of phenylalanine, is the mainstay of treatment. PKU is rare and is diagnosed by routine newborn screening. A strict diet low in phenylalanine can prevent complications of the disease, and afflicted patients can lead a relatively normal life. Neurologic deficits, such as intellectual disability and seizures, can occur if untreated. If an afflicted patient does not adhere to the special diet, they may begin having cognitive deficits and/or seizures. Administration of growth hormone may occur in cases of congenital adrenal hypoplasia. It has no role in PKU. Cognitive behavioral therapy is an excellent treatment for several psychiatric disorders, but it has no role in newborns and infants, and it generally cannot prevent the cognitive decline in PKU. Oxygen therapy and surgery are not appropriate treatments for PKU.

Case A 22-year-old man is brought to the Emergency Department with nausea, fever, loss of appetite, and right lower quadrant pain. He was in his usual state of good health until about 1 day prior to admission when he began having poorly localized abdominal pain. The pain worsened and became sharp and localized to the right lower quadrant. His fever also worsened, and he was found to have an elevated white blood cell count. Question What is the most common early symptom of the patient's condition?

Correct answer: Diffuse abdominal pain Explanation Appendicitis is an inflammation of the appendix. Appendicitis is a common infection of the abdomen that can lead to surgical intervention. The appendix is attached to the first part of the colon, called the cecum. It is a blind pouch and is actually called the vermiform appendix (which means "worm-like appendage"). The appendix is part of the immune system and contains lymphatic tissue. The appendix produces mucus, and as objects (usually stool) enter the appendix, they can become trapped. In addition, mucus can thicken and accumulate. As a result, the appendix can become blocked by the resulting fecalith. After the appendix becomes blocked, bacteria can increase in numbers and cause infection. The infection can spread from the appendix out into the abdomen. The appendix can rupture, and the spread of infection starts as an abscess around the appendix. Occasionally, the body can contain the infection and resolve. Differential diagnosis includes: kidney disease, pelvic inflammatory disease, gall bladder disease, right-sided diverticulitis, and Meckel's diverticulitis. The most common complication of appendicitis is perforation, which causes the infection to spread outside the appendix. Other complications include peritonitis, sepsis, and intestinal blockage. Symptoms

Case A 50-year-old man presents with a lump in his groin for 2 weeks. He states he was lifting an air conditioner and he felt a pop in his groin and began to notice an outpouching in his lower abdomen that has become mildly tender over the last week. Resting and lying flat appears to help; standing and lifting aggravate it. He denies any fevers, nausea, vomiting, or changes in bowel habits. Patient denies any previous abdominal surgeries or procedures. Upon examination, you identify a soft, reducible mass in the lower abdomen and hernia examination reveals a mass pushing against the side of your finger. You order an ultrasound of the lower abdomen and find the intestinal sac has traversed through a weakened area of the abdominal wall and through Hesselbach's Triangle. Question What type of hernia does this patient have?

Correct answer: Direct inguinal hernia Explanation Direct inguinal hernia is correct because the direct inguinal hernia enters through the weakened abdominal fascia and into the anatomic region known as Hesselbach's Triangle. This area is bordered by the rectus abdominus, the inferior epigastric artery, and the inguinal ligament. Femoral hernias, indirect inguinal hernias, and umbilical hernias do not traverse through Hesselbach's triangle. There was no previous surgery for an incisional hernia to occur. If the hernia strikes the pad or side of the finger, it is usually direct, if it strikes the distal aspect of the finger it is usually indirect.

Case A full-term neonate with an Apgar score of 9 at birth developed abdominal distension with watery diarrhea and flatulence following the initiation of milk feeding. The infant became normal when the milk feeding was terminated. Question What is the most likely diagnosis?

Correct answer: Disaccharidase deficiency Explanation The disaccharidases, the most important of which is lactase, are located in the apical cell membrane of the villous absorptive epithelial cells. A deficiency of disaccharidase leads to the incomplete breakdown of the disaccharide lactose into its monosaccharides, glucose and galactose, leading to diarrhea from the unabsorbed lactose. Bacterial fermentation of the unabsorbed sugars leads to increased hydrogen production, which is readily measured in the exhaled air by gas chromatography. There are both hereditary and acquired forms. Histologically, there is no abnormality of the mucosal cells of the bowel. Malabsorption is promptly corrected when exposure to milk and milk products is terminated. Celiac sprue is a rare chronic disease in which there is a characteristic mucosal lesion of the small intestine in the form of blunting of the villi and an overall increase in plasma cells, lymphocytes, macrophages, and eosinophils in the lamina propria. Also known as the gluten-sensitive enteropathy, non-tropical sprue, and celiac disease, it is due to sensitivity to gluten, which is an alcohol-soluble water-insoluble protein component called gliadin of wheat and other closely related grains. Detection of circulating anti-gliadin or anti-endomysial antibodies strongly favors the diagnosis. Both the symptoms and the mucosal histology improve on withdrawal of gluten from the diet. Tropical sprue is a celiac-like disease occurring in people living or visiting the tropics. Bacterial overgrowth by an enterotoxigenic organism is said to be the cause of this disease. Intestinal changes are similar to celiac sprue, but injury is seen at all levels of the small intestine. This is in contrast to celiac sprue, where it is concentrated in the proximal small intestine. It is mainly treated with broad-spectrum antibiotics. Whipple disease is a rare condition that may involve any organ of the body, but principally affects the intestine, central nervous system, and joints. It is caused by Gram-positive actinomycetes called Tropheryma whippelii. Histologically, the small intestinal mucosa is laden with distended macrophages in the lamina propria. These macrophages contain periodic acid-Schiff positive granules and rod-shaped bacilli by electron microscopy. Patients usually present with a form of malabsorption, including diarrhea and weight loss, and they generally respond to antibiotic therapy. Amebic dysentery is caused by Entamoeba histolytica, a protozoan, and generally causes diarrhea with blood and mucus. A stool examination will show the cysts and trophozoites of Entamoeba histolytica, which is diagnostic.

Case A 22-year-old African-American woman comes to the emergency department because of low-grade fever, abdominal pain, and severe, watery, sometimes blood-tinged mucous diarrhea, with 10-15 stools per day for the last 48 hours. She says she has been drinking lots of liquids since the symptoms started, but remained diarrheic. One week ago, she had rhinorrhea with a purulent discharge, for which she received a prescription of amoxicillin. Her family history is unremarkable. Vital signs on admission are BP 110/70 mmHg, HR 86 bpm, RR 16/min, and temperature 38.0°C (100.4°F). Her abdomen is soft, diffusely tender to palpation, with mild rebound tenderness. The remainder of the physical examination shows no abnormalities. A complete blood count shows Hct 46%, Hb 13.1 mg/dL MCV 82 fl, MCHC 30 g/dL, WBC 16.200 /mm3. Na+ 138, K+ 4.8, Cl- 101. A plain abdominal radiograph is normal. Antibiotic-associated colitis is suspected, and a rapid diagnosis is sought. Question What is the most appropriate next step?

Correct answer: ELISA assay for Clostridium difficile toxin Explanation Antibiotic-associated colitis is an important complication of antibiotic use, and is estimated to occur in 1% of hospitalized patients. The etiologic agent is Clostridium difficile, a Gram-positive bacillus that colonizes the bowel of 2-3% of healthy adults. Antibiotic use, particularly clindamycin, penicillins, and cephalosporins, can lead to proliferation of C. difficile. This organism produces 2 toxins, termed A (enterotoxic) and B (cytotoxic), which cause colitis. The most common symptoms are watery diarrhea and abdominal pain. Patients with severe disease can have intense diarrhea, with up to 30 stools per day, low-grade fever, and abdominal pain. A minority of cases progress to severe colitis and toxic megacolon. An ELISA assay for C. difficile toxin is one of the most popular tests for diagnosing C. difficle colitis because it yields rapid results (usually within 24 hours). It is highly specific (99%), but obtaining maximum sensitivity may require repeat testing. The estimated sensitivity of ELISA testing is 60-95%. An alternative to ELISA is PCR analysis for toxic. Compared to ELISA, PCR has a relatively higher sensitivity. Common antigen testing tests for GDH, an enzyme that is specifically produced by the C. difficle organism. This test is sometimes used to screen samples with follow-up toxin assays performed on positive samples. It is not a routine first-line test for the diagnosis of C. difficle colitis. Anaerobic culture for C. difficile take a long time, is costly, and is primarily reserved for epidemiological purposes. Fecal leukocytes have low sensitivity for antibiotic-associated colitis and are found in only 50% of cases. Sigmoidoscopy or colonoscopy may be used as an adjunct to laboratory diagnosis in certain settings, such as when there is a high clinical suspicion of C. difficle colitis despite a negative toxin assay. It should be emphasized that endoscopic bowel examinations are more dangerous in patients with megacolon and should be used carefully if this complication is suspected.

Case A 52-year-old Caucasian man presents with easy fatigability, poor concentration, and "stronger than usual" effort-related palpitations. He denies any constipation or weight loss. His past medical history includes type II diabetes, which was diagnosed 10 years ago; it is well controlled with diet and metformin, and there are no complications. Both his parents are in good health; his sister had a malignant breast nodule removed 2 years ago, but she showed no signs of metastatic disease. His physical examination shows mild obesity (BMI 32 kg/m2), but it is otherwise normal. A laboratory workup shows glucose 91 mg/dL, Na+ 138 mEq/L, K+ 4.9 mEq/L, Cl- 100 mEq/L, hemoglobin 11 g/dL, hematocrit 32%, mean corpuscular volume(MCV) 82 fl, mean corpuscular hemoglobin concentration (MCHC) 24 pg/mL, leukocytes 7,400 /mm3 w/normal differential. Platelets 310,000 /mm3, ferritin 29 ng/L (25-300 ng/mL), and a total iron binding capacity of 450 ng/dL. Question What is the most appropriate next step in diagnosis?

Correct answer: Fecal occult blood testing (FOBT) Explanation The correct response is fecal occult blood testing. The pattern of low mean corpuscular volume and hemoglobin concentration, low ferritin, and elevated total iron-binding capacity (TIBC) is diagnostic of iron deficiency. Lack of iron inhibits hemoglobin synthesis, thereby reducing the volume and hemoglobin concentration of erythrocytes. Some patients have thrombocytosis. The mechanism is unclear, but hypotheses include stimulation by high circulating erythropoietin concentrations or circulating cytokines. Besides asthenia and pallor, other symptoms include pagophagia (craving ice or cold foods), pica (clay eating), and leg cramps; physical examination may reveal esophageal webbing, koilonychia (spoon-shaped nails), glossitis, angular stomatitis, and gastric atrophy. New-onset anemia with hypochromia and microcytosis in an adult should be considered secondary to gastrointestinal bleeding until proved otherwise.The most common etiologies are diverticular disease, angiodysplasia, colitis (including from NSAIDs), and colon cancer, particularly left-sided colon cancer. Testing for occult blood in a stool specimen is the initial diagnostic method of choice. If the fecal occult blood test is positive, the patient should undergo upper endoscopy and colonoscopy examinations. Colonoscopy is a useful exam for identifying sources of occult gastrointestinal bleeding. It is more sensitive than barium enema or CT scans for early lesions and permits biopsy when indicated (e.g., lesions suspicious for malignancy or colitis). It would be considered among a next step treatment plan, and is used as screening after the age of 50, but a first step would be a fecal occult blood test. Bone marrow biopsy can evaluate bone marrow iron stores and reveal erythroid hyperplasia. However, it has been practically abandoned in the workup of iron-deficiency anemia, as there is considerable interobserver variability and assays for iron, ferritin, and TIBC permit evaluation of the body's iron stores less invasively. Capsule endoscopy is the most sensitive method for diagnosing small bowel bleeding. However, small bowel bleeding is much less common than upper (proximal to the angle of Treitz) or colonic bleeding. Therefore, it should only be undertaken after colonoscopy and upper endoscopy are both negative.

Case A 71-year-old woman who was admitted for jaundice, nausea, and vomiting. Reviewing the abdominal ultrasound and note the following interpretation: "dilated and edematous common bile duct with evidence of chronic scarring." The patient has achieved some relief from her symptoms in the last 24 hours after initiation of intravenous antibiotics and hydration. She has had similar occurrences of this at least once yearly for the past 3 years. Question Due to the patient's medical history, what intervention would be appropriate at this time?

Correct answer: ERCP with stent placement Explanation The patient described above is most likely having acute attacks due to accumulated gallstones in her gallbladder, which are leading to blocked common bile ducts. Over time, as the patient has another acute attack, this will cause the walls of the gallbladder to shrink; this will potentially lead to the gallbladder not being able to function as efficiently as it should, leading it to not being able to concentrate, release, or store bile as it should. Each acute attack causes active inflammation that eventually leads to permanent scarring of the common bile duct; over time, the scarring in itself may lead to a blocked duct. The most common clinical intervention for a patient such as the one above would be a laparoscopic cholecystectomy; however, our patient has an obstruction in the common bile ducts. For this reason, the most viable option to help decrease these repeated instances of acute attacks will be performing an endoscopic retrograde cholangiopancreatography (ERCP) with a common bile duct stent placement. Then followed up by a scheduled laparoscopic cholecystectomy once symptoms resolve. Magnetic resonance angiography with angioplasty is not an appropriate intervention for bile ducts and is usually reserved for blocked blood vessels. A CT guided biopsy would only be indicated if potentially an unusual mass was found on the imaging studies to assess if it is malignant but this was not seen. Daily oral antibiotics may help decrease the acute inflammation due to the blocked bile duct; however, this will not resolve the permanent cellular changes that have occurred due to the scarring.

Case A 33-year-old man with no past medical history presents with groin mass. He denies pain and trauma to the region. When the patient stands, there is the presence of a round swelling in the inguinal area. If the patient is supine, the mass disappears. Question What is the appropriate intervention in this patient?

Correct answer: Elective surgical repair Explanation Elective surgical repair is correct. The patient has an inguinal hernia, which can be reduced easily. Immediate surgical repair is incorrect. The patient's hernia should be repaired. However, the hernia is not currently incarcerated and can be treated electively. Oral antibiotic therapy and IV antibiotic therapy are not indicated in the treatment of hernias. No intervention is needed is incorrect. The presence of a hernia signals a risk for potential incarceration in the future. The patient is young with no comorbidities. He should be scheduled for elective repair to avoid the development of incarceration.

Case An 11-year-old boy presents with increasing cognitive trouble in school. His parents state that he has had more trouble with grades and schoolwork since starting middle school. He also seems slightly more defiant. His history is significant for a metabolic disorder; it was diagnosed at birth, but it has been controlled with diet. He is interviewed while his parents are out of the room; mild cognitive impairment is noted. He tells you that, since he started middle school, he has been "cheating" on his diet. Question Based upon the information above, what laboratory finding would be expected in this patient?

Correct answer: Elevated serum phenylalanine Explanation Phenylketonuria is an autosomal recessive metabolic disorder, which causes abnormal metabolism of phenylalanine, resulting in elevated serum phenylalanine. It is a rare condition, and it is diagnosed by routine newborn screening. A strict diet low in phenylalanine can prevent complications of the disease, and afflicted patients can lead a relatively normal life. Neurologic deficits, such as intellectual disabilities and seizures, can occur if it is untreated. If an afflicted patient does not adhere to the special diet, he or she may begin having cognitive deficits and/or seizures. Chronic decreased serum vitamin D can lead to poor bone development and rickets. It is not diagnosed at birth. In severe cases, there can be some cognitive deficits, but overall this patient's history does not suggest vitamin D deficiency. Elevated serum gastrin is associated with Zollinger-Ellison syndrome (ZES), in which a gastric-secreting tumor leads to acid secretion and peptic ulcer disease. It affects older individuals; furthermore, it would not cause the cognitive changes seen in the patient. Elevated sweat chloride is the classic test finding with cystic fibrosis (CF), a chronic and progressive lung disease. CF could be diagnosed in the newborn, but it is not controlled by diet, and it does not cause cognitive decline. Presence of Bence Jones protein in the urine is primarily associated with multiple myeloma, a bone malignancy. Multiple myeloma is neither diagnosed at birth, nor is it controlled by diet.

Case A 70-year-old Caucasian man presents with a 2-month history of rapidly progressive jaundice, severe gnawing epigastric pain radiating to the back, loss of 15% of body weight, nausea, vomiting, loss of appetite, and asthenia. On physical examination, he is alert and oriented, emaciated, and is markedly jaundiced. Cardiovascular and respiratory systems are normal. The epigastrium is painful to palpation, but there is no well-defined palpable mass. The liver is palpable 2 cm below the right costal border, with a hardened consistency and irregular contours. The remainder of the examination shows no other abnormalities. An abdominopelvic CT scan shows an irregular mass originating in the pancreatic head, enlarged gallbladder, and small nodules throughout an enlarged liver. Question What is the most appropriate next step in management?

Correct answer: Endoscopically insert a biliary stent Explanation Pancreatic cancer is more common in men (1.2 to 1.5 male-to-female ratio). The disease is rare before the age of 50 years, and the median age of diagnosis is 69 in whites and 65 in blacks. Risk factors include smoking (heavy smokers have a two to threefold higher incidence), chronic pancreatitis, long-standing diabetes, family history, and obesity, The most common signs and symptoms are jaundice (in obstructive lesions), weight loss, and abdominal pain. More rarely, diabetes mellitus, splenomegaly, a palpable gallbladder (Courvoisier's sign), and migratory thrombophlebitis (Trousseau's sign) are seen. The delayed diagnosis explains the disease's 98% lethality rate. Several imaging modalities (e.g., transabdominal ultrasound, computed tomography, magnetic resonance imaging, endoscopic ultrasound, and endoscopic retrograde cholangiopancreatography [ERCP]) can visualize these tumors, but the vast majority of them are diagnosed late in their course because symptoms only occur in advanced disease stages. This patient has unresectable, stage IV pancreatic cancer (see pancreatic cancer staging) and biliary obstruction. Treatment is palliative, and the initial goal is to re-establish patency of the biliary tract. Several approaches are possible: endoscopic stenting using endoscopic retrograde cholangiopancreatography, percutaneous drain placement, and operative redirection of the bile ducts. Endoscopy is usually the preferred method. Several techniques are available, including plastic and expandable metal stents, lasers, and photodynamic therapy. Duodenopancreatectomy consists of removal of the pancreatic head, duodenal arch, gallbladder, and gastric antrum. Reconstruction is performed by anastomosing the remaining jejunal loop end-to-end to the pancreatic remnant, a biliary-enteric anastomosis, and a Billroth-II gastric reconstruction. It's the standard procedure for pancreatic cancer resection with curative intent. However, this patient has unresectable disease. Gemcitabine affords symptomatic improvement but improves survival only slightly. While it might be useful in this patient, establishing biliary drainage takes precedence over chemotherapy. Radiotherapy is sometimes useful for palliation of pain. However, pancreatic cancer is relatively radioresistant, and survival is not improved.

Case A 50-year-old woman presents with a 4-day history of diarrhea, vomiting, and muscle pains. The stools are watery. She has just returned from vacationing in Asia and Africa. Examination of the stool reveals no fecal leukocytes or trophozoites. Question What is the causative organism?

Correct answer: Enterotoxigenic Escherichia coli Explanation Patients with enterotoxigenic Escherichia coli food poisoning usually present with watery diarrhea. Other symptoms include vomiting, abdominal pain, and muscle pains with fever. Onset is 12-72 hours after ingesting contaminated water, raw fruit and/or vegetable salads. As it is the most common cause of traveler's diarrhea, patients may give a history of travel. Examination of the stool reveals no fecal leukocytes or trophozoites. The mainstay of treatment is fluid replacement, as most cases are mild and self-limiting. They typically last for 1-5 days. Antibiotics (e.g., Trimethoprim-sulfamethoxazole) are indicated for severe diarrhea, especially if it is associated with vomiting and fever. Patients with Shigella dysenteriae food poisoning present with bloody diarrhea; symptoms usually start 16 hours after ingesting contaminated foods (e.g., potato salad). A stool examination reveals polymorphonuclear leukocytes. In cases of emetic Bacillus cereus food poisoning, patients develop vomiting 1-6 hours after ingesting contaminated food (e.g., reheated fried rice). In the diarrheal form, they develop watery diarrhea 8-16 hours after ingestion. There are no fecal leukocytes or trophozoites. In adults with Norwalk virus food poisoning, the watery diarrhea is more prominent than vomiting. In children, the reverse is true. The incubation period is 1-3 days. There are no fecal leukocytes or trophozoites. Entamoeba histolytica colitis develops 2-6 weeks after ingesting contaminated water or food. Patients usually present with bloody and mucoid diarrhea. Stool examination reveals hematophagous trophozoites and cysts.

Case A 27-year-old man has been experiencing heartburn on and off for approximately 6 years. At this current visit, the patient is also describing symptoms of chest pain, dysphagia, and even early satiety. Because of the persistence of his symptoms despite pharmacologic therapy with both proton pump inhibitors and various histamine blockers, you decide to consult with a local gastroenterologist group. The patient undergoes an endoscopy, with biopsies of tissue. Results of the biopsy show no ulcers and are H. pylori negative; the tissue samples themselves are prominently infused with an abundant amount of eosinophils. Question Based on the history and physical exam findings above, what is the most effective pharmaceutical intervention at this time?

Correct answer: Fluticasone metered dose inhaler Explanation The correct response is fluticasone metered dose inhaler. The patient described in the above scenario has the diagnosis of eosinophilic esophagitis (EE). Because this is a relatively new diagnosis, the prevalence of EE is unknown, although the incidence is increasing in both children as well as adults. These patients will have symptoms typically 4 - 5 years before they have a confirmed diagnosis. EE is more commonly seen in male patients who are Caucasian, and up to 70% of patients with EE have personal history of atopy to environmental or foods. The most common symptoms found in adults with EE are dysphasia, refractory heartburn, chest pain, and food impaction. Usually, no physical exam abnormalities will be detected. An endoscopy with esophageal biopsy is the only definitive method to diagnose EE. The presence of eosinophils of 15 or more per high power field is required for a diagnosis. Other physical findings that may be visualized during an endoscopy that are highly suggestive of EE include stacked circular rings ('feline esophagus'), proximal strictures, linear furrowing, as well as white exudates or even papules that signify eosinophilic purulent material. Currently there are no FDA approved medications for the treatment of EE. The most efficient treatment for symptoms is topical corticosteroids, either via fluticasone metered dose inhaler or viscous budesonide. These medications are swallowed and therefore coat the esophagus; the symptoms as well as the overall eosinophilia improve fairly quickly. Patient response is usually in the first few days of treatment; however, symptoms will reoccur once the medication regimen is discontinued. Topical fluconazole is not the correct option; EE is not a fungal infection. Oral ampicillin is not correct because EE's underlying pathology is not a bacterial infection. Oral omeprazole is not specifically recommended for treatment of EE; however, it may treat the co-existent GERD. Oral cetirizine is also not an appropriate choice; if there is evidence of severe atopy, therapy should be initiated, but no evidence presents in the case scenario showing that the treatment of airborne allergic disorders improves EE.

Case A 30-year-old man presents with a 1-hour history of severe nausea and vomiting. Prior to falling ill, he had been at a party where he ate pudding, along with other food. Physical examination reveals a normal temperature with mild, diffuse tenderness of the abdomen. The organism isolated is a Gram-positive coccus that occurs in grape-like clusters, is catalase and coagulase-positive, and forms a golden-yellow colony on agar. Question What toxin released by the causative organism is responsible for the patient's symptoms?

Correct answer: Enterotoxin Explanation Enterotoxin produced by Staphylococcus aureus is an exotoxin; it is responsible for the patient's condition. The clinical presentation and the laboratory findings (e.g., a short incubation period of 1 - 6 hours with predominant emesis) are suggestive of Staphylococcal food poisoning. S. aureus organisms are Gram-positive cocci that occur in grape-like clusters. They are catalase and coagulase-positive, and they form golden-yellow colonies on agar. Staphylococcal food poisoning results from the ingestion of preformed enterotoxins on food contaminated with S. aureus. Bacteria growing in carbohydrates and meat products produce enterotoxins that, upon ingestion, diffuse into the circulation and cause emesis by stimulating the vomiting center in the central nervous system. Toxic shock syndrome toxin (TSST-1) has superantigen activity, and when expressed systemically it causes life-threatening toxic shock syndrome. The clinical presentation includes fever, hypotension, and diffuse macular erythema; there is involvement of 3 or more of the organ systems (i.e., gastrointestinal, renal, hepatic, musculoskeletal, and nervous system). Exfoliatin toxin (ET) elaborated by S. aureus causes scalded skin syndrome; this is manifested as widespread blistering and loss of the epidermis, revealing a red base. ETA and ETB are the 2 antigenically distinct forms of the toxin. ET has esterase and protease activity, which targets a protein involved in maintaining epidermal integrity; this causes separation of the epidermis. Leukocidin is a membrane-damaging toxin expressed by S. aureus that acts on polymorphonuclear leukocytes. Alpha toxin, or alpha hemolysin, is the most potent membrane-damaging toxin of S. aureus; it causes hemolysis.

Case A middle-aged woman presents with profuse, bloody diarrhea, and abdominal cramps that started this afternoon. She does not remember eating any chicken, pudding, or ice cream in the past day; however, she went to a restaurant yesterday and ate a rare steak. She appears ill from from cramps, but she is not dehydrated. She doesn't have a fever. Her blood pressure and pulse are within normal limits. Question What is the most likely cause of her condition?

Correct answer: Escherichia coli O157: H7 strain Explanation Serotype O157: H7 Escherichia coli is commonly involved in enterohemorrhagic (verotoxin-producing) E. coli gastroenteritis. It has been shown to produce a Shiga-like toxin, which is also called verotoxin (verocytotoxin) because of the cytotoxic effect on Vero cells in culture. The diarrhea usually is rapid-onset, profuse, bloody, and self-limiting. One of the severe complications is hemolytic-uremic syndrome (i.e., acute renal failure, thrombocytopenia, and microangiopathic hemolytic anemia). See the table for differential diagnosis of food-born bacterial diseases.

Case A 69-year-old man presents with a 1-day history of diarrhea that was initially watery; it is now bloodstained. He lives on a farm and reports that the drinking water had recently been contaminated by cow manure. A stool examination reveals polymorphonuclear leukocytes, but no trophozoites. Question What is the causative organism?

Correct answer: Escherichia coli serotype Explanation In Escherichia coli serotype O157:H7 (also termed enterohemorrhagic E. coli food poisoning), patients present with severe, crampy abdominal pain and watery diarrhea which can become bloodstained within 24 hours. Sources of infection include drinking unpasteurized milk and water contaminated by cow manure, as well as ingesting raw or undercooked beef. A stool examination reveals polymorphonuclear leukocytes; E. coli can be isolated from stool cultures. Complications include hemolytic-uremic syndrome. Management includes supportive care with rehydration. Patients with Staphylococcus aureus food poisoning may present with vomiting and watery diarrhea 1-6 hours after ingesting contaminated food (e.g., processed meat). There are no fecal leukocytes or trophozoites. In Clostridium perfringens food poisoning, patients present with watery diarrhea 8-24 hours after ingesting contaminated food (e.g., inadequately cooked meat or legumes). No fecal leukocytes or trophozoites are present on stool examination. Giardia lamblia can be acquired by drinking contaminated water; patients present with watery diarrhea. The incubation period is around 1 week. There are trophozoites in the stool, but no leukocytes. Patients with Salmonella typhi food poisoning usually present with bloody diarrhea. Sources of infection include beef, poultry, eggs, and dairy products. Examination of the stool reveals mononuclear leukocytes.

Case A 53-year-old man presents with increased difficulty swallowing and occasional regurgitation of his meals. His symptoms have been occurring with greater frequency and severity over the last 4 months. He also gets some shortness of breath but attributes that to his weight and lack of physical activity. His past medical history is remarkable for chronic heartburn, which he treats intermittently with over-the-counter antacids. He takes no regular medications and he has no allergies. He has not had any surgeries. He is a smoker, but he denies use of alcohol and drugs. He works as a building inspector and he lives with his wife and children. The patient is obese, but the rest of his physical exam is normal. Blood tests, electrocardiogram, and chest X-ray are done in the clinic; they are normal. He is referred for endoscopy, and esophageal biopsy shows specialized intestinal metaplastic cells (of columnar epithelium). Question What is the medication of choice for this patient's condition?

Correct answer: Esomeprazole Explanation This patient is presenting with Barrett's esophagus, which is a type of chronic esophagitis in which the normal squamous epithelium is replaced by columnar epithelium. Barrett's esophagus is a complication of chronic gastroesophageal reflux disease (GERD) and can develop into esophageal adenocarcinoma. Use of proton pump inhibitors (PPIs), such as esomeprazole, reduces the risk of cancer. In order to monitor for the development of cancer, a routine endoscopy should be periodically performed in patients with known Barrett's esophagus. Alendronate is a bisphosphonate; it is used for both treatment and prevention of osteoporosis. Alendronate can cause multiple gastrointestinal adverse effects, including esophagitis and esophageal ulcerations and strictures. This medication could dramatically worsen this patient's current condition. Dicyclomine is an anticholinergic medication that relaxes smooth muscle. It is used for spasms associated with irritable bowel syndrome. It does not have a role in treating GERD or Barrett's esophagus. Famotidine antagonizes H2 histamine receptors (H2 blocker). It is used in treating duodenal and gastric ulcers as well as GERD. H2 blockers are usually considered less potent than PPIs. PPIs are the treatment of choice for Barrett's esophagus. Simethicone is sold in various over-the-counter products for flatulence. It has no role in treating GERD or Barrett's esophagus.

Case A 45-year-old chronic alcoholic presents with history of massive hematemesis. This hematemesis followed a bout of prolonged vomiting. Patient has been a known alcoholic for 20 years. On examination, he has a pulse rate of 100/min and a BP of 90/70 mm Hg with cold extremities. Question These findings are typical for what condition?

Correct answer: Esophageal laceration Explanation Esophageal laceration extends to the submucosal veins that bleed profusely, also known as Mallory-Weiss syndrome. Hiatus hernia may cause a reflux along with inflammation and possible ulceration, but bleeding is not typically massive. Diverticula of the esophagus usually do not bleed. Barrett's esophagus (metaplasia with gastric mucosa) is associated with reflux and inflammation and possible ulceration, but bleeding is not usually massive. Esophageal squamous cell carcinoma will cause ulceration, but massive bleeding is not common.

Question A 43-year-old man presents with fever, productive cough, and wheezing, leading to an intial diagnosis of bilateral patchy pneumonia and admission to the hospital. The patient's history is positive for increasing dysphagia for both solids and liquids over the past 2-3 years, and an approximate weight loss of 12 lbs recently. He occasionally gets severe chest pain for which he uses OTC meds but he has not consulted a physician. At night he experiences bouts of coughing. 8 months ago he had bronchitis and 4 months ago he had pneumonia. Both were treated on an outpatient basis. A barium swallow shows a dilated flaccid esophagus filled partly with fluid. Endoscopy reveals erythematous esophageal mucosa with resistance to passage of the endoscope at the lower esophageal sphincter. No structural lesions are found. What is the next step in the definitive diagnosis of this condition?

Correct answer: Esophageal manometry Explanation This patient is suffering from achalasia of the cardia. This is a motility disorder of the esophagus that occurs due to a neurological deficit from fibrosis, scarring in the Auerbach's plexus, and degeneration of vagal fibers in the esophagus. This causes decreased peristalsis in the body of the esophagus and incomplete relaxation of the lower esophageal sphincter. Dysphagia to solids and liquids, weight loss, chest pain, nocturnal cough due aspiration of esophageal contents, and recurrent bronchitis and pneumonia are classic symptoms. Radiography and endoscopy are performed to rule out anatomic pathology. Manometry is needed for the definitive diagnosis, which shows elevated lower esophageal sphincter pressure, absence of normal peristalsis in the entire esophagus, and incomplete relaxation of the lower sphincter. Treatment is with pneumatic dilatation, endoscopic injection of botulinum toxin, or Heller myotomy. Biopsy of the mucosa may rule out malignancy at the esophagogastric junction (which can cause secondary achalasia or pseudoachalasia) will not assist in the definitive diagnosis of achalasia. Swallow evaluation and video fluoroscopy permit evaluation of the cervical esophagus only, and therefore are inadequate. CT scan of the chest is indicated in malignancy related and obstructive esophageal conditions. 24 hour pH monitoring is a tool used for diagnosis of reflux esophagitis.

Question A 45-year-old man presents with a 24-hour history of severe anal pain and swelling. The pain started after straining at defecation and has worsened over the course of the day. There is no history of fever. Examination of the anal area reveals a swollen ecchymotic mass in the perianal skin, very close to the anal verge. What is the treatment of choice for this condition?

Correct answer: Excision of thrombosed external hemorrhoid Explanation The clinical case described most likely represents a patient with a thrombosed external hemorrhoid. External hemorrhoidal thrombosis is not infrequent and can be seen in patients with no prior history of hemorrhoidal disease. The cause is thought to be from elevated venous pressures related to excessive straining with constipation and physical exertion following a bout of diarrhea or change in diet. It is an acute painful event with the pain generally lasting 7-14 days and resolving with the resolution of the thrombosis. The clinical presentation is usually as described above, with acute swelling and pain located at or around the anal verge. When the overlying skin of the thrombosed hemorrhoid is eroded, bleeding occurs. The pain usually lasts for a few days and then spontaneously resolves. The swelling may take several weeks to resolve. Treatment depends on the time of presentation and the severity of the condition. Acute thrombosed external hemorrhoid presenting within 72 hours of the onset of symptoms should be excised. Excision of the thrombosed external hemorrhoid can be performed under local anesthesia and is associated with a low recurrence and complication rate. Patients presenting after 72 hours of onset of symptoms should be managed by conservative/symptomatic medical therapy. Antibiotics, immediate surgical drainage, colonoscopy, and sphincterotomy are not the treatment options for thrombosed external hemorrhoid.

Case A 75-year-old woman presents with heartburn and dyspepsia. She was diagnosed with osteoarthritis 4 years prior to presentation, and for the past 18 months, she has been managing pain with naproxen. The gastroenterologist suggests that the patient be tested for Helicobacter pylori infection. Question What is the most sensitive and specific noninvasive method to diagnose this infection?

Correct answer: Fecal antigen test Explanation The long-term use of non-steroidal anti-inflammatory drugs (NSAIDS) in the treatment of arthropathies carries about a 4-fold increase in the relative risk of duodenal or gastric ulcers. This risk is multifactorial; one risk factor for the development of gastric ulcers in patients taking NSAIDS is infection with Helicobacter pylori. Fecal antigen testing is an easy noninvasive method to test for the presence of H. pylori. It has a sensitivity and specificity >90%. A culture of H. pylori is difficult and requires considerable microbiologic expertise. However, indicators in the cultivation medium can detect urease. Histologic examination of gastric biopsies obtained during endoscopy is the gold standard for diagnosis of H. pylori. In older patients, these biopsies should be obtained from several sites in the body and antrum of the stomach in order to enhance detection. In addition, the question calls for a noninvasive method of detection as a first test. The Schilling test is a method to detect abnormal B12 absorption. It is not used to detection H. pylori infection.

Case A 42-year-old woman presents for a post-operative visit 6 weeks following abdominal hysterectomy for fibroids. Her only current medications include the Norco (acetaminophen and hydrocodone) that she was prescribed after surgery, and she continues to take it sparingly. Today, she notes several new symptoms; they include decreased appetite, nausea, "bloating", and mild abdominal pain. These symptoms have been gradually getting worse over the course of the last 2 weeks. Her bowel movements have consisted of small amounts of liquid stools every few days. Question What is the most likely diagnosis?

Correct answer: Fecal impaction Explanation Fecal impaction presents with abdominal pain, nausea, vomiting, decreased appetite, abdominal distention, and diarrhea from liquid stool escaping around the impaction. Opiates can predispose a patient to fecal impaction. The remaining diagnoses are incorrect because they present differently than the patient in this case. C. diff infection typically presents with copious watery diarrhea many times a day. Ascites may present with bloating and abdominal pain, but it is typically associated with chronic liver disease and is not associated with diarrhea. Endometriosis is often asymptomatic, but it can be associated chronic pelvic pain that worsens with menses. While a bowel obstruction can be associated with fecal impaction, in such cases, it is typically a large bowel obstruction. A small bowel obstruction would present with paroxysmal severe mid-abdominal pain and vomiting.

Case An 85-year-old woman who lives in a local nursing home is sent to your primary care clinic due to a 3-week history of inability to pass a bowel movement. The patient has a diagnosis of dementia, and she is delivered via a transportation service and not able to give you much more information about the acute issue. The only notes with the patient are she has had a decreasing amount of bowel movements in the last 3 months. Because she is a long time patient of your practice, you are able to obtain past historical information from her patient chart. Her past medical history includes a longstanding history of depression, hypertension, and more recently, congestive heart failure. For these diagnoses, she is on a multitude of various medications daily, 12 total. On physical examination, you auscultate hypoactive bowel sounds and palpate hard stool in the descending colon. Digital rectal examination is performed, and it reveals copious amount of stool in the rectum. Guaiac test is negative for blood. Question What is the most likely diagnosis?

Correct answer: Fecal impaction Explanation The patient described in the above scenario is most likely to be suffering from fecal impaction secondary to constipation. Using Rome III criteria, this patient is diagnosed as having functional constipation due to the hard stool palpated on exam and the decrease in stool frequency for the last 3 months. Risk factors include female gender, physical inactivity, low education and income, concurrent medication use, depression, and the fact that older adults often eat fewer calories and meals. The patient above is highly likely to be falling under the majority of these risk factors. Constipation in itself plays a major role in the development of fecal impaction, especially in institutionalized adults. It results largely from the person's inability to sense and respond to the presence of stool in the rectum, decreased mobility, and lowered sensory perception. A key finding to fecal impaction is during the digital rectal examination finding copious amounts of stool in the rectum. Colon cancer, Crohn's disease, and ulcerative colitis should always be one's radar when deciphering differentials; however, the more common red flags that would lean a clinician towards needing a more extensive workup for these more serious diagnoses are missing from the patient scenario, including a positive fecal occult blood test, hematochezia, obstructive symptoms, severe persistent constipation unresponsive to treatment, a change in stool caliber, weight loss of at least 10 lbs, and a family history of colon cancer or inflammatory bowel disease. Irritable bowel syndrome (IBS) is not as likely to be the diagnosis due to the fact that the symptoms of IBS usually begin in late teens to early 20's. Part of the main approach to IBS would be to exclude all other organic causes of the symptoms, so it would be premature at this time to even label this as IBS.

Case An 82-year-old woman is brought in by her daughter due to fecal incontinence. The patient has been living with her daughter since a hip fracture 1 month earlier, and she did not have any problems with bowel continence until the last week. The patient is now noticing small amounts of runny stool in her underwear. The patient admits some abdominal discomfort, but she has trouble differentiating abdominal, pelvic, and hip discomforts after her recent fracture. She denies a known history of any gastrointestinal disorders, but she admits some constipation since her hip surgery. She denies any spinal cord trauma. She reports a history of chronic urinary incontinence, which has been controlled on oxybutynin. The patient has no other medical history. She has been taking oxycodone for post-surgical pain, over-the-counter diphenhydramine to help her sleep, and her usual oxybutynin. Her only surgery is the hip fracture repair. She is a non-smoker, and she denies alcohol and drug use. On physical exam, the patient appears slightly sedated; she is thin, with a slightly distended abdomen. She is diffusely tender over her abdomen. Rectal exam reveals a very large amount of stool in the rectal vault. The remainder of her exam is normal. Question What is the most likely diagnosis?

Correct answer: Fecal impaction Explanation This patient's fecal incontinence is likely due to a fecal impaction, as evidenced by the large amount of stool in the rectum. An impaction can lead to leakage of stool around the impaction. This patient has numerous risk factors for constipation, which leads to impaction; 3 of her medications all contribute to constipation and decreased mobility following hip fracture. An enema and/or manual disimpaction may be needed to relieve the impaction; her constipation should then be addressed to prevent recurrence. An anorectal fistula may cause soilage of underwear. However, a skin lesion/tract and drainage would have likely been identified on the patient's physical exam. An impaction is clearly a more likely diagnosis. If the impaction was resolved and fecal leakage continued, a fistula could be considered. Irritable bowel syndrome (IBS) is a chronic condition; it is characterized by abdominal cramping and diarrhea and/or constipation. While this patient's switch from constipation symptoms to leaky stools may suggest IBS, the timing and onset of symptoms so late in life is not consistent with IBS. IBS also would not explain the large amount of stool in the rectal vault. Neurologic dysfunction of the anal sphincter is a possible cause of fecal incontinence. We can assume the sphincter tone was normal on exam (the remainder was noted to be normal), and the patient has no history that suggests a spinal cord or neurologic condition that would correspond with bowel symptoms in the last 1 week. Rectal cancer may cause a mass in the rectum, along with bowel changes, such as the changes the patient in this case described. If the provider is unable to differentiate stool from a solid mass on this patient's rectal exam, further evaluation (imaging and colonoscopy) should be performed.

Case A 55-year-old woman presents with poor appetite and nausea. She has vomited twice over the past week. She has lost 4 pounds in the past month. History is significant for a 15-year history of alcoholism and a 5-year history of hypertension and diabetes, for which she receives no treatment. She has been drinking 2 pints of beer almost every day for the past 20 years, and she consumes greater quantities on weekends. Vitals: BP 140/100 mm Hg, HR 96/min, RR 20/min, and temp 96.8 F. Physical exam reveals hepatomegaly. Lab tests reveal AST of 293 U/L and ALT of 201 U/L. Question What is a major risk factor for the patient's condition?

Correct answer: Female gender Explanation The most probable diagnosis in this patient is alcoholic liver disease (ALD), as deduced from the history of alcoholism, her symptoms, finding of hepatomegaly, and elevated AST and ALT. The risk factors for this condition include female gender, quantity and duration of alcohol consumption, hepatitis C virus, genetic factors, and malnutrition. Women have greater susceptibility to ALD at lower levels of alcohol intake. Typically, more than 20-40 g/d of alcohol for 10 years is the threshold for women to develop ALD; however, for men, the threshold is 3 times this quantity. The quantity and duration of alcohol intake is the most important risk factor, and consumption of 160 g/d increases the risk of developing ALD by 25 times. Therefore, the most important aspect of treatment of this condition is total abstinence from alcohol.

Case A 50-year-old woman presents with "swelling in my right groin" when she stands. On physical examination you note a reducible bulge that is 3 cm below her right groin crease and is lateral to her pubis. The bulge is on the ventromedial surface of the anterior thigh. Question What is the most likely diagnosis?

Correct answer: Femoral hernia Explanation What this patient is experiencing is a femoral hernia. This is typically an acquired lesion and is usually due to the short medial attachment of the transversus abdominis muscle on the Cooper's ligament. This results in an enlarged femoral ring that unfortunately can predispose patients who have this entity to herniate. During physical examination a femoral hernia bulge will be located much lower in the groin than other hernias (well below where the inguinal ring would be located, and even onto the anteromedial thigh of the patient.) Direct inguinal hernia is not the correct diagnosis. This type of hernia bulges directly through the posterior inguinal wall. At times a direct inguinal hernia may also be termed a Hesselbach's hernia due to the fact that it bulges through this area anatomically. This is not nearly as common in women as femoral hernias are. These are also usually acquired pathologies. Epigastric hernia is incorrect. This type of hernia protrudes though the crossing midline fibers over the umbilicus. It is due to a congenital or acquired weakness of the midline linea alba. Indirect inguinal hernia is not the correct option. This type of hernia is the most common abdominal wall hernia that is seen in clinical practice and more often than not will occur in younger, male patients. The reason it occurs is a patent processus vaginales. During examination, this type of hernia will appear as if it is descending directly into the scrotum, which is inconsistent with our patient above. Spigelian hernia is not the correct answer. This is a rare type of hernia that occurs though the semilunar line, located at the lateral margin of the rectus muscle. This will appear to lie cephalad to the inferior epigastric vessels.

Case A 41-year-old woman presents due to rectal pain. The pain is brought on by bowel movements and is described as an intense tearing pain. She notes a small amount of blood on the toilet paper, but none in the stool itself. She has a bowel movement approximately every 2 to 3 days and notes that her stool is somewhat hard. On examination, a small crack is noted in the posterior midline. Hemoccult is negative. There are no masses noted on rectal examination. Question What is the recommended initial treatment for the patient's condition?

Correct answer: Fiber supplementation and sitz baths Explanation Fiber supplementation and sitz baths is correct. The patient has an anal fissure. Anal fissures are associated with tearing pain during defecation, constipation, hematochezia, and the presence of a crack in the epithelium on examination. Since anal fissures are associated with constipation, fiber treatment is recommended to make the stool easier to pass, therefore placing less strain on the affected area. Sitz baths help alleviate the discomfort and irritation associated with anal fissures. Oral miralax is incorrect. Fiber should be trialed before using miralax to treat constipation. Topical nitroglycerin ointment is effective in the treatment of anal fissure, but it is considered a second line therapy. Oral prednisone is incorrect. Anal fissures that occur off the midline can potentially be caused by Crohn's disease, which could be treated with oral prednisone. However, the patient's fissure is located in the posterior midline, and she does not have any of the other typical symptoms of Crohn's disease, such as diarrhea and abdominal pain. Surgical intervention may ultimately be required for anal fissures, but is not considered a first line therapy.

Case A mother presents with a 5-week-old male infant for excessive vomiting. He has been having consistent episodes of projectile vomiting 30-40 minutes after feeding and is not gaining weight. The infant has no significant past medical history, although the mother mentions that she began renovating their Victorian-era house while pregnant. He has a 3-year-old sister, who is healthy and has no significant past medical history. Pyloric stenosis is suspected given the infant's projectile vomiting. Question What has congenital pyloric stenosis been most commonly associated with?

Correct answer: First-born male child Explanation Pyloric stenosis is the most common cause of gastric outlet obstruction in infants. Clinical features characteristic of pyloric stenosis include projectile vomiting, weight loss, and dehydration beginning around the third week of life. Diagnostic tests show a pylorus with hypertrophic pyloric musculature. Although the disease is easily treated with surgery, the exact cause of congenital pyloric stenosis is unknown. The etiology is thought to be multifactorial due to various environmental and genetic factors. There is a higher prevalence of pyloric stenosis in boys, with first-born boys having the highest risk. There is a generalized occurrence for siblings of 5-9% and a high concordance rate in monozygotic twins. It is most common in Caucasians and less prevalent in African Americans and Asians. A rare association with developmental delay has also been reported. Suspect environmental factors include infantile hypergastrinemia, myenteric plexus innervation abnormalities, cow's milk protein allergy, and exposure to macrolide antibiotics. Paint exposure has not been shown to have any effect on the development of hypertrophic pyloric stenosis (HPS). Therefore, both genetic and environmental factors are involved in the development of hypertrophic pyloric stenosis (HPS).

Case A 21-year-old Caucasian woman presents due to weakness, fatigue, palpitations, and postprandial abdominal pain with bloating, flatulence, and occasional diarrhea with voluminous stools. She received oral iron for iron-deficiency anemia 4 years ago but says she has been feeling weak again. Three months ago, pruritic papules and vesicles appeared on both elbow surfaces. Her family history is unremarkable. Vital signs are within normal ranges. On physical examination, there is noticeable skin pallor. Excoriated papules and blisters are seen on both elbows. The remainder of the examination is normal. A complete blood count shows the following: Ht 31% Hb 9.8 g/dL MCV 78 fl MCHC: 25 g/dL WBC 8.200 /mm3. w/ normal differential iron 54 µg/dL ferritin 5 µg/dL. Question What is displayed in the attached image from the endoscopic examination of her duodenum?

Correct answer: Flattened vili and a submucosal lymphocytic infiltrate Explanation The hallmark of celiac sprue is an immunologic reaction to gluten. Endoscopic examination of the duodenum shows discrete but characteristic mucosal granulation (see image in the question text). Histopathology reveals villus flattening and lymphocytic proliferation in the submucosa. These mucosal alterations also produce secondary lactase deficiency. Celiac disease is a common cause of steatorrhea. The classical presentation consists of diarrhea with bulky, foul-smelling stools, abdominal distention, and weight loss. However, most patients don't develop the full-blown syndrome. The presence of a subset of these symptoms should trigger suspicion of celiac disease. This patient's dermatologic findings are typical for dermatitis herpetiformis, an extraintestinal manifestation that is highly specific for celiac disease. It consists of a maculopapular, pruriginous eruption with a predilection for extensor surfaces of the elbows, knees, trunk, buttocks, scalp, and neck. Laboratory examinations may reveal anti-gliadin, anti-endomysial IgA or transglutaminase antibodies, or deficiencies in micronutrients such as iron, folate, B12, and vitamin K (prolongation of PT time). More rarely, celiac disease manifests itself as isolated micronutrient deficiencies, and diagnosis required a high index of suspicion. Primary intestinal lymphangiectasia manifests itself as non-bloody diarrhea and peripheral edema starting in the first decade of life. Secondary lymphangiectasia occurs when intra-abdominal diseases disrupt lymph flow. The patient's clinical presentation is incompatible with this diagnosis PAS-positive macrophages containing Gram-positive cocci are specific for Whipple's disease, a rare entity caused by proliferation of Tropheryma whippleii. This disease causes steatorrhea and prominent extra-intestinal symptoms such as arthritis, uveitis, endocarditis and, in severe cases, dementia. This uncommon diagnosis is unlikely in this patient. Giardia lamblia is the most common intestinal waterborne pathogen in the United States. Most infections are asymptomatic, and most patients who do develop symptoms have acute watery diarrhea. However, a small percentage develops chronic infections with recurrent diarrhea and steatorrhea, as well as weight loss, micronutrient deficiencies and, more rarely, reactive arthritis and urticaria. The relative rarity of chronic giardiasis makes this diagnosis much less likely than celiac disease in this case. Primary duodenal lymphoma is a rare entity. Most are of the mucosa-associated lymphoid tissue (MALT) type, and are related to H. pylori infection. This diagnosis would not explain the patent's findings. Bacterial overgrowth syndrome is secondary to stasis, fistulas, low gastric acidity, reduced, malabsorption, or mucosal damage. None of these features are present in this case.

Case A 42-year-old homeless man is admitted to the hospital with fatigue and chest pain. He states that he has been having fever, night sweats, nausea, vomiting, diarrhea, weight loss, and a burning sensation in the mouth. He also notes a sensation of skin burning, increased skin pigmentation, and emotional instability over the past 3-4 months. Physical examination reveals a bronze skin tone, several missing teeth, and a low-grade fever. Laboratory analysis revealed a normal CBC, WBC, and urinalysis. A PPD was performed and was found to be negative. Serum transferrin is normal. Question What statement concerning this patient's condition is true?

Correct answer: Foods rich in tryptophan may help to compensate for this deficiency. Explanation Foods rich in tryptophan may help compensate for niacin deficiency. Niacin (nicotinamide, nicotinic acid) deficiency is uncommon in the United States. It is often found in people who live on a diet that consists mainly of corn (maize). This is due to the fact that the niacin in corn cannot be absorbed unless it is chemically treated with alkali first. If a person consumes a diet rich in tryptophan but low in niacin, they are able to compensate since tryptophan can be converted into niacin. Deficiency may also result from alcoholism, cirrhosis, or diarrhea. Men and women are affected equally. Symptoms of niacin deficiency include nausea, vomiting, diarrhea, rash, glossitis, stomatitis, depression, and psychosis. The rash may be found on any part of the body, but it is commonly found on the arms and legs. Niacin deficiency, also known as pellagra, manifests as the "3 Ds": diarrhea, dermatitis, and dementia. If this diagnosis is suspected, supplemental niacin should be given immediately. Diagnosis can be made by from a good history and from the physical symptoms. Laboratory tests can confirm the diagnosis. Treatment consists of niacin supplementation. This should be started as soon as the deficiency is suspected. Some patients deficient in niacin may also be deficient in other vitamins (such as B vitamins), so they should be given as well.

Case A 42-year-old man presents with severe abdominal pain in the mid-epigastric region. The pain began 3 days ago with an intensity of 10/10, concomitant nausea, and vomiting (on two occasions). The patient denies any significant past medical history. BP is 86/58, and pulse is 112; temp is 101.5°F. There is rebound tenderness in the midepigastric area; bowel sounds are absent, and there is a positive Cullen's sign. The rest of the physical exam is within normal limits. Question What are the two most common causes of acute pancreatitis?

Correct answer: Gallstones and alcohol abuse Explanation More than 70% of cases of acute pancreatitis are due to alcohol abuse or gallstones. Less common causes include postoperative pancreatitis, abdominal trauma, hyperlipidemia, certain drugs, hypercalcemia, peptic ulcer disease, uremia, cystic fibrosis, and viral infections. Complications such as adult respiratory distress syndrome, hemorrhagic pancreatitis, pancreatic abscess, pancreatic pseudocyst, and pancreatic ascites account for the 10% mortality rate associated with acute pancreatitis. Hemorrhagic pancreatitis can lead to retroperitoneal hemorrhage and widespread tissue necrosis, and it may require surgical intervention or peritoneal lavage. This retroperitoneal bleeding can be responsible for the discoloration or hematoma in the periumbilical area (Cullen's sign).

Case Ico-delete Highlights A 65-year-old Caucasian man presents due to losing 15 lb over the past 6 months; he is also experiencing postprandial fullness, hematemesis, abdominal pain, and melena. He also has a 35-year history of daily alcohol intake and smoking. On examination, the patient is found to have pallor and several hyperpigmented velvety papillomatous plaques on the back of his neck and axilla, as shown in the image. Question What condition is suspected based on patient's signs and symptoms?

Correct answer: Gastric carcinoma Explanation Gastric carcinoma is the correct answer. The weight loss, abdominal pain, post-prandial fullness, hematemesis and melena are important signs and symptoms of gastric carcinoma. The sudden appearance of extensive skin lesions suggestive of seborrheic keratosis or acanthosis nigricans must include workup for intra-abdominal malignancy; in particular, gastric cancer should be considered. It is a para-neoplastic syndrome; it has been given the eponym "Leser-Trelat sign". Chronic liver disease is incorrect. Skin manifestations of cirrhosis include spider naevi and caput medusae. Inflammatory bowel disease is incorrect. The skin manifestations most commonly seen with the condition are erythema nodosum and pyoderma gangrenosum. Pancreatitis is incorrect. Severe acute pancreatitis is sometimes associated with ecchymoses of the abdominal wall; they are known as Grey-Turner's sign and Cullen's sign. Colon cancer is incorrect. It is rarely associated with dermatomyositis.

Case A 72-year-old woman presents because she "threw up fresh blood" 1 hour ago. She denies any heartburn or gastric symptoms. Her past medical history includes type 2 diabetes controlled with diet and glyburide and naproxen for severe chronic osteoarthritis. On initial examination, she appears anxious, but she is ambulatory and has no noticeable pallor. Family history is unremarkable. Vital signs are as follows: BP 140/80 mm Hg without postural hypotension, HR 104 bpm, and RR 18 bpm. The rest of the physical examination shows no abnormalities. Question The patient is admitted and undergoes upper endoscopy. What finding is most probable?

Correct answer: Gastritis Explanation Non-steroidal anti-inflammatory agents (NSAIDs) cause ulcer diathesis because they inhibit prostaglandin synthesis. Virtually all NSAID users develop some degree of gastritis, and hemorrhagic gastritis is a common cause of upper gastrointestinal bleeding in these patients. Among common over-the-counter medications, naproxen and aspirin carry a higher risk than ibuprofen. Patients with gastritis often experience upper abdominal pain, but many people with gastritis are asymptomatic. Erosions and ulceration, sometimes with multiple ulcers, are also common. Endoscopic studies have shown that 15-20% of chronic NSAID users develop ulcers, but most of these lesions are not associated with serious complications (bleeding, perforation, etc), and screening is not indicated. Chronic NSAID users have a 2-4% per year risk of upper gastrointestinal bleeding. It is estimated that NSAID-associated bleeding causes 2600 deaths annually in the United States. Its incidence is rising because of population aging, which has led to an increased number of people with chronic rheumatic conditions. The following factors are associated with a higher risk of bleeding:

Question A 44-year-old white male comes in with complaints of heartburn, substernal pain, regurgitation, and difficulty in swallowing. He relates that he likes to eat foods that have substance to them, such as hamburgers, steaks, fries, rich desserts, etc. He also tells you that his wife is a great cook and fixes all his favorite dishes with extra butter. He tells you he has had heartburn for years. To relieve the heartburn he has taken antacids. This time the pain is worse. He has just eaten a large fatty meal in the last hour. He denies other medical problems. He does not smoke and only occasionally uses alcohol. On physical exam you note he weighs 280 pounds. He is 5 feet 10 inches tall. While waiting for test results, what do you think is the most likely diagnosis?

Correct answer: Gastroesophageal Reflux Disease Explanation The most likely diagnosis is gastroesophageal reflux disease or GERD. The essential features of GERD are: Heartburn; may be exacerbated by meals, bending, or recumbency. Endoscopy demonstrating abnormalities such as erythema and friability of the squamocolumnar junction in <50% of patients The typical symptom is heartburn. This most often occurs 30-60 minutes after meals and upon reclining. Certain foods or beverages like coffee or alcohol, drugs like calcium channel blockers or beta blockers, and obesity are contributing factors. Patients often report relief from taking antacids or baking soda. When this symptom is dominant, the diagnosis is established with a high degree of reliability. Overall, a clinical diagnosis of GERD has a sensitivity of 80% but a specificity of only 70%. Severity is not correlated with the degree of tissue damage. Untreated, symptoms progress to regurgitation, dysphagia, and esophagitis. Esophageal neoplasms are either squamous cell carcinomas or adenocarcinomas. Adenocarcinomas arise most often in the lower third of the esophagus and occur almost exclusively in the context of Barrett's esophagus. Squamous cell esophageal carcinoma is more common in African Americans and patients who drink alcohol and smoke heavily. Additional risk factors for esophageal carcinoma are as follows: Smoking and tobacco use (chewing) Alcohol abuse Drug abuse (e.g., morphine, opium) Malnutrition Environmental carcinogens (e.g., nitrosamines, silica, fungi) History of cancer of the larynx or pharynx History of chronic inflammation of esophagus, achalasia Dysphagia is the most common presenting symptom. Regurgitation and weight loss may also occur. The etiology is not well defined but is associated with chronic esophageal irritation. Peptic ulcer disease usually presents with gnawing or burning pain that is relieved by eating. The pain may occur at night. Untreated peptic ulcers may cause bleeding. Esophageal Varices can be caused by portal hypertension (elevated portal pressure beyond 10 mm Hg) in association with cirrhosis, liver parenchymal disease, duodenal ulcer, or acute pancreatitis, Budd Chiari syndrome, schistosomiasis, etc. Bleeding esophageal varices eventually develop in about 50% of individuals with cirrhosis. Portal veins narrow and become obstructed as a result of the underlying disease process. As the lumen narrows, the venous blood returning to the right atrium from the intestine and spleen seeks new routes through collateral vessels. These collateral vessels enlarge and become tortuous, and the mucosa ulcerates. Complications can include esophageal rupture, with massive hemorrhage and death being the most common complication. With acute bleeding, the mortality rate is about 50%. Approximately 60% of individuals die within a year of the first episode of bleeding. Esophageal varices do not typically cause pain. Biliary colic usually presents as right upper quadrant pain that occurs shortly after eating and persists for several hours. It may be accompanied by vomiting. Obesity and a history of eating fatty meals is often present.

Case A 45-year-old man presents with a 30-minute history of substernal chest pain. He describes the pain as burning. He denies any trauma to the chest. He has had similar episodes like this many times. He denies any additional symptoms such as shortness of breath or diaphoresis, but he states that his voice is often hoarse. His medical issues include diabetes mellitus and heavy alcohol use. Question What is the most likely cause of his chest pain?

Correct answer: Gastroesophageal reflux Explanation Gastroesophageal reflux pain is a substernal pain that is usually described as burning. It may last for up to 1 hour. It is relieved by antacids, and it may be aggravated by alcohol and postprandial recumbency. Gastroesophageal reflux may cause hoarseness due to irritation of the vocal cords by acid. Pneumonia pain is usually described as a sharp pain that is aggravated by inspiration. Other signs and symptoms of pneumonia, such as cough, fever, dyspnea, and rales on auscultation, may also be present. Aortic dissection pain is of abrupt onset and is usually described as severe and tearing. It may radiate from the anterior chest wall posteriorly to the interscapular region. On examination, the blood pressure may be elevated. A pericardial rub and the diastolic murmur of aortic insufficiency radiating along the right sternal border may be auscultated. Pulmonary embolism pain is usually of abrupt onset and pleuritic in nature. It has a duration of several minutes to a few hours. There may be predisposing factors, such as deep venous thrombosis or long periods of immobilization. Patients may also report hemoptysis. On examination, the patients are dyspneic and tachypneic with a tachycardia.

Question A 36-year-old software consultant presents with complaints of a burning sensation in his chest for the last two weeks. He has had similar symptoms in the past, on and off for about 6 months. His wife has noticed episodes of regurgitation and coughing at night. There is no dysphagia, weight loss, hematemesis, or melena. He has no other past medical history. He is a nonsmoker and does not drink alcohol. Family history is significant only for HTN in father. On exam, his BP is 120/80 mm of Hg, pulse 76/min, SPO2 92%, height 6'4, and weight 242 pounds. Physical exam is unremarkable. What would be the likely diagnosis?

Correct answer: Gastroesophageal reflux disease Explanation Gastroesophageal reflux disease, or GERD, is the most common cause of noncardiac chest pain. The classic symptoms are burning in the chest, especially at night; regurgitation; and dysphagia. Choking and coughing during sleep may also be seen. Symptoms of dyspepsia like pain or discomfort centered on the upper abdomen, belching, abdominal fullness, and satiety are common. Obesity, as in this patient, is an important aggravating factor. Other aggravating factors include caffeine, alcohol, and drugs like beta blockers, calcium channel blockers, and progesterone. Endoscopy is not necessary for diagnosis. It is only indicated in patients with long standing GERD to rule out Barrett's esophagus and for assessment of dysphagia. A barium esophagogram is done in case of dysphagia. Esophageal manometry is done in case of resistance to treatment, frequent relapses, and atypical symptoms, such as cough or asthma. Treatment should be initiated in mild to moderate GERD with H2 receptor blockers, which are effective in healing 70-80% cases. In severe cases, treatment should be with a proton pump inhibitor like omeprazole or lansoprazole for complete relief from symptoms. Peptic ulcer disease has similar symptoms of dyspepsia but is associated with nausea, vomiting, hemetemesis, melena, and occasionally acute bleeding from an ulcer with hypotension requiring emergent treatment. Endoscopy is indicated for diagnosis as well as treatment. H2 blockers or proton pump inhibitors can be used as first line of treatment. Esophageal stricture most commonly occurs in patients with long standing reflux esophagitis, as well as a variety of other clinical disorders like scleroderma, malignancy, nasogastric intubation, surgery, etc. Patients present with insidious onset of dysphagia, initially to solids, then to liquids with dyspepsia-like symptoms and regurgitation. A barium swallow should be done to assess the location and length of the stricture, since it is the least invasive and is 90-100% sensitive in detecting the subtle narrowings. Endoscopy and dilatation are the usual modes of treatment. Acute gastritis is an inflammation of the gastric mucosa usually caused by drugs like NSAIDS, alcohol, caustic ingestion, and infections like Helicobacter pylori. Symptoms include epigastric burning, nausea, vomiting, and, rarely, acute episode of gastrointestinal bleeding with hypotension and shock. Other regimens for this infection are also available. Diagnosis is based on endoscopic findings, and therapy includes removal of offending agents, proton pump inhibitors, and surface acting agents like sucralfate. If H.pylori is found to be the causative factor, then triple therapy with amoxicillin, metronidazole, and clarithromycin is used. Other regimens for this infection are also available. Fluid resuscitation and blood replacement may be needed in cases of shock. Cholelithiasis usually presents with acute epigastric or right upper quadrant pain radiating to the back and occasionally may be associated with symptoms of dyspepsia. There is right upper quadrant tenderness known as Murphy's sign. Liver enzymes and bilirubin may be raised. Ultrasound of the gallbladder is diagnostic, and treatment is surgical removal of gall bladder with complete resolution of symptoms. Surgery is recommended only in cases of symptomatic stones.

Question A 60-year-old Caucasian man comes to your office to establish care. He has no known medical problems and his only medication is daily ibuprofen for 5 years for musculoskeletal aches associated with his work as a handyman. He is at most risk for which following due to chronic NSAID use?

Correct answer: Gastrointestinal bleed Explanation The patient described in this question is at most risk for a gastrointestinal bleed secondary to his chronic non-steroidal use. Patients in their 60s are at an approximate 3-fold increased risk for a gastrointestinal bleed when they take non-steroidal anti-inflammatory medications compared to not taking them. The non-steroidals inhibit the prostaglandins responsible for homeostasis, and therefore can lead to gastrointestinal mucosal injury. Chronic analgesic consumption is associated with chronic kidney disease. Analgesic nephropathy is still a relatively uncommon cause of chronic kidney disease; its prevalence is higher in Europe and Australia than in the United States, where it accounts for only 2-4% of the cases of end stage kidney disease. End-stage kidney disease is the most advanced stage of kidney disease, for which patients typically require dialysis. Some studies have debated causality, however, a recent New England Journal of Medicine article addressed the risk in patients with chronic consumption, and the article found a 2.5-5 fold increase risk of chronic renal failure in patients who chronically used acetaminophen vs. those who did not. Cumulative lifetime use increased the odds of renal failure, and results were similar for patients who took aspirin. The highest risk was noted among patients who took >500 g/yr acetaminophen (>1.4g/day). Other studies have found increased risk among users of >3-5 gram per lifetime and >6 tablets/day for 3 years.

The delivery room calls to report a newborn has been delivered with a protruding upper midline abdominal mass. You find that it is a dark red, matted, stiff mass of abdominal viscera without a covering lining. Question What is the most likely diagnosis?

Correct answer: Gastroschisis Explanation This is gastroschisis, a serious congenital abnormality, and the outcome is determined by the condition of the protruding viscera. Both gastroschisis and omphalocele have a protrusion of abdominal viscera from a ventral body wall defect. Gastroschisis occurs above the umbilicus and is not protected by membranous sacs, so the bowel is usually more severely affected. Omphalocele occurs at the level of the umbilicus and is covered with peritoneal lining and layers of the umbilical cord, so the bowel is usually in better condition. Bladder exstrophy is an eversion of the mucosa of the urinary bladder and is usually a lower midline defect that occurs just above the pubis. Prune belly is due to congenital lack of the rectus abdominus muscles and is covered by skin. Eventration refers to a chronic condition of the diaphragm not used in the context of this newborn's congenital anomaly.

Case A 65-year-old woman presents with a 3-week history of stomach pain. Her family history is remarkable for her father's death due to prostate cancer and the deaths of her mother and sister due to breast cancer. Her 2 brothers experienced myocardial infarctions; both of them have had coronary bypass operations. The patient suffers from hypertension, hyperlipidemia, and coronary heart disease. She underwent angioplasty and stent placement 1 year prior to presentation. She denies having any GI problems in the past. Her daily medications include aspirin (81 mg daily), simvastatin (3 x 5 mg daily), metoprolol (3 x 50 mg daily), vitamin E (400 I.U. daily), and a multivitamin supplement. In addition, for the past 12 weeks, she has been taking diclofenac (3 x 50 mg daily) for osteoarthritis of the right knee. She admits to sometimes taking up to 5 x 50 mg of diclofenac for management of intense pain. She states that she always used to be constipated; lately, her stool is occassionally black. Her temperature is 36.8° C, blood pressure is 135/90 mm Hg, pulse rate is 105/min, and respiratory rate is 18/min. Physical examination reveals an obese, pale woman in no apparent distress. ECG shows slight ST depression and atrial fibrillation. There is pain upon pressure in the epigastric area. The rest of the examination is unremarkable. Lab analysis is as follows: hemoglobin 10.8 g/dL (normal 14 to 18 g/dL); hematocrit 35.2% (normal 42% - 52%); mean corpuscular volume 92 μm3 (normal 80 - 94 μm3), white blood cell count 26,900/mm; and platelet count 233,000/mm3. Question What is the most appropriate next step?

Correct answer: Gastroscopy Explanation Considering the history, physical examination and lab results, the patient most likely suffers from a NSAID-induced GI bleed; therefore, gastroscopy is the next logical step. Minor bleeding can be treated during the procedure by an injection of epinephrine (1:10,000) or by using a bipolar probe. Since the patient has melena, the bleeding is most likely occurring in the upper GI tract. If no bleeding source in the upper GI tract is found, a colonoscopy or barium X-ray should be done. The examination choice depends on availability, the expertise of the medical staff, and the patient's wishes. Since there has not been a test done for Helicobacter pylori infection, treatment is not justified. Helicobacter pylorus specifically and selectively colonizes human gastric mucin-secreting cells; it is the most common cause of non-erosive gastritis. It can be diagnosed by a urease breath test or serum IgG test. Eradication is achieved by treatment with clarithromycin (500 mg/day) and omeprazole (2 x20 mg/day for the 1st 2 weeks, and 1 x 20 mg/day for the next 2 weeks). The next step would be rectosigmoidoscopy; however, it is very unlikely that melena originates in the end part of the bowel.

Case An 18-year-old woman presents with symptoms of mild jaundice, fatigue, nausea, and abdominal pain. Other than that, the history is unremarkable. Physical exam reveals normal vital signs and no other symptoms beside mild scleral icterus. Labs reveal elevated unconjugated bilirubin and normal conjugated bilirubin levels. Question What is the most likely diagnosis?

Correct answer: Gilbert's syndrome Explanation Gilbert's syndrome is a multifactorial inherited disorder that affects the way bilirubin is processed by the liver and causes jaundice. The symptom of jaundice appears under conditions of exertion, stress, fasting, and infections. The condition is usually benign. Symptoms include mild jaundice, fatigue, and nausea. The appearance of jaundice is often related to dehydration and stress. A serial serum indirect bilirubin shows changes consistent with Gilbert's disease. Usually no treatment is necessary, but phenobarbital can reduce the bilirubin level and relieve the jaundice. Jaundice may fluctuate and persist throughout the patient's life. It usually causes no health problems. Crigler-Najjar syndrome is inherited as an autosomal recessive trait. Infants who inherit the trait from both parents (this is called homozygous inheritance) develop severe jaundice (hyperbilirubinemia) a few days after birth. If these infants are not treated, they may develop kernicterus (a condition in the newborn marked by severe neural symptoms, associated with high levels of bilirubin in the blood). The jaundice will persist into adult life and require daily treatment. The constantly elevated levels of bilirubin eventually will produce an adult form of kernicterus despite daily treatment. Symptoms include yellow skin and eyes (jaundice) that begins on the 2nd or 3rd day of life and progressively worsening jaundice that persists beyond 2 weeks without an obvious cause, as well as confusion (resulting from brain damage). Liver transplantation may be considered for some people with this disorder. Breast milk jaundice is a persistent jaundice in the newborn caused by a hormone found in breast milk. Bilirubin is eliminated by chemically hooking a sugar molecule to the bilirubin molecule, which then makes the bilirubin water-soluble. In its water-soluble form, bilirubin can be excreted in the urine. Breast milk contains a hormone, pregnanediol, which interferes with the body's ability to hook the sugar onto the bilirubin. Symptoms include jaundice in a breast-fed newborn that lasts longer than a week. Cessation of nursing for 24 to 48 hours will result in a rapid drop of bilirubin. Newborn jaundice is a condition of yellowish skin color during the newborn period. This results from the immaturity of liver function combined with the destruction of redblood cells present in the newborn infant. The jaundice usually appears between the 2nd and 5th days of life and clears by 2 weeks. Symptoms include yellow color of the skin (jaundice), poor feeding (may be present), and lethargy (may be present). An elevated serum bilirubin level will be present. Usually treatment is unnecessary. Sometimes artificial lights (called bili lights) are used on infants whose levels are very high, or on premature infants. The jaundice resolves without treatment within 1 to 2 weeks. Dubin-Johnson syndrome is an inherited disorder characterized by mild jaundice throughout life. The transport of bilirubin from the liver into the biliary system is abnormal, and the bilirubin accumulates in the liver. An elevation of conjugated bilirubin is observed. Affected people have lifelong low-grade jaundice that may be aggravated by alcohol, pregnancy, infection, and other environmental factors. The symptoms include a mild jaundice, which may not appear until puberty or adulthood. The liver, if biopsied, is characteristically black.

Case A 2-year-old boy has been admitted to the hospital due to lethargy. His weight and height are below the fifth percentile. Vital signs are within normal limits. Abdominal examination reveals hepatomegaly; the liver is palpable 4 cm below the costal margin. There is no splenomegaly. Other systems are normal. Lab reports indicate hypoglycemia and hyperlipidemia. Question Based on these findings, what is the most likely diagnosis?

Correct answer: Glycogen storage disease type VI (Hers disease) Explanation Glycogen storage disease type VI is also known as Hers disease. The diagnostic feature of this disorder is a deficiency of the hepatic phosphorylase enzyme. This enzyme is required for glycogenolysis (glycogen breakdown) in the first step of conversion of glycogen to glucose-1-phosphate. An enlarged liver is indicative of glycogen buildup. Children usually present with growth retardation. Symptoms of hypoglycemia are possible when the liver is unable to maintain normoglycemia during starvation. Hyperlipidemia is also a characteristic finding. Glycogen storage disease type V, also known as McArdle's disease, is caused by a deficiency of myophosphorylase in muscle cells. Patients typically present with muscle weakness and fatigue, especially with exertion. The liver is not involved. The muscular dystrophies are hereditary degenerative disorders of skeletal muscle. They are mainly caused by structural defects in the muscle cell membrane. Duchenne muscular dystrophy is the most common type of muscular dystrophy. Patients usually present by 3 years old with a profound weakness of the proximal muscles followed by the distal muscles. Becker muscular dystrophy is genetically similar to Duchenne muscular dystrophy, but the symptoms are less severe. Both are X-linked disorders. Typical labs include elevated CK levels, indicative of muscle breakdown. Myotonic dystrophy is a disorder of muscles. Features include weakness and muscle wasting. An early symptom may be slow relaxation of hand grip after a forced voluntary closure. CK levels may be normal or elevated. Myotonic dystrophy does not affect the liver.

Case A 42-year-old man presents with acute abdominal pain. He describes the pain as constant and severe. Upon further questioning, he affirms that the pain radiates to his back. He leans forward in an attempt to alleviate the pain. On physical examination, his temperature is 101°F, and his pulse is 94. His abdomen is tender, but there is no guarding or rigidity. There is discoloration in the flanks and discoloration around his umbilicus. His nose is bulbous and red, and he appears malnourished. He states that he does not use illicit drugs. His lab results are as follows: TEST RESULTS REFERENCE RANGE Potassium 4.2 mEq/L 3.5-5.0 mEq/L Calcium 8.1 mg/100 mL 8.7-10.5 mg/100 mL Amylase 378 U/L 25-125 U/L Lipase 630 U/L 30-210 U/L Question The discoloration of the flanks in this patient is indicative of what sign?

Correct answer: Grey Turner's sign Explanation Grey Turner's sign is ecchymosis of the flanks that can sometimes be seen with acute pancreatitis. This patient has signs and symptoms of acute pancreatitis, which include non-colicky abdominal pain that radiates to the back and fever. An elevated amylase, an elevated serum lipase, and hypocalcemia are all also consistent with pancreatitis. The most frequent underlying cause of pancreatitis is alcoholism or biliary disease. Cole's sign is a deformity of the duodenal contour seen in radiographs and is a sign of the presence of duodenal ulcer. Lisker's sign is tibial bone tenderness that can sometimes be elicited with deep vein thrombosis. Kehr's sign is pain in the left shoulder secondary to splenic rupture. Blumberg's sign is abdominal rebound tenderness. This is an indication of peritoneal irritation.

Case A 72-year-old man has a history of medically-treated COPD for 20 years, chronic constipation, benign prostatic hypertrophy, and obesity; he presents to his primary care office with a "bulge" in his abdomen, which became noticeable 1 week ago following a coughing episode. He states that it enlarges when he coughs, sneezes, laughs, stands, and bends forwards; it lessens in size when he lays supine. He admits to a pressure sensation, but he denies any pain, nausea, vomiting, diarrhea, melena, hematochezia, a change in bowel or dietary habits, hematuria, flank pain, or dysuria. His physical exam demonstrates an obese body habitus, a barrel chest, and the following physical exam finding. Question What is a correct health maintenance strategy regarding this patient?

Correct answer: He should avoid straining at defecation and lifting heavy objects Explanation This patient's most likely diagnosis is an umbilical hernia. Patients should be counseled to avoid those activities that increase intra-abdominal pressure (e.g., straining at defecation and lifting heavy objects). This may require restrictions on work or school-related activities. Weight loss should be encouraged; obesity and an overweight state contribute to hernia development. An umbilical hernia in an adult should be repaired expeditiously to avoid incarceration and strangulation. Repairs utilizing mesh result in the lowest recurrence rate. Even with asymptomatic hernias, repair at an early stage (before the hernia enlarges) is preferred. Referral to a general surgeon for discussion of the available types of hernia repair is therefore warranted. This patient's pathology is not consistent of an underlying malignancy; therefore, chemotherapy or radiotherapy is indicated in this patient's management plan. If a patient refuses operative repair or when there are absolute contraindications to operation, a truss should be fitted to provide adequate external compression over the defect in the abdominal wall. It should be taken off at night and put on in the morning before the patient arises. The use of a truss does not preclude later repair of a hernia, although it may cause fibrosis of the anatomic structures; therefore, subsequent repair may be more difficult. Trusses place pressure on the skin and bowel, induce related injury, and mask signs of incarceration and strangulation. Strapping, with or without a coin, is not indicated in the treatment of umbilical hernia, due to of problems with skin erosion and lack of effectiveness.

Case A 60-year-old hypertensive man presents with constipation; he states that he has not had a single bowel movement in the past 2 days. 1 month ago, he had a myocardial infarction, but he is now stable and is on a low-fat, low-salt diet. He refuses a docusate sodium enema and is prescribed docusate tablets. Question What precaution should the patient take while using this laxative?

Correct answer: He should not use it long-term Explanation Docusate should not be used long-term; generally it should not be used for more than 1 week. Docusate is a stool softener which acts in the small and large intestines by increasing the water and fat content in the stool; this facilitates its easier movement. It is ideally suited for short-term treatment of constipation because long-term use may increase the risk of a deficiency of fat-soluble vitamins due to decreased absorption. Other contraindications include appendicitis, acute surgical abdomen, intestinal obstruction, fecal impaction, and undiagnosed abdominal pain. Mineral oil should not be prescribed along with docusate as the former may be absorbed, leading to toxicity. Docusate tablets are taken with a glass of water. A high-salt diet is not required for docusate to be effective. Dehydration is a side effect of sodium picosulfate, which is a contact laxative used to treat constipation or to prepare the bowel prior to surgery.

Case A 50-year-old man presents with weakness and abdominal discomfort. Upon questioning, he acknowledges a lack of sexual desire. He denies any photosensitivity. On physical examination, his liver is enlarged. He has an abnormal skin pigmentation that is located on his face, neck, and elbows; it gives his skin a slate-gray hue. His laboratory results are in the chart. TEST RESULTS REFERENCE RANGE TIBC 275 μg/dL 250 - 375 μg/dL Plasma iron 220 μg/dL 50 - 150 μg/dL Transferrin saturation 90% 20 - 40% Question What is the most likely diagnosis?

Correct answer: Hemochromatosis Explanation This patient has signs and symptoms consistent with hemochromatosis. Hemochromatosis is due to an increase in iron within the tissues. The liver is commonly affected, with hemochromatosis; hepatomegaly can be seen. There is an abnormal skin pigmentation that can be seen with hemochromatosis. Plasma iron will be elevated, total iron binding capacity will be normal or low, and transferrin saturation will be elevated. Lead poisoning can cause abdominal pain; however, the laboratory results of this patient are not consistent with lead poisoning. Wilson's disease is due to an accumulation of copper. The liver is also affected with Wilson's disease. The laboratory tests in this scenario do not support a diagnosis of Wilson's disease. Ceruloplasmin is the blood test that should be done in evaluation of a patient for Wilson's disease. Protoporphyria is due to a defect in heme synthesis and metabolism. Photosensitivity is seen with protoporphyria. The lab results are not consistent with iron deficiency anemia. Serum TIBC would likely be increased in iron deficiency anemia, not normal (as in the case here). With iron deficiency anemia there also would be a decrease in serum iron level, not an increase (as is seen in this case).

Question A 42-year-old man presents with severe abdominal pain in the mid-epigastric region. The pain began 3 days ago with an intensity of 10/10; there is concomitant nausea and vomiting (on 2 occasions). The patient denies any significant past medical history, but he states that he "only" drinks 10 beers a day, and he has been doing so for the past 15 years. BP is 86/58. Pulse is 112, and temp is 101.5F. There is rebound tenderness in the midepigastric area, absent bowel sounds, and a positive Cullen's sign. The rest of the physical exam is within normal limits. In cases of acute pancreatitis, a positive Cullen's sign suggests what complication?

Correct answer: Hemorrhagic pancreatitis Explanation The correct response is hemorrhagic pancreatitis. More than 70% of cases of acute pancreatitis are due to alcohol abuse or gallstones. Complications, such as adult respiratory distress syndrome, hemorrhagic pancreatitis, pancreatic abscess, pancreatic pseudocyst, and pancreatic ascites, among others, account for the 10% mortality rate associated with acute pancreatitis. Hemorrhagic pancreatitis can lead to retroperitoneal hemorrhage and widespread tissue necrosis, and it may require surgical intervention or peritoneal lavage. This retroperitoneal bleeding can be responsible for the discoloration or hematoma in the periumbilical area (Cullen's sign).

Question A 68-year-old man presents for a routine physical exam. A soft venous hum over the epigastric and umbilical area is noted on auscultation. The venous hum is due to increased collateral circulation resulting from what type of condition?

Correct answer: Hepatic cirrhosis Explanation A venous hum is due to an increased collateral circulation between the systemic and portal venous system resulting from hepatic cirrhosis. Chlamydial and gonococcal perihepatitis, liver tumor, and a recent liver biopsy cause grating sounds called "friction rubs" along the hepatic and splenic borders of the abdomen.

Case A 28-year-old woman presents with a 5-day history of progressive jaundice, vomiting, nausea, and malaise. She has developed a fever and has a 2-day history of dark urine. She has no history of intravenous drug use and has had no recent transfusions; she denies any sexual contact in the last 3 months. The patient has recently returned from a trip to Mexico where she consumed various foods from street vendors. Physical examination reveals a jaundiced patient. An abdominal examination is significant for a palpable liver edge 2 cm below the right costal margin and a total liver span of 12 cm below the mid-clavicular line. Blood is drawn for cultures, a complete blood cell count, and liver function tests. Blood studies are significant for a direct bilirubin of 13.0 mg/dL, aspartate aminotransferase of 1,800 U/liter, and alanine aminotransferase of 2,500 U/liter. Blood cultures are negative and serological studies are conclusively diagnostic. Question What is the most probable cause of the patient's symptoms?

Correct answer: Hepatitis A Explanation Hepatitis A is a member of the Picornaviridae family; it is a nonenveloped virus approximately 27 - 28 nm in diameter. It is spherical, with a surface structure that suggests icosahedral symmetry. It is a single-stranded RNA virus. Signs and symptoms of an infection include diarrhea, dark urine, jaundice, and flu-like symptoms. Incubation is 15 - 50 days, with an illness duration of 2 weeks - 3 months. The virus is associated with shellfish harvested from contaminated waters, raw produce, uncooked foods, and cooked foods that are not reheated after contact with an infected food handler. Confirmation is based on the presence of anti-hepatitis A antibodies, positive IgM, and increases in serum ALT and bilirubin. Treatment is supportive, and prevention is by vaccination. Norwalk virus is a single-stranded RNA virus that is approximately 29 - 35 nm in diameter with spiky ill-defined outlines. The virus produces gastroenteritis-like symptoms that include nausea, vomiting, and watery/large volume diarrhea. Fever is usually absent. The incubation period is 24 - 48 hours, and the illness duration is 24 - 60 hours. Sources of infection include poorly cooked shellfish and ready-to-eat foods (e.g., salads and sandwiches) touched by infected food handlers. Diagnosis is based on negative stool cultures, stool that is negative for white blood cells, and serological studies for the presence of fourfold or greater increases in antibody titers of Norwalk virus antibodies. Treatment is mainly supportive; bismuth sulfate may be given. Rotavirus is a double-stranded RNA virus belonging to the Reoviridae family. The virus is 70 nm in diameter and appears as a wheel. The virus infection has signs and symptoms that include vomiting, watery diarrhea, and low-grade fever. There may be temporary lactose intolerance. The virus is especially common in infants and children, the elderly, and the immunocompromised. The incubation period is 1 - 3 days, and the duration of illness is 4 - 8 days. The virus is associated with ready-to-eat foods touched by infected food handlers (e.g., salads and fruits) and fecally-contaminated foods. Confirmation is based on the detection of the virus antigen in stool. Routine stool cultures are negative, and stools are negative for the presence of white blood cells. Treatment is supportive in nature. Fluid electrolyte replacement may be needed in severe cases of diarrhea, especially in infants. Adenovirus is a virus with DNA as the genetic material; it is approximately 70 nm in diameter. The virus causes gastroenteritis; symptoms include vomiting, nausea, malaise, diarrhea, headache, and fever. Incubation is 10 - 70 hours, with a duration of 2 - 9 days for symptoms. The organism is associated with ingesting food contaminated with feces and ready-to-eat foods touched by infected food workers. The virus is also found to be transmitted through the ingestion of some shellfish. The virus can be detected antigenically in acute stool samples. Stool staining for white blood cells is negative. Supportive care is the treatment of choice. The disease is usually mild and self-limiting. Parvovirus is a single-stranded DNA virus. Infections with this virus are mostly asymptomatic, although the virus produces erythema infectiosum (fifth disease) in small children (a benign rash appears on the face, trunk, and extremities). A fever also develops. The disease can cause intrauterine infection and fetal death due to the vertical transmittance of the virus from mother to fetus. In individuals with hemoglobinopathies, there can be transient aplastic crisis. Immunodeficient patients can develop chronic anemia.

Case A 26-year-old Caucasian man went on a tour to Asia. He traveled to India, Sri Lanka, and Hong Kong over a period of 10 days and then returned to the USA. About 1 week later, he started feeling tired and complained of malaise. Two days later he develops a low grade fever associated with nausea, vomiting, and right upper quadrant pain. His roommate thinks his eyes look yellow and insists on taking him to the ER. The patient has no significant past medical history and is on no medications. He is a non-smoker and drinks alcohol only occasionally. On examination, he has a temperature of 100.4 F; pulse is 84/minute, and BP 120/80 mm Hg. His sclera is icteric, and he has no lymphadenopathy. Lungs are clear, and heart sounds normal. Abdominal exam reveals an enlarged liver 3 cm below the subcostal margin, with mild tenderness in the right upper quadrant. There is no evidence of ascites, caput medusae, or splenomegaly. Neurological exam shows a fully oriented patient without any deficits. Labs reveal a Hb of 14g/dL, WBC of 13,000/uL, platelets 350,000/uL, ALT 3200 U/L, AST 2650 U/L, alkaline phosphatase 675 U/L, total bilirubin 10mg/dL, and direct bilirubin 7mg/dL. Blood cultures are drawn and hepatitis profile sent. Ultrasound of the liver is pending. Question Which of the following would you consider the most likely diagnosis?

Correct answer: Hepatitis A Explanation Hepatitis A is a common cause of acute hepatitis all over the world caused by Hepatitis A virus (HAV), an RNA virus. It is more prevalent in the developing countries like India, due to overcrowded areas with poor hygiene and sanitation and in the lower socio-economic strata. It is spread by the fecal-oral route and has an incubation period of 2-6 weeks. The prodrome is characterized by malaise, fatigue, fever, nausea and vomiting, and right upper quadrant pain, as in this patient. Labs show liver enzymes to be higher than 1000 mg/dL and direct hyperbilirubinemia. ALT is higher than AST. Leukocytosis may be seen. Detection of serum IgM anti-HAV is diagnostic and remains positive for 3-6 months. IgG anti-HAV appears after 1 month of disease and may persist for years. Presence of IgG alone indicates previous exposure to HAV, immunity to recurring HAV infection, and noninfectivity. The disease is self limiting, and treatment is supportive. Almost all patients recover fully, and fulminant hepatitis is rare. There are of various types of hepatitis A vaccines including inactivated vaccines, live attenuated vaccines, and combined Hepatitis A and B vaccines. The 2 approved for use in USA include HAVRIX and VAQTA, both of which have to be given in a single dose followed by a booster dose within the next 6-12 months. The indications for vaccination include travel to highly endemic areas like Africa, Asia, Middle East, Mexico etc; children residing in areas where the incidence is twice the national average like Arizona, Alaska, Oregon, Utah, New Mexico, Washington etc; people with high risk sexual activities; intravenous drug abusers in areas with history of Hepatitis A breakouts; healthcare workers with high risk of exposure; people with chronic liver disease, especially with Hepatitis C infection, in whom Hepatitis A infection can be disastrous; and susceptible people receiving clotting factor concentrates. Hepatitis B is caused by the Hepatitis B virus (HBV), a DNA virus. It usually causes subclinical, subacute, or chronic hepatitis. It is spread by sexual contact, vertical transmission from mother to fetus (especially in some Asian countries like China and Korea), and blood or blood products. It is common in IV drug users and healthcare personnel. Only 30% patients have acute icteric hepatitis. Incubation period is 2-6 months. Fulminant hepatitis is rare. Chronic hepatitis can occur leading to cirrhosis of liver and hepatocellular carcinoma. The prevalence of this is 90% in perinatally infected children, 20-50% in children 1-5 years old, and less than 5% in adult-acquired infection. Extra hepatic manifestations like arthritis, rash, and fever may occur. Carrier state may occur and have a good prognosis. Patients with chronic HBV infection are treated with interferon with or without steroids. Lamivudine and other antiviral drugs may be used. Hepatitis C virus is an RNA virus previously known as non A-non B virus. It is transmitted by blood transfusions, IV drug abuse, and needlestick exposure. Transmission by sexual or maternal-neonatal routes is uncommon. It is usually a silent infection associated with a chronic asymptomatic elevation of aminotransferases with fluctuations. Diagnosis is by positive anti-HCV antibody. Cirrhosis of liver, liver failure, and hepatocellular carcinoma may occur. Treatment includes interferon therapy and liver transplant. Hepatitis D virus (HDV), or the delta agent, is a small RNA particle that requires the presence of hepatitis B surface antigen to cause infection. It may occur as a co-infection with HBV or as a superinfection in an already HBV infected person. It is strongly associated with IV drug abuse and may cause benign acute hepatitis, fulminant hepatitis, asymptomatic carrier state, and rapidly progressive chronic liver disease. Diagnosis is by detection of anti-HDV antibody. Treatment includes interferon alpha, ribavarine, lamivudine, and liver transplant. Alcoholic hepatitis occurs in chronic alcoholics, which this patient is not. It is associated with insidious onset of anorexia, nausea, vomiting and weight loss with ascites, jaundice, hepatomegaly, splenomegaly, and encephalopathy in later stages. It can progress to cirrhosis of liver. Liver enzymes are usually less than 400 U/L with AST higher than ALT. Gamma glutamine transferase (GGT) is typically high. Treatment includes abstinence from alcohol, nutritional support, steroids, and liver transplant.

Question A 55-year-old male from Southeast Asia is visiting his son in the US for several months. While here, he loses weight. He starts to develop abdominal pain, anorexia, and ascites. His son takes him to the family doctor. On physical examination, ascites is present and his liver is enlarged. Interestingly, he is not jaundiced. Additional laboratory results are as follows: TEST RESULTS REFERENCE RANGE SGOT 134 IU/L 5-40 IU/L SGPT 121 IU/L 5-35 IU/L bilirubin (total) 2.8 mg/dL 0.2-1.5 mg/dL alkaline phosphatase 113 U/L 20-70 U/L A liver biopsy is done, and he is found to have hepatocellular carcinoma. What virus is associated with the development of hepatocellular carcinoma?

Correct answer: Hepatitis B Explanation Consistent with hepatocellular carcinoma are the weight loss, abdominal pain, and anorexia that this patient experienced. The physical findings of ascites and hepatomegaly are consistent as well. The lab results, reflecting elevated transaminases, elevated bilirubin, and elevated alkaline phosphatase, are consistent as well. Hepatitis B virus is a DNA virus. It is also referred to as the Dane particle. There is an association of hepatitis B infection and the development of hepatocellular carcinoma. Epstein Barr virus is a herpesvirus. Epstein Barr virus is the cause of infectious mononucleosis. Epstein Barr virus is associated with the development of the African form of Burkitt's lymphoma. Epstein Barr virus is also associated with nasopharyngeal carcinoma. Human papilloma virus is a papovavirus. Human papilloma virus is responsible for warts and is associated with the development of cervical cancer. Varicella-zoster virus is a herpesvirus. Varicella zoster virus is responsible for chicken pox and shingles, among other things. Varicella-zoster virus is not associated with the development of cancer in man. HTLV is a retrovirus. HTLV-I is associated with T-cell leukemia/lymphoma.

Case A 27-year-old man is seen for evaluation of a small mass in his right groin. His history reveals that this mass has been present for about 2 years, but he has suddenly been motivated to seek medical evaluation by his wife who has fears of cancer. He believes he 1st became aware of the mass when getting dressed, and notes occasional aching in the right groin. He denies trauma to the region. His past medical history is significant for chronic low back pain, secondary to a motor vehicle accident (MVA) 9 years ago. He takes over-the-counter ibuprofen as needed, and has no known drug allergies. He has had a tonsillectomy, appendectomy, and L4-L5 fusion, post-MVA. There is a family history of lung cancer in his paternal grandfather and diabetes in his maternal grandparents. The patient smokes, but denies alcohol and illicit drugs. He is married with 3 children, but admits to >10 sex partners previous to marriage. He works as a pipe fitter, which involves heavy lifting and frequent bending. His physical exam is as follows: BP 115/80, pulse 72, temp 98.0 degrees, weight 190 pounds, and height 69". Inspection of the patient in the supine position reveals no visible mass in the right inguinal region, corresponding with his description. There is no erythema or swelling in either inguinal region or abdomen. Abdominal exam reveals no tenderness, masses, hepatosplenomegaly, acites, or bruits. Inspection of the scrotum and penis shows no lesions, erythema, or tenderness. The patient is asked to stand for a testicular exam, which reveals normal-sized testes without nodules or masses. With a finger placed in the inguinal ring an impulse is felt on the right, but not on the left, when the patient is asked to cough. Although many conditions comprise the differential for a mass in the groin, you suspect he has an inguinal hernia. Question What aspect of this patient's history or physical is most specific to the diagnosis of hernia?

Correct answer: Impulse felt with cough Explanation The impulse the examiner felt with the cough most strongly supports the diagnosis of hernia; this is a classic finding associated with inguinal hernias. Other conditions in the differential would be much more likely to present as an inguinal mass that does not come and go with straining, coughing, or Valsava maneuvers. The individual's history and exam are most consistent with the indirect type of inguinal hernia, but distinguishing direct versus indirect does not significantly change management. While absence of fever may help to exclude acute infectious etiology (e.g., epididymitis, orchitis, and cat-scratch disease), the differential of a groin mass in an afebrile patient still includes malignancy, hydrocele, varicocele, as well as hernia. The chronic nature of this patient's mass helps the examiner focus away from the acute scrotal emergencies, such as testicular torsion and infectious conditions. Patients with hernias may exhibit either chronic or acute presentations. Other items on the chronic groin mass differential include malignancy, undescended testes, and dermatologic conditions; the acute versus chronic course of the complaint is not helpful in fully narrowing the differential. While long-term heavy work seems to have an association with groin hernia, this patient's history of heavy lifting is not specific enough upon which to base the diagnosis of hernia. The patient's history of multiple sex partners would be associated with a risk for epididymitis or orchitis. This aspect of the patient's history does not support the suspected diagnosis of inguinal hernia. Unilateral groin mass suggests a very broad differential (e.g., lymphadenopathy, scrotal mass, vascular mass, and infectious processes). This aspect of the patient's history does not narrow the differential to hernia alone.

Case A 50-year-old man presents with a 3-month history of weakness, fatigue, and abdominal discomfort. Upon further questioning, he acknowledges a lack of sexual desire. He denies any photosensitivity. On physical examination, his liver is enlarged, and his spleen is palpable. He has abnormal skin pigmentation on his face, neck and his elbows and which gives his skin a metallic gray hue. His laboratory results are in the chart. TEST RESULTS REFERENCE RANGE TIBC 275 μg/dL 250 - 350 μg/dL Plasma iron 220 μg/dL 80 - 160 μ/dL Transferring saturation 90% 16 - 57% Question What serious complication is associated with the patient's condition?

Correct answer: Hepatocellular carcinoma Explanation This patient has signs and symptoms consistent with hemochromatosis, which is associated with hepatocellular carcinoma. Hemochromatosis is due to an increase in iron within the tissues. Hemochromatosis is a disorder of iron overload; it could be due to genetic or non-genetic causes. In hereditary hemochromatosis, there is absorption of a few milligrams of iron each day, in excess of need. As such, clinical manifestations often occur only after the age of 40 years, when body iron stores have reached 15 to 40 g. The symptoms of hemochromatosis are nonspecific and include arthralgia; abdominal pain; fatigue, weakness; impotence; weight loss; amenorrhea and early menopause; abnormal skin pigmentation; damage to the pancreas leading to diabetes; cardiomyopathy; and cirrhosis. The liver is commonly affected with hemochromatosis, and hepatosplenomegaly is commonly seen. There is an abnormal skin pigmentation that is seen with hemochromatosis. In hemochromatosis, the plasma iron will be elevated; total iron binding capacity will be normal or low; and transferrin saturation will be elevated. Hepatocellular carcinoma is the most serious complication, and it is a major cause of death in patients with hemochromatosis. A large percentage of patients with hemochromatosis will develop problems with their pancreas; however, the pancreatic pathology commonly seen with hemochromatosis is insulin dependent diabetes, not pancreatic cancer. Bronchogenic carcinoma, pancreatic carcinoma, lymphoma, and leukemia are not known complications of hemochromatosis.

Case Ico-delete Highlights A 20-year-old woman presents with anorectal pain; there are streaks of blood on her stool and toilet paper. She states that "she has a tearing pain during each bowel movement". She dreads having a bowel movement, and she attempts to hold it as long as she can. The symptoms have been occurring over the past 2 weeks. Her history is also significant for breaking her leg in a skiing accident 4 weeks ago; she was prescribed Percocet for the first few days due to her pain. An anoscope is used; you find an acute anal fissure. Question What should be the next step in regard to treatment?

Correct answer: High fiber content/stool softeners Explanation A high fiber diet/stool softeners should be tried first for about 6 weeks; doing so typically heals acute anal fissures. Percocet use is known to cause constipation and constipation is a cause for anal fissures. Unless she continues to have significant pain from her leg fracture, her Percocet should be discontinued to avoid further constipation. Topical nitroglycerin should be considered if the high fiber diet fails to heal the fissure. Botulinum toxin can be used in conservative treatment, but it is limited in regard to availability and expense. When all conservative therapies have been exhausted, lateral internal sphincterotomy should be used as a last resort.

Case A 40-year-old man is known to have glycogen storage disease Type IA (von Gierke's disease); he presents with constant pain in his right lower leg. The pain started a few hours ago, and it is located at the mid-tibia level. He denies a previous history of a similar pain, and he does not remember any recent trauma to the area. Examination reveals a well-developed man with obvious discomfort in his right leg. His vital signs are unremarkable. He has numerous tophi on the extensor surfaces of his limbs. Pertinent positive findings are confined to his swollen right lower leg; the patient holds it still. The leg has point tenderness to palpation and motion along the anterior tibia, approximately midway between the tibial tuberosity and the anterior ankle crease. Radiological examination of the leg shows a non-displaced fracture of the right tibia corresponding to the point of maximum tenderness. All bone elements show demineralization by radiological examination. Chronic metabolic acidosis is the cause of these bone changes in this patient. Question What is the main cause of chronic metabolic acidosis in this case?

Correct answer: High lactate levels Explanation Chronic metabolic acidosis in von Gierke's disease is due to elevated lactic acid levels. In von Gierke's disease, there is a deficiency of glucose 6-phosphatase, which produces a block in the conversion of glucose 6-phosphate to glucose. Hence, glycogen breakdown, which normally produces glucose, diverts the glucose 6-phosphate into glycolysis. The clinical features include hepatomegaly, cherubic face, growth retardation, delayed onset of puberty, weakness, convulsions, bleeding tendencies due to altered platelet function, xanthomas, osteopenia, and fractures. The metabolic effects include hypoglycemia, hyperlactacidemia, hyperuricemia, and hyperlipidemia. Definitive diagnosis is by assessment of glucose 6-phosphatase activity in fresh frozen liver tissue specimen, which is obtained by liver biopsy. The aim of the treatment is prevention of hypoglycemia and metabolic acidosis by frequent small carbohydrate-rich meals and restriction of galactose and fructose intake, which increases the lactic acid levels. Persistent elevated uric acid levels may be treated with allopurinol. In von Gierke's disease, there is an excess of lactic acid and a chronic lactic acidemia, which produces a chronic metabolic acidosis. In an attempt to buffer the chronic metabolic acidosis, the calcium phosphate salts in the bones disassociate to free inorganic phosphate anions into the bloodstream. The bones become demineralized, and they easily fracture from ordinary daily stress, as in the case of this patient. In this disease, blood uric acid levels are increased due to increased generation and decreased excretion of uric acid by the kidneys. It may be that the excess glucose 6-phosphate is also diverted into the pentose pathway and ultimately into the formation of phosphoribosyl-pyrophosphate, which is the starting substrate for the formation of pyrimidines. Because in this disease there is a bottleneck in glycogen breakdown at the glucose 6-phosphate step, glycogen stores build up in the liver and in the kidney; neither event in and of itself is a cause of the chronic acidosis.

Case A 36-year-old Jewish man presents with abdominal pain and diarrhea, as well as a 2-day history of a low-grade fever. He is initially treated with fluids and antibiotics, but his symptoms worsen. On evaluation, he is diagnosed with Crohn's disease. He is a nonsmoker, but has alcoholic drinks on alternate days; he underwent an appendectomy 2 years ago. His mother was diagnosed with irritable bowel syndrome (IBS) 10 years ago. Question What in his history is a risk factor for Crohn's disease?

Correct answer: His Jewish ancestry Explanation Crohn's disease (CD) is more common in the Jewish population; it is more common in whites than African-Americans. Environmental factors may play a role in its etiopathogenesis because African-Americans seem to have the same risk as whites, but African blacks have a lower risk. The age of onset is between 15 to 30 years with a second peak between 60 to 80 years. CD is an inflammatory bowel disease affecting any part of the GIT from mouth to anus; the most commonly affected region is the ileocecal region. Symptoms include abdominal pain, diarrhea (which may be bloody), vomiting, fever, and weight loss. Extraintestinal symptoms (e.g., arthritis, uveitis) may also develop. Smoking, not alcohol, is a risk factor for CD. A family history of IBS is unrelated to CD; however, a family history of CD itself is a risk factor. An appendectomy neither increases the risk nor is protective in CD.

Question Excessive doses of some vitamins can cause toxicity. For a certain vitamin, early manifestations of chronic toxicity include dermatological findings like dry scaly skin and hair loss. Anorexia, vomiting, and hyperostosis may also be seen. What hypervitaminosis can cause these symptoms?

Correct answer: Hypervitaminosis A Explanation Very large doses of the fat-soluble vitamins are definitely toxic. Acute vitamin A intoxication was first described by Arctic explorers, who developed headache, diarrhea, and dizziness after eating polar bear liver. The liver of this animal is particularly rich in vitamin A. Nausea, vomiting, papilledema, and lethargy may be seen in acute toxicity. Chronic toxicity can occur when a person consumes 50,000 units per day for 3 months or more. Mouth sores may also be seen as an early manifestation along with those symptoms mentioned in the question. Hypercalcemia, increased intracranial pressure with papilledema, and decreased cognition may be seen in more serious cases. Hepatomegaly leading to cirrhosis may occasionally occur. Hyperostosis is a diffuse spondyloarthropathy caused by excessive calcium levels. It is associated with pain, decreased range of motion, and an increased risk of fracture. Elevated vitamin A in the blood is the only clue to the diagnosis. Elimination of vitamin A from the diet usually results in rapid recovery. Large doses of water-soluble vitamins (C and B) have been thought to be less likely to cause problems because they can be rapidly cleared from the body; however, it has been demonstrated that ingestion of large doses of vitamin B6 can produce peripheral neuropathy and large doses of niacin (B3) may cause skin flushing. Very large doses of vitamin C can cause gastric irritation, flatulence, or diarrhea. There is a theoretical risk of oxalate stones, as oxalate is metabolized by vitamin C; however, not many such cases have been reported. Vitamin E is the least toxic of the fat-soluble vitamins. Large doses taken over extended periods of time have not produced toxicity, except for a few GI disturbances.

Case A 5-month-old girl presents with a 3-day history of vomiting. She is exclusively breastfed, and her mother states that today she has vomited within 15 minutes of each feeding. Her last wet diaper was 10 hours ago. On physical examination, she is afebrile, tachycardic, irritable, and does not express tears when crying. She was a full-term infant born via vaginal delivery. She has no significant past medical history. Her 3-year-old sister has had gastroenteritis for the past few days. Question What is the most likely laboratory finding?

Correct answer: Hypokalemia Explanation This infant is showing signs of moderate dehydration due to emesis and limited breast milk intake. Gastric and urinary potassium losses can both contribute to hypokalemia. Dehydration leads to hemoconcentration with an increased hemoglobin and increased urine specific gravity. BUN and creatinine could be increased with volume depletion, and they could be increased further with renal insufficiency. Not eating for an extended period of time could lead to hypoglycemia in a 5-month-old infant. Hypernatremia implies a deficit of total body water. Major symptoms are thirst, confusion, neuromuscular excitability, seizures, and coma.

Case Ico-delete Highlights A 55-year-old African-American man presents as febrile with massive swelling of the abdomen and diarrhea. He has a 20-year history of heavy alcohol use. A fluid wave is elicited on physical examination of the abdomen by striking one flank and feeling the transmitted wave on the opposite flank. Question In what case would ascitic fluid analysis suggest cirrhosis as a cause of ascites in this patient?

Correct answer: If the fluid has a protein concentration below 3 g/dL Explanation The correct response is if the fluid protein concentration below 3/dL. After the diagnosis of ascites is made by physical examination, all patients with new-onset ascites should undergo abdominal paracentesis and ascitic fluid analysis. The most important tests to order for fluid analysis include protein concentration and cell count. Fluids with protein concentration above 3 g/dL are designated as exudates. Those with values below 3 g/dL are designated as transudates. Diseases usually associated with transudates include congestive heart failure, cirrhosis, constrictive pericarditis, inferior vena cava obstruction, hypoalbuminemia, Meigs syndrome, and some cases of nephrotic syndrome. The amount of albumin in the ascitic fluid compared to the serum albumin (the Serum Ascites Albumin Gradient, SAAG) can be indicative of the cause of ascites. Ascites related to hypertension, cirrhosis, or congestive heart failure generally shows a SAAG greater than 1.1 g/dL. Exudates are more commonly seen with peritoneal neoplasm, pancreatic ascites, myxedema, and tuberculous peritonitis. A large number of red blood cells in the fluid or grossly bloody ascites suggests a diagnosis of neoplasm. An ascitic fluid and leukocyte count of more than 500/mm3 strongly suggests a peritoneal infection or inflammatory process. Other tests that should be ordered in the appropriate clinical setting include cytologic examination, lactic dehydrogenase (LDH), specific tumor markers, glucose, and cultures for bacteria, mycobacteria, and fungi.

Case A 60-year-old man presents with severe abdominal pain that started 10 hours ago. It is increasing in severity and is colicky in nature. The patient has not had a bowel movement for 3 days. In the last 2 days, he vomited 4 times. He looks tired and dehydrated. On examination, increased bowel sounds were noted. There is also a mass in the right inguinal area. The patient said this mass has been present for 10 years, but until now, it has always disappeared when he lay on his back. Question What is the best treatment option?

Correct answer: Immediate exploratory laparotomy Explanation Intestinal obstruction occurs when bowel movement is encountering an obstacle in the passage of the bowel. This can be caused by any mass, adhesion, or any other means of mechanical obstruction. The symptoms of obstruction vary according to the site of obstruction, but generally there is increased bowel movement proximal to obstruction with the reflected increase in bowel sounds. There may be vomiting. There may be no bowel movement, from absolute constipation to feces and flatus. In this case, the patient has inguinal hernia, which is recently irreducible and obstructed. The best diagnostic procedure is erect X-ray film which shows multiple fluid levels and gives confirmation of the diagnosis. The treatment is exploratory surgery and there is no place for any conservative treatment without surgery. Manual reduction is contraindicated for fear of reduction of gangrenous bowel segment. Paralytic ileus is a type of intestinal obstruction that usually follows general anesthesia and is characterized with silent abdomen with no bowel sounds, usually treated by bowel rest and proper hydration. Total parental nutrition and bowel rest is the treatment of choice in paralytic ileus. Conservative treatment with antispasmodics can be used in acute abdomen due to cholecystitis. Conservative treatment with rehydration is used in acute diverticulitis.

Case A 33-year-old woman presents for an annual physical. She has past medical history of GERD, asthma, and irritable bowel syndrome. She drinks 1-2 alcoholic beverages per week and has never smoked; she does not use illicit drugs, and she consumes a vegetarian diet. Her past surgical history includes an appendectomy at age 14. Her father passed away from a heart attack at age 63. Her mother is alive with history of colorectal cancer, which was diagnosed at age 41. Question What is the recommended colorectal cancer screening for this patient?

Correct answer: Immediate screening colonoscopy Explanation The above patient should have an immediate screening colonoscopy. The American College of Gastroenterology currently recommends that patients with positive family history of colorectal cancer should begin screening colonoscopy at either age 40 or 10 years younger than the age at diagnosis of their relative. The patient's mother was 41 years old when she was diagnosed with colorectal cancer, which means that the patient should have undergone her first screening colonoscopy at age 31. Since the patient is currently 33 years old, she should undergo colonoscopy as soon as possible. Screening colonoscopy beginning at age 40, 45, and 50 are incorrect because of the age the patient's mother was when she was diagnosed with colorectal cancer. Screening colonoscopy if the patient becomes hemoccult positive is an incorrect answer. Screening should begin immediately; the primary care physician should not wait for signs of colorectal cancer to develop prior to screening.

Case A 35-year-old man presents for initial history and physical at his new primary care provider office. He is currently asymptomatic but based on his sexual history, you decide to screen with hepatitis lab tests in addition to other screening tests. The following results are received from the laboratory: Hep A Ab negative HBcAb negative HBeAg negative HBsAg negative HBsAb positive Hep C Ab negative Question What is your most likely diagnosis based on these results?

Correct answer: Immune due to the Hepatitis B vaccination Explanation These lab results are consistent with current vaccination but no present or past active Hepatitis B infection. The positive Hepatitis B surface antibody (HBsAb) is the antibody that denotes immunity after administration of the hepatitis B vaccine. Hepatitis B surface antigen (HBsAg) is the most frequently and easily performed serology test for hepatitis B, and is the first test to become abnormal after being exposed to or infected by the Hepatitis B virus. Hepatitis B core antibody (HBcAb) is detected approximately 1 month after infection with HBsAg, and declines over a several year period. HBcAb is also frequently positive in patients with chronic hepatitis. Hepatitis B e-antigen (HBeAg) is typically not used for diagnosing hepatitis infection, but rather to evaluate the index of infectivity. The presence of HBeAg correlates with early and active (and highly infective) disease. Hep A Ab and Hep C Ab are both incorrect, as they are not interpreting the immunity status of Hepatitis B.

Case A 24-year-old man presents with a painful ulcer on his left leg and systemic symptoms. The lesion started 1 week ago as a small pustule that developed at the site of mosquito bite, and it rapidly developed into a painful ulcer. Patient describes pain as "stabbing" and debilitating. He also develops symmetrical joints pain, muscle pain, fever, and malaise. His past medical history is significant for ulcerative colitis, which is currently in remission. On examination, you find a deep exudative ulceration, with a well-defined violet border and a worn, erythematous, and indurated edge. There are signs of pathergy as well. Question What will you recommend in addition to meticulous wound care?

Correct answer: Immunosuppressant (systemic corticosteroids) Explanation Your patient most probably has Pyoderma gangrenosum, which is a ulcerative skin lesion of an uncertain etiology; in more than 50% of cases, it is associated with systemic diseases, most commonly inflammatory bowel disease. It usually develops rapidly and can progress from a pimple to an ulcer in 1 or 2 days. In a process termed pathergy, new ulcerations may occur after trauma or injury to the skin. Pain is the predominant symptom, but symmetrical arthritis, myalgias, and malaise are also common. When the lesions heal, they usually leave the scars that are often cribriform. Immunosuppression is the mainstay of treatment; it is believed that dysregulation of the immune system (specifically, altered neutrophil chemotaxis) is involved. Most clinicians use both topical and systemic therapy. The most commonly used treatments include topical potent corticosteroids or tacrolimus to treat early lesions; systemic corticosteroids, TNF-α inhibitors, or other anti-inflammatories or immunosuppressants are used to treat more severe manifestations. Surgery or debridement is contraindicated because of the presence of pathergy. Even if there was no pathergy, surgery is contraindicated because skin trauma can trigger the pathergy. Acyclovir is not indicated; the clinical picture is not consistent with acute herpes infection and the patient is not immunocompromised. Fluconazole is not indicated; the clinical picture is not consistent with tinea (or any other fungal infection). Paracetamol alone probably will not relieve the debilitating pain. Sufficient pain medication in this case will probably include paracetamol and some other medication, sometimes opioids.

Case A 35-year-old man presents with a painful perirectal lump. It began 6 days ago as a small firm mass, and it has gradually increased in size. As the mass has grown, it has become more tender. On examination, there is a 4-cm fluctuant red perirectal mass. Question What is the appropriate treatment?

Correct answer: Incision and drainage Explanation Incision and drainage is correct. The patient has a perirectal abscess. When an abscess becomes fluctuant, incision and drainage should be performed. Fistulotomy is used in the treatment of anal fistula. An anal fistula is the communication of a granulated tract between the anal canal and the skin. A perirectal abscess may develop into an anal fistula. There are no signs of anal fistula on examination. Even if a fistula were present, the abscess would need to be drained before fistulotomy could be performed. Sphincterotomy is used in the treatment of anal fissure. In the early stages of abscess development, moist compresses may help bring the abscess to a head. This patient's abscess has developed into a fluctuant mass and needs to be drained. Intravenous antibiotics may be given in conjunction with incision and drainage of a perirectal abscess. They are not useful in monotherapy because it is difficult for the antibiotics to reach the walled-off infection. Antibiotics are often given following incision and drainage. Patients who are otherwise in good health can often be treated with oral antibiotics or may not require any antibiotic therapy.

Case A 28-year-old man presents with a 2-week history of a lump in his groin. While lifting a heater unit, he felt a "pop" in his groin and began to notice an outpouching in his lower abdomen; it has become mildly tender over the last week. Resting and lying flat appears to help; standing and lifting aggravate it. He denies any fevers, nausea, vomiting, or changes in bowel habits. The patient denies any previous abdominal surgeries or procedures. On examination, you identify a soft reducible mass in the lower abdomen; hernia examination reveals a mass pushing against your finger. An ultrasound of the lower abdomen shows the intestinal sac has traversed the deep inguinal ring. Question What is the patient's condition called?

Correct answer: Indirect inguinal hernia Explanation The correct answer is indirect inguinal hernia; these hernias are characterized by the intestinal sac entering through the deep inguinal ring. They are typically the result of a congenital defect, and there is incomplete obliteration of processus vaginalis. A direct inguinal hernia enters through the weakened abdominal fascia and into the anatomic region known as Hesselbach's triangle. This area is bordered by the rectus abdominus, the inferior epigastric artery, and the inguinal ligament. Femoral hernias and umbilical hernias do not traverse through the deep inguinal ring, and femoral hernias occur lower in the body than inguinal hernias near the leg crease. Umbilical hernias occur higher in the abdomen around the umbilicus. The patient did not have surgery, so an incisional hernia could not have occurred.

Case A 44-year-old woman is referred to a hospital because of occasional headache accompanied by irritability, confusion, sweating, and hunger. She states that symptoms appeared approximately 3 months ago; they frequently tend to appear early in the morning before breakfast or between meals. On one occasion, during the crisis period, a decreased blood glucose level (55 mg/dL) was detected despite the fact that she consumes a diet rich in glucose. She admits to excessive abuse of alcohol, and she occasionally used aspirin for headaches up to 6 days before admission. Her medical records suggest she was previously treated as a psychopath who was prone to malingering. The patient herself confirms that she was haloperidol-treated for more than 1 year, but she discontinued haloperidol use 2 weeks ago. Her general physical findings are unremarkable. Routine laboratory tests taken on admission reveal no abnormalities. During echosonographic examination of abdomen, no abnormalities were noted. Fasting test was attempted, but it had to be discontinued due to a fall in blood glucose level from baseline value of 75 to 33 mg/dL 8 hours later. It required intravenous glucose administration and termination of the test. Laboratory analyses taken at the time of test termination also revealed elevated serum insulin (9 uU/ml), elevated serum proinsulin (6.3 pmol/l), and elevated C peptide (0.3 mmol/ml) levels. Insulin receptor antibodies are not present in the patient's serum, and sulfonylurea is absent in the patient's sera and urine. Question What disorder is the most likely cause of patient's hypoglycemia?

Correct answer: Insulin-producing tumor Explanation An insulin producing tumor (insulinoma) is characterized by an increased endogenous production of insulin by insulin-secreting tumor. Since a proportion of insulin is released into the blood in the form of proinsulin, proinsulin production is also increased in patients with insulinoma. Also, proinsulin production is increased in patients with liver cirrhosis or diabetes mellitus type II. Therefore, an increased proinsulin level in a hypoglycemic patient who has no liver cirrhosis indicates hypoglycemia is caused by increased production of endogenous insulin. Deliberate insulin intake causes hypoglycemia. However, if hypoglycemia is caused by intake of exogenous insulin, serum proinsulin level tends to be normal or low. Also, serum level of C peptide that is produced by insulin-secreting cells together with insulin is increased in patients with insulinoma. C peptide level is, however, decreased if hypoglycemia is caused by exogenous insulin administration. Since the patient has both elevated proinsulin and C peptide levels, it is unlikely that hypoglycemia is caused by deliberate patient's administration of insulin. Chronic alcohol intake may cause hypoglycemia by decreasing glucose release from the liver. However, it is not likely that patient's hypoglycemia can be attributed to alcohol intake because her diet was rich in glucose. Also, elevated serum levels of insulin, proinsulin, and C peptide, as well as high serum insulin/glucose ratio (9/33=0.33), point to the existence of insulinoma.

Case A 60-year-old man presents with severe abdominal pain that started 10 hours ago. It is increasing in severity and is colicky in nature. The patient has not had a bowel movement for 3 days. In the last 2 days, he vomited 4 times. He looks tired and dehydrated. On examination, increased bowel sounds were noted. There is also a mass in the right inguinal area. The patient said this mass has been present for 10 years, but until now, it has always disappeared when he lay on his back. Question What is the most likely diagnosis?

Correct answer: Intestinal obstruction Explanation Intestinal obstruction refers to an obstacle in the normal passage of the bowel. This can be caused by any mass, adhesion, or any other means of mechanical obstruction. The symptoms of obstruction vary according to the site of obstruction, but generally there is increased bowel movement proximal to obstruction with the reflected increase in bowel sounds. There may be vomiting. There may be no bowel movement; absolute constipation to feces and flatus. Here the patient has inguinal hernia, which is recently irreducible and obstructed. The best diagnostic procedure is erect X-Ray film which shows multiple fluid levels and gives confirmation of the diagnosis. The treatment is exploratory surgery to correct the cause, and there is no place for any conservative treatment without surgery. Paralytic ileus is a type of intestinal obstruction usually following general anesthesia. It is characterized by silent abdomen with no bowel sounds, usually treated by bowel rest and proper hydration. Colon cancer may be a reason for obstruction. Here the patient has a clear reason, which is obstructed irreducible hernia. Appendicitis does not present with absolute constipation. It presents with pain, vomiting, rigidity, and rebound tenderness. The pain is usually localized to the right lower quadrant.

Question A 40-year-old man presents with a 2-year history of severe, burning epigastric pain. A detailed history reveals that the pain is greatest in the early hours of the morning and wakes him up from sleep. The pain is also felt 2-3 hours after meals. He reports diarrhea for the past 2 years and black stools for the past 3 days. On examination, his pulse is 74/min and blood pressure 136/84 mm Hg. There is slight epigastric discomfort on palpation. Lab examination shows occult blood in the stool and hyperchlorhydria. What could be a complication in this patient?

Correct answer: Intestinal ulcers Explanation The diagnosis in this case is gastrinoma (Zollinger-Ellison disease). Gastrinomas are small tumors that continually release excessive amounts of gastrin. Hypergastrinemia has a trophic effect on the gastric mucosa and enhances the secretion of hydrochloric acid from gastric parietal cells in both basal and stimulated states. The continued delivery of acid to the upper small intestine exceeds the capacity of acid-neutralizing mechanisms and results in the erosion and ulceration of the intestinal mucosa. In addition to acid secretion, parietal cells release intrinsic factor in response to gastrin. Intrinsic factor is a glycoprotein required for the normal intestinal absorption of vitamin B12. Red blood cell maturation is retarded in the absence of intrinsic factor resulting in pernicious anemia. Intestinal acidosis in patients with hypergastrinemia also reduces the activity of pancreatic enzymes, particularly lipases. This inhibits digestion and reabsorption of fat resulting in steatorrhea (fat in the feces) and weight loss. Gastrinomas may be either sporadic or associated with autosomal dominantly inherited Multiple Endocrine Neoplasia (MEN) type I.

Case A 6-week-old male infant presents with a 2-day history of vomiting after every feeding of cow's milk-based formula with iron, in the amount of 4 ounces per feeding. There has been no fever, diarrhea, or other symptoms except increased crying. The child appears alert and hungry. The mother describes the vomiting as forceful, traveling about 2 feet. Physical evaluation reveals minimal tear production with mild skin tenting. Bowel sounds are decreased. BUN was 29 mg/dl; serum sodium was 129 mg/dl; serum potassium was 3.4 mg/dl; serum chloride was 89 mg/dl; and serum bicarbonate was 34 mg/dl. Question What is the next step in this patient's care?

Correct answer: Intravenous correction of dehydration and electrolyte imbalance with simultaneous abdominal ultrasonography Explanation While the above scenario is highly suggestive of infantile hypertrophic pyloric stenosis (IHPS) (age 6 weeks, male, projectile vomiting with each feeding, hypochloremia with metabolic alkalosis), the need for surgery is not emergent. The child should be placed NPO, and dehydration and electrolyte abnormalities should be corrected, after which a pyloromyotomy can be performed. Surgery in the face of uncorrected dehydration and electrolyte abnormalities can be deleterious. While Helicobacter pylori has been implicated as a possible contributing factor in the development of IHPS, it is unproven. Furthermore, this would not be an immediate step in managing this patient. Methylprednisolone has been used as adjunctive treatment in nonhypertrophic pyloric stenosis in older individuals treated endoscopically, but is not used in IHPS. IHPS is a form of gastric outlet obstruction, and oral rehydration should not be employed.

Question A 54-year-old woman presents to the emergency department with nausea, vomiting, right-upper quadrant abdominal pain, fever, and jaundice starting 4 hours ago. During the last 6 months, she suffered several bouts of upper-abdominal pain accompanied by nausea, vomiting, and occasional jaundice, for which medical attention was sought. Her past medical history includes hyperlipidemia, for which she first took simvastatin; she switched to cholestyramine because of side effects. Vital signs on admission are as follows: BP 110 / 80 mm Hg, HR 90 bpm, RR 20 rpm, temperature 38.1°C (100.6 F). She is alert and oriented, and mildly jaundiced. Her right-upper abdomen is diffusely tender to palpation. An upper abdominal ultrasound is performed and reveals a thickening and calcifications of the gallbladder wall, but there are no signs of air within the peritoneal cavity or the bile ducts. What is the most appropriate next step in management?

Correct answer: Intravenous hydration and antibiotics Explanation The presence of fever, jaundice, and right-upper quadrant pain defines Charcot's triad, which is the classical presentation of acute cholangitis. Reynolds' pentad consists of Charcot's triad plus sepsis/shock and mental status changes. 95% of patients presenting with these syndromes have common duct stones. In most cases, there is a favorable response to conservative treatment, which consists of interrupting oral feeding, analgesia, intravenous hydration, and antibiotics. The following antibiotic schemes can be used: A third-generation cephalosporin + an aminoglycoside Piperacillin or ampicillin + metronidazole + an aminoglycoside Monotherapy with imipenem, meropenem, mezlocillin, ampicillin-sulbactam, ticarcillin-clavulanate, or piperacillin-tazobactam After clinical improvement, cholecystectomy is undertaken in the first few days after the initial hospitalization in order to avoid recurrence. Surgery is warranted because the recurrence rate is as high as 10% per year in patients whose gallbladders are not removed. The use of cholestyramine is another risk factor for recurrence in this patient, and gallbladder calcifications (porcelain gallbladder) are thought to pose a risk of malignization and are considered an indication for cholecystectomy. When the disease progresses despite the initial conservative treatment (i.e., there is worsening fever, leukocytosis, abdominal pain, and guarding), emergency biliary drainage is warranted because of the possibility of gallbladder perforation or gangrene. Some possible approaches are cholecystectomy (conventional or laparoscopic), cholecystostomy, or percutaneous drainage. The latter 2 are usually reserved for sicker patients, who are less likely to tolerate surgery well.

Case A 36-year-old woman presents with confusion, a lack of control over her body, and unusual eye movements. She appears to be malnourished, and she shows signs of poor hygiene. She has been vomiting recently. Her pulse is 80/min, blood pressure is 122/70 mm Hg, temperature is 98.4° Fahrenheit (36.9 °C), and respiration is normal. Question What is the next best step in the management of this patient?

Correct answer: Intravenous thiamine Explanation The correct response is intravenous thiamine. Based on the symptoms, this patient has Wernicke encephalopathy, part of a syndrome caused by vitamin B1 deficiency; it is a common consequence of poor nutrition and alcoholism. To prevent chronic brain damage and the onset of Korsakoff psychosis, a condition that often follows a Wernicke encephalopathy episode, the patient needs to be administered vitamin B1 (thiamine) immediately. Thiamine supplementation can provide complete resolution of Wernicke symptoms, especially if alcohol misuse is not the underlying cause. Intravenous glucose is not advised for patients with Wernicke-Korsakoff syndrome; it can cause the exhaustion of their thiamine and accelerate the disorder. Lack of vitamin B12 causes a different nutritional disease which can also lead to damage within the brain. It results in the subacute combined degeneration of the spinal cord (SCDSC); however, symptoms of this disease do not include confusion and ophtalmoplegia. The patient's symptoms are associated with alcoholism, and participation in an alcohol recovery program should be included in the treatment. Diazepam is commonly used for relief of the withdrawal symptoms, but this is not the 1st step in the treatment of Wernicke encephalopathy. Toxicological screening is often performed in emergency situations, but given the symptoms in this case, the test will not provide more information or change the treatment steps.

Case A 2-year-old boy presents with acute abdominal pain. The boy has not passed stool for 2 days. On examination, the abdomen is distended and tender. Bowel sounds are absent. The boy is taken to surgery. Refer to the image for the gross appearance of the surgical specimen. Question What is the most likely diagnosis?

Correct answer: Intussusception Explanation Intussusception is the telescoping of a segment of intestine into the segment below due to peristalsis. The telescoped segment is called the intussusceptum and the lower receiving segment is called the intussuscipiens. The condition commonly occurs in infants and young children, more often in the ileocecal region. Less commonly, ileo-ileal and coli-colic intussusception occurs. In children, the cause is not known, though enlargement of the lymphoid tissue in the terminal ileum has been suggested. In adults, the usual causes are foreign bodies and tumors. The main complications include intestinal obstruction, infarction, gangrene, perforation, and peritonitis. The picture shows cecum, which has been cut open to show the ileum that has intussuscepted inside. Acute appendicitis is the most common acute abdominal condition confronting the surgeon. The most common etiological factor is the obstruction of the lumen, which leads to increased intraluminal pressure. This presses upon the blood vessels to produce ischemic injury, which in turn favors bacterial proliferation and acute appendicitis. Grossly, the appendix is swollen and hyperemic during early stages. The serosa is then covered with fibrinopurulent exudate, called acute suppurative appendicitis. As the disease advances, there is necrosis and ulceration of the mucosa, which extends through the wall so that the appendix becomes soft and friable, and the surface shows greenish-black gangrenous necrosis (known as acute gangrenous appendicitis). Microscopically, the diagnostic criterion is the neutrophilic infiltration of the muscularis. Clinically, the patient presents with acute abdominal pain. If not adequately managed, complications like peritonitis, appendicular abscess, adhesions, mucocele, and portal pylephlebitis occur. Meckel diverticulum is the most common congenital anomaly of the gastrointestinal tract. It occurs in 2% of the population, and it is more common in men. It is commonly situated on the antimesenteric border of the ileum, about 1 meter above the ileocecal valve. Meckel diverticulum is an outpouching containing all the layers of the intestinal wall in their normal orientation. It is usually lined by a small intestinal type of epithelium. At times, it may contain gastric mucosa or pancreatic tissue. Meckel diverticulitis is a common complication of the diverticulum. If not treated, it may lead to perforation and hemorrhage. Volvulus is the twisting of a loop of intestine upon itself through 180 degrees or more. This leads to obstruction of the intestine, as well as cutting of vascular supply to the affected region. It is usually due to bands, adhesions, and long mesenteric attachment. The condition affects mainly the sigmoid colon. Ischemic bowel disease applies to the structural changes in the colon that occur due to deprivation of blood supply. The causes for deprivation of blood could be arterial occlusion, venous occlusion, and non-occlusive ischemia. The causes of non-occlusive ischemia include cardiac failure, shock, dehydration, and vasoconstrictive drugs. Depending on the severity and the presentation, it is classified into transmural, mural and mucosal infarction, and chronic ischemic colitis. In transmural infarction, there is hemorrhagic infarction of the bowel with gangrenous changes and a risk of perforation. In the mucosal and mural type, the mucosa is hemorrhagic and edematous but the remaining layers will remain intact. In chronic ischemia, mucosal ulceration and inflammation develop, and sub-mucosal chronic inflammation and fibrosis may lead to stricture formation. In the setting of transmural infarction, patients generally present with acute abdominal pain, vomiting, fever, and bloody diarrhea. The patient may progress to shock and vascular collapse within hours.

Question What mineral is essential to prevent goiter?

Correct answer: Iodine Explanation Although the body needs only a trace amount of the mineral iodine, it is essential for the synthesis of the thyroid hormone, thyroxine. The thyroid gland enlarges in iodine deficiency and sometimes become enlarged enough to be visible as a lump in the neck called a goiter. Vitamin B12 is essential in maintaining and protecting the myelin sheaths of nerve fibers. Calcium is vital to bone formation. Most calcium is stored in teeth and bones. 1% of this mineral is in cellular fluid, which aids in cell membrane transport, blood clotting, muscle contraction, and maintaining blood pressure. Iron aids in transporting oxygen throughout the body. Only 1% of magnesium can be found in cellular fluids. The majority of magnesium is found in bones and muscles, and the rest of the mineral can be found in the liver, muscle, and heart. Magnesium is essential in numerous cellular functions, such as potassium, calcium, and vitamin D metabolism.

Case A 23-month-old boy presents with a sudden onset of bright red blood in his bowel movements. He is afebrile and does not seem to be in any pain. A technetium-99m pertechnetate scan after enhancement with cimetidine shows a 'hot spot' in the area of his ileum. Question What is the most likely cause?

Correct answer: Meckel diverticulum Explanation This patient is most likely suffering from a common congenital abnormality of the development of the ileum called a Meckel's diverticulum. In the embryo, the vitelline duct is a communication between the yolk sac and the lumen of the gastrointestinal tract at the midgut. Normally, it degenerates completely, but the persistence of a portion of the vitelline duct leads to the development of a cul-de-sac on the ileum (Meckel's diverticulum). This congenital birth defect follows a rule of 2s. It occurs in 2% of the population, but only 2% show symptoms. It is usually located about 2 feet from the ileocecal valve and is about 2 inches long. It can contain 2 types of ectopic tissue: gastric or pancreatic. It usually presents by 2 years old. The presence of ectopic gastric mucosa in the diverticulum can lead to secretion of stomach acid downstream from the duodenum, which has bicarbonate-secreting submucosal Brunner glands to neutralize gastric acid. Ectopic gastric tissue in the ileal diverticulum can lead to ulceration and bleeding of the adjacent ileal mucosa. Vascular malformations, intussusception, and coagulation disorders are more likely to manifest themselves in the first year of life. Cow's milk colitis is a problem of the first year of life and spontaneously resolves by the end of that year for most children. NEC is a problem of the stressed, usually premature, newborn. Rectal polyps are likely to present in older children, as is HSP. Babies and older children can have gastric bleeding from gastritis or gastric ulcers. Duodenal ulcers are much more common in older children.

Case A 20-year-old woman presents with a 1-week history of dizziness. She reports that her menstrual periods have been heavy since menarche at the age of 14 years. On examination, she is pale and has koilonychia. Laboratory investigations reveal normal prothrombin time, normal activated partial thromboplastin time, and a normal platelet count. Question What is the most likely diagnosis?

Correct answer: Iron deficiency Explanation In cases of iron deficiency, patients can present with feeling weak, dizzy, and tired; they may experience syncope. On examination, they have pale conjunctivae and koilonychia (spooning of the nails). Causes include inadequate dietary intake as well as blood loss in menorrhagia and during delivery or after trauma. Dietary sources of iron include beef, liver, and dark green vegetables. In riboflavin or vitamin B2 deficiency, patients present with angular stomatitis and cheilosis. On examination, they are pale, have atrophic glossitis, and the tongue may appear magenta. Vitamin B1 (or thiamine) deficiency results in beriberi, which is characterized by: a bilateral symmetric peripheral neuropathy beginning in the legs Wernicke-Korsakoff syndrome, which is comprised of nystagmus, ophthalmoplegia, ataxia, memory loss, and confabulation congestive heart failure with tachycardia, peripheral edema, and cardiomegaly In cases of vitamin K deficiency, patients present with bleeding tendencies such as epistaxis, menorrhagia, and hematuria. The prothrombin time (PT) and the activated partial thromboplastin time (aPTT) are usually prolonged. In cases of vitamin D deficiency, children can present with inability to walk unsupported due to muscle weakness and lower limb skeletal deformities, such as genu varum and genu valgum.

Case A 20-year-old woman presents with intractable constipation and bloating for the last several weeks. Her stools are hard, and bowel movements occur 1-2 times per week. She has diffuse crampy abdominal pain after eating, which is relieved with defecation. She denies diarrhea, nausea, vomiting, weight loss, fever, dyspepsia, or melena. Past medical history includes depression, for which she has been inconsistently taking Venlafaxine. Family history is significant for mother having depression as well. She does not smoke or drink. On exam she is afebrile and vitals are normal. Abdominal exam shows no specific findings. Pelvic exam is unremarkable. Labs show a hemoglobin of 12 g/dL, WBC 7000, and platelets 400,000. TSH is pending. Liver enzymes are normal and ultrasound of the gallbladder reveals no pathology. Obstructive series of the abdomen shows copious stool in the colon. The patient is given stool softeners without relief. Question What is her most likely diagnosis?

Correct answer: Irritable bowel syndrome Explanation Irritable bowel syndrome (IBS) is an intestinal motor disorder of unknown cause. The symptoms of constipation (or chronic diarrhea), abdominal pain relieved with defecation, and bloating are typical. It is a diagnosis of exclusion, however, and other common disorders of the alimentary tract, like bowel obstruction, motility disorders of the colon, and medication-induced constipation must be excluded prior to definitive diagnosis. Patients with diarrhea must be evaluated for celiac disease, lactose intolerance, and medication-induced diarrhea. It is associated with functional disorders like depression. Colonoscopy may reveal spastic colon. Treatment includes reassurance, reduction of stress, exercise, fiber supplementation, fluids, pro-motility drugs and antispasmodics as needed. Intestinal obstruction is characterized by constipation, vomiting, abdominal pain, and abdominal distension, with abnormal bowel sounds on auscultation. Obstructive series shows multiple air-fluid levels. Acalculous cholecystitis is an inflammation of the gallbladder due to causes other than gallstones. It is associated with acute right upper quadrant abdominal pain, nausea, vomiting, and sometimes dyspepsia. Hypothyroidism can be associated with moderately severe constipation, but there is an absence of abdominal pain and bloating. Other symptoms like weight gain, menstrual irregularities, and cold intolerance may be seen. Depression may also be present. Drug-induced constipation can be caused by several antidepressants, antihistamines, and atropine-like substances, but abdominal pain is uncommon. It can be confirmed by stopping the medication, considerably relieving symptoms.

Case A 68-year-old woman presents with a 4-hour history of severe left-sided abdominal pain. The pain was initially associated with several episodes of diarrhea with some hematochezia. Since then, she has had a few more stools, but she has not seen any more frank blood. Her pain is somewhat improved from what it was at the onset. Past medical history is positive for renal lithiasis and atrial fibrillation. Current medications are digoxin and aspirin. She is a recovering alcoholic who last had a drink 15 years prior to presentation. She recently returned home from a trip to Mexico; she was visiting relatives for 2 months. On exam, you see a well-nourished woman in extreme discomfort. Auscultation of her lungs reveals good breath sounds bilaterally; her heart has an irregularly irregular rhythm with a rate of 92 beats per minute; there is a soft II/VI systolic murmur. Her abdomen is mildly obese and tender on the left side; there is no appreciable mass or rebound. There is no flank tenderness. Rectal exam reveals guaiac positive mucus mixed with flecks of bright red blood. 12 hours after presentation, her left upper quadrant pain is still present; however, it is much improved, and she had only 2 more stools with small flecks of blood in them. Question What is the most likely diagnosis?

Correct answer: Ischemic colitis Explanation This is a classic presentation of ischemic colitis. Ischemic colitis occurs when there is obstruction of the colonic blood supply, causing ischemia with inflammation and ulceration of the colonic mucosa. The area of the splenic flexure is most at risk for ischemia because it is the watershed area of the arterial supply to the colon. Ischemic colitis is most commonly seen in elderly individuals. These patients typically present with an acute onset of pain associated with bloody diarrhea, and they have an area of tenderness corresponding to the ischemic segment of colon. Patients may experience recurrent bouts of ischemic colitis. Sigmoidoscopy makes the diagnosis in 85% of patients; 15% will have ischemia in areas proximal to the reach of a sigmoidoscopy. The majority of these patients will improve with supportivecare (hydration and prophylactic antibiotics, in case of bacterial transmigration). However, some patients have much more severe disease, which may even require emergency colectomy. The most important condition to differentiate from ischemic colitis is that of small bowel ischemia. Small bowel ischemia can lead to gangrene of the intestine. Most commonly, these patients have developed occlusion of some of the distribution of the superior mesenteric artery, either from acute thrombosis in an area of atherosclerosis, or from embolic disease. These patients classically have pain out of proportion to their exam, at least early in the process, and they do not usually have an area of localized tenderness on palpation. These patients can have leukocytosis on the level of 20,000 to 30,000/μL, and they may exhibit a lactic acidosis and elevated amylase levels. Their stool is usually positive for occult blood, but frank blood is only rarely seen. Early diagnosis is very important, although often difficult, and urgent surgery is the only treatment. Renal or ureteral lithiasis can cause severe abdominal pain, and these patients can have diarrhea as well, but it should not cause hematochezia or localized anterior abdominal tenderness. Diverticulitis is more common on the left, which is the case with this patient's pain, but it typically does not cause hematochezia. Additionally, the sudden onset of very severe pain would not be typical of diverticulitis. Infectious colitis can cause abdominal pain and hematochezia, but the improvement in her pain over such a short time period would be atypical for an enterotoxic infection. Nevertheless, infection should be considered in this patient because of her recent travel history.

Question Why is McBurney's point an important anatomic landmark?

Correct answer: It is the somatic site of pain when acute appendicitis has involved the parietal peritoneum. Explanation McBurney's point is the cutaneous point in the right lower quadrant of the abdominal wall where an inflamed appendix causing peritoneal irritation will cause pain. In the context of an appropriate preceding clinical history, it is most suspicious for acute peritonitis. McBurney's point is located 2/3 of the distance between the umbilicus and the right anterior superior iliac crest. It can also be painful in cases of other lower abdominal pathology, such as a ruptured ovarian cyst or Meckel's diverticulum.

Question Ico-delete Highlights What is the clinical presentation of erosive gastritis?

Correct answer: It typically occurs 7 days after the initial injury Explanation Erosive gastritis typically will present 7 to 10 days after the initial injury. The initial presentation is bleeding, either as hematemesis or as blood in the nasogastric tube effluent. Diagnosis is usually confirmed by endoscopic evaluation of the gastric lumen. Since erosive gastritis is a condition affecting the mucosa and not the deeper parts of the gastric wall, patients will not have complaints or findings of perforation or peritoneal irritation such as is found in peptic ulcer disease.

Case A 53-year-old man presents with a 2-day history of jaundice and malaise. His history is significant for mucosal candidiasis, for which the patient is on oral ketoconazole 200 mg daily for the past 3 weeks. He occasionally has headaches, and sometimes takes acetaminophen in a daily dose of 1 gram to achieve headache relief. Two weeks before the appearance of symptoms, the patient was treated with flucloxacillin for respiratory tract infection. Except for the appearance of jaundice and malaise, the patient denies the presence of any other symptoms, and the remainder of his personal history is unremarkable. Physical examination reveals a mildly jaundiced patient, 180 centimeters tall, 82 kilograms in weight. His blood pressure is 110/86 mmHg, and the remainder of his general physical examination revealed no abnormalities. Laboratory analyses reveal the presence of hyperbilirubinemia and elevated serum transaminases, normal alkaline phosphatase and γ-Glutamyl transferase levels. Other routine laboratory analyses reveal no abnormalities. Serological testing does not reveal the presence of antibodies against human immuno-deficiency, hepatitis A, C, D and E, or viruses in patient's serum. Also, HBsAg, anti-HBs or anti-HBc antibodies are not present in patient's serum. Anti-LKM-1, antinuclear, anti-thyroid, antimitochondrial, and anti-smooth muscle antibodies are absent. Question What is the most likely cause of liver dysfunction in this patient?

Correct answer: Ketoconazole-induced hepatitis Explanation Ketoconazole is an antifungal imidazole used in treating systemic mycoses. The common adverse effect of ketoconazole is hepatotoxicity. The severity of ketoconazole-induced hepatotoxicity is linked to the exposure level of the drug. Ketoconazole-induced hepatotoxicity is probably mediated through a reactive metabolite N- deacetyl ketoconazole (DAK). The latter appears to be the major metabolite, which is a hepatic cytotoxic. Ketoconazole administration results in a significantly dose-dependent increase in serum transaminase activities, as well as cloudy swelling, ballooning degeneration and centrilobular confluent necrosis of the hepatocytes. The histological feature ranges from acute hepatitis to confluent centrilobular, or massive necrosis. Acetaminophen hepatotoxicity is dose-related (i.e., signs of liver dysfunction appear only when a high dose of acetaminophen, which exceeds liver metabolizing capacity for this drug, is taken). In such a case, toxic intermediary metabolites do accumulate, causing liver damage. It is considered that a daily dose of 10 - 15 mg/kg may be considered as a safe, therapeutic dose. Therefore, liver dysfunction, in the presented case, is unlikely to be the result of acetaminophen-induced hepatitis. Primary sclerosing cholangitis is frequently accompanied with elevation of serum alkaline phosphatase, and gamma-glutamyl transferase levels. Since increased serum levels of those enzymes are not noted in the presented case, it is unlikely that the patient suffers from primary sclerosing cholangitis. Flucloxacillin administration causes cholestatic hepatitis (i.e., liver dysfunction in patients with flucloxacillin-induced hepatitis is associated with elevated alkaline phosphatase and gamma-glutamyl transferase levels, as well as with disproportionate rise of conjugated serum bilirubin level). Since hepatitis in the presented patient is not of cholestatic type, it is unlikely that he suffers from flucloxacillin-induced hepatitis. Autoimmune hepatitis is characterized with the appearance of auto-antibodies, including liver kidney microsomal (LKM) antibodies. LKM antibodies in patients with genuine (primary) autoimmune hepatitis are of LKM1 type, not of LKM2 type.

Question A 52-year-old man presents with heartburn, associated with reflux of sour-tasting material into the mouth. Some of these episodes are accompanied by increased salivation, coughing, and occasional regurgitation of food. He denies any bleeding or abnormalities in his stools. He is slightly overweight and has mild hypertension, which is well controlled with enalapril. Vital signs are within normal ranges and the physical examination is unremarkable. An upper endoscopy reveals a large hiatal hernia and coalescing linear erosions throughout the esophageal circumference and a 5.5-cm-long, circumferential cherry-red patch above the gastroesophageal junction. Biopsy of the patch reveals columnar metaplasia, but no dysplasia. What surgical treatment can improve the patient's symptoms most efficiently?

Correct answer: Laparoscopic fundoplication Explanation Laparoscopic fundoplication is the surgical treatment of choice for people with gastroesophageal reflux disease (GERD) that is unresponsive to medication. The most common technique is the Nissen procedure (i.e., suturing the gastric fundus to the lower esophagus throughout the full esophageal circumference). Side effects are uncommon and mostly minor: abdominal bloating, inability to belch, paraesophageal herniation, dysphagia, and diarrhea. Barrett's esophagus sometimes disappears after fundoplication, but this procedure does not eliminate the risk of dysplasia and carcinoma in all patients. Therefore, endoscopic screening after the procedure is mandatory. The other major therapeutic option for patients with GERD is long-term proton pump inhibitors (PPIs). Patients treated with these drugs have esophageal healing and symptom relief rates comparable to those of operated patients. However, about 30-40% of patients only improve after dose escalation. Therefore, PPI users take longer on average to achieve symptom control. Esophagectomy would be an option if there were an esophageal carcinoma or high-grade dysplasia, particularly in younger patients without significant comorbidities. The decision to operate must take into account the patient's general health and comorbidities, which are common given that Barrett's esophagus often occurs in elderly patients. However, advanced age by itself does not contraindicate esophagectomy. Whipple procedure is a complex operation involving removal of part of the pancreas, duodenum and gallbladder used most commonly in the treatment of pancreatic tumors. This is not a procedure for the treatment of gastrointestinal reflux. Several endoscopic techniques (e.g. mucosectomy, laser ablation, and photodynamic therapy) have had some success in the treatment of Barrett's esophagus. However, complication such as bleeding after extensive mucosectomies, laser perforations, and skin burns from photoporphyrin-induced photosensibility are common. Incomplete mucosectomies, after which small areas of high-grade dysplasia or even cancers remained in the esophageal wall, have also been documented. These procedures would not be effective in treating the hiatal hernia. Radiofrequency thermal coagulation of the esophageal wall (Stretta procedure) consists of using a special catheter to induce controlled heating of the esophageal wall at the level of the gastroesophageal junction. This induces fibrosis and contraction of the gastroesophageal junction, which increases sphincter pressure and improves reflux. However, recurrence is high after this procedure, and it cannot be performed when there is a hiatal hernia, as in this case.

Case A 16-year-old girl is brought to the pediatrician by her mother with a history of chronic diarrhea. She gives a history of large volumes of watery fecal output that is non-bloody and can be easily flushed; the diarrhea is painless and persists with fasting. The patient's mother complains that she has been eating less and has lost weight. She has no history of fever, flushing, wheezing, intolerance to heat/cold, or infection. Her family history and menstrual history are noncontributory. On examination, her weight is 130 lb; height is 5 feet 5 inches; pulse is 80/min; BP is 100/74 mm Hg; and temperature is 98.8°F. The tongue is dry, but the rest of her physical examination is within normal limits. Question What is the most likely diagnosis?

Correct answer: Laxative abuse Explanation The most likely diagnosis is laxative abuse. A large volume of watery fecal output that is non-bloody, painless, and persists with fasting suggests secretory diarrhea that can result from the use of stimulant laxatives like senna, bisacodyl, or castor oil. The patient is young, dehydrated, and based on her symptoms, is likely taking laxatives to lose weight and get rid of unwanted calories. There are no signs and symptoms of other causes of secretory diarrhea, such as carcinoid syndrome, and the rest of the options do not have conditions that cause secretory diarrhea. Many patients might not admit the use of laxatives to lose weight. Carcinoid tumors can cause secretory diarrhea, but it is unlikely in this patient; she has no associated symptoms, such as episodic wheezing or flushing. Hyperthyroidism results in diarrhea due to dysmotility. However, the patient does not have any symptoms of hyperthyroidism, such as unexplained weight loss or heat intolerance. Inflammatory diarrhea can be ruled out in this patient; there is no fever or signs of inflammation or bloody stools. Celiac disease causes diarrhea due to mucosal malabsorption. Patients present with multiple nutritional deficiencies and fatty diarrhea, which are not present in this patient, making this diagnosis unlikely.

Case A 28-year-old man presents for a barium enema; he has a 6-month history of abdominal cramping, pain, tenesmus, bloody diarrhea, and painful urgency. He also reports arthralgias, but denies a history of traveling abroad or the use of antibiotics. On physical exam, he has indurated and painful erythematous plaques and nodules on both shins. An abdominal exam reveals a mild tenderness in the lower left quadrant (LLQ). Stool exam is positive for occult blood and negative for ova and parasites. Lab analysis reveals mild leukocytosis, microcytic anemia, positive p-ANCA antibodies, and elevated ESR. Question What would be the most likely result of a barium enema?

Correct answer: Lead pipe appearance Explanation Lead pipe colon is the appearance on barium enema of a foreshortened, narrow colon with loss of redundancy and haustral markings. This sign is specific for ulcerative colitis (UC), an inflammatory disease of the rectum and colon. The peak incidence of UC is around 25 to 45 years. P-ANCA antibodies are associated with UC. Iron deficiency can lead to microcytic anemia. Common extra intestinal manifestations of UC are erythema nodosum (described as erythematous plaques and nodules on pretibial areas), arthritis, pyoderma gangrenosum, uveitis, episcleritis, and sclerosing cholangitis. Toxic megacolon is a complication of UC characterized by the dilation of the colon; there is a risk of perforation. There is an increased incidence of colon cancer (10%) after 10 years of this disease. The inflammation involves the mucosal tissues only, which are friable and show ulcerations on colonoscopy. There is also a characteristic continuous involvement and pseudopolyp appearance caused by mucosal regeneration. Sulfapyridine, mesalamine, corticosteroids, 6-mercaptopurine, and azathioprine are used for treatment. Colectomy is curative and is indicated for intractable disease, or in the case of dysplasia. An apple core appearance on barium enema is seen in patients with colon cancer. This sign refers to the shouldered margins of the stricture caused by the neoplasmic invasion. A cobblestone appearance and string sign are characteristic for Crohn's disease, another inflammatory disease of the digestive tract. The cobblestone appearance refers to the discontinuous areas of inflammation separated by healthy bowel. String sign, seen in the small bowel series, is the appearance caused by stricture areas. Thumb printing is a radiological sign seen in ischemic colitis, a disease most likely found in elderly patients with left lower abdominal pain and gross rectal bleeding.

Question A patient with a history of alcohol abuse presents with a histopathological report; he is asking about the 'bodies found in his liver cells'. What type of bodies is he talking about?

Correct answer: Mallory bodies (hyaline inclusions) Explanation Mallory bodies (alcoholic hyaline) are cytoplasmic inclusions found most often in alcoholic hepatitis. They are abnormal aggregations of cytoskeletal proteins. Russell bodies are found in reactive or malignant plasma cells, not in liver cells. They represent excess immunoglobulin. Lewy bodies are abnormal aggregates of protein inside nerve cells in Parkinson's disease (PD) and some other degenerative brain disorders. They are not found in liver cells. Negri bodies are eosinophyllic, viral, intracytoplasmic inclusions found in hippocampal nerve cells. They are pathognomonic for rabies infection. Heinz bodies are inclusions within red blood cells that are composed of denatured hemoglobin. Although they can be found in liver disease, as well as alpha thalassemia and glucose-6-phosphate dehydrogenase, they are not found in liver cells. Guarnieri bodies, or B-type inclusions, are the sites of viral replication; they are found in all poxvirus-infected epithelial cells, but not in the hepatocytes.

Case A 25-year-old man presents with a 3-hour history of retrosternal chest pain. He reports that he has been vomiting since the previous evening and his last vomitus had fresh blood in it. He denies any trauma to the chest. He has a history of heavy alcohol use. On examination, his BP is 90/60 mm Hg and P 110/min. Question What is the most likely cause of this patient's chest pain?

Correct answer: Mallory-Weiss syndrome Explanation In cases of Mallory-Weiss Syndrome, patients present with hematemesis, especially after repeated severe retching and vomiting, which results in a mucosal tear at the gastroesophageal junction. They may also have retrosternal chest pain. Tuberculous pleuritis chest pain is pleuritic and aggravated by coughing. It is described as sharp. Other symptoms (e.g., fever, dyspnea, and weight loss) may also be present. On examination, there may be dullness on percussion as well as absent breath sounds on the affected side. In Tietze's syndrome, patients present with anterior chest pain that is aggravated by taking a deep breath, sneezing, and turning motions. On examination, the affected costochondral junctions are erythematous, warm, and tender on palpation. The most commonly affected are the second or third costochondral joints. Patients with myocardial infarction usually present with a left-sided or retrosternal pain that may radiate to the jaw, neck, and shoulder. They describe it as heaviness or a squeezing sensation. It is of variable duration and often lasts for more than 30 minutes. There is a gradual intensification of the pain. The onset of the pain may be during physical exertion or at rest. It is not relieved by nitroglycerine. On examination, they may be dyspneic and diaphoretic. Chest pain due to a panic disorder has a variable presentation. It can be retrosternal or localized, brief or over 30 minutes in duration. It can be described as aching and/or sharp. Other symptoms of a panic disorder include lightheadedness, shortness of breath, nausea, paresthesias, palpitations, derealization, and the fear of losing control. The history helps elicit precipitating factors and prior panic attacks.

Question A 19-year-old man was on a drinking binge over the weekend with his friends. He was brought to the ED after he had several bouts of retching and vomiting followed by an episode of bloody emesis. He has no other past medical history and is not on any medications. He smokes a half pack of cigarettes daily and is a weekend drinker. Family history is insignificant. On exam, he has a temperature of 99 F, pulse of 96/min, BP 110/60 mm Hg, and SPO2 of 92%. Mucosa is somewhat dry, and his breath smells of alcohol. Lungs are clear and he has some epigastric tenderness. Labs reveal Hb of 12g/dl, WBC 11,000/uL, and platelets 346,000/uL. Metabolic panel is normal. Chest X ray is clear and EKG normal. Fluid resuscitation is started and emergency GI consult placed. He is taken by the gastroenterologist for upper endoscopy. What condition would an endoscopy confirm?

Correct answer: Mallory-Weiss syndrome Explanation Mallory-Weiss syndrome is a mucosal tear in the esophagus that occurs due to forceful retching and increased intra-abdominal pressure. It can occur at any age and is common in alcoholics and binge drinkers. It can, however, occur after any event that can cause an increase in the intra-abdominal pressure or gastric prolapse into the esophagus. Patients usually, but not always, give a history of non-bloody vomiting or retching before the onset of hematemesis. The amount of blood loss is usually small and self-limited. However, massive hemorrhage requiring transfusions, and even death, can occur. Endoscopy is indicated for diagnostic as well as therapeutic purposes. The mucosal tear can be visualized within or near the gastroesophageal junction with or without an actively bleeding vessel, an adherent clot, or a fibrin crust. Surgery is needed in less than 10% cases. Most tears heal spontaneously. Patients may need injection with epinephrine to stop the bleeding. Epinephrine causes vasoconstriction and edema; it therefore decreases arterial blood flow. Electrocoagulation is also popular, effective, inexpensive, and safe. It is contraindicated in portal hypertension and esophageal varices, since it may precipitate or worsen bleeding. Sclerosing agents, hemeclipping, and angiotherapy are other modalities of treatment. Usually they are discharged within 24 hours. Boerhaave's syndrome is a rupture of the esophagus, usually above the esophagogastric junction, that can occur spontaneously due to increased intra-esophageal pressure, stricture dilatation, pill esophagitis, caustic ingestion, etc. Chest X-ray most commonly shows a left-sided pleural effusion and may show air in the left mediastinum (unlike this patient's CXR). Immediate surgical intervention is necessary for survival. Zenker's diverticulum is a mucosal herniation above the cricopharyngeal level. It is not a true diverticulum and can cause obstructive symptoms. It is usually seen in older men with chronic symptoms for years including regurgitation of food into the mouth, pulmonary aspiration, gurgling in the throat, or appearance of a mass in the neck. It is not usually associated with hematemesis, as in this patient. Surgery is needed for large diverticula. Dieulafoy's ulcer occurs due to an abnormal vascular lesion in the gastric fundus. There is absence of the normal decrease in the caliber of the submucosal vessel as it passes through the layers of the stomach, and this dilated vessel erodes through the mucosa to cause recurrent GI bleed, usually in men older than 50. It is uncommon and difficult to diagnose until the bleeding vessel is visualized. Treatment is endoscopic hemostasis or occasionally surgery. This patient's age and single episode of bloody emesis goes against the diagnosis. Esophageal varices are seen in patients with portal hypertension, most commonly due to cirrhosis of liver. Bleeding can be catastrophic and fatal. Treatment includes endoscopic hemostasis as well as therapy for the underlying condition. This patient does not have signs and symptoms or supportive lab data to diagnose cirrhosis and portal hypertension.

Case A 55-year-old Caucasian man presents to the emergency department vomiting bright red blood. He tells you he is not taking any prescription or over-the-counter medications. He does not use herbs and has no known drug allergies He does drink alcohol on a daily basis; he consumes a 6-pack of beer on weekdays after work and a fifth of whiskey on the weekends. He does not smoke. The client denies weight loss. His weight today is 190 pounds at 5 foot 11 inches. He tells you that he has vomited blood at least twice before and it stopped on its own. This time he got scared because it was not stopping. It started when he choked on his drink and started to retch. He denies any injuries or trauma. He lives alone and has no close friends. An upper endoscopy is performed when the client is stabilized. It shows a 0.5-4 cm linear mucosal tear located just below the esophageal junction. Question What would be the most likely cause of the frank bleeding?

Correct answer: Mallory-Weiss syndrome Explanation Mallory-Weiss syndrome is a non-penetrating mucosal tear at the gastroesophageal junction. The syndrome is brought on by events that suddenly raise trans-abdominal pressure, such as lifting, retching, or vomiting, ulcers, malrotation, volvulus, or inflammatory conditions of the stomach and intestine. Any disorder that brings on vomiting may result in the development of Mallory-Weiss tears. The disorder develops as a linear laceration at the gastroesophageal junction. The round shape makes longitudinal tears occur more easily. Alcoholism is a strong predisposing factor for Mallory-Weiss syndrome. Hiatus hernia is seen in 35-100% of the patients. Mallory-Weiss tears appear most often in the 50 to 60-year-old client. The tears can occur also in children. It should be noted that Mallory-Weiss tears occur equally in men and women; however, in women of childbearing age, pregnancy should always be ruled out as the cause, due to hyperemesis gravidarum in the first trimester. Esophageal neoplasm comprises less than 60% of squamous cell carcinomas arising from the surface epithelium, most commonly in the middle and lower esophagus. Adenocarcinomas, which constitute 35%, arise from the gastric fundus and develop in the lower third of the esophageal tract. This cancer is most common in older adults, with African Americans affected 3 times as often as Caucasians, and men 3 times as often as women. On endoscopy, the neoplasm would be visible as a mass rather than a tear. Risks factors: Smoking and tobacco use (chewing) Alcohol abuse Drug abuse (e.g., morphine, opium) Malnutrition Environmental carcinogens (e.g., Nitrosamines, silica, fungi) History of cancer of the larynx or pharynx History of chronic inflammation of the esophagus, achalasia (failure of esophageal sphincter to relax), tylosis, or caustic burns to the esophagus. Dysphagia is the most common presenting symptom. Regurgitation and weight loss may also occur. Esophageal varices are enlarged veins that can be caused by portal hypertension (elevated portal pressure beyond 10 mmHg) in association with cirrhosis, liver parenchymal disease, duodenal ulcer, or acute pancreatitis, as well as Budd Chiari syndrome, and infections such as schistosomiasis. Bleeding esophageal varices eventually develop in about 50% of individuals with cirrhosis. Portal veins narrow and become obstructed as a result of the underlying disease process. As the lumen narrows, the venous blood returning to the right atrium from the intestine and spleen seeks new routes through collateral vessels. These collateral vessels enlarge and become tortuous, and the mucosa ulcerates. With acute bleeding, the mortality rate is about 50%. Approximately 60% of individuals die within a year of the first episode of bleeding. On endoscopy, these enlarged veins are visible on the mucosal surface; they are not tears in the mucosa. Esophageal stricture formation occurs in about 10% of clients with esophagitis. It is manifested by the gradual development of solid food dysphagia, progressive over months to years. Often there is a reduction in heartburn because the stricture acts as a barrier to reflux. Most strictures are located at the gastroesophageal junction. Endoscopy may reveal a passage that is difficult to get through with the endoscope. In gastroesophageal reflux disease the essential features are: Heartburn; may be exacerbated by meals, bending, or recumbency. Clinical diagnosis; typical uncomplicated cases do not require diagnostic studies. Endoscopy demonstrates abnormalities, such as erythema and friability of the squamocolumnar junction in <50% of patients. Barium esophagography is seldom helpful. The typical symptom is heartburn. This most often occurs 30-60 minutes after meals and upon reclining. Certain foods or beverages like coffee or alcohol, drugs like calcium channel blockers or beta blockers, and obesity are contributing factors. Patients often report relief from taking antacids or baking soda. When this symptom is dominant, the diagnosis is established with a high degree of reliability. Overall, a clinical diagnosis of gastroesophageal reflux has a sensitivity of 80% but a specificity of only 70%. Severity is not correlated with the degree of tissue damage. Clinical Manifestations: Early heartburn (particularly with spicy or fatty meals, exercise, and recumbent positions) that is relieved by antacids. Midcourse heartburn accompanied by high epigastric and substernal pain, regurgitation, and dysphagia. Late Bleeding, dysphagia, disappearance of heartburn, 50% patients develop esophagitis. Endoscopy reveals mucosal irritation and sometimes atypia but not a tear as in Mallory-Weiss syndrome.

Question A 1-year-old boy presents with a 1-week history of bloody diapers. The child has been crying almost constantly. On clinical exam, there is abdominal tenderness. Guaiac test is positive, hematocrit is 39.0, and hemoglobin is 13.0. What is the most likely diagnosis?

Correct answer: Meckel's diverticulum Explanation The Meckel diverticulum appears to be a remnant of developmental structures that were not fully reabsorbed. It is a common congenital abnormality that consists of a small pouch called a diverticulum located off the wall of the small bowel. Symptoms generally occur during the 1st few years of life. Symptoms include passing of blood either with or without stool and abdominal discomfort ranging from mild to severe. Tests should include stool smear for occult blood (stool guaiac), hematocrit, hemoglobin, and technetium scan to demonstrate diverticulum. Surgery to remove the diverticulum is recommended if bleeding develops. Iron replacement may be needed to correct anemia. If bleeding is significant, blood transfusion may be necessary. Diverticulitis is inflammation of an abnormal pouch (diverticulum) in the intestinal wall, usually found in the large intestine (colon). Small protruding sacs of the inner lining of the intestine (diverticulosis) may occur in any part of the intestine. They occur with increasing frequency after the age of 40. Diverticulitis is an inflammatory condition where gross or microscopic perforation (hole) of the diverticula has occurred. A low-fiber diet may be a contributing factor to the development of diverticula. Symptoms include left lower abdominal pain, constipation or diarrhea, chills, fever, swallowing difficulty, stools (clay colored or bloody), nausea and vomiting, heartburn, cough, and breath odor. Tests should include colonoscopy, sigmoidoscopy, barium enema, rectal examination (showing bleeding), abdominal palpation (showing left lower quadrant mass), and stool hemoccult test (revealing blood). Treatment should include increasing the bulk in the diet with high-fiber foods and bulk additives. Peutz-Jeghers syndrome is transmitted as an autosomal dominant trait. Pigmented spots, brownish or bluish gray, develop from infancy through childhood around the lips, gums, and mucus membranes in the mouth. Symptoms include crampy abdominal pain, vomiting, occasional gross blood in the stool, and intussusception (a telescoping of one portion of the intestine into another). Intestinal polyps also develop that can be detected with special studies. Diagnostic tests include X-ray of abdomen (shows polyposis), occult blood in stool, CBC, serum iron, serum, total iron binding capacity, and biopsy of polyps. Treatment includes surgery to remove polyps that cause chronic problems. Iron replacement therapy (iron supplements) helps counteract blood loss. Periodic studies are recommended to watch for malignant changes in polyps. Crohn's disease (also called ileitis or enteritis) causes inflammation in the small intestine. The most common symptoms of Crohn's disease are abdominal pain (often in the lower right area) and diarrhea. Rectal bleeding, weight loss, decreased appetite, and fever may also occur. Bleeding may be serious and persistent, leading to anemia. Tests should include CBC (increased white count), upper GI, and colonoscopy. The goals of treatment are to control inflammation, relieve symptoms, and correct nutritional deficiencies. Surgery is indicated to relieve chronic symptoms that do not respond to treatment. Ulcerative colitis is chronic, episodic, inflammatory disease of the large intestine and rectum characterized by bloody diarrhea. Ulcers form in the inner lining, or mucosa, of the colon or rectum, often resulting in diarrhea, blood, and pus. The inflammation is usually most severe in the sigmoid and rectum and usually diminishes higher in the colon. It may affect any age group, although there are peaks at ages 15 to 30 and then again at ages 50 to 70. Symptoms include diarrhea of between 10 and 25 times a day (in which blood and pus may be present), abdominal pain and cramping that usually subsides after a bowel movement, abdominal sounds (borborygmus, a gurgling or splashing sound heard over the intestine), fever, weight loss, stools (foul smelling), and tenesmus (pain while passing stool). Tests include colonoscopy with biopsy, and barium enema. The goals of treatment are to control the acute attacks and prevent recurrent attacks. Corticosteroids are prescribed to reduce inflammation. Sulfasalazine may decrease the frequency of attacks. Surgery may be indicated in refractory disease.

Case A 52-year-old man presents with a 3-day history of persistent diarrhea, having up to 7 watery bowel movements without bleeding a day. He has associated lower-abdominal cramping and notes mild nausea. He denies travel outside of the country. He does not recall eating anything unusual, and none of his family members are sick. Past medical history is significant for GERD, for which he takes protonix daily. He recently had an upper respiratory infection and completed a course of oral levofloxacin approximately 3 weeks ago. A stool sample is obtained. Stool is negative for ova and parasites, but PCR testing is positive for Clostridium difficile. Question What is the appropriate first-line treatment in this patient?

Correct answer: Metronidazole Explanation Metronidazole is correct. The patient is infected with Clostridium difficile, a Gram-positive, spore-forming bacillus. It is manifested by watery diarrhea, which left untreated can progress to fulminant colitis. Previous antibiotic use is the leading cause of Clostridium difficile, as the use of antibiotics alters the normal colonic flora, allowing Clostridium difficile to proliferate. Patients who have confirmed infection should first discontinue any offending antibiotics. The patient is then treated with either oral metronidazole or oral vancomycin. Due to its cost, oral vancomycin is typically reserved for severe or recurrent cases of Clostridium difficile. Loperamide and lomotil are incorrect. These medications are both anti-motility agents. Their use is not recommended in the treatment of Clostridium difficile, as anti-motility agents have been associated with the development of toxic megacolon and systemic infection. Prednisone is incorrect. While corticosteroids have been used in severe Clostridium difficile colitis as adjunctive treatment to reduce inflammation, their use is not recommended for first-line treatment. In order to clear Clostridium difficile, the patient must undergo treatment with either metronidazole or vancomycin. Clindamycin is incorrect. Clindamycin is not active against Clostridium difficile. In fact, its use has been associated with development of Clostridium difficile.

Case A young woman presents with pneumonia; a course of cephalosporins is given to treat this condition. A few days after starting the treatment, she develops a non-bloody, watery diarrhea with abdominal cramps. She returns to the doctor's office. Her stool is sent to the lab, and a colonoscopy is performed to find out the cause of this diarrhea. The colonoscopy shows yellow-white plaques and membranes on colonic mucosa. Question What antibiotic best treats this condition?

Correct answer: Metronidazole Explanation The correct response is metronidazole. This young woman is presenting with a typical presentation of antibiotic-associated pseudomembranous colitis. The cause of this condition is Clostridium difficile, which is a gram-positive spore-forming rod. It is found in the gastrointestinal tract in approximately 3% of the general population. Antibiotics (e.g., clindamycin, ampicillin, cephalosporins) suppress drug-sensitive members of the normal flora, allowing C. difficile to multiply and produce cytopathic toxins called cytotoxin A and B. Initially, the treatment is to withdraw the causing antibiotic; if the diarrhea is severe and patient is ill, treatment with metronidazole is recommended. Vancomycin can be used as well, but it is not preferred over metronidazole because of the risk of developing resistant enterococci. Cytotoxin A is an enterotoxin causing an outpouring of fluid resulting in watery diarrhea. Cytotoxin B causes damage to the colonic mucosa, leading to the pseudomembranous formation. This is a heat and acid-labile protein, and it attaches to the intestinal epithelial cells at the microvilli level; it causes cell death. Clindamycin, ampicillin, and cephalosporins can cause pseudomembranous colitis, so they cannot be used for treatment. Tetracycline and trimethoprim-sulfamethoxazole do not have any effect on Clostridium difficile.

Case A 31-year-old man presents with burning substernal chest pain following meals. He is not experiencing nausea or vomiting. A stool sample is negative for occult blood. An upper endoscopy reveals some erythema of the lower esophagus. Esophageal biopsies from this area demonstrate neutrophils infiltrating a hyperplastic squamous epithelium. Question This biopsy finding by itself is most consistent with what?

Correct answer: Mild reflux esophagitis Explanation The findings in this patient are consistent with acute esophagitis. If no other findings are present, the most likely cause is reflux esophagitis. Barrett's esophagus is characterized by the appearance of columnar metaplasia with chronic reflux esophagitis. There is little inflammation with scleroderma, only mucosal and submucosal fibrosis. A 'punched out' ulcer of the esophagus is characteristic for herpes, but the appearance of cells with intranuclear inclusions and multinucleation helps with diagnosis. Atresia is uncommon (about 1 in 1000 live births), and it leads to problems with feeding in neonates.

Question A person deficient in the enzyme lactase would have a problem if they consumed what?

Correct answer: Milk Explanation The surface of the small intestine has enzymes, intestinal disaccharidases, which hydrolyze disaccharides to monosaccharides that can then be absorbed. Lactase is one of these enzymes and hydrolyzes the disaccharide lactose to glucose and galactose. Lactose is the sugar found in milk products. In the absence of the enzyme lactase, lactose cannot be hydrolyzed and absorbed in the upper small intestine. The bacteria in the lower small intestine ferment lactose, resulting in gas production. In addition, water is drawn into the intestinal lumen because of the osmotically active solutes. This leads to diarrhea. Commercial sources of lactase are now available; it can be added to milk to pre-digest the lactose. During the process of making yogurt, the lactose present is partially hydrolyzed; therefore, patients lacking lactase often do not have problems eating yogurt.

Case A 60-year-old man presents with a 6-month history of pain in his knee joints. It has been gradually increasing, and now limits his activities. X-ray reveals osteoarthritic changes in both knee joints. You consider treating him with a non-steroidal anti-inflammatory drug (NSAID); however, you note that he has a history of gastric ulcers. In the past, the patient has not tolerated proton pump inhibitors well; he has had several episodes of severe headache, diarrhea, and dizziness when treated with them. Question What is the best way to reduce the risk of NSAID-induced gastric mucosal injury in this patient?

Correct answer: Misoprostol Explanation Misoprostol is a prostaglandin analogue that acts on the gastric parietal cells to reduce gastric acid secretions. Proton pump inhibitors (PPI) are better tolerated than misoprostol and may be preferred over misoprostol, even in NSAID-induced gastric injury. Ideally, this patient should be treated with a selective COX-2 inhibitor NSAID with a PPI; however, considering his poor tolerance to PPI (which is rare), misoprostol is the next best choice. Misoprostol causes nausea, diarrhea, abdominal pain, and dysmenorrhea. It should be avoided in pregnancy, as it may cause an abortion. Milk of magnesia is an antacid that acts by neutralizing the acid in the stomach; it is commonly used in dyspepsia. Octreotide is a somatostatin analogue that inhibits gastric and pancreatic secretion. It is used in Zollinger-Ellison syndrome. H2 receptor blockers (e.g., ranitidine) are used to treat gastroesophageal reflux disease, as well as gastric, duodenal, and stress ulcers; however, they are not thefirst choice in NSAID-induced injury because a double dose would be required. Sucralfate is an ulcer protective; it forms a coating on the ulcer crater and acts as a barrier to acid and pepsin.

Case A 62-year-old man passed a foul-smelling tarry stool 1 week ago. 3 weeks ago, he began taking diclofenac because of worsening osteoarthritis. His past medical history includes primary hypertension, coronary heart disease, and 2 episodes of venous thrombosis in the calf. Besides diclofenac, his medications include warfarin, enalapril, low-dose aspirin, propranolol, and sublingual nitroglycerin. A screening colonoscopy performed 2 weeks ago was entirely normal. Vital signs are stable, and the physical examination is unremarkable. Upper endoscopy shows only pangastritis and erosions. Question What is the most effective way of reducing his risk of rebleeding?

Correct answer: Misoprostol Explanation The correct response is misoprostol. Besides NSAID use, this patient has several risk factors for gastric bleeding (a previous episode, advanced age, use of anticoagulants and low-dose aspirin). Therefore, prophylaxis against bleeding must be seriously considered. Prostaglandin analogs are the most effective drugs for reducing the risk of rebleeding. One study described reductions of up to fivefold in the incidence of gastric ulcers on endoscopy, but this drug requires 2-4 daily doses and has several side effects. The most common are diarrhea (up to 30% of patients) and abdominal discomfort. It is also a potent abortifacient. Starting the drug at a low dose and then progressively raising it might reduce the incidence of these side effects and improve tolerance. Proton-pump inhibitors such as omeprazole also reduce the incidence of gastrointestinal complications in chronic users of NSAIDs. They are slightly less effective than prostaglandin analogs, but their once-daily dosage is more convenient and the incidence of side effects is lower. The use of low-dose aspirin abrogates the protective effects of COX-2-selective anti-inflammatory drugs (celecoxib, rofecoxib, or valdecoxib). Therefore, switching to COX-2 inhibitors would not be advantageous for this patient. H2-blockers such as famotidine have only marginal efficacy in reducing the incidence of NSAID-associated gastric ulcers and should not be prescribed in this case. The medications that should be taken with at least 250 mL of water are the bisphosphonates. This precaution reduces the risk of pill esophagitis. This drug should also be taken on an empty stomach and with the patient in an upright position. If these precautions are not followed, bisphosphonates can cause severe esophagitis, which can even lead to strictures or bleeding.

Case A 20-year-old female college student presents due to a 7-day history of daily heartburn. She has never experienced heartburn as bad as she does currently. She is treating this with over-the-counter histamine-2 receptor antagonist, famotidine, 1 tablet daily. The patient denies any other significant past medical history and is currently taking a daily multivitamin and an antihistamine for seasonal allergies. She recently admits to having increased episodes of headaches that she believes are due to stress. For this reason, she has been taking ibuprofen 600 mg every 8 hours. She states that she has been taking this consistently every 8 hours for the last 10 days; she hopes that after finals are finished, her headaches will subside. She also states that she was given amoxicillin 2 weeks ago for a middle ear infection, which resolved without any further intervention. She denies any difficulty swallowing, weight loss, night sweats, chest pain, black tarry stool, use of tobacco/alcohol, or coughing up blood. Physical examination is unremarkable for any abnormalities. Question Which of the following medications that the patient mentioned is most likely causing her increased gastrointestinal symptoms?

Correct answer: NSAID Explanation NSAID is the correct response. Patients classified as having mild or intermittent symptoms of gastroesophageal reflux disease (GERD) typically are seen as not adversely affecting the patient's quality of life. Initial action includes modification of behaviors, such as eating smaller meals and eliminating acidic foods or known foods that precipitate the reflux (fatty foods, alcohol, chocolate, or peppermint). Weight loss has also been shown to help decrease issues, as well as advising patients to avoid lying down at least 3 hours after eating meals. Elevation of the head of the bed is also suggested to initiate relief. The next step in terms of treatment would be to discontinue any medications that may be increasing the symptoms of GERD. Medications that potentially irritate the esophagus and cause gastritis symptoms include: tetracycline, bisphosphonates, iron supplements, Non-Steroidal Anti-inflammatories (NSAIDs), and potassium supplements. Medications that increase acid reflux and worsen the condition of GERD include: anticholinergics, calcium channel blockers, narcotics, progesterone, quinidine, benzodiazepines, or even theophylline.

Question A 65-year-old female is brought to the ER after she complained of sudden onset of dizziness early this morning when she was in the bathroom and fell to the floor, hitting her head. She had 3 episodes of dark semi-solid stool since last night but no nausea or vomiting. She has a history of hypertension and chronic osteoarthritis, and she takes amlodipine 10 mg daily as well as ibuprofen 400 mg 3-4 times daily as needed for the last 4-5 years. She does not smoke or drink. Her family history is insignificant. On exam she is afebrile, BP is 90/52 mm Hg, pulse is 112/min, and SPO2 is 92%. She looks pale and tired. Her lungs are clear, and she has some epigastric tenderness. Chronic arthritic changes are noted in her knees and fingers. Neurologically she is awake but somewhat lethargic and fully oriented. No other deficit is found. Labs show Hb 8.2 g/dl, WBC 12,000/uL and platelets 350,000/uL. Chest X ray is clear, EKG shows sinus tachycardia, and cardiac enzymes are normal. Metabolic panel is within normal limits, except BUN being 32. The likely diagnosis is

Correct answer: NSAID-induced peptic ulcer Explanation This patient is using NSAIDs chronically for arthritis without any protective therapy with proton pump inhibitors or H2 receptor blockers. She has developed a bleeding peptic ulcer, as evidenced by melena, which can be catastrophic in the elderly. The patient now has anemia due to acute GI bleed with hypotension and hypovolemia. Therapy includes vigorous fluid resuscitation, intravenous proton pump inhibitor, and life-saving blood transfusion followed by urgent upper endoscopy. Long term follow-up includes indefinite proton pump inhibitor therapy and acetaminophen for arthritis. Even though the patient has leukocytosis, there is no obvious focus of infection or fever. This can occur in response to the acute bleeding. Amlodipine will not cause melena and in supervised treatment of hypertension, it is unlikely to cause such a severe hypotension. AV malformation usually presents with lower GI bleed, not melena. It may be a chronic slow bleed causing iron deficiency in the elderly.

Question A 2-week-old female infant presents as a post treatment for yellow skin and eyes and poor eating. Symptoms began when she was 3 days old. A CBC showed red blood cell destruction while a second test of blood work revealed elevated serum bilirubin. After 2 days of phototherapy and hydration treatment, the symptoms resolved. What is the most likely diagnosis?

Correct answer: Newborn jaundice Explanation Newborn jaundice is a condition of yellowish skin color during the newborn period. This results from the immaturity of liver function combined with the destruction of red blood cells present in the newborn infant. The jaundice usually appears between the 2nd and 5th days of life and clears by 2 weeks. Symptoms include yellow color of the skin (jaundice), poor feeding (may be present), and lethargy (may be present). An elevated serum bilirubin level will be present. Usually treatment is unnecessary. Sometimes artificial lights (called bili lights) are used on infants whose levels are very high, or on premature infants. The jaundice resolves without treatment within 1 to 2 weeks. Crigler-Najjar syndrome is inherited as an autosomal recessive trait. Infants who inherit the trait from both parents (this is called homozygous inheritance) develop severe jaundice (hyperbilirubinemia) a few days after birth. Symptoms include yellow skin and eyes (jaundice) that begins on the 2nd or 3rd day of life and progressively worsen, jaundice that persists beyond 2 weeks without an obvious cause, and confusion (resulting from brain damage). Liver transplantation may be considered for some people with this disorder. Breast milk jaundice is a persistent jaundice in the newborn caused by a hormone found in breast milk. Bilirubin is eliminated by chemically hooking a sugar molecule to the bilirubin molecule, which then makes the bilirubin water-soluble. In its water-soluble form, bilirubin can be excreted in the urine. Breast milk contains a hormone, pregnanediol, which interferes with the body's ability to hook the sugar onto the bilirubin. Symptoms include jaundice in a breast-fed newborn that lasts longer than a week. Tests should include a blood test for bilirubin and a breast milk pregnanediol assay (seldom); bilirubin conjugation (the process of hooking the sugar molecule onto bilirubin) will be high. Cessation of nursing for 24 to 48 hours will result in a rapid drop of bilirubin. Gilbert's syndrome is a multifactorial inherited disorder that affects the way bilirubin is processed by the liver and causes jaundice. The symptom of jaundice appears under conditions of exertion, stress, fasting, and infections. The condition is usually benign. Symptoms include mild jaundice, fatigue, nausea, and abdominal pain in the upperright quadrant. A serial serum indirect bilirubin shows changes consistent with Gilbert's disease. Dubin-Johnson syndrome is an inherited disorder characterized by mild jaundice throughout life. The transport of bilirubin from the liver into the biliary system is abnormal, and the bilirubin accumulates in the liver. Affected people have lifelong low-grade jaundice that may be aggravated by alcohol, pregnancy, infection, and other environmental factors. The symptoms include a mild jaundice, which may not appear until puberty or adulthood.

Question Pellagra, known by many medical students to produce the "4 D's" (diarrhea, dermatitis, dementia, and death), is caused by what deficiency?

Correct answer: Niacin Explanation Niacin deficiency has been linked to pellagra. This condition, now quite rare, consists of nausea with vomiting, diarrhea, dermatitis, and cognitive decline (such as irritability and memory loss). Vitamin B6 (pyridoxine) deficiency and excess has been associated with neurological disease. Both lead to a distal symmetrical polyneuropathy. A deficiency of B6 is commonly found in patients taking INH, an anti-tuberculosis drug. INH inhibits pyridoxine phosphorylation, thereby decreasing the concentration of the active compound-pyridoxal phosphate. Excessive use of B6 leads to an excess of pyridoxine, which competes with pyridoxal phosphate for binding sites on the enzyme. Vitamin B12 deficiency causes both a peripheral neuropathy and a degeneration in the spinal cord known as subacute combined degeneration. This affects the dorsal column sensory functions (position sense and vibration) and the lateral corticospinal tract. The resulting symptoms include paresthesias of the feet and hands with subsequent weakness and stiffness of the legs and a spastic gait. Vitamin B1 (Thiamine) deficiency has been linked to the neurological syndrome of Wernicke's encephalopathy. This condition, although often associated with alcoholism, is a true nutritional deficiency, causing neuropathological lesions in areas such as the thalamus, mammillary bodies, and the periaqueductal areas. A polyneuropathy has also been linked to Vitamin B1 deficiency, especially in alcoholism. This deficiency plays at least a role in the development of this disease, along with the possible neurotoxic role of alcohol. Vitamin E deficiency occurs usually with malabsorption of the fat-soluble vitamins A, D, E, and K. This can occur with conditions such as cholestatic liver disease or celiac disease. Neurological symptoms include a peripheral neuropathy and spinocerebellar degeneration. The spinocerebellar degeneration can occur as a defect in abetalipoproteins, as occurs in Bassen-Kornzweig syndrome. This is also associated with acanthocytes and retinitis pigmentosa.

Case A 37-year-old man presents with diarrhea, nausea, vomiting, and a rash. He also complains of memory problems and depression. He states that he and his family live on a farm that produces corn. Due to recent financial problems, his diet has become more limited. He eats mainly corn products and only small amounts of fish and vegetables. He takes no vitamin supplementation, and he states that his symptoms began about 1 month ago. He began having trouble with his memory about 1 week ago. He has no significant past medical history, but he does admit to daily alcohol consumption. His physical examination is significant for stomatitis, glossitis, and a rash over his arms and legs. His mini-mental status exam score is 24. Question What deficiency does this patient most likely have?

Correct answer: Niacin Explanation This patient most likely has niacin deficiency. Niacin (nicotinamide, nicotinic acid) deficiency is uncommon in the United States. It is often found in people who live on a diet that consists mainly of corn (maize). This is due to the fact that the niacin in corn cannot be absorbed unless it is chemically treated with alkali first. If a person consumes a diet rich in tryptophan but low in niacin, they are able to compensate since tryptophan can be converted into niacin. Deficiency may also result from alcoholism, cirrhosis, or diarrhea. Men and women are affected equally. Symptoms of niacin deficiency include nausea, vomiting, diarrhea, rash, glossitis, stomatitis, depression, and psychosis. Niacin deficiency, also known as pellagra, manifests as the "3 Ds": diarrhea, dermatitis, and dementia. Treatment consists of niacin supplementation. The causes of zinc deficiency include malnutrition, chronic debilitating diseases, chronic renal disease, alcoholism, drugs such as penicillamine and diuretic, and genetic disorders such as sickle cell disease. Clinical manifestations in severe cases include alopecia, diarrhea, weight loss, infections, dermatitis, hypogonadism in men, and intercurrent infections. Supplementation with zinc is the treatment of choice. Folate deficiency causes megaloblastic macrocytic anemia, as folate plays a key role in nucleic acid synthesis. The early manifestation of folate deficiency, especially in its suboptimal state, predisposes to occlusive vascular disease and thrombosis. These manifestations are linked to increased homocysteine levels found in folate deficiency. Neurological disturbances, such as mood disturbance and spinal cord syndromes, are also seen. It is also associated with predisposition to neoplasia and interferes with immunologic status. Folate replacement is the option to prevent and to treat the deficiency. Vitamin A deficiency occurs because of inadequate dietary intake of preformed retinol in fish oils, liver, and vitamin A-fortified foods. Beta-carotene, the vitamin A precursor, is abundant in carrots, yellow fruits and vegetables, and in dark green leafy vegetables. Deficiency occurs because of inadequate dietary intake and is commonly seen in hospitalized patients suffering from fat malabsorption. Vitamin A deficiency is characterized by night blindness, xerophthalmia, keratomalacia (dry ulcerated corneas) as well as hyperkeratosis of the skin. Treatment consists of vitamin A supplementation. Excess intakes 10x the RDA have been associated with liver toxicity, hypercalcemia, anorexia, and fetal anomalies. Vitamin B1 (thiamine) deficiency causes beriberi, occurring mostly in malnourished and alcoholic patients. The deficiency manifests with acute heart failure, neurologic deficits, and epilepsy. Empiric use of thiamine and prophylactics in high-risk population is strongly recommended even before blood reports are obtained, as the treatment is inexpensive and prevents major catastrophes.

Case A 30-year-old man presents with a 1-week history of a painful tongue. He has been having diarrhea and forgetting things more easily. He recently emigrated from India and lives alone on a maize-based diet. On examination, you note that the tip and margins of his tongue are a bright scarlet color. He also has a symmetrical erythematous rash on his forearms. Question What is the most likely diagnosis?

Correct answer: Niacin deficiency Explanation Niacin deficiency causes pellagra, which is characterized by: Symmetrical dermatitis, usually on parts of the body exposed to sunlight Scarlet glossitis and stomatitis Diarrhea Mental aberrations, such as memory impairment, depression, and dementia. These may appear alone or in combination. Causes include inadequate dietary intake, especially in patients with corn-based diets or alcoholism. Dietary sources include legumes, yeast, meat, and enriched cereal products. In cases of vitamin A deficiency, patients can present with an inability to see well in dim light or night blindness. There may also be conjunctival and corneal xerosis, as well as pericorneal and corneal opacities, and Bitot's spots. Bitot's spots are a collection of keratin appearing as triangular foamy spots on the conjunctiva. The patient may also have xeroderma, hyperkeratotic skin lesions, and increased susceptibility to infections. Causes include inadequate dietary intake and malabsorption. Dietary sources of vitamin A include fish, liver, egg yolk, butter, cream, dark green leafy vegetables, as well as yellow fruits and vegetables. In cases of pyridoxine or (vitamin B6 deficiency), patients can present with peripheral neuropathy, seborrheic dermatosis, glossitis, and cheilosis. Laboratory investigations reveal anemia with lymphopenia. Causes include malabsorption as well as medications such as isoniazid and penicillamine. Dietary sources of vitamin B6 include liver, legumes, whole grain cereals, and meats. In cases of vitamin C (or ascorbic acid) deficiency, patients can present with bleeding tendencies (as a result of weakened capillaries) and impaired wound healing due to impaired formation of connective tissue. On examination, the gums may be swollen and friable; the teeth may be loose. There may also be multiple splinter hemorrhages on the nails and ecchymoses, especially over the lower limbs. Causes include inadequate dietary intake and certain conditions such as pregnancy and lactation that increase vitamin C requirements. Dietary sources of vitamin C include citrus fruits, such as oranges, lemons, and tangerines, as well as tomatoes and potatoes. Vitamin E deficiency may cause a hemolytic anemia in premature infants. Laboratory investigations reveal low plasma tocopherol levels, a low hemoglobin level, reticulocytosis, hyperbilirubinemia, and creatinuria. Causes of vitamin E deficiency in premature infants include the limited placental transfer of vitamin E and the resultant low levels at birth combined with its relative deficiency in the infant diet. Dietary sources for older children and adults include wheat germ, vegetable oils, egg yolk, and leafy vegetables.

There has been no vomiting. He has a recurrent knee sprain on the left, and he takes over the counter ibuprofen on a regular basis to relieve the pain. He has no other prior medical or surgical history. He keeps a large supply of famotidine (Pepcid AC) at his bedside. Social history is significant for moderate alcohol use and significant caffeine intake; he has a 30-pack per year smoking history. He currently smokes 1 pack daily. There is no significant family history. The patient denies diarrhea or constipation. The pain is somewhat postprandial, particularly after a heavy meal and alcohol. Physical examination reveals a man in no acute distress. Blood pressure is 140/90 mm Hg, and pulse is 75/min; temperature is 97.8 degrees Fahrenheit, and respiratory rate is 15/min. HEENT examination is within normal limits. Abdominal exam is difficult due to the patient's body habitus, but there is no obvious organomegaly. Bowel sounds are normal. There is no tenderness or distention, and Murphy's sign is negative. Laboratory values are performed: White blood cell count 8.0 per mcL Hemoglobin 14.0 g/dL Hematocrit 52.7 Sodium 141 mEq/L Potassium 4.5 mEq/L Creatinine 1.1 mg/dL AST (SGOT) 35 units/L ALT (SGPT) 37 units/L Alkaline phosphatase 94 IU/L Amylase 100 units/L Helicobacter Pylori Antibody - negative Upper GI series (barium swallow) test is suggestive of hiatal hernia, but there are no definite signs of reflux. An abdominal ultrasound is performed and the following findings are identified: normal common bile duct; mild fatty replacement of the liver. pancreas not well visualized. Question What is the pathophysiology most likely causing this patient's symptoms?

Correct answer: Nicotine and caffeine, causing decreased lower esophageal sphincter tone Explanation This patient presents with classic symptoms of gastroesophageal reflux (GERD), a condition where the lower esophageal sphincter tone is decreased. There are many risk factors for reflux. The weight of the pannus in an obese patient puts direct pressure on the stomach, increasing the risk of reflux. Nicotine, alcohol, and caffeine can decrease lower esophageal sphincter tone. Large meals and lying down tend to exacerbate the symptoms. Temporary relief can often be obtained through the use of H2 blockers, such as famotidine. Barium swallow has poor sensitivity in detecting reflux; therefore, a negative test does not rule out the diagnosis. Endoscopy could be performed if the diagnosis is uncertain. Gallbladder pain is often postprandial, but the abdominal ultrasound did not show any stones or dilatation of the common bile duct, so this diagnosis is less likely. Alcohol can cause pancreatitis, but the patient's amylase was normal, so this is unlikely. Esophageal spasm can be very painful, but a filling defect would most likely have been detected on the barium swallow. Fatty liver is not a painful condition, and it would not be relieved by antacids.

Case A 51-year-old woman presents with difficulty swallowing. She reports a 2-month history of problems swallowing liquids and solids. She also brings up undigested food. X-ray reveals a bird's beak appearance of the esophagus. Question What medication would be most appropriate in this patient?

Correct answer: Nifedipine Explanation Nifedipine is correct. This patient has achalasia, the absence of peristalsis in the lower esophagus with impaired relaxation of the lower esophageal sphincter. Achalasia causes a characteristic "bird's beak" appearance of the esophagus with distal narrowing and proximal dilation. Nifedipine, a calcium channel blocker, is used to relax the lower esophageal sphincter. Pantoprazole and famotidine are incorrect. Pantozprazole and famotidine are both used in the treatment of gastroesophageal reflux disease (GERD). Ciprofloxacin is incorrect. The patient does not have signs of bacterial infection. Nystatin is incorrect. Nystatin is used in the treatment of Candida esophagitis.

Case A 28-year-old man presents with rectal bleeding. The patient had noticed blood with bowel movements 3 times. The blood is described as bright red in color and small in amount. He also complains of rectal pain, especially with passing hard stools. He has tried some over-the-counter hemorrhoid creams without relief. The patient admits episodic constipation. He denies dark, tarry stools, easy bruising, and prior episodes of rectal bleeding. He has not noticed blood in his urine or with brushing his teeth. He denies nausea, vomiting, diarrhea, fevers, and weight loss. He has no known medical conditions. Family history is negative for gastrointestinal disorders. Social history reveals he is in a heterosexual relationship and denies anal intercourse. On physical exam, abdomen is normal. The anus has no visible protrusions or rash; however, there is a very small, erythematous and tender area that appears like a "paper cut" or crack in the skin. The patient experiences pain with digital rectal exam (DRE). No masses are noted in the rectal vault. Question What is the most appropriate prescription treatment for this patient's current condition?

Correct answer: Nitroglycerin ointment Explanation This patient most likely has an anal fissure. Fissures are small, painful lesions in which the skin has a "cut" or "torn" appearance. The fissure most likely occurred secondary to anal sphincter stretching and tearing with passing of large, firm bowel movements. Fissures can be associated with a small amount of bright red rectal bleeding. Intra-anal use of nitroglycerin ointment is a first-line treatment for anal fissures. The nitroglycerin improves perfusion of the surrounding tissue and lowers elevated resting pressures. Bacitracin ointment is a topical antimicrobial, used for mild, superficial skin infections and is widely available over-the-counter. It would not treat anal fissures. Minoxidil topical is a treatment used to stimulate hair growth for alopecia. Its use is not appropriate for this patient. Nystatin topical is an antifungal, commonly used for treatment of candida infections, such as thrush. It would not treat anal fissures. Tretinoin topical is a retinoid medication used for dermatologic conditions such as acne and wrinkles. It would not treat anal fissures.

Case A 51-year-old man presents with a 3-day history of severe pain associated with bowel movements. The pain lasts up to 1 hour following each bowel movement. His stools are described as "hard", but they retain their normal brown color, with the occasional presence of a few drops of bright red blood. His dietary history is remarkable for a low-fiber diet lacking in raw fruits and vegetables. He denies any sexual contact, drug use, or other gastrointestinal symptoms. He takes ferrous sulfate for anemia and oxycodone for chronic joint pain. His rectal exam reveals the following finding in the attached image. Question What is the most appropriate diagnostic test to order at this time?

Correct answer: No diagnostic tests are necessary Explanation This patient's diagnosis is a posterior anal fissure. Along with a history, the diagnosis can usually be made based on findings from a gentle perianal examination with inspection of the anal mucosa. In this case, no diagnostic procedures are required. A digital rectal examination is painful and often can be deferred. Anoscopy may be required for lesions that are not well-visualized; however, this is not well tolerated by a patient with an acute anal fissure. In this case, anoscopy can often be deferred; the patient can be treated based only on the symptoms. If an ordinary anal fissure is suspected, and if it is located in the posterior or anterior midline, then no laboratory or imaging tests are necessary. If the fissure is off the midline or irregular, or if an underlying illness (e.g., Crohn disease, squamous cell cancer, AIDS) may be present, then the appropriate tests should be ordered. These tests include erythrocyte sedimentation rate, stool and viral cultures, human immunodeficiency virus (HIV) testing, and biopsy of the lesion/fissure.

Case A 2-month-old infant who is breastfeeding presents with low hemoglobin levels. The infant was born at home and the mother received no prenatal care; she did not, and does not, take any medications. Family history is unremarkable. On examination, the infant appears healthy. Question What is the underlying mechanism of low hemoglobin levels in this infant?

Correct answer: Normal process Explanation Intrauterine hypoxia stimulates erythropoietin. Infants are born with high levels of hemoglobin and erythrocytes in their blood which downregulates erythropoietin. A progressive drop in hemoglobin during the 1st 2 - 3 months of life is the signal to the infant's body to increase erythrocyte production. Called physiologic anemia of infancy, this is considered normal and no treatment is needed. Folate deficiency in the mother would more likely result in low birth weight, prematurity, and/or neural tube defects. Vitamin K deficiency is the cause of hemorrhagic diathesis; its manifestations would be obvious. Breastfeeding when the mother did not, and does not, take any drugs cannot be the cause of anemia in a 2-month-old infant. Common infections in a 2-month-old infant include the digestive tract, ear, skin, and upper respiratory tract; all will have relevant symptoms.

Case A 35-year-old woman presents with watery diarrhea. It began 1 day after eating at a party. Her 6-year-old daughter ate the same food and is vomiting, but she does not have diarrhea. On examination, the woman is not dehydrated or febrile. A stool examination is negative for leukocytes and trophozoites. Question What is the causative organism?

Correct answer: Norwalk virus Explanation Norwalk virus can cause waterborne and foodborne outbreaks. The incubation period is 1-3 days. Patients present with watery diarrhea and vomiting. They may also report abdominal and muscle pains. In adults, diarrhea is usually more prominent than vomiting. In children, the reverse is true. On examination, they may be febrile. The stool examination is negative for leukocytes, ova, and trophozoites. The mainstay of therapy is hydration, as symptoms usually resolve within 2 days. Entamoeba histolytica colitis develops 2-6 weeks after ingesting contaminated water or food. Patients usually present with bloody and mucoid diarrhea. Stool examination reveals hematophagous trophozoites and cysts. Patients with enterotoxigenic Escherichia coli food poisoning present with watery diarrhea 12-72 hours after ingesting contaminated water or raw fruit salads. Examination of the stool reveals no fecal leukocytes or trophozoites. Patients with Shigella dysenteriae food poisoning present with bloody diarrhea; symptoms usually start 16 hours after ingesting contaminated foods like potato salad. A stool examination reveals polymorphonuclear leukocytes.

Case A 63-year-old woman presents with a 6-month history of worsening difficulty in swallowing. Shortly after swallowing, she feels like something is getting stuck in her upper chest, and the sensation lasts long enough to begin causing significant chest discomfort just behind her breastbone. The difficulty swallowing is often extremely variable and intermittent; it has not been progressive. It gets to the point that she feels like she is going to regurgitate her food, and she is also experiencing substantial episodes of acid reflux. The difficulty swallowing seems to be worsened by when the patient is extremely stressed and when she eats hot or cold food. She denies any weight loss, night sweats, or other significant issues. Physical examination of the patient is otherwise noncontributory. An extensive gastrointestinal evaluation, which included a comprehensive endoscopic evaluation, is negative. Question What is the first-line prescribed medication that would help alleviate this patient's signs and symptoms?

Correct answer: Oral proton pump inhibitor (PPI) Explanation The correct response is oral proton pump inhibitor. The patient in this presented scenario is showing signs and symptoms that are most consistent with esophageal dysphagia. Difficulty swallowing is the main complaint and is typically caused by 1 of 2 entities: localized neuromuscular disorders or obstructive lesions. Our patient specifically is showing issues of diffuse esophageal spasm. This is a motility disorder that is defined by simultaneous uncoordinated contraction of several esophageal segments. This can lead to potentially retrosternal pain and is worsened or precipitated by acid reflex, rapid eating, stress/anxiety, and hot/cold food. The dysphagia found in these patients is intermittent, non-progressive and does not cause weight loss. These patients have no documented abnormality in the distribution of myenteric neurons, normal lower esophageal sphincter relaxation, and no evidence of obstruction. All of these components are consistent with the patient described. In terms of a first-line treatment, acid suppression with a proton pump inhibitor (PPI) is the first-line pharmaceutical intervention that should be initiated. Sublingual nitroglycerin and calcium channel blockers may also be considered as treatment, but they are not typically considered first-line options. Botulinum toxin is not considered first-line treatment for diffuse esophageal spasms; it may be considered to be administered/injected in the lower esophageal sphincter and lower esophagus to reduce chest pain caused by diffuse esophageal spasm but again is not the first line treatment. Also, the patient described above is experiencing other symptoms, not just the chest pain. Prokinetic agents, such as metoclopramide, are considered treatment options for reflux esophagitis and Barrett esophagus-related causes of dysphagia. Oral corticosteroids may be considered in patients with diagnosed eosinophilic esophagitis.

Case A 35-year-old man presents with a groin mass. The patient states that the mass is painless; there is no known trauma to the region. The mass is present when the patient stands, and it disappears when the patient is lying flat. Past medical history is significant for obesity, hypertension, and hyperlipidemia. Past surgical history is significant for lipoma removal from the left shoulder. The patient denies tobacco use; he tells you that he drinks about 6 beers per week. In office ultrasound confirms diagnosis and surgical repair is scheduled. Question What is the most significant risk factor for this patient's condition?

Correct answer: Obesity Explanation The patient has an inguinal hernia. Hernias are more likely to occur in conditions in which there is excess pressure on the abdomen. Obesity increases abdominal pressure, so it increases the risk of hernia. Neither hypertension nor hyperlipidemia is associated with an increased risk of hernia development. While previous surgery to the abdomen may increase the risk of hernia, surgery on the shoulder (lipoma removal) would not. Tobacco use increases the risk for hernia, but alcohol use does not.

Case A 48-year-old Caucasian man presents with severe epigastric pain radiating to the back after a bout of drinking. Two weeks later, a repeat ultrasound showed a round thin-walled hypoechoic lesion near the pancreas tail measuring 4 cm in its largest diameter and with some calcifications in its walls. MRCP visualized a communication between this cavity and the pancreatic duct consistent with the diagnosis of pancreatic pseudocyst. Question What is the most appropriate next step in management?

Correct answer: Observation only Explanation Pancreatic pseudocysts complicate about 10% of attacks of acute pancreatitis. A pseudocyst is formed when pancreatic fluid leaks and is confined by organs adjacent to the pancreas. Eventually, a fibrous wall forms around the collection. Most cysts regress spontaneously over a period of several weeks, but in some cases, complications such as bleeding, abscess formation, and intractable pain may occur. Therefore, observation is the best approach for most patients. Several interventional approaches—surgical, radiologic, and endoscopic (ERCP and transpancreatic drainage or EUS-guided transgastric drainage)—may be attempted if spontaneous resolution does not occur or complications warrant; internal drainage procedures are preferred to external approaches. Indications for intervention are intractable pain, expanding lesions, and infection. Sphincteroplasty is used to treat scarring of the pancreatic duct sphincter or sphincter of Oddi, which usually occurs as a result of chronic pancreatitis. It is not a treatment for a pancreatic pseudocyst. CT angiography is useful if there is suspicion of a pancreatic pseudoaneurysm, in which case coil embolization may be considered. A Puestow procedure is a surgical procedure that involves the creation of a side-to-side anastomosis of the pancreatic duct and the jejunum. It is used sometimes as a treatment for chronic pancreatitis, but not for a pseudocyst.

Case A 45-year-old woman presents with yellow discoloration of the skin, clay-colored stool, and dark urine. She is obese but otherwise healthy. She has long-standing digestive troubles and an intolerance to fatty meals, for which she takes antispasmodic medications. On examination, there is mild tenderness in the upper right abdominal quadrant; serum bilirubin shows elevated total and conjugated types, as well as normal unconjugated bilirubin. Question What is the most likely cause of her jaundice?

Correct answer: Obstructive reasons Explanation With obstructive jaundice, bilirubin is formed and conjugated normally, but the process of biliary drainage is impaired; an increase in serum conjugated bilirubin is seen. This can be due to a stone or tumor in the bile ducts, liver, or pancreas. Jaundice (or icterus) is a yellowish discoloration of the skin and mucous membranes due to increased serum bilirubin. Jaundice can occur due to a deficit in the bilirubin clearance pathway. Normally, bilirubin is unconjugated in the blood; it undergoes conjugation in the liver and is secreted in the bile. Increased unconjugated serum bilirubin occurs due to increased hemolysis, and it is usually mild. When there is a defect in liver conjugation due to congenital or acquired reasons (e.g., hepatitis), conjugation decreases; this results in increased unconjugated serum bilirubin. In hepatitis, there is also some defective biliary drainage, so both unconjugated and conjugated bilirubin increases. There is also clay-colored stool, dark urine, and itching.

Case An 88-year-old male with abdominal pain presents to the emergency room with his wife. A mini-mental status exam reveals pronounced forgetfulness and confusion. The patient is discovered to have acute appendicitis requiring immediate surgery. He is unable to understand the situation and cannot provide informed consent. Question What further action should the physician take?

Correct answer: Obtain consent from his wife to perform surgery Explanation In cases in which an emergency exists, the patient is incompetent to give consent, and the withholding of treatment would be potentially life-threatening, the physician must seek out close relatives of the patient to supply consent. The physician should proceed with treatment, assuming the patient would want the treatment had he or she understood the situation. Not performing surgery could cost the patient's life. Having another doctor confirm the necessity of surgery is favorable (if done immediately), but not mandatory, and does not change the patient's consent status. Obtaining a court order is not necessary with the patient's wife immediately accessible. Trying to persuade the patient to consent to surgery would not only waste time and prove futile, but might agitate the patient as well.

Case A 66-year-old man with a history of obesity and hyperlipidemia presents with an 8-month history of progressing "burning in his chest". This sensation is noted in the midline of his chest, and it is provoked when he bends over, wears tight clothing, after he eats a large meal, and when lying supine. He denies any chest pressure, cough, shortness of breath, palpitations, dizziness, lightheadedness, and diaphoresis. His physical examination is unremarkable. An upper endoscopy is performed, which reveals the attached image. Question Of the choices below, what would be the most appropriate pharmacotherapeutic intervention for this patient?

Correct answer: Omeprazole Explanation The correct response is Omeprazole. This patient's presentation is most consistent with gastroesophageal reflux disease (GERD). Either proton pump inhibitors (PPIs), such as omeprazole, or histamine-2 receptor antagonist (H2RA) therapy, such as ranitidine, are recommended as the initial treatment for esophagitis associated with GERD, depending on the severity of symptoms and extent of esophagitis on endoscopy. If the patient only has symptoms a few times a week and no esophagitis present on endoscopy, H2RA treatment would be more appropriate initial treatment. If they have severe symptoms or significant esophagitis noted on endoscopy, omeprazole would be a more appropriate initial treatment. This patient is not experiencing fungal esophagitis (which is most commonly caused by Candida albicans). Medical therapy includes topically active agents, such as nystatin or clotrimazole. Oral amphotericin B, fluconazole, or itraconazole are also useful. Gastroprokinetic agents, such as metoclopramide or cisapride, and sucralfate, a coating agent, are less effective, but they may be useful in selected patients or as second-line agents. Cisapride is only available through an investigational limited-access program because of its potential for risk of serious cardiac arrhythmias and death.

Case A 49-year-old Caucasian woman presents with a 2-month history of worsening abdominal pain. The patient describes the pain as crampy, unrelenting abdominal pain that begins in the right upper and middle quadrant of the abdomen; very occasionally, it will go into right shoulder area. The pain usually occurs shortly after especially heavy meals, usually within 30 minutes; it then eventually subsides, although this takes anywhere from 60 - 120 minutes. Physical examination reveals significant right upper quadrant tenderness and rigidity with a palpable, firm mass. Ultrasound performed in the office reveals complete calcification of the gallbladder wall; there is a hyperechoic semilunar appearance with posterior acoustic shadowing. Question A symptomatic patient as described above with such severe calcification as seen on ultrasound would most likely be treated via what method?

Correct answer: Open cholecystectomy Explanation Patients with symptomatic porcelain gallbladders are much more commonly female than male, with the typical age range being from 38 - 70 years old. In general, patients will describe a history of biliary type pain. Diagnostically, an ultrasound or CT scan will most efficiently give the visualization of the calcification necessary to confirm a porcelain gallbladder. Those found to have complete mural calcification (complete type) generally will need to have it treated with an open cholecystectomy. This is because the mucosal calcification that creates the porcelain characteristic makes the gallbladder very thick and fibrotic, which in turn makes it potentially difficult to grab by forceps and dissect out in a laparoscopic approach. Endoscopic cholecystectomy is not an appropriate surgical procedure and would not be indicated for this patient's diagnosis. Anytime a porcelain gallbladder is found, there has to be a heightened alert for the suspicion of gallbladder carcinoma. The more advanced and severe the calcification is of the gallbladder, the heightened potential of it being malignant. Reassurance only and periodic ultrasounds would not be appropriate in this scenario. This approach may be followed in some part in a patient with less severe signs and symptoms. Percutaneous biliary stent placement is not the appropriate choice for this clinical scenario, as it is not treating the main pathology being presented. References Varadarajulu S, Z

Case A 32-year-old woman presents with a 2-week history of left lower quadrant abdominal pain while being treated with ampicillin for a urinary tract infection. The pain is accompanied by malaise and diarrhea in the form of 6-7 watery stools every day. A colonoscopy reveals the presence of an inflamed mucosa with the appearance of pseudomembranes and Clostridium difficile toxin (isolated from the patient's stools). The undertaken therapy—which consists of ampicillin withdrawal and the prescription of oral metronidazole in a dose of 250 mg qid for 10 days—results in the disappearance of the patient's symptoms and normalization in the number and consistency of the patient's stools. 2 days ago, metronidazole therapy was discontinued, and the patient now has symptoms in the form of 2 watery stools a day, again associated with mild discomfort in left iliac area. Physical examination reveals no abnormalities. Laboratory analysis points to the presence of an elevated erythrocyte sedimentation rate (45 mm/first hour), as well as the existence of Clostridium difficile toxin in the stool. Question What therapeutic approach should be recommended to the patient?

Correct answer: Orally administered metronidazole Explanation After discontinuation of treatment for Clostridium difficile infection, relapses may appear. Reappearance of diarrhea and other symptoms related to a Clostridium difficile infection, accompanied by the presence of Clostridium difficile toxin in the stool, indicates that a relapse is present. In the case of relapses, oral administration of metronidazole represents the therapy of choice if the patient has favorably responded to it in the past. Usually, patients who favorably respond to metronidazole during the initial attack of Clostridium difficile colitis also favorably respond when relapses appear, so re-utilization of metronidazole in this patient is justified. Orally administered vancomycin may be effective in the treatment of Clostridium difficile-induced colitis, but metronidazole is the preferred drug for the treatment of Clostridium difficile colitis at present. It is as effective as vancomycin, but much cheaper, so oral vancomycin is prescribed only in metronidazole-resistant cases. Intravenous administration of vancomycin is not utilized for the treatment of a Clostridium difficile infection; the intestinal concentration of vancomycin after its intravenous administration is low, and the bactericidal effect of this antibiotic on Clostridium difficile in such a case is lacking. Orally administered Lactobacillus has a suppressive effect on Clostridium difficile, but utilization of this type of bacteria is restricted only to patients who have had multiple (≥2) relapses of Clostridium difficile diarrhea despite the utilization of metronidazole or vancomycin. In such a case, concomitant administration of lactobacillus and an antibiotic is indicated. Intravenous administration of metronidazole may be effective in the treatment of Clostridium difficile-induced colitis, but such a therapeutic approach is less practical and is restricted only to patients who cannot tolerate oral metronidazole medication.

Question A 46-year-old woman presents with a 2-month history of heartburn, epigastric discomfort, nausea, and occasional vomiting. She has a history of hyperlipidemia, controlled with diet and exercise, as well as asthma, for which she takes inhalers as needed. She takes no other medications, including over-the-counter analgesics. Family history is noncontributory. On exam she is afebrile, BP120/70 mm Hg, pulse 74/min, and SPO2 92%. Lungs are clear, and she has minimal epigastric tenderness. Otherwise, physical examination is unremarkable. She is advised by her physician to take lansoprazole once daily, which provides only partial relief. Endoscopy is then recommended, which shows a duodenal ulcer. Biopsy reveals infection with Helicobacter pylori. What would be the recommended regimen at this time?

Correct answer: PPI (proton pump inhibitor), amoxicillin, and clarithromycin twice daily for 2 weeks Explanation There are several regimens recommended for H. pylori infection, which is an important cause of peptic ulcer disease and should be treated if found associated with symptoms. The choice of the regimen depends on considerations such as cost, side effects, and ease of administration. Allergy to one of the medications, as well as intolerance, should also be taken into account. Any proton pump inhibitor (PPI) with amoxicillin 1000 mg twice daily and clarithromycin 500 mg twice daily for 2 weeks or PPI with metronidazole 500mg twice daily and clarithromycin 500mg twice daily for 2 weeks are recommended. These are the triple drug therapies available. The other regimens suggested are bismuth, metronidazole, and tetracycline 4 times daily for 2 weeks along with PPI twice daily for 2 weeks or H2 receptor antagonist twice daily for 4 weeks (quadruple drug therapy). Dual therapy with a proton pump inhibitor and an antibiotic (amoxicillin or clarithromycin) is not recommended as primary therapy, since eradication rates are much lower than the above regimens. The most common side effect is a metallic taste in the mouth due to clarithromycin or metronidazole. Amoxicillin can cause diarrhea or a rash. Clarithromycin can also cause nausea, vomiting, abdominal pain, and (rarely) QT prolongation. Metronidazole can cause peripheral neuropathy, seizures, and a disulfiram-like reaction when taken with alcohol. Tetracycline is teratogenic and causes photosensitivity.

Question You are working in an adolescent health clinic. Protocols have been established to screen certain teens for hepatitis B immune status with blood tests. Based on test results alone, what teen has developed immunity from immunization? Test Patient A Patient B Patient C Patient D Patient E HBsAg Negative Negative Negative Positive Positive Anti-HBc Positive Negative Positive Positive Positive Anti-HBs Negative Positive, >10 mIU/mL Positive Negative Negative IgM anti-HBc Negative Negative Positive Negative

Correct answer: Patient B Explanation Most people who have antibodies from immunization have only anti-HBs (surface antibody). The patient with negative hepatitis B surface antigen, but positive antibodies to both hepatitis B surface antigen and to core antibody (IgM anti-HBc) is immune from prior infection. Anti-HBs is a protective, neutralizing antibody. In persons who recover from an acute infection, hepatitis B surface antigen is generally eliminated from the blood within 3-4 months. Anti-HBs develops during convalescence. The patient with a negative hepatitis B surface antigen and surface antibody, but positive hepatitis B core antibody may be recovering from an acute infection. This patient has demonstrated a decline in surface antigen, but has not yet begun to produce antibody to the surface antigen. Alternatively, the patient may have a low level of immunity that cannot be detected at the test sensitivity level, a false positive anti-HBc, or the patient may be chronically infected with an undetectable level of hepatitis B surface antigen. The presence of anti-HBs following acute HBV infection generally indicates recovery and immunity from re-infection. Anti-HBs can also be acquired as an immune response to hepatitis B vaccine, or passively transferred by administration of HBIG. Using radioimmunoassay (RIA), a minimum of 10 sample ratio units should be used to designate immunity. Using enzyme immunoassay (EIA), the manufacturer's recommended positive should be considered an appropriate measure of immunity. The level of anti-HBs may also be expressed in milli-International units/mL (mIU/mL). 10 mIU/mL is considered to indicate a protective level of immunity. The patient with a positive hepatitis B surface antigen is infected. HBsAg is the most commonly used test for diagnosing acute HBV infections or detecting carriers. HBsAg can be detected as early as 1 or 2 weeks and as late as 11 or 12 weeks after exposure to HBV when sensitive assays are used. The presence of HBsAg indicates that a person is infectious regardless of whether the infection is acute or chronic. Positive anti-hepatitis B core antibody and IgM anticore antibody with negative antibodies to hepatitis B surface antibody are indicators of acute infection, while negative IgM and negative antibodies to hepatitis B surface antigen indicate chronic infection. Anti-HBc (core antibody) develops in all HBV infections, and it appears shortly after HBsAg in acute disease at the time of onset of symptoms or liver test abnormalities; it indicates HBV infection at some undefined time in the past. Anti-HBc only occurs after HBV infection and does not develop in persons whose immunity to HBV is from vaccine. Anti-HBc generally persists for life and is not a serologic marker for acute infection. IgM anti-HBc appears in persons with acute disease about the time of illness onset and indicates recent infection with HBV. IgM anti-HBc is generally detectable 4-6 months after onset of illness, and it is the best serologic marker of acute HBV infection. Although not used for screening immune status to hepatitis B, the hepatitis "e" antigen (HBeAg) is a peptide and normally detectable in the bloodstream when the hepatitis B virus is actively reproducing, indicating that the viral load is high. This, in turn, leads to the person being much more infectious and at a greater risk of progression to liver disease. The exact function of this non-structural protein is unknown, but it is thought that HBe may be influential in suppressing the immune system's response to HBV infection. HBeAg is generally detectable at the same time as HBsAg and disappears before HBsAg. The presence of HBeAg in chronic infection is generally taken to indicate that HBV is actively reproducing and that there is a higher probability of liver damage. In acute infection, HBeAg is generally only transiently present, but mutant strains of HBV exist that replicate without producing HBeAg. Hepatitis B DNA (HBV DNA) is one of the first things that can be detected in the bloodstream after initial infection. The virus consists of an inner core and icosahedral nucleocapsid surrounded by a lipid envelope, with surface proteins encoded by the virus. It can be detected as soon as 1 week after infection using sensitive tests. It is believed that the level of HBV DNA may indicate how fast the virus is replicating. Hepatitis B virus replicates by reverse transcription of an RNA intermediate. Levels are also high during exacerbation and reactivation of the disease. The test for HBV DNA is performed using PCR, which is expensive and difficult to perform and therefore not frequently used. Tests for HBV DNA are generally not performed as standard because other hepatitis B markers such as the "e" antigen can be used to determine viral activity. It is generally only used for research purposes, but it can be used to confirm the presence of hepatitis B and/or measure viral load for viral mutants that do not produce the "e" and/or normal surface antigens. Current recommendations in the United States from the Advisory Committee on Immunization Practices is to immunize all children and adolescents up to age 18 against Hepatitis B; adults at risk for hepatitis B infection, or who need protection from Hepatitis B infection, should also be immunized, but identification of a risk factor is not a prerequisite for immunization. Risk factors that would indicate a need to determine hepatitis B status, or to vaccinate if hepatitis B status is not known include: Persons with more than 1 sex partner in a 6-month period Persons diagnosed with a sexually transmitted disease Men who have sex with men Sex partners of infected persons IV drug users Household contacts of chronically infected persons Infants born to infected mothers Immigrants and children of immigrants from areas with elevated hepatitis rates, including Asia, Africa, the Pacific Islands, Eastern Europe, the Middle East, and the Amazon Basin Healthcare and public safety workers who might be exposed to blood, including those who work in facilities treating patients for sexually transmitted diseases, HIV testing facilities, HIV treatment facilities, facilities providing drug abuse treatment and prevention, chronic hemodialysis facilities, healthcare settings serving men who have sex with men, and institutions and non-residential daycare facilities for persons with developmental disabilities Chronic hemodialysis patients Patients receiving blood transfusions frequently (may not be applicable except in high-prevalence areas).

Case A 60-year-old man presents with a history of alcohol consumption; he has consumed at least 3 beers per day for the past 15 years. He has been unable to carry out his regular activities for the past 6 months. He just sits around and drinks all day. He has lost 10 pounds over the last 3 months. Vitals are as follows: Temp 99 degrees F, RR 24/min, HR 90/min, and BP 130/90 mm Hg. Physical exam is significant for icterus, digital clubbing, and hepatomegaly. Lab and ultrasound reports are pending. Question What is the best treatment for this patient?

Correct answer: Pentoxyfylline Explanation A long history of alcohol consumption along with hepatomegaly on examination points to a diagnosis of alcoholic liver disease (ALD). Pentoxyfylline is used in the treatment of ALD, as it reduces the production of tumor necrosis factor and other proinflammatory cytokines. It has been found to decrease portal hypertension in experimental animals. It also appears to decrease short-term mortality by reducing the risk of hepatorenal syndrome. Erythromycin is a macrolide antibiotic that would be useful only if the patient had an infection. Ribavirin and lamivudine are antivirals. Interferons are mainly used in the treatment of hepatitis C. None of these drugs are useful in alcoholic cirrhosis.

Question What is the most common cause of upper gastrointestinal bleeding?

Correct answer: Peptic ulcer Explanation A peptic ulcer is the most commonly uncovered lesion in the diagnostic workup of a patient with upper gastrointestinal bleeding. The location of the ulcer responsible for the bleeding is most often the duodenum; ulcers of the stomach and esophagus are less frequently found. Rarely, ulceration of the small intestine may be the origin of upper gastrointestinal bleeding. Hemorrhage from a peptic ulcer may be acute or chronic. Gastritis, esophageal varices, Mallory-Weiss syndrome (esophageal mucosal tear), and cancer (mostly stomach and esophagus, rarely small bowel) are less common causes of upper gastrointestinal bleeding.

Case A 62-year-old woman presents with a 1-day history of coffee ground emesis. She has also had black tarry stools for a few days. Her past medical history is significant for osteoarthritis, hypertension, coronary artery disease, and depression. Her hemoglobin is 9.3 g/dl. Question What is the most likely cause of her upper gastrointestinal bleeding?

Correct answer: Peptic ulcer disease Explanation Upper gastrointestinal bleeding is a cause of significant morbidity and mortality. The most common cause of upper gastrointestinal bleeding is peptic ulcer disease, and this is the etiology in up to 62% of cases. Duodenal ulcers are more likely to cause severe bleeding and are more common than gastric ulcers in the United States. There is a high prevalence of Helicobacter pylori infection in patients with peptic ulcer bleeding. H. pylori adheres to the gastric epithelium and renders the underlying mucosa more vulnerable to damage by producing enzymes and toxins, affecting gastrin levels and acid output. Use of non-steroidal anti-inflammatory drugs (NSAID) also increases the risk of bleeding, but the risk varies based on the type of NSAID used. Esophageal varices cause about 6% of upper gastrointestinal bleeding. Usually, there is an underlying history of liver cirrhosis and portal hypertension. Gastrointestinal malignancy is the etiology in about 2% of cases. Pertinent clinical features include a history of weight loss, smoking, or alcohol consumption. It is more common in Asians. Arteriovenous malformations are seen in about 10% of cases. It causes painless bleeding in older patients. Dieulafoy's ulcer is an uncommon cause of bleeding, seen more commonly in men. It is named after the French surgeon Georges Dieulafoy, who described the lesion in 1898. It is considered to be a congenital lesion that histologically consists of an abnormal submucosal caliber-persistent artery. The artery fails to diminish to the usual minute size of mucosal capillaries and can give rise to severe gastrointestinal hemorrhage.

Case A 46-year-old man presents with intermittent painless rectal bleeding. He denies any difficulty passing stools, tenesmus, weight loss, or fatigue, but he does endorse occasional stools where he sees blood mixed with the stool. His only past medical history is primary hypertension, which is well controlled with metoprolol, and there are no signs of end-organ damage. His family history is unremarkable. Vital signs are: BP 110/70 mm Hg, HR 62 bpm, RR 16 bpm, and temperature 36°C (96.8°F). Physical examination, including a rectal examination, shows no abnormalities except for a painless purplish lesion in the anal verge. The lesion protrudes when the patient is asked to strain, but it is manually reducible. Basic chemistries and a complete blood count are normal. Question What is the most appropriate next step?

Correct answer: Perform colonoscopy Explanation In this patient, a colonoscopy should be performed. Hemorrhoids (which this patient has) are certainly the most common cause of anal pathology, but other causes of anal disease such as rectal prolapse, pelvic floor dysfunction, perianal fistulas, inflammatory bowel disease, psychogenic symptoms, and (in particular) colon cancer must be actively sought in patients presenting with anal complaints. In younger patients at low risk for colorectal cancer, it is reasonable to first treat the hemorrhoids and performing a further workup if the bleeding persists, but the incidence of colon cancer begins to rise after age 40. Painless rectal bleeding should raise suspicion of rectal cancer, a suspicion that is further supported in this case by the report of visible blood mixed with stool (rather than blood visualized only at the end of defecation or on the toilet paper). Endoscopic methods such as sigmoidoscopy and colonoscopy are the preferred screening tests because they permit biopsy of the lesion to confirm the diagnosis. If sigmoidoscopy is chosen it should be followed by a barium enema unless a source of bleeding is identified. Hemorrhoids consist of an accumulation of hemorrhoidal tissue in the perianal region. Their pathophysiology is related to constant straining and repeated trauma to the anal canal due to prolonged straining, hardened stools, lack of pelvic floor support, and increased intra-abdominal pressure. The high prevalence of these factors in western societies makes hemorrhoids an extremely common disease, with 1 million cases per year. Hemorrhoids can be internal (above the dentate line) or external hemorrhoids (below the line). Internal hemorrhoids have no pain innervation and can be destroyed in an office setting without local anesthesia. Some modalities are sclerosis, banding, cryotherapy, and laser ablation. The following classification is used for hemorrhoids: Grade Characteristics I Enlargement; lesion remains above the dentate line II Protrusion with straining and spontaneous reduction III Protrusion with straining and manually reducible IV Irreducible protrusion The most common symptoms are bleeding (usually described as painless, live bleeding, often seen on wiping, pain (usually dull and aching, caused by vascular engorgement), protrusion, and difficulties with anal hygiene. Thrombosis and strangling of a protruding hemorrhoid by the anal sphincter are very painful and require urgent attention. Some patients bleed persistently and can even develop iron-deficiency anemia.

Case Mandatory newborn screening has been conducted on an infant who was born 36 hours ago. The abnormality found in the screening has resulted in counseling on initiating lifelong modification to the newborn's diet. It is explained to the mother this action must be taken in order to help decrease the incidence of severe intellectual disability, hyperactivity, and even seizures. Question Considering the most likely diagnosis of this patient, which of the following must be limited in order to prevent these consequences?

Correct answer: Phenylalanine Explanation A mandatory newborn screening panel is done on all newborns prior to their initial discharge after birth. This is due to the fact that precautions, treatment, diet modifications, or actions must begin as soon as possible to ensure the infant has little to no incidence of adverse effects caused by that individual pathology. Phenylketonuria is an extremely well known amino acid metabolism disorder that can be found in newborns. In classic phenylketonuria, there is decreased activity of phenylalanine hydroxylase, which is the enzyme that converts phenylalanine to tyrosine; this leads to excessively high levels of phenylalanine accumulating in the patient. It is an autosomal recessive trait, and it has an incidence in Caucasian infants of around 1:10,000 live births. Patients with untreated phenylketonuria can develop such issues as severe intellectual disability, hyperactivity, seizures, or eczema. Treatment requires a special meal plan that is low in phenylalanine; it should be started as early as possible, usually within the first 7 - 10 days of life. If this treatment plan is put into place and followed throughout the patient's life, there is a very high likelihood that little or no issues will be seen in terms of the adverse effects of this condition. Tyrosine is an incorrect choice; this amino acid is in fact what is lacking in patients who have phenylketonuria. Thyroid hormone would be limited only in the case of some type of hyperthyroidism. Dietary leucine restriction would be required in a newborn found to have what is known as Maple Syrup Urine Disease, or Branched-Chain Ketoaciduria. Fructose avoidance should be followed in patients who are diagnosed with hereditary fructose intolerance.

Case Mandatory newborn screening has been completed on a Caucasian infant who was born 36 hours ago. The birth was relatively benign without any complications; the newborn was born at exactly 40 weeks gestation. An abnormal result found in the screening tests has resulted in you ordering required counseling for the parents due to the fact that they must initiate lifelong modification in their child's diet to decrease the incidence of abnormal intellectual development. Question What disease state was most likely found during this patient's screening?

Correct answer: Phenylketonuria Explanation All disease states listed above are mandatory to be tested in newborns prior to their initial discharge after birth. This is due to the fact that precautions, treatment, meal plans, or actions, if started as soon as possible, ensure the infant has little to no incidence of adverse effects caused by that individual pathology. Phenylketonuria is an extremely well-known amino acid metabolism disorder that can be found in newborns. In classic phenylketonuria, there is decreased activity of phenylalanine hydroxylase, which is the enzyme that converts phenylalanine to tyrosine; this leads to excessively high levels of phenylalanine accumulating in the patient. It is an autosomal recessive trait and it has an incidence in Caucasian infants of around 1:10,000 live births. Patients with untreated phenylketonuria can develop issues such as severe intellectual disability, hyperactivity, seizures, or eczema. Treatment requires a meal plan that is low in phenylalanine; it should be started as early as possible, usually within the first 7 - 10 days of life. If this treatment plan is put into place and followed throughout the patient's life, there is a very high likelihood that little or no issues will be seen in terms of the complication due to this condition. Galactosemia is caused by a complete deficiency of galactose-1-phosphate uridyltransferase, resulting in excessive accumulation of galactose-1-phosphate. High levels of this leads to such issues as hepatic parenchymal disease, renal Fanconi syndrome, cataracts, delayed apraxic speech, and even ovarian failure if not recognized and treated appropriately. Developmental delays, tremor, and ataxia occur rarely in patients who have this pathology. Treatment consists of a galactose-free diet, usually through lifelong avoidance of dairy products. Congenital hypothyroidism, if not detected early and treated promptly, will lead to an increased incidence of intellectual impairment; however, treatment is not with modification of diet. Instead, it should be treated with external replacement of thyroxine, levothyroxine. Sickle cell disease is categorized as a hemoglobinopathy, and although its complications are extensive, it does not directly lead to abnormal intellectual development. Tyrosinemia of the newborn is almost exclusively found in premature infants; it is due to the immaturity of 4-hydroxyphenylpyruvic acid oxidase that results in an increase in tyrosine and the precursor phenylalanine. Typically, there is no sequela from this condition; it resolves on its own within 3 months.

Question A new patient is in your office for a routine physical. They inform you that they have a genetic disease. Upon questioning, they tell you that it was identified at birth and its complications were minimized with a specific diet. The patient asks if it is okay to drink diet soda drinks. You advise them against consuming foods containing aspartame. You do this because the patient has what disease?

Correct answer: Phenylketonuria Explanation Aspartame is a commercially synthesized dipeptide used as an artificial sweetener in many foods, including soft drinks. The structure of aspartame is L-aspartyl-L-phenylalanyl methyl ester. This can be broken down in the body to release phenylalanine. The disease phenylketonuria (PKU) results from decreased levels of the enzyme phenylalanine hydroxylase that is part of the phenylalanine degradation pathway. Untreated, PKU leads to intellectual disability. Appropriate intervention with a very low phenylalanine diet, especially in the very young, can prevent the intellectual disability greatly. Maple syrup urine disease is another disease that results from a defect in an enzyme involved in amino acid degradation—in this case, that of the aliphatic amino acids leucine, isoleucine, and valine. The block is in the oxidative decarboxylation (α-keto acid dehydrogenase is the enzyme) of these amino acids. Similarly, alkaptonuria also results from a defect in an enzyme involved in an amino acid degradation pathway. This is due to a defect in the enzyme homogentisate oxidase. Foods containing aspartame are often recommended for patients with diabetes since it is a way to have something that tastes sweet but does not contain sugar.

Case Mandatory newborn screening has been conducted on an infant who was born 36 hours ago. The abnormality found in the screening results requires counseling in order to initiate lifelong modification of the diet in order to help decrease the incidence of abnormal intellectual development. Question What disease state was most likely found during this patient's screening?

Correct answer: Phenylketonuria Explanation It is mandatory that all of the disease states listed above are tested in newborns prior to their initial discharge after birth. This is due to the fact that precautions, treatment, meal plans, or actions if begun as soon as possible ensure the infant has little to no incidence of adverse effects caused by that individual pathology. Phenylketonuria is an extremely well-known amino acid metabolism disorder that can be found in newborns. In classic phenylketonuria, there is decreased activity of phenylalanine hydroxylase, which is the enzyme that converts phenylalanine to tyrosine; this leads to excessively high levels of phenylalanine accumulating in the patient. It is an autosomal recessive trait, and it has an incidence in Caucasian infants of around 1:10,000 live births. Patients with untreated phenylketonuria can develop such issues as severe intellectual disability, hyperactivity, seizures, or eczema. Treatment requires a special meal plan that is low in phenylalanine; it should be started as early as possible, usually within the first 7 - 10 days of life. If this treatment plan is put into place and followed throughout the patient's life, there is a very high likelihood that little or no issues will be seen in terms of the adverse effects of this condition. Congenital hypothyroidism, if not detected early and treated promptly, will lead to an increase incidence of intellectual impairment. However, treatment is not with modification of diet; instead, it should be treated with external replacement of thyroxine, levothyroxine. Sickle cell disease is categorized as a hemoglobinopathy, and although its complications are extensive, it does not directly lead to abnormal intellectual development. Galactosemia is caused by a complete deficiency of galactose-1-phosphate uridylyltransferase, resulting in excessive accumulation of galactose-1-phosphate. High levels of this leads to such issues as hepatic parenchymal disease, renal Fanconi syndrome, cataracts, delayed apraxic speech, and even ovarian failure if not recognized and treated appropriately. Developmental delays, tremor, and ataxia occur less frequently in patients who have this pathology. Treatment consists of a galactose-free diet, usually through lifelong avoidance of dairy products. Tyrosinemia of the newborn is almost exclusively found in premature infants; it is due to the immaturity of 4-hydroxyphenylpyruvic acid oxidase that results in an increase in tyrosine and the precursor phenylalanine. Typically, there is no sequela from this condition; it resolves on its own within 3 months.

Case A family friend calls you for medical advice. She just gave birth to her 3rd baby 1 week ago. She is worried after a nurse from the hospital called her. The nurse said the baby tested high on a newborn screening for phenylalanine and that she would set up an appointment at a specialty center for the family. Your friend could not remember the name of the disorder, but she was told that it could prevent her baby from normal neurological development if left untreated. The friend reports that her pregnancy was uncomplicated, and the baby appears healthy so far. No one in the infant's immediate family has any health problems. Question What is the most likely diagnosis?

Correct answer: Phenylketonuria Explanation Your friend's newborn most likely has phenylketonuria (PKU), which is an autosomal recessive metabolic disorder that causes abnormal metabolism of phenylalanine and results in elevated serum phenylalanine. It is a rare condition, and it is diagnosed by routine newborn screening. A strict diet low in phenylalanine can prevent complications of the disease, and afflicted patients can lead a relatively normal life. Neurologic deficits, such as intellectual disability and seizures, can occur if it is untreated. If an afflicted patient does not adhere to the special diet, he or she may begin having cognitive deficits and/or seizures. Acromegaly is a disorder of increased growth hormone effects (through insulin-like growth factor I, or IGF-I). It is typically caused by a pituitary adenoma, and it causes enlargement of the extremities if bone epiphyses are closed; it causes gigantism if an individual is affected in childhood. Newborns are not routinely screened for acromegaly, and it would not be expected to impair neurological development. Treatment is usually removal of the adenoma. Cystic fibrosis is a chronic and progressive lung disease. CF could be diagnosed in a symptomatic newborn, but screening is not routinely done on healthy babies. CF is not controlled by diet, and it would not impair neurological development. Marfan syndrome is systemic connective tissue disease that can affect the aorta, thorax, heart valves, and joints. It is present at birth in affected individuals, but screening of healthy, asymptomatic newborns is not done for Marfan syndrome. Marfan syndrome is not controlled by diet, and it would not impair neurological development. Von Willebrand disease is a genetic bleeding disorder. Newborn screening is not routinely performed; it is not controlled by diet, and it would not impair neurological development.

Case A 35-year-old Hispanic man presents to your office but is too embarrassed to tell the nurse his chief complaint. You enter the room, and he admits to severe, intense itching around his anus that has been worsening the last several weeks. He further states that he has noticed increasingly severe and tearing pain in the anal area with each bowel movement. He would rank this pain as a 10/10 on a pain scale. This intense pain makes him not want to have any bowel movements. He admits to only one episode of a small amount of bright red blood on the toilet paper as well as on the stool itself. The patient denies fever, diarrhea, or ever being diagnosed with inflammatory bowel disease. Question Considering the findings in the patient above, what is best choice of diagnostic study to assist in confirmation of diagnosis?

Correct answer: Physical examination Explanation This patient is most likely suffering from an anal fissure. Anal fissures most often affect infants and middle-aged individuals. The majority of fissures are considered primary and caused by local trauma, such as passage of hard stool, prolonged diarrhea, vaginal delivery, or anal sex. Presentation of anal fissures is a tearing pain accompanying bowel movements as well as bright red rectal bleeding that is limited to a small amount noted on the toilet paper or surface of the stool. The patient described all of these components. Patients will also complain of perianal pruritus or irritation, which he also admits to experiencing. The best diagnostic approach to confirming an anal fissure is usually based on the history of pain with defecation and the physical examination finding of a superficial tear noted in the anoderm; this type of tear, no matter if located posteriorly or laterally, is considered a pathognomonic feature of an acute anal fissure. The examiner should carefully spread the buttocks apart and examine the area gently. Digital rectal examination, or anoscopy, is not recommended for diagnosis of this condition; patients will not be able to tolerate them due to the severe pain they will be experiencing from the anal fissure. Sigmoidoscopy or colonoscopy are both inappropriate diagnostic tools in this patient case scenario. These studies should only be considered if there is presence of otherwise unexplainable rectal or gastrointestinal bleeding. Either a sigmoidoscopy or a colonoscopy can assist in evaluating the corresponding source of bleeding.

Case A 55-year-old man visits his family physician for an accidental cut to the forearm with an old rusty wrench. He works as a tank mechanic for the US army and has been an army recruit since the age of 18. The patient received his influenza vaccine a few months ago. He also received a tetanus and diphtheria toxoid approximately 2 months ago when he had an unrelated injury. The patient has been consuming a lot of alcohol over the past few years. He admits to wanting to cut down. He currently drinks a half bottle of red wine every day. He also admits to taking a shot of vodka in the morning to "tide" him through the day. The wound is cleaned and dressed appropriately and his liver function is tested, which reveals the following: Alanine transaminase (ALT) 35 units/L Aspartate transaminase (AST) 76 units/L Question What prophylactic treatment is indicated?

Correct answer: Pneumococcal vaccination and hepatitis A and B Explanation Alcoholism predisposes patients to pneumonia (streptococcus pneumoniae infection). Therefore, administering the annual influenza vaccination and the pneumococcal vaccination is recommended in patients with alcoholism. Hepatitis A or B can be particularly severe in persons with chronic liver disease; therefore, many authorities recommend vaccination against both conditions for persons with chronic liver disease of any cause (particularly those with hepatitis C). This patient is an alcoholic with elevated AST and ALT levels; therefore, he requires administration of pneumococcal vaccine along with Hepatitis A and B vaccines. The MMR vaccine is routinely given to all healthy children at age 12 to 15 months with a second dose at age 4 to 6 years. A second dose is especially important for military recruits, college students, and susceptible health-care personnel. The tetanus toxoid is not required for this patient because he received them 2 months ago. The tetanus toxoid is indicated in clean minor wounds if the person has not received the toxoid in the past 10 years. For other wounds, it is indicated if the person has not received the toxoid in the past 5 years.

Case A 47-year-old man presents with abdominal pain and difficulties breathing. He has a history of alcohol abuse and confirmed cirrhosis of the liver. On examination, you see a malnourished, jaundiced patient with a distended belly. Percussion of the abdomen reveals a huge amount of fluid and wave sign. Question What is the primary cause of the ascites?

Correct answer: Portal hypertension Explanation In cirrhosis the fibrotic changes in the liver tissue increase the resistance to blood flow through the organ and this results in increased portal pressure. The lesion is intrahepatic and sinusoidal. Hepatic synthetic failure causes hypoalbuminemia and decreased albumin production, which leads to reduced portal oncotic pressure in cirrhotic patients. Ammonia levels may be elevated, but elevated levels do not cause ascites. Ammonia is a highly toxic metabolic product of the urea cycle. The urea cycle is the only major pathway to remove waste nitrogen, and it usually gets converted to non-toxic urea in the liver. Increased portal pressure, reduced portal oncotic pressure, as well as elevated aldosterone level with sodium retention cause ascites. Decrease in fluid intake is actually one of the therapeutic strategies for ascites. Blockage of the common bile duct results in jaundice, but it has nothing to do with the development of ascites.

Case A 23-year-old Caucasian woman presents with weakness, fatigue, and occasional palpitations starting 6 months ago. She took a multivitamin supplement with transient improvement. In the last 3 months, blisters appeared on her elbows, which were associated with intense itching. Her only medication is an implantable contraceptive inserted 3 years ago. There is no relevant past medical history, and her family history is also unremarkable. Vital signs are BP 120 / 80 mm Hg, HR 94 bpm, RR 18 rpm temperature 36.3° C (97.2° F). On physical examination, she is alert and oriented, with no dyspnea or cyanosis. Examination of the skin reveals bullous lesions in her elbows. The remainder of the examination shows no abnormalities. A complete blood count shows hemoglobin 12 g/dL, hematocrit 31%, Mean Corpuscular Volume (MCV) 78 fl, Mean Corpuscular Hemoglobin Concentration (MCHC) 29 pg/L, Leukocytes 7.400 /mm3 w/normal differential, and platelets 218,000 /mm3. A skin biopsy reveals a neutrophilic infiltrate, with fibrin and microblisters, most dense at the tips of the dermal papillae and containing IgA on immunofluorescence staining. Question What finding is most probable on subsequent workup?

Correct answer: Positive transglutaminase antibodies Explanation Positive transglutaminase antibodies is the correct response. Dermatitis herpetiformis produces erythematous, pruriginous bullous lesions on the extensor surfaces of the elbows, knees, scalp, neck, and buttocks. These lesions, sometimes called skin celiac disease, are found in 10% of patients with celiac disease, and are highly specific for it; nearly all patients with dermatitis herpetiformis have histologic evidence of celiac disease on small-bowel biopsy. However, in many cases the enteropathy is subclinical. Histopathologic examination of the bullae reveals deposits of anti-gliadin IgA antibodies in the basal membrane of the epidermis. In most cases, the skin lesions involute after the patient is put on a gluten-free diet. However, this can take several months. In patients with severe pruritus, the mainstay of treatment is dapsone, which is highly effective in relieving this symptom. Second-line options include sulfasalazine and sulfapyridine. Celiac disease can present with isolated nutrient deficiencies (e.g. iron, folate, B-complex vitamins), and therefore could explain this patient's iron deficiency anemia. As transglutaminase antibodies are found in 70% of cases of celiac disease, there is a high probability that this assay will be positive in this patient, whether or not the anemia is caused by celiac disease. The more florid presentation of celiac disease - with overt steatorrhea, weight loss, and malnutrition - is becoming less common in the United States. Therefore, the physician must be aware of the possibility of celiac disease in patients with milder symptoms, such as occasional diarrhea with bulky stools, bloating, or more rarely, isolated micronutrient deficiencies. Anti-Saccharomyces (ASCA) antibodies are associated with Crohn's disease (60-70% of patients are positive). Lesions in these patients include oral aphthous ulcers, pyoderma gangrenosum, and rheumatoid nodules, as well as signs of non-cutaneous disease (e.g. enterocutaneous fistulas, malnutrition, micronutrient deficiencies). A 99mTc-red cell scintigraphy could be positive because occult gastrointestinal bleeding is still a possibility. Causes of a negative iron balance include: Deficient intake, which is most prevalent in poor countries. Pregnancy. Chronic bleeding (gynecologic, gastrointestinal, hematuria). Iron deposition in tissues (e.g., cardiac hemosiderosis) The deficiency in hemoglobin synthesis in the bone marrow tends to cause a compensatory rise in erythropoietin levels, which produces erythroid hyperplasia in the bone marrow. Other findings include absence of sideroblasts and stainable iron in the bone marrow's reticular network. However, bone marrow biopsy has been largely abandoned for diagnosis of iron-deficiency anemia because of interobserver variability and the availability of less invasive tests of iron status, particularly serum ferritin and total iron binding capacity (TIBC).

Case A 77-year-old woman presents with a cough of several weeks duration. The patient has seen 2 physicians for the cough; the 1st time was about 6 weeks ago, and the 2nd was 4 weeks ago. She was given antibiotics and a steroid inhaler. The cough is still there throughout the day, and it is particularly bothersome at night. She states the cough started after a mild upper respiratory tract infection that has cleared. She denies wheezing, shortness of breath, or chest pain. She has no symptoms referable to the head and neck; in particular, she has no throat pain or postnasal drip. Her past medical history is significant for celiac disease, and she follows a strict gluten-free diet. She also has mild hypertension; it is controlled with thiazides. She denies a history of asthma, smoking, or exposure to second-hand smoke. On exam, her vital signs are normal. You observe an elderly woman in no acute distress; her HEENT, heart, abdominal, and lung exams are within normal limits. A chest X-ray is also within normal limits. Question What would be the next step in the management of this patient?

Correct answer: Prescribe a proton pump inhibitor Explanation The patient should be prescribed a proton pump inhibitor. The most common causes of a chronic cough in immunocompetent adults (explaining 95% of cases) are post-nasal drip syndrome, gastroesophageal reflux disease (GERD), asthma, chronic bronchitis from cigarette smoking, bronchiectasis, eosinophilic bronchitis, or the use of an angiotensin-converting enzyme inhibitor (ACEI). The rest of the patients (5%) have chronic cough caused by bronchogenic carcinoma, carcinomatosis, sarcoidosis, left ventricular failure, or aspiration. A psychogenic cough, or a cough persisting from habit, is rare and is a diagnosis of exclusion. Post-nasal drip syndrome could be due to vasomotor rhinitis, allergic or non-allergic rhinitis, or chronic bacterial sinusitis. It would be unusual for a patient to present with a chronic cough due to a post-nasal drip syndrome with no symptoms referable to the HEENT (e.g., the absence of post-nasal drip, sore throat, nasal symptoms, or a feeling of fullness/gagging in the throat); therefore, the prescription of a decongestant/antihistamine is unwarranted. Sinus X-rays are indeterminate in the diagnosis of chronic sinusitis. Asthma may present with chronic cough in adults, without wheezing or shortness of breath; however, even if used suboptimally, the steroid inhaler should have some effect. It should be noted that a steroid inhaler is also the treatment for eosinophilic bronchitis. Moreover, if a further trial of steroids were to be indicated, it would be prudent to try the steroid inhaler again (this time with a spacer) instead of subjecting the patient to possible complications of oral steroids. Of the patients who present with a chronic cough who are subsequently found to have GERD as a cause, about 75% do not have symptoms of heartburn or regurgitation. Given this patient's presentation, it seems reasonable to approach her as if she had GERD and prescribe a proton-pump inhibitor as a trial for 2 - 3 weeks. It is very likely that this will control her cough, and over time the proton-pump inhibitor (PPI) can be downgraded to less aggressive treatment with H2-blockers. This would avoid both the high cost of PPIs, as well as the chronic use of PPIs in patients. The use of codeine in an elderly patient is not a good idea unless there is a high degree of assurance that its use will substantially help the patient. Excessive sedation, falls, and constipation are common side effects of narcotics in the elderly.

Case A 45-year-old man is admitted to the medical floor of a hospital with increasing jaundice, swollen legs, and episodes of disorientation for the last several weeks. His abdomen is distended and he looks ill. According to the family, he is an alcoholic and has recently been diagnosed with cirrhosis of liver. On examination, he has a temperature of 99°F, BP 100/72 mm Hg, and his pulse is 86/minute. Sclerae are icteric, and he has 2+ pitting pedal edema. Lungs are clear, and heart sounds are normal. Abdomen is distended with moderate ascites, caput medusae, and no tenderness. Liver and spleen are not palpable. He is awake but drowsy and oriented to person and place but not to time. He does have a fine tremor in his hands. Labs show WBC 8000/uL, platelets 100,000/uL, Hb 12g%, AST 76 U/L, ALT 56 U/L, AP 62 U/L, and ammonia is 124 mg/dL. Question In order to avoid full decompensation into hepatic encephalopathy, which of the following is the most important action to take at this time?

Correct answer: Prevent constipation. Explanation This patient is already in the initial phases of hepatic encephalopathy due to alcoholic liver disease, as evidenced by the disorientation and tremors, also known as asterixis. The primary cause of hepatic encephalopathy is unclear. Metabolic abnormalities due to liver dysfunction—resulting in a spectrum of neuropsychiatric signs and symptoms—are seen. High levels of ammonia are found in the blood. Constipation causes increased ammonia production and absorption due to prolonged intestinal contact and aggravates the condition.Lactulose should be administered frequently to eliminate the ammonia in the stool. It is an indigestible sugar that acts as an osmotic laxative by increasing the water content of the stool and promoting bowel movements. It is digested by the colonic bacteria, and the acidic remains convert ammonia into ammonium ions in the colon, which Lactulose should be administered frequently to eliminate the ammonia in the stool. It is an indigestible sugar that acts as an osmotic laxative by increasing the water content of the stool and promoting bowel movements. It is digested by the colonic bacteria, and the acidic remains convert ammonia into ammonium ions in the colon, which are then excreted in the stool. Oral antibiotics can lower blood ammonia levels by decreasing ammonia production and absorption. The commonly used ones are neomycin, metronidazole, vancomycin, and, lately, rifaximin. The last 3 are better tolerated than neomycin. However, antibiotics have their side effects and can cause bacterial overgrowth syndromes. Their main use continues to be in patients who cannot tolerate disaccharides like lactulose. Acarbose and fermentable fiber can also cause decrease in intestinal ammonia production and absorption. Newer studies with sodium benzoate are ongoing. Benzoate and glycine react to form hippurate, and for every mole of benzoate utilized this way, one mole of nitrogen is excreted in the urine, thereby enhancing ammonia metabolism. This, however, still needs to be studied further to be used widely. Studies with ornithine-aspartate are also being done as a stimulator of ammonia metabolism.All the products mentioned are yet to replace lactulose as the first line of treatment but are potentially useful once more studies are done. Other precipitating factors for hepatic encephalopathy include azotemia, hypokalemia, gastrointestinal bleeding, high protein diet, alkalosis, infection, sedatives, and other hepatotoxic agents. All the products mentioned are yet to replace lactulose as the first line of treatment but are potentially useful once more studies are done. Other precipitating factors for hepatic encephalopathy include azotemia, hypokalemia, gastrointestinal bleeding, high protein diet, alkalosis, infection, sedatives, and other hepatotoxic agents. High protein diet is a contraindication in this condition, as protein catabolism causes increase in ammonia levels. Daily protein should be restricted to 40 g/day. Sedation of the patient should also be avoided since sedatives cause cerebral depression and worsening of encephalopathy. These drugs are also not metabolized adequately by the diseased liver. Though infection is an important precipitating factor for hepatic encephalopathy, empiric treatment is not recommended. However, early and adequate treatment of an infection should be done, especially for spontaneous bacterial peritonitis. In fact, multiple randomized control trials have been done regarding antibiotic prophylaxis for SBP, and they have shown not only a decrease in bacterial infections but also a significant reduction in mortality. Prophylaxis is recommended in cirrhotic patients with risk factors for SBP like GI bleeding, prior history of SBP, and low ascitic fluid protein. Thiamine and folic acid should be added for nutritional support for all alcoholic patients since they are malnourished and vitamin-depleted. However, this will not change the outcome in hepatic encephalopathy as quickly or as much as avoiding constipation.

Case A 35-year-old man presents with a 4-week history of difficulty swallowing food. On detailed history, he says that the regurgitation contains undigested fragmented food material; it is not associated with a foul smell. He has lost 3 kg over the past month. On examination, his pulse is 94 bpm and blood pressure is 130/86 mm Hg. Mild pallor is noted, but there is no icterus. Epigastric tenderness is present. CBC shows a hemoglobin of 9.5 g/dL (total count 7400 cells/mm3 and ESR 12 mm/hr). An X-ray of the abdomen shows the absence of a fundic air shadow. The patient undergoes barium studies that show a dilated esophagus; the lower end appears beak-shaped. Question What is the most common cause for the above condition?

Correct answer: Primary idiopathic Explanation Gradual onset of difficulty in swallowing food, regurgitation of undigested food, the absence of fundic air on plain abdominal X-ray, and dilated esophagus with beak-shaped lower end are diagnostically indicative of achalasia. This condition is due to degeneration of inhibitory neurons in the lower end of the esophagus. The most common cause of achalasia is primary idiopathic. There is a deficiency of inhibitory neurons, vasoactive intestinal peptide, and nitric oxide synthase in this condition. Other rarer causes of achalasia include Chagas disease, lymphoma, neurodegenerative disorders, esophageal gastroenteritis, and certain viral infections. Diabetes mellitus is not associated with achalasia. Patients with diabetes may have gastroparesis, the term for delayed gastric emptying.

Case A 54-year-old man is admitted to the intensive care unit and intubated on a ventilator after a serious motorcycle accident where he sustained multiple fractures, spinal cord trauma, and a liver laceration. On his fifth week of hospital admission, blood cultures from a central line reveal a staphylococcal septicemia. After another month in the intensive care unit, the patient is found to have ascites and pitting edema. Vitals are within normal limits. Ultrasound demonstrates a fatty liver and unremarkable gallbladder. Echocardiogram shows no structural defects and a normal ejection fraction. Question What is the most likely diagnosis?

Correct answer: Protein deficiency Explanation This patient is most likely to have a protein deficiency. In developed countries, protein deficiency most often occurs in the face of acute severe illness such as sepsis. The typical findings in these patients are ascites and pitting edema due to hypoalbuminemia. Hypovitaminosis B1 (thiamine) is not the correct answer choice. This deficiency is known as wet beriberi. Though it may cause edema, there will be dilated cardiomyopathy, which is not found in this case. Acalculous cholecystitis is a condition that may be seen in critically ill patients. Clinically, there is fever and leukocytosis. Ultrasound reveals gallbladder wall distention. Marasmus is a total caloric and protein deficiency. These patients typically present with broomstick extremities (muscle wasting). These patients do not present with ascites and pitting edema. Primary sclerosing cholangitis (PSC) is commonly associated with patients who have ulcerative colitis. PSC is a chronic liver disease; it presents as inflammation, destruction, and fibrosis of both the intrahepatic and extrahepatic bile ducts, with the resultant destruction leading to cirrhosis of the liver.

Question A 6-month-old infant is brought to your office. The mother indicates that the infant had several scratches that took a long time to stop bleeding. After ruling out various forms of hemophilia, you suspect that the infant has a vitamin deficiency that is causing this problem. What proteins would be affected by this vitamin deficiency?

Correct answer: Prothrombin Explanation Prothrombin and many of the other blood clotting factors contain the modified amino acid γ-carboxyglutamic acid (Gla). This modification occurs via an enzymatic reaction requiring vitamin K and converts glutamic acid, a weak calcium chelator, to Gla, a much stronger calcium chelator. Vitamin K is an isoprenoid compound, classified as a fat-soluble vitamin; it is found in green plant leaves and is also synthesized by intestinal bacteria. The formation of Gla residues is required for the "active" form of prothrombin. These Gla residues are found in the amino-terminal region of the protein and function by chelating calcium ions. This forms part of the binding site to phospholipid membranes, where the conversion from the zymogen prothrombin to active thrombin takes place. A vitamin K deficiency results in an increased coagulation time. A 2,3-epoxide derivative of vitamin K is an intermediate in the formation of Gla residues. Other proteins that contain Gla residues include Factors VII, IX, and X, several other anticoagulation proteins, and the bone proteins (Osteocalcin and matrix-Gla protein). Dicumarol is a naturally occurring anticoagulant that is an antagonist of vitamin K. It inhibits the reductase that converts the epoxide form of vitamin K back to the active vitamin.

Case A 30-year-old man presents with a 1-month history of severe indigestion. He has tried over-the-counter antacids for the last 2 weeks to help relieve the pain, but they provided only mild relief. He describes the pain as a dull, aching, "hunger-like" pain. He also states that a few hours after eating the pain lessens, and then worsens a few hours later. His physical examination is unremarkable except for epigastric pain on deep palpation of the abdomen. His stool guaiac test is negative. Question What therapeutic regimen should be prescribed?

Correct answer: Proton pump inhibitor therapy and dietary modifications Explanation The clinical picture is suggestive of peptic ulcer disease (PUD). The treatment of choice is proton pump inhibitors due to their ease of use and efficacy. Dietary modifications will assist in the decrease of stomach acid production. The hallmark symptom of PUD is epigastric pain (dyspepsia); the pain can be described as aching, dull, or gnawing. About 50% of patients do get some relief with over-the-counter antacids or other treatments. Some patients may have gastric pain that awakens them at night. Duodenal ulcers are usually indicated when there is relief of pain for a few hours after eating. H2-blocker therapy is effective in treating PUD, but it has been replaced by proton pump inhibitors. Dietary modifications alone are not helpful in this situation due to the patient experiencing severe dyspepsia. Continuing antacids would not be helpful since they offer only mild relief; they are no longer used as first-line therapy for active ulcers. Bismuth therapy (Pepto-Bismol), another over-the-counter antacid, is no longer used as first-line therapy for active ulcers.

Case A 57-year-old African-American woman presents with retrosternal burning sensation. The symptoms started during the day several months ago, and lately they have been occurring at night. Heavy meals and sweets seem to trigger them. She also mentions that swallowing solid foods is uncomfortable and she needs to "wash" everything down with water. The burning starts in the epigastrium, which is sensitive to pressure; it radiates to the lower jaw, and it is accompanied by heartburn. The patient describes her job as very stressful, and she complains about never having enough time for herself. She is 165cm tall, weighs 95kg, her blood pressure reads 145/90 mm Hg, pulse is 75/min, and respiration rate is 23/min. A stool sample tested positive for hemoglobin. Question What step should be taken next in order to corroborate the suspected diagnosis?

Correct answer: Upper endoscopy Explanation The symptoms described let you suspect GERD (gastroesophageal reflux disease). Diagnostic measurements depend upon symptoms. If symptoms are of short duration and there are no systemic manifestations, empiric treatment should be tried. With good response to that, chronic maintenance treatment should be considered. Poor response should lead to endoscopy of the upper gastrointestinal tract. With dysphagia, the first diagnostic step is barium swallow or upper endoscopy. If the endoscopy shows reflux, chronic treatment for GERD will be necessary. With an unclear diagnosis after history and endoscopy, if empiric treatment does not show any response or surgical treatment is considered, esophageal manometry with Bernstein test and 24-hour esophageal pH monitoring need to be performed. An ECG would be appropriate if you suspected angina pectoris as the cause of the symptoms. Angina pectoris, a common manifestation of coronary artery disease, can present as heartburn with radiating pain from the epigastrium to jaw, left shoulder, and/or left axilla. It can be triggered by physical and mental stress, cold temperatures, or heavy meals. One kind of angina, Prinzmetal's angina, typically occurs in bed at night; it is caused by coronary spasms, and it is not precipitated by cardiac work. Dysphagia, however, is not a symptom of angina. When there is no dysphagia, but there are chronic symptoms and/or signs of blood-loss (heme-positive stool, anemia, or hematemesis), an esophagogastroscopy would be proper. Helicobacter pylori (HP) plays a role in gastritis and peptic ulcers. There is no proof of any connection to GERD. Acute HP gastritis is characterized by epigastric pain, nausea, and vomiting. Ulcus duodeni classically presents with burning epigastric pain 1 - 3 hours after meals and at night. It tends to wax and wane over months. Additionally, ulcer patients may present with hemorrhages. Esophageal manometry, which measures intraesophageal pressure, is performed if a patient with symptoms of GERD has a normal upper endoscopy after barium swallow or empiric treatment for GERD. Gastric secretory testing and serum gastrin measuring is indicated in patients with refractory ulcer disease; it is also indicated if Zollinger-Ellison syndrome (caused by a gastrin producing endocrine tumor, which is usually located in pancreas or duodenum) is suspected, or in patients who will undergo elective surgery for duodenal ulcers. Treatment of GERD First line lifestyle modification (avoid spicy foods and foods that delay gastric emptying, smaller meal sizes, weight loss, avoid lying down after meals Second line H2-receptor antagonist for 8 - 12 weeks Third line Proton pump inhibitors Fourth line Surgical fundoplication

Case A 45-year-old woman developed a severe urinary tract infection following a 3-week vacation to the Caribbean islands. She was treated with ampicillin for 10 days. On the 7th day of her treatment, she developed acute abdominal pain; it was accompanied by cramps and diarrhea. Her temperature was normal. Sigmoidoscopic examination revealed yellow-white plaques on the mucosal surface and areas of necrosis. Question What is the most likely diagnosis?

Correct answer: Pseudomembranous colitis Explanation Pseudomembranous colitis may follow the use of almost any antibiotic. The causative organism is C. difficile, which produces an enterotoxin responsible for necrosis of the mucus membrane of the sigmoid colon and rectum. Diagnosis is made by the demonstration of the toxin in fecal extract. An infectious cause of colitis, such as E. coli or amebic colitis, though possible, is less likely given the onset of the patient's symptoms shortly after taking antibiotics. Toxic megacolon is severe colonic dilatation that develops in some patients with ulcerative colitis. Ischemic colitis is due to acute blockage of the mesenteric artery. It is usually presents with the abrupt onset of severe abdominal pain. The condition is most common in patients with risk factors for arterial embolization, such as atrial fibrillation.

A 52-year-old man presents with rectal pain during bowel movements. Patient describes his pain as tearing. Physical examination reveals a tear in the anal mucosa. Question What is considered first-line treatment in this case?

Correct answer: Psyllium Explanation Psyllium is the correct response. The patient has an anal fissure. Anal fissure is exacerbated by constipation. First-line treatment is fiber supplementation, such as psyllium and sitz baths. Topical nitroglycerin is an incorrect response. Nitroglycerin is used in the treatment of anal fissure, but is not a first line treatment. Oral ciprofloxacin is an incorrect response. The patient has no signs of infection, so there is no role for antibiotics. Botox and surgery are incorrect responses. These therapies are reserved for refractory cases.

Case A newborn male infant begins vomiting 3 weeks after birth. The mother states that her infant nurses eagerly, but vomits forcefully after feeding. The mother also stated that she notices her infant's abdomen becomes distended after feeding and prior to vomiting. He has begun to lose weight. This is her first child and she is concerned. On examination, you note a palpable 7 mm oval mass in the abdomen. Question What is the most likely diagnosis?

Correct answer: Pyloric stenosis Explanation The clinical picture is suggestive of pyloric stenosis. This disorder usually begins between 2 and 4 weeks of age, and rapidly becomes projectile after feeding. An olive sized mass, 5 - 15mm in longest dimension, can be felt upon deep palpation in the right upper abdomen, especially after the child has vomited. Typical signs and symptoms of congenital megacolon are failure to pass meconium, followed by vomiting, abdominal distention, and reluctance to feed. Esophageal atresia (EA) is a congenital medical condition where the esophagus connects to the stomach in a blind-ended pouch rather than connecting normally. The presence of EA is suspected in an infant with excessive salivation (drooling), and in a newborn with drooling that is frequently accompanied by choking, coughing, and sneezing. When fed, these infants swallow normally but begin to cough and struggle as the fluid returns through the nose and mouth. In duodenal atresia, bile-stained vomiting and abdominal distention usually begin in the first 48 hours of life. Copious salivation associated with choking, coughing, and cyanosis coincident with the onset of feeding in a newborn is suggestive of tracheoesophageal fistula.

Case A 20-year-old Caucasian man presents with a 1-month history of tingling sensations in his legs. He is on isoniazid treatment for pulmonary tuberculosis. On examination, you note a greasy yellow scale over his scalp and eyebrows. Laboratory investigations reveal a hemoglobin level of 9 g/dL. Question What is the most likely diagnosis?

Correct answer: Pyridoxine deficiency Explanation The correct answer is pyridoxine deficiency. In cases of pyridoxine or (vitamin B6 deficiency), patients can present with peripheral neuropathy, seborrheic dermatosis, glossitis, and cheilosis. Laboratory investigations reveal anemia with lymphopenia. Causes include malabsorption as well as medications such as isoniazid and penicillamine. Dietary sources of vitamin B6 include liver, legumes, whole grain cereals, and meats. In cases of vitamin A deficiency, patients can present with the inability to see well in dim light or night blindness. There may also be conjunctival and corneal xerosis, as well as pericorneal and corneal opacities, and Bitot's spots. Bitot's spots are a collection of keratin appearing as triangular foamy spots on the conjunctiva. The patient may also have xeroderma, hyperkeratotic skin lesions, and increased susceptibility to infections. Causes include inadequate dietary intake and malabsorption. Dietary sources of vitamin A include fish, liver, egg yolks, butter, cream, dark green leafy vegetables, and yellow fruits and vegetables. In cases of vitamin C (or ascorbic acid) deficiency, patients can present with bleeding tendencies (as a result of weakened capillaries) and impaired wound healing due to impaired formation of connective tissue. On examination, the gums may be swollen and friable; the teeth may be loose. There may also be multiple splinter hemorrhages on the nails and ecchymoses, especially over the lower limbs. Causes include inadequate dietary intake and certain conditions such as pregnancy and lactation that increase vitamin C requirements. Dietary sources of vitamin C include citrus fruits, such as oranges, lemons, and tangerines, as well as tomatoes and potatoes. Niacin deficiency causes pellagra, which is characterized by: Symmetrical dermatitis, usually on parts of the body exposed to sunlight Scarlet glossitis and stomatitis Diarrhea Mental aberrations, such as memory impairment, depression, and dementia. These may appear alone or in combination. Causes include inadequate dietary intake, especially in patients with corn-based diets or alcoholism. Dietary sources include legumes, yeast, meat, and enriched cereal products. Vitamin E deficiency may cause a hemolytic anemia in premature infants. Laboratory investigations reveal low plasma tocopherol levels, a low hemoglobin level, reticulocytosis, hyperbilirubinemia, and creatinuria. Causes of vitamin E deficiency in premature infants include a limited placental transfer of vitamin E and the resultant low levels at birth combined with its relative deficiency in the infant diet. Dietary sources for older children and adults include wheat germ, vegetable oils, egg yolk, and leafy vegetables.

Case A mother has brought in her 2-year-old son with a 2-day history of bleeding of the rectum. Bleeding was very little to begin with, but today the child has soaked 3 diapers. There is no history of fever, vomiting, or loose bowel movements. Child is active and playful and has no obvious discomfort. Stool is brick red in color. Abdomen is soft with no distension or organomegaly. Perianal examination is normal, and there is no fissure. Question What is the investigation of choice to confirm the diagnosis?

Correct answer: Radionuclide scan Explanation The clinical presentation of the child with intermittent painless bleeding per rectum is suggestive of Meckel diverticulum (MD), which accounts for 50% of lower GI bleeds in children below 2 years of age. Radionuclide scan is the most sensitive study that can confirm the presence of Meckel diverticulum. It is performed after intravenous infusion of technitium-99m pertechnetate. The mucous secreting cell of the ectopic gastric mucosa in the Meckel diverticulum takes up pertechnetate so that it can be visualized on the scan. The uptake can be enhanced by ranitidine, cimetidine, or glucagon. Sensitivity of the enhanced scan is about 85%, and specificity is approximately 95%. During active GI bleeding, the radioactive red cells leak into the intestines where bleeding is occurring and will appear as 'hot areas' on the scan. 'Hot areas' are not seen on the scan in the absence of active bleeding. Meckel diverticulum is a remnant of the omphalomesenteric duct, which connects the yolk sac with the gut in the developing embryo and provides nutrition to the embryo until the placenta develops. Later this duct is obliterated and separates from the intestine. The lining of the yolk sac is similar to the lining of the stomach. Failure of involution of the omphalomesenteric duct results in various residual structures, of which Meckel diverticulum is the most common. It is a 3-6 cm long pouch of ileum, about 50-75 cm from the ileocecal valve. This distance depends on the age of the patient. Meckel diverticulum is lined by the acid-secreting mucosa that can cause intermittent, painless rectal bleeding due to ulceration of the diverticular and the adjacent ileal mucosa. Meckel diverticulum can act as a lead point for intussusception or may lead to a volvulus of itself and of the small intestines, thus presenting with bowel obstruction. Routine barium studies usually do not fill the diverticulum; hence, they are not helpful in the diagnosis. Double contrast barium enema can be performed for evaluation of unexplained lower GI bleeds. However, it should not be undertaken during acute hemorrhagic phase, as it makes subsequent diagnostic evaluation like colonoscopy very difficult. Small bowel barium examination has a low yield because the diverticulum fills transiently and the surrounding loops of small bowel overlap and obscure the diverticulum. CT scan is rarely used as a primary imaging modality where Meckel diverticulum is suspected. If Meckel diverticulum is associated with intussusception, it may be revealed as an intraluminal mass, but it cannot be differentiated from intussusception due to other causes. Ultrasound has a limited role in evaluating GI hemorrhage. Sometimes intussusception due to Meckel diverticulum has been diagnosed by ultrasound, but the sensitivity and specificity is generally low. Colonoscopy cannot positively diagnose bleeding from a Meckel diverticulum because the colonoscope usually cannot reach the part of small intestines where the Meckel diverticulum is located. It can be helpful in its diagnosis if blood-filled colon is seen without another source of bleeding, particularly if it is accompanied by an abnormal Meckel radionuclide scan.

Case A 45-year-old woman presents with diarrhea and vomiting that started this afternoon. She says she warmed up leftover rice for supper last night and symptoms began shortly thereafter. She has no fever, and her blood pressure and pulse are within normal limits. Question What would be the best next step?

Correct answer: Reassure the patient and send home with oral rehydration Explanation This patient is suffering from acute food poisoning. The history gives us the clue that she acquired this food poisoning from reheated rice. The most common organism that might cause the condition in this case would be Bacillus cereus. Bacillus cereus is a Gram-positive, spore-forming rod. Spores on grains, such as rice, survive steaming and rapid frying. The spores germinate when rice is kept warm for many hours. B. cereus produces 2 enterotoxins. The mode of action of 1 of the enterotoxins is the same as that of cholera toxin (i.e., ADP-ribosylates G protein, which stimulates adenylate cyclase and leads to an increased concentration of cyclic AMP within the enterocyte). The mode of the other enterotoxin is uncertain. Clinically, it is a self-limiting situation and requires supportive treatment only. The incubation period is about 18 hours. The best advice for this patient would be to reassure her that it will get better and encourage her to get oral rehydration. There is no sign of hemodynamic instability, so intravenous hydration or admission to the hospital is not necessary. Stool samples are not necessary in the case of an isolated, self-limited food poisoning episode. Bacterial diseases transmitted by foods:

Case A 43-year-old man presents with a "sore" near his rectum. He reports rectal pain with bowel movements; there is some anal discomfort with sitting and physical activity. He occasionally notes some drainage from the lesion and rectal pruritus. He denies anal intercourse, unusual travel, and other skin lesions. He has tried over-the-counter hemorrhoid treatments without relief. His past medical history is significant for Crohn's disease; he is not being treated for it currently. He has no known allergies and takes no medications. He is married/monogamous; he works as a bartender and denies the use of tobacco. He reports occasional alcohol use, but no drug use. On physical exam, a single 4mm pustule-like lesion is visualized on the perianal skin; it is located about 2 cm away from the anus. No other skin lesions are found. Rectal exam is normal. Question What is the most likely diagnosis?

Correct answer: Rectal fistula Explanation This patient most likely has a rectal fistula. A fistula (or fistula-in-ano) is a communicating tract (usually from an anal crypt) to the skin surface, and it often develops from a rectal abscess. Rectal fistulas are more common in patients with Crohn's disease. Fistulas are associated with rectal pain, itching and tenderness. External hemorrhoids are a common condition and can be associated with anal pain, constipation and bright red rectal bleeding. Hemorrhoids are dilated blood vessels and appear as protrusions, either singly or several clustered together, such as a bunch of grapes. (Internal hemorrhoids would not be visualized on this patient without anoscopy or colonoscopy.) Genital herpes simplex lesions are caused by sexual transmission of herpes simplex virus types 1 or 2 (HSV-1 and HSV-2). The classic presentation is a tingling, burning, and/or painful sensation from 1 or more vesicular or ulcerated lesions on the genitals. This patient's single perianal pustule and history of Crohn's disease make fistula a much more likely diagnosis. Lymphogranuloma venereum (LGV) is a sexually transmitted disease; it presents with a primary painless lesion on the genitals/anus. LGV is rare is developed countries, with greater incidence in the tropics. This patient noted a painful lesion, making LGV unlikely. Similarly, a syphilitic chancre is the primary lesion of the sexually transmitted Treponema pallidum. It presents as a solitary, painless anogenital lesion.

Case A 45-year-old man presents with hematemesis. He has had 2 episodes of vomiting 'coffee-ground'-appearing material; the vomiting began 45 minutes prior to presentation. Additionally, he reports passing black, sticky stools for the past 3 or 4 days. Past medical history is positive for occasional headaches; they have been coming more frequenly lately. Social history reveals alcohol use (1 case of beer each weekend) and tobacco (1 pack per day). Medications include ibuprofen as needed for headaches; he has been taking 800 mg 3 times a day for the past week. You place a nasogastric tube and find bright red blood that fails to clear with saline irrigation. Hemoglobin is 8.9 g/dL. Evaluation of his blood pressure and pulse reveals orthostatic changes that resolve with an intravenous fluid bolus of 500 cc of Lactated Ringer's solution. Question What should you do next?

Correct answer: Refer for emergency upper endoscopy Explanation He should be referred for an emergency upper endoscopy. This patient is most likely bleeding from a gastric ulcer. His recent NSAID use, as well as his alcohol and tobacco habits, make him at risk for peptic ulcer disease. His symptoms of melena and hematemesis, along with his anemia, make the diagnosis quite straightforward. It appears that this patient is still actively bleeding based on the results of the nasogastric tube irrigation; therefore, the priority should be getting the ulcer to stop bleeding. Upper endoscopy should be performed so that the bleeding site can be identified and treated with electrocautery, coagulation, or an injection of epinephrine or a sclerosing agent. If the bleeding cannot be stopped with endoscopic interventions, angiographic embolization should also be tried. If these interventions do not succeed, the patient has rapid deterioration, or if he requires more than 6 units of blood in a 24-hour period, then emergency surgery may be indicated. The other choices are not the best options for immediate management. This individual cannot be followed simply with transfusions and serial CBC's because he appears to still be actively bleeding. Helicobacter pylori infection may very well be playing a part in the etiology of this man's ulcer, but evaluation for H. pylori can be done with a biopsy at the time of his endoscopy; it will not help in his immediate management. A barium esophagram will not identify actively bleeding ulcers and cannot treat active bleeding. While NSAID, alcohol, and tobacco use may have precipitated this man's GI bleed, counseling about his use of these substances will not sufficiently treat his immediate bleed.

Case A 52-year-old man presents for a wellness examination. He has no present symptoms or any significant past medical or surgical history. He consumes a healthy diet and exercises for at least 30 minutes per day. His immunizations are up to date. He has no significant family history for any major illnesses. He undergoes a screening colonoscopy; 2 tubular polyps (0.4 cm and 0.6 cm) are discovered and resected. Question What is the correct statement about future screening for this patient based on the National Comprehensive Cancer Network guidelines?

Correct answer: Repeat colonoscopy within 5 years Explanation This patient is at an increased risk of colon cancer due to polyps found on colonoscopy. He has low-risk adenomatous polyps (tubular, <1 cm); therefore, he should undergo a repeat colonoscopy within 5 years as per the National Comprehensive Cancer Network (NCCN) colon cancer screening guidelines. In cases of advanced or multiple adenomatous polyps (villous, >1 cm, 3 - 10 polyps, high-grade dysplasia), colonoscopy should be repeated within 3 years. For incomplete polypectomy, or polypectomy of large sessile polyps, colonoscopy should be repeated within 2 - 6 months; timing is also dependent on endoscopic and pathologic findings. Double contrast barium enema is reserved for patients who cannot undergo colonoscopy. This patient is healthy and does not have contraindications for colonoscopy.

Question A 4-year-old boy presents with colicky pain, a history of irritability, and a 2-day history of lethargy. There is also history of rectal bleeding and passage of "currant jelly" stool for the past 2 days. Plain abdominal film shows evidence of obstruction, and barium enema detects coiled-spring appearance to the bowel. Intussusception is diagnosed. Investigations have revealed that the lead point causing the intussusception is an ileal lipoma. What is the definitive management in this case?

Correct answer: Resection of the bowel Explanation Intussusception is a common cause of childhood intestinal obstruction in which there is invagination of a segment of the bowel (usually small bowel) into the lumen of a more distant bowel segment. Peristalsis causes the invaginated segment, called the intussusceptum, to be carried distally and along with it the mesentery and vessels also become involved with the intraluminal loop. This segment is squeezed within the engulfing segment, which is called the intussuscipiens. Most often the cause is idiopathic. However, in neonates and in children older than 3 years, a mechanical lead point is usually the cause. Lead points are nonidiopathic causes of intussusception and include Meckel's diverticulum, lymphoma or leukemia of the bowel, Peutz-Jeghers syndrome, polyps, lipomas, post-abdominal trauma, and Henoch-Schönlein purpura. Bowel resection and primary anastomosis are required when pathological lead points are present. Also, surgery may be required when intussusception occurs outside the ileocolic region. Delay in presentation also necessitates surgery, as reduction by air or contrast would be ineffective or associated with complications. Air enema reductions are less likely to succeed in patients less than 3 months old and in those more than 3 years old. Supportive therapy by rehydration is not the definitive management. Adequate rehydration and stabilization are required for all cases of intussusception. Reduction by contrast enema and observation for spontaneous reduction would not be beneficial in a child with a lead point presenting late.

Case A 2-year-old girl has a 2-week history of small wounds on the sides of her mouth. On examination, you note she has pale conjunctivae, a magenta tongue, and macerated lips (in addition to the angular stomatitis). Her mother notes that she is a very fussy eater; she is underweight and was diagnosed in infancy with a malabsorptive syndrome. Question The child's symptoms are most likely the result of what nutrient deficiency?

Correct answer: Riboflavin deficiency Explanation In cases of riboflavin (or Vitamin B2) deficiency, patients present with angular stomatitis and cheilosis. On examination, they are pale, have atrophic glossitis, and the tongue may appear magenta. They may also have a sebaceous dermatosis with greasy material in their nasolabial folds, alae nasi, and genitals. Causes include inadequate dietary intake and malabsorption. Dietary sources of riboflavin include milk, cheese, meat, green vegetables, and enriched cereal products. Vitamin B1 (or thiamine) deficiency results in beriberi, which is characterized by: a bilateral symmetric peripheral neuropathy beginning in the legs Wernicke-Korsakoff syndrome, which is comprised of nystagmus, ophthalmoplegia, ataxia, memory loss, and confabulation congestive heart failure with tachycardia, peripheral edema, and cardiomegaly In cases of vitamin K deficiency, patients present with bleeding tendencies, such as epistaxis, menorrhagia, and hematuria. The prothrombin time (PT) and the activated partial thromboplastin time (aPTT) are usually prolonged. In cases of vitamin D deficiency, children can present with an inability to walk unsupported due to muscle weakness and lower limb skeletal deformities, such as genu varum and genu valgum. They may also have prominent costochondral junctions. In cases of iron deficiency, patients can present with feeling weak, dizzy, and tired; they may experience syncope. On examination, they have pale conjunctivae and koilonychia (spooning of the nails).

Case A woman brings in her exclusively breastfed, 7-month-old male infant for a routine checkup. You see a well-fed, African American boy with pigeon breast, depression along the line of diaphragmatic insertion into the rib cage, and costochondral thickening that looks like a string of beads. On an x-ray, the bones are translucent, and the skull bones are thinning. Question What is the most likely diagnosis?

Correct answer: Rickets Explanation These symptoms are classic for rickets. Pigeon breast is pectus carinatum (sternum protrusion), Harrison's groove is a depression along the line of diaphragmatic insertion into the rib cage, rachitic rosary is costochondral thickening, and craniotabes is a thinning of skull bones and radiolucency of bones. This child has 2 risk factors for rickets. His dark skin pigmentation interferes with the penetration of ultraviolet light in the skin, and he is exclusively breastfed. Breast milk of African American women contains only about 25 IU/L vitamin D, which is far below the 400 IU/day recommended for infants by the American Academy of Pediatrics. Osteomalacia would be the right diagnosis for an adult showing these symptoms. Primary hyperparathyroidism is caused by either tumor or hyperplasia of the parathyroid glands. In this case, it would be secondary hyperparathyroidism caused by hypocalcemia. Osteitis deformans or Paget's disease is an idiopathic increase of osteoblast and osteoclast activity, usually in elderly people. It presents with diffuse fractures and bone pain. Silverman syndrome is congenital pigeon breast often combined with heart abnormalities. Though theoretically possible, it would not explain the patent's other signs and symptoms, and there is no evidence of heart problems.

Case A 38-year-old man was diagnosed with ulcerative colitis 7 years ago. He has been relatively stable on a course of daily sulfasalazine. The patient's initial diagnosis was made via colonoscopy, which he described as a traumatic experience. He is reluctant to undergo any other colonoscopic exams. Question Regarding screening colonoscopy for colon cancer in a patient with ulcerative colitis, which of the following statements is most accurate?

Correct answer: Screening should be performed 8 - 10 years after the initial diagnosis. Explanation There is an increased risk of cancer in patients with ulcerative colitis. Surveillance colonoscopy with multiple biopsies is recommended every 1 to 2 years beginning 8 - 10 years after the diagnosis of pancolitis or 12 to 15 years after diagnosis of left-sided colitis according to guidelines of the American Gastroenterological Association.

Case A 3-year-old boy presents with his mother; she reports that he is unable to walk without support. He is an only child who rarely leaves his tenth-floor apartment home. On examination, he has a rachitic rosary and genu valgum. Question What is the most likely diagnosis?

Correct answer: Vitamin D deficiency Explanation In cases of vitamin D deficiency, children can present with an inability to walk unsupported due to muscle weakness and lower limb skeletal deformities such as genu varum (bow legs) and genu valgum (knock knees). On examination, they may have prominent costochondral junctions (rachitic rosary) and indentation of the lower ribs at their diaphragmatic attachment (Harrison's groove). Causes include inadequate dietary intake and inadequate exposure to sunlight. They should be encouraged to be exposed to ultraviolet irradiation of the skin as well as increase their dietary sources of vitamin D, including fortified milk, fish liver oils, butter, egg yolk, and liver. Vitamin B1 (or thiamine) deficiency results in beriberi, which is characterized by: a bilateral symmetric peripheral neuropathy beginning in the legs Wernicke-Korsakoff syndrome, which is comprised of nystagmus, ophthalmoplegia, ataxia, memory loss, and confabulation congestive heart failure with tachycardia, peripheral edema, and cardiomegaly In cases of riboflavin (or vitamin B2) deficiency, patients present with angular stomatitis and cheilosis. On examination, they are pale, have atrophic glossitis, and the tongue may appear magenta. In cases of vitamin K deficiency, patients present with bleeding tendencies, which include epistaxis, menorrhagia, and hematuria. The prothrombin time (PT) and the activated partial thromboplastin time (aPTT) are usually prolonged. In cases of iron deficiency, patients can present with feeling weak, dizzy, and tired; they may experience syncope. On examination, they have pale conjunctivae and koilonychia.

Case A 32-year-old woman presents with fatigue, pruritus, and yellowing of the white of her eyes. Investigations reveal anti-mitochondrial antibodies and elevated alkaline phosphatase (ALP), γ -glutamyl transpeptidase (GGTP) levels. Primary biliary cirrhosis is diagnosed, and she is treated with ursodeoxycholic acid, corticosteroids, and cholestyramine. 1 month later, she returns with abdominal discomfort and constipation. Her symptoms cease when she stops taking cholestyramine. However, the itching is severe when the drug is stopped. Question In order to relieve this patient's itching, what drug may be used as an alternative to cholestyramine?

Correct answer: Rifampin Explanation Rifampin has been found to relieve pruritus in biliary disease. Rifampin is an antimycobacterial agent that can be used in patients who cannot tolerate cholestyramine. It is started at a dose of 150 mg per day and may be gradually increased to 600 mg per day. The mechanism of action of rifampin in pruritus of biliary disease is explained as follows: Being a strong enzyme inducer of the microsomal enzyme inducer system, it promotes the metabolism of endogenous pruritogenic compounds. It competes with the uptake of bile salts in the hepatocytes. It may alter the synthesis of secondary bile acids in the intestinal lumen by its antimicrobial action and thus reduce the amount of lithocholic acid. The other choices are all antimycobacterial agents with no role in pruritus of biliary disease.

Case In the winter, an 11-month-old infant presents with a 2-day history of vomiting, diarrhea, and fever. His temperature is 104° Fahrenheit. Clinically, he is significantly dehydrated; his white blood cell count is 5400 cells/mm3 with a normal differential. His stool and urine are negative for white blood cells. Question What is the most likely cause of gastroenteritis in this child?

Correct answer: Rotavirus Explanation The most common cause of acute diarrhea is a viral infection; a viral etiology is also suspected in a patient with a normal white blood cell count. In the first 2 years of life, especially in the winter months, rotavirus is a common cause of gastroenteritis. Epidemics are not uncommon; therefore, rotavirus vaccinations are now considered standard at 2, 4, and 6 months of age. Toxigenic E. Coli is rare in developed nations. Clostridium difficile is common with prior antibiotic use. Norwalk-like viruses are more common in school-age children and can cause sporadic outbreaks of gastroenteritis in food and waterborne infections. Shigella gastroenteritis occurs as a sporadic outbreak due to food or water contamination; it is associated with an elevated white blood cell count.

Case Ico-delete Highlights Early one afternoon, a male adolescent presents with lower right abdominal pain, anorexia, nausea, and vomiting. The pain has been worsening since the onset of symptoms of nausea and vomiting in the morning. There is no known gastrointestinal disease in the history or in the immediate environment. His temperature is 101 degrees F. The attending physician suspects appendicitis and begins the physical examination. Question What sign would support the diagnosis of appendicitis?

Correct answer: Rovsing's sign Explanation The correct answer is Rovsing's sign. Rovsing's sign refers to pain in the right iliac fossa on palpation of the left iliac fossa. It indicates local peritoneal irritation. When the left iliac fossa is palpated, the bowel contents are pushed towards the ileocaecal valve, increasing the pressure around the inflamed appendix. This produces pain in the right iliac fossa. Presence of Rovsing sign makes the diagnosis of appendicitis more likely; however, its absence does not rule out appendicitis. Cullen sign and Grey Turner sign are seen in severe pancreatitis. Cullen sign is the bluish discoloration of the periumbilical area, and Grey Turner sign refers to the bluish discoloration of the flanks. Both are caused by retroperitoneal blood leak from a hemorrhagic pancreas. Murphy sign is a specific but not sensitive test for cholecystitis. On palpation of the right upper quadrant, tenderness and inspiratory pause are elicited. Dance sign refers to the emptiness in the right lower quadrant in intussusception. It is a difficult sign to palpate and is best elicited when the infant is not crying, between spasms of colic.

Case A 48-year-old woman presents with a 6-month history of difficulty in swallowing. A barium swallow reveals an area of stricture in the lower esophagus just above the gastroesophageal junction. She has an upper endoscopy performed, and a biopsy of the lower esophagus is taken; it shows no acute inflammation or ulceration, only submucosal atrophy along with fibrosis of smooth muscle. Question What do these findings most strongly suggest?

Correct answer: Scleroderma Explanation Patients with progressive systemic sclerosis, with either the CREST syndrome or diffuse scleroderma, have fibrosis in the gastrointestinal tract; most often, it is located in the esophagus. Scleroderma is a serious, progressive disease that affects the skin and connective tissue (e.g., cartilage, bone, fat, and the tissue that supports the nerves and blood vessels throughout the body). Scleroderma is also frequently called systemic sclerosis. Iron-deficiency anemia can be associated with esophageal webs. This is called Plummer-Vinson syndrome, and it is quite rare. Barrett's esophagus is a columnar metaplasia of the esophageal epithelium that occurs with chronic reflux; the esophagus is lined with gastric mucosa, and it is usually diagnosed endoscopically. Portal hypertension leads to the appearance of esophageal varices in the lower esophagus that can erode and bleed profusely. Mallory-Weiss syndrome is an esophageal mucosal laceration; it usually presents with marked vomiting, and it is typically found in alcoholics. Mallory-Weiss syndrome is bleeding from an arterial blood vessel in the upper gastrointestinal tract, and it is caused by a mucosal gastric tear at or near the point where the esophagus and stomach join.

Question A 44-year-old woman has symptoms of alternating constipation and diarrhea for 18 months. These symptoms are followed by rectal pain for the next 6 months. She seeks treatment, but she is prescribed only topical medicine and pain killers. She ends up with metastatic rectal cancer and dies within 2 months. Her 41-year-old sister is furious with what she conceives as poor medical care for her sibling and decides that she will not go through the same. What advice should you give the sister regarding screening for colon cancer?

Correct answer: Screening colonoscopy right away and repeat every 3-5 years if normal Explanation The recommendations for screening someone who has a first degree relative who has had colorectal cancer diagnosed at less than 60 years of age include screening colonoscopy starting at age 40 years, or 10 years younger than the earliest diagnosis in their family, whichever comes first. It should be repeated every 5 years. The same is advised for anyone with 2 first degree relatives with colorectal cancer at any age. Fecal occult testing annually alone is not a sensitive screening test for colon cancer. It should be used with sigmoidoscopy every 3-5 years, or with colonoscopy every 10 years for screening in the general population, starting at age 50.

Question A 72-year-old man presents with swollen and bleeding gums. The examining doctor finds anemia, perifollicular hemorrhages, and loose teeth. A careful history reveals that the patient has been living alone since the death of his wife 18 months ago. His diet consists largely of cola and hot dogs or cheese sandwiches from a nearby deli. What is the most likely diagnosis?

Correct answer: Scurvy Explanation This is a classic case of vitamin C deficiency. Vitamin C (ascorbic acid) is a water-soluble antioxidant. It is necessary for hydroxylation of proline in collagen synthesis, degradation of tyrosine, epinephrine synthesis, and bile acid formation. Defective collagen in vessel walls leads to bleeding, producing ecchymoses and petechiae. Vitamin C is found in almost all fruits and vegetables, especially in citrus fruits. Deficiency is seen mainly in infants fed boiled milk, the elderly, and single people. Treatment involves administration of 250 mg of ascorbic acid daily, followed by 40 mg daily maintenance doses. The patient should be encouraged to eat fresh fruit and vegetables.

Question A 56-year-old man presents with a 10-day history of epigastric pain and vomiting. He states that similar symptoms have appeared frequently throughout the past year and that he was diagnosed with a duodenal ulcer 8 months ago. He was given eradication therapy for a Helicobacter pylori infection and urea breath test confirmed successful eradication. With the discontinuation of protein pump inhibitors, his symptoms reappeared. He has a surgical history of an incomplete vagotomy at age 22 for the treatment of duodenal ulcer. He denies intake of salicylates or any other drugs that could damage duodenal mucosa or be a causative agent for recurrent ulcers. General physical findings are unremarkable, except for the presence of epigastric and periumbilical pain on palpation. Neither routine laboratory analyses nor an abdominal echo-sonogram reveal abnormalities. Endoscopy reveals the presence of an ulcer in the first portion of the duodenum. His BAO is 15.3 mEq/hr, and his BAO/MAO (basal/maximal acid output) ratio is 0.75. His serum gastrin level is 268 pg/mL (non-fasting normal is 200 pg/mL). What test should be used first to confirm the suspected diagnosis of Zollinger-Ellison syndrome?

Correct answer: Secretin stimulation test Explanation Hypergastrinemia is always present in patients with Zollinger-Ellison syndrome but may also be present in other diseases and conditions. In order to discriminate patients with Zollinger-Ellison syndrome from patients with hypergastrinemia caused by other factors, a stimulatory test with secretin is performed. The test, which is highly sensitive and specific, is based on the fact that secretin administration causes a striking elevation of serum gastrin level in patients with Zollinger-Ellison syndrome that cannot be noted in persons with hypergastrinemia caused by other factors. The repeated determination of serum gastrin level is of no diagnostic value, particularly if vagotomy (which influences serum gastrin level) was formerly performed. Calcium infusion test may also be utilized for the diagnosis of Zollinger-Ellison syndrome. However, this test is less sensitive and less specific than secretin test, and its performance is associated with more side effects and potential false negatives. Endoscopic echo-sonography may be of valuein detection of tumor site once diagnosis of Zollinger-Ellison syndrome is firmly established. Also, CT scan of the pancreas may be helpful in detecting tumor lesiona, but it is of no value in detection of hormonal potential of the tumor.

Case A 65-year-old man presents with a bulge in his lower abdomen that has been present for 5-10 years. He can usually push this bulge back, but, about 4 hours earlier, it came out and he was not able to push it back as easily at home. There is a dull, aching pain, but no nausea or vomiting. He admits to straining at the stool and getting up twice a night to urinate. He smokes 1 pack of cigarettes per day. On physical examination, his vital signs are within normal limits; his pulse oximetry is 91%. There is no jugular vein distention. Chest shows an increased A-P diameter, but breath sounds are good. Heart sounds are distant, and the rate and rhythm indicate normal sinus. There is an obvious bulge in the right lower quadrant extending into the right scrotal sac. The abdomen itself is soft, flat, and non-tender. Bowel sounds are slightly hyper active; there are no rushes or tingling sounds. The bulge is tender to palpation and the patient squirms in pain during your attempts to manually reduce the bulge and he asks you to stop. Bowel sounds are present within the mass. A somewhat enlarged, non-tender prostate is detected on rectal examination. Brown stool is obtained and is negative for blood. There is no abnormality on EKG. Chest x-ray shows some hyperinflation, but no infiltrates. An x-ray, flat and upright, of the abdomen shows some dilated small bowel extending into the right scrotal sac, but signs of obstruction are absent. Routine lab and urine studies are within normal limits. You start an intravenous line of 0.9 N/S to run at keep-open rate. Question Ico-delete Highlights

Correct answer: Sedate the patient well and attempt to manually reduce the mass Explanation The concern in this patient is that he may have an incarcerated indirect hernia. Direct hernias do not usually incarcerate and come straight through the abdominal wall. Indirect hernias, by definition, come down the inguinal canal into the scrotal sac. Intestinal contents accompany the sac in either one. This patient's lung signs and signs of prostate enlargement are both impediments to emergency surgery, and they are additional reasons to delay repair to a more elective time if possible. Though patient tried to reduce the hernia, it is wise to attempt reduction in the clinic. Every reasonable attempt should be made to reduce this hernia using manual compression after adequate sedation. A warm compress on the area may help reduce muscle spasm and relax the abdominal musculature. There are times when either a nasogastric tube and/or a long intestinal tube are warranted. This patient has no signs of intestinal obstruction (vomiting, constipation, etc.). The minimal dilatation of the small bowel is an accompanying sign of an incarcerated hernia. An obstructed segment of bowel may become incarcerated and/or strangulated; when reduced or replaced in the peritoneum, signs and symptoms of mechanical obstruction occur. Of course, should it not be possible to reduce the hernia, obstruction and strangulation both may develop. Strangulation occurs when blood flow (venous, arterial, or both) is closed off, and the bowel segment dies. Then an emergency exists, and repair must occur upon recognition. A Foley catheter has no value unless there is urinary outlet obstruction.

Case A 47-year-old man presents to an urgent care center with 18 hours of abdominal pain, nausea, vomiting, and chills. He is a single construction worker, denies smoking, and has at least a 10-year history of drinking 2 - 4 alcoholic beverages daily. A series of lab work is performed on the patient to evaluate his abdominal pain prior to abdominal imaging. Question What laboratory results would be most indicative of the patient suffering from acute pancreatitis?

Correct answer: Serum amylase 310 U/L and serum lipase 760 U/L Explanation The correct answer is a serum amylase of 310 U/L and a serum lipase of 760 U/L, as these levels are significantly elevated, which is indicative of acute pancreatitis. The normal range for serum amylase is 30-220 U/L, and the normal range for serum lipase is 0-160 U/L. Both levels being significantly elevated is typically seen in a patient with acute pancreatitis. Serum lipase is both more sensitive and more specific than serum amylase for diagnosis of acute pancreatitis, but more so when the serum lipase is at least 3 times the normal level (as is with this case). However, various other biliary and intestinal diseases can also alter these lab results. Blood work results with a serum amylase of 250 U/L and a serum lipase of 110 U/L do not indicate acute pancreatitis. The serum amylase is slightly elevated, but the serum lipase is normal. This can occur for many gastrointestinal-related disorders and renal failure, but these lab results could also commonly be found in a patient with chronic (rather than acute) pancreatitis. Lab results of serum aspartate aminotransferase (AST) 32 U/L and serum alanine aminotransferase (ALT) 29 U/L are actually normal results for these particular lab tests. The AST and ALT can be used to identify hepatocellular diseases of the liver. However, mildly increased levels can also be seen in patients with acute pancreatitis. Lab results showing serum aspartate aminotransferase 120 U/L and serum alanine aminotransferase 40 U/L can be seen in patients with various hepatocellular levels as both are increased from normal. These results are possible in a patient with acute pancreatitis, but are not the most indicative of the answer choices. Cirrhosis is a disease of the liver that can often occur in patients with large alcohol intake and often causes lab results such as these. Specifically, results that show an AST level that is 3 times that of the ALT level. Lab results showing serum white blood cell count 14,000/mm3 and serum total bilirubin 1.8 mg/dl could be seen with various infections separately. However, the combination of both results limits the possible disorders. The white blood cell count could be elevated in a patient with acute pancreatitis, and the serum bilirubin could also be elevated in patients with acute pancreatitis when it is associated with alcoholic hepatitis. However, these results are not the most indicative of acute pancreatitis when compared to the other options.

Case A 20-year-old woman presents with a 1-day history of passing bloody diarrhea. On examination, her temperature is 39°C (102.2°F). A stool examination reveals polymorphonuclear leukocytes, but no trophozoites. Question What is the causative organism?

Correct answer: Shigella dysenteriae Explanation In cases of food poisoning caused by Shigella dysenteriae, patients present with bloody diarrhea; symptoms usually start 16 hours after ingesting contaminated foods. They may also be febrile. Common sources of infection include potato salad and raw vegetable salads. A stool examination reveals polymorphonuclear leukocytes. The diagnosis is confirmed by culturing the organism from stool. Management includes rehydration and, in severe cases, antibiotics like trimethoprim-sulfamethoxazole or ampicillin. In cases of emetic Bacillus cereus food poisoning, patients develop vomiting 1-6 hours after ingesting contaminated food (e.g., reheated fried rice). In the diarrheal form, they develop watery diarrhea 8-16 hours after ingestion. There are no fecal leukocytes or trophozoites. In adults with Norwalk virus food poisoning, the watery diarrhea is more prominent than vomiting. In children, the reverse is true. The incubation period is 1-3 days. There are no fecal leukocytes or trophozoites. Entamoeba histolytica colitis develops 2-6 weeks after ingesting contaminated water or food. Patients usually present with bloody and mucoid diarrhea. Stool examination reveals hematophagous trophozoites and cysts. Patients with enterotoxigenic Escherichia colifood poisoning present with watery diarrhea 12-72 hours after ingesting contaminated water or raw fruit salads. Examination of the stool reveals no fecal leukocytes or trophozoites.

Case A 27-year-old man presents with a burning sensation in his chest, dry cough, hoarseness, and a sensation of a lump in his throat. He also experiences belching followed by a sour liquid taste in the mouth. Symptoms started 2 hours ago, and he thinks that they are worsening. He has had several similar episodes over the past 2-3 years that recently have become more frequent, probably because there is more stress in his life now. Namely, each episode of chest pain is associated with an anticipated or experienced stressful event (exams, job interviews, etc.). The rest of his history, family history, and physical are noncontributing. His EKG and chest X-ray are normal. Question What will be the next step in management?

Correct answer: Start proton pump inhibitor therapy Explanation The patient most likely has gastroesophageal acid reflux or gastroesophageal reflux disease (GERD). In a 27-year-old patient, coronary insufficiency is not high on your list of differentials of the chest pain, particularly with a normal EKG. The same applies to the patient who has neither symptoms nor signs of pulmonary disorder and has a normal chest X-ray. High-dose proton pump inhibitor (PPI) given over 4 weeks is empiric therapy that should both relieve the pain and confirm your diagnosis of GERD. Chronic cough is a very common symptom of GERD. The most common causes of chronic cough in adults are upper airway cough syndrome, asthma, and gastroesophageal reflux disease, alone or in combination. Screening for Barrett esophagus is not routinely recommended, but may be considered in Caucasian men 50 years or older who have had GERD symptoms for at least 5 years. Esophageal PH or impedance and pH studies will be indicated in patients who have persistent symptoms and no convincing response on proton pump inhibitor therapy. It may also differentiate GERD from the nonacid reflux episodes and chest pain. Those investigations are not necessary in your patient. Endoscopy will probably be the preferred procedure when patients with GERD have "red flag" symptoms: dysphagia, odynophagia, weight loss, persistent or progressive heartburn, the presence of symptoms while on antacid therapy, and a patient age of more than 45 years. Your patient does not have these 'red flags.' Even your patient claims that stress is the main cause of his problems, and a significant number of patients with GERD have underlying depression, anxiety, and panic disorder. Psychiatric consultation (if any) comes only after PPIs. TSH will show you if your patient has problems with the function of the thyroid gland. Some of the symptoms (dry cough, hoarseness, and a sensation of a lump in a neck) may be associated with thyroid disease, but when symptoms come and go, they are not likely the consequence of thyroid disease.

Question What statement is true concerning balloon tamponade of esophageal varices?

Correct answer: Should be preceded by endotracheal intubation to protect the airway from aspiration Explanation The use of balloon tamponade, most commonly with the Sengstaken-Blakemore tube, for esophageal varices is temporarily effective in controlling bleeding in 80-90% of patients. However, rebleeding will occur in 60% of patients and therefore is not effective for long term control. This is not a benign procedure, there is a significant risk of aspiration and esophageal or gastric perforation. Therefore, elective endotracheal intubation prior to placement of the tube is preferential. Careful placement of the tube with meticulous monitoring of balloon pressures is essential to minimize the risks of perforation. Radiologic confirmation of the tube position should always precede inflation of the balloons and frequent radiologic studies should be performed to assure maintenance of this position. In a patient with a history of esophageal varices with a new upper gastrointestinal bleed, 25% of the time it will be from a source other than the varices. Therefore, if the patient is not having exsanguinating hemorrhage, it is useful to perform upper endoscopy to confirm the source of bleeding and possibly treat the varices with sclerotherapy, which can control the bleeding in 90-95% of cases.

Case A 46-year-old woman presents with nausea, vomiting, crampy abdominal pain, and loud bowel sounds for the past several hours. She denies weight loss. She has had 1 normal bowel movement since the symptoms began, but it did not help her symptoms. She has a past surgical history of an abdominal hysterectomy 7 years prior. On physical exam, she is afebrile, with hyperactive and high-pitched bowel sounds localized to the left upper quadrant. She also has mild, diffuse abdominal tenderness. Question What is the most likely diagnosis?

Correct answer: Small bowel obstruction Explanation Small bowel obstruction typically presents with localized high-pitched bowel sounds with crampy abdominal pain, nausea, and vomiting. The bowel sounds will diminish if complete obstruction occurs. Whipple's disease is an infectious disorder characterized by fever, lymphadenopathy, arthralgias, weight loss, and chronic diarrhea. Patients with diverticulitis often have more severe, localized abdominal tenderness on exam; they typically present with fever and (possibly) peritoneal signs. Acute paralytic ileus typically presents with no bowel sounds or very diminished bowel sounds. Irritable bowel syndrome is not typically associated with nausea and vomiting, and the abdominal pain is relieved with defecation.

Case A 35-year-old man presents with intense itching around his anus that has been worsening over the last several weeks. He states that he has noticed an increasingly severe and tearing pain in the anal area with each bowel movement. He would rank this pain as a 10/10 on a pain scale. This intense pain makes him not want to have any bowel movements. He admits to only 1 episode of a small amount of bright red blood on the toilet paper as well as on the stool itself. The patient denies fever, diarrhea, or ever being diagnosed with inflammatory bowel disease. Question After confirming the suspected diagnosis via physical examination, you educate the patient to avoid what in order to help initiate healing?

Correct answer: Spicy foods Explanation The patient should avoid spicy foods. This patient is most likely suffering from an anal fissure. Anal fissures most often affect infants as well as middle-aged individuals. The majority of fissures are considered primary and caused by local trauma such as passage of hard stool, prolonged diarrhea, vaginal delivery, or anal sex. Presentation of anal fissures is a tearing pain accompanying bowel movements as well as bright red rectal bleeding that is limited to a small amount noted on the toilet paper or surface of the stool. The patient described all of these components. Patients will also note perianal pruritus or irritation, which he also admits to experiencing. The best diagnostic approach to confirming an anal fissure is usually based on the history of pain with defecation as well as simply the physical examination finding of a superficial tear noted in the anoderm; this type of tear, whether located posteriorly or laterally, is considered a pathognomonic feature of an acute anal fissure. The examiner should carefully spread the buttocks apart and examine the area gently. Due to the trauma and irritation that has occurred once an anal fissure is established, the key component in this patient is to have him perform actions that will allow healing and hopefully prevent recurrence or other complications in the future. Consuming both an appropriate amount of stool softeners (fiber and fiber containing products) as well as staying appropriately hydrated daily is urged to these patients. It should be emphasized the patient should utilize these healthy habits every day. Physical activity is also recommended due to the regular activity helping regulate bowel movement. Sitz baths, or sitting in/taking warm baths for a few minutes (up to 20 minutes) a few times a day is also strongly recommended to aid the overall healing process. Patients should also be instructed to avoid spicy foods, especially if this is an acute fissure that patient presents with, due to the fact that spicy food can worsen symptoms.

Case A 35-year-old woman presents with vomiting and watery diarrhea that began 4 hours after eating a chicken sandwich at her workplace cafeteria. She reports that 2 other workmates have also developed similar symptoms. There are no leukocytes or cysts on examination of her stool. Question What is the causative organism?

Correct answer: Staphylococcus aureus Explanation In cases of Staphylococcus aureus food poisoning, patients present with severe nausea and vomiting. They may also develop diarrhea, abdominal pain, and occasionally headache and fever. Stools are watery but not bloodstained. The onset of symptoms is 1-6 hours after ingesting the contaminated food. Common culprits include processed meat, dairy products, and potato salads, especially when prepared by food handlers with skin infections that are left at room temperature. Several persons may be similarly affected. There are no fecal leukocytes or trophozoites on examination of the stool. Management includes intravenous replacement of fluid and electrolytes. In cases of Clostridium perfringens, food poisoning, patients present with watery diarrhea 8-24 hours after ingesting contaminated food like inadequately cooked meat or legumes. No fecal leukocytes or trophozoites are present on stool examination. In cases of Escherichia coli serotype O157:H7 food poisoning, patients present with watery diarrhea, which can become bloodstained. Sources of infection include undercooked beef. A stool examination reveals polymorphonuclear leukocytes. Giardia lamblia can be acquired by drinking contaminated water. Patients present with watery diarrhea. The incubation period is around 1 week. There are trophozoites in the stool, but no leukocytes. Patients with Salmonella typhi food poisoning usually present with bloody diarrhea. Sources of infection include beef, poultry, eggs, and dairy products. Examination of the stool reveals mononuclear leukocytes.

Case A 27-year-old man eats a meal consisting of ham casserole with potatoes and green beans. 3 hours later, he abruptly begins vomiting; he experiences profuse watery diarrhea, abdominal cramping, and headache. Examination determines that he is diaphoretic and salivating; he has normal vital signs. The patient is encouraged to rest and take copious fluids. The following evening his symptoms subside, and he is able to return to routine activities within 24 hours of becoming ill. Question This food-borne illness is most likely to be caused by what organism?

Correct answer: Staphylococcus aureus Explanation The correct response is Staphylococcus aureus. Staphylococcal food poisoning results from the ingestion of heat-stable enterotoxin B. An acute clinical syndrome of salivation, vomiting, and abdominal cramps is followed by watery diarrhea. The onset of vomiting and nausea within the 1st hours following ingestion is characteristic of staphylococcal intoxication (refer to the table). S. aureus are tolerant of high-salt concentrations, making salted meats (e.g., ham) a common source of intoxication. Clostridium perfringens produces several enterotoxins that cause diarrhea. 1 of the most common agents of food-borne illness, it is transmitted primarily by meat and poultry. Symptoms include watery diarrhea, vomiting, and abdominal cramps (usually without fever). Unlike the rapid onset of S. aureus, symptoms of clostridial food poisoning occur 12 - 18 hours after ingestion. Clostridium difficile is also capable of producing an enterotoxin and a cytotoxin, and it is not associated with food-borne illness. This organism causes colitis frequently associated with antibiotic use. Patients, usually the elderly or hospitalized, present with mild to moderate diarrhea and abdominal cramps; they may develop more serious illnesses such as pseudomembranous colitis or an acute abdomen. Shigella causes enteritis within 12 - 18 hours after ingestion of as few as 200 organisms. This clinical syndrome consists of acute onset fever and abdominal cramps which are followed by an enterotoxin-mediated acute onset of copious watery diarrhea. Within 24 - 48 hours, dysentery occurs as a result of mucosal invasion. Shigella sonnei is the most common etiologic agent of shigellosis in industrialized countries. Yersinia enterocolitica is transmitted by milk, water, or solid foods; it is often outbreak-associated. Patients experience diarrhea, fever, and abdominal pain 16 - 72 hours after consuming food. Y. enterocolitica may mimic acute appendicitis as a result of mesenteric adenopathy or terminal ileitis. Refer to the information below.

Case Ico-delete Highlights A 39-year-old woman (CFO of a finance company) is seen for upper abdominal discomfort. She experiences heartburn, belching, and epigastric burning pain on and off for the last several months. She does say that her food habits are irregular, due to the high stress at her job, and her caffeine intake is more than she thinks is healthy. She does not smoke or drink, has no medical problems, and she is not taking any medications. On exam, she is afebrile, pulse is 78/min, BP 110/70, height is 5'6", and weight 135 pounds. Further exam reveals some epigastric tenderness but is otherwise normal. She is advised a trial of lansoprazole by her doctor and follow-up in 2 weeks. However, she says that she has been reading about her symptoms and is concerned about infection with Helicobacter pylori. Question What is the most cost-effective initial measure to diagnose this?

Correct answer: Stool antigen test Explanation The American College of Gastroenterology (ACG) H. pylori guidelines recommend the following patients should be tested for H. pylori infection: anyone with active peptic ulcer disease (PUD) or a past history of PUD (unless documentation of previous H. pylori cure) patients with low-grade gastric mucosa-associated lymphoid tissue (MALT) lymphoma patients with a history of endoscopic resection of early gastric cancer Non-endoscopic testing is recommended for patients under 60 who have uninvestigated dyspepsia and no alarm features. Of the available non-invasive tests, stool antigen testing is the most cost-effective test and has a sensitivity of 94% and specificity of 97%. It can be used to both establish an initial diagnosis of H. pylori and to confirm eradication. Referral to a specialist for endoscopy is indicated in patients with known peptic ulcer disease, in suspected MALT lymphoma, and follow-up of ulcers. Endoscopic based diagnostic testing of H. pylori should only be done if endoscopy is indicated for other reasons. The American College of Gastroenterology (ACG) H. pylori guidelines further recommend that patients who test H. pylori positive should be offered treatment for the infection. Empiric treatment for H. pylori, however, is not recommended; treatment should not be undertaken

Case A 40-year-old woman presents with rectal pain. She reports that the pain is especially severe with bowel movements. On exam, she is noted to have an anal fissure. Question What is the best initial treatment?

Correct answer: Stool softeners Explanation The correct response is stool softeners. Anal fissures are usually caused by trauma to the anal canal due to the passage of hard stool. Most are found in the posterior midline of the anus, but some are found in the anterior midline. The ones found laterally may be associated with TB, syphilis, or AIDS. Hydrocortisone creams are used to decrease the inflammatory process usually associated with inflamed hemorrhoids. Anal fissures are initially treated conservatively using stool softeners and bulking agents to ease the passage of a bowel movement. If there is no improvement after 6 weeks of conservative therapy, surgery is usually required. The operation for non-healing anal fissures is an internal anal sphincterotomy. Fissurectomy is not a procedure used for the treatment of anal fissures. Hemorrhoidectomy is the operative resection of hemorrhoids.

Case A 38-year-old man presents with a 2-day history of a mass and severe pain in his scrotum. Physical examination reveals that his right testicle appears much larger than his left. On palpation, you note a small hole in his inguinal canal, and you are unable to place the contents into the canal. The contents of the hernia appear ischemic. Question What is the best description of this hernia?

Correct answer: Strangulated Explanation The clinical picture is suggestive of an inguinal hernia. The hernia has become ischemic, which is better known as a strangulated hernia. In a reducible hernia, the contents would be able to be placed back into the abdominal cavity with simple manipulation. A recurrent hernia is a hernia that has been previously repaired surgically and has now returned. There is no history of a previous hernia in this patient. An irreducible (also known as incarcerated) hernia is when the hernial contents cannot be returned to their normal site with simple manipulation. This type of hernia would cause edema and entrapment due to impaired venous return.

Case A 15-year-old boy was healthy until 3 months ago. He has been having episodes of crampy abdominal pain and explosive liquid stools 2-3 times a day. He has had intermittent fevers and has noted blood in the stool. He has a documented 15-lb weight loss. Biopsied obtained at colonoscopy revealed inflammation with distorted crypt architecture and erosion of the mucosal later. Question What is the best way to start treatment?

Correct answer: Sulfasalazine Explanation This patient has ulcerative colitis. Ulcerative colitis is an inflammatory bowel disease that affects the rectum and colon. The exact cause is unknown, but the gut immune system is inappropriately triggered and leads to T-cell mediated inflammation and damage to the superficial and mucosal layers of the larger bowel. Classic symptoms include bloody diarrhea and abdominal pain. Biopsies of the colon, especially during acute exacerbations, reveal diffuse inflammation, distorted crypt architecture, and damage to the lining of the colon that is restricted to the mucosal layer. Patients with lactose intolerance do not usually present with bloody stools and fever. The microscopic findings are also not characteristic of food intolerance. Given the diagnosis, it would be most appropriate to start treatment with sulfasalazine at 50-75 mg/kg/day in 2-4 divided doses. Sulfasalazine is a sulfa drug that has important anti-inflammatory properties because it inhibits the synthesis of mediators of the inflammatory response. In some patients, sulfasalazine is poorly tolerated, in which case mesalamine (50-100 mg/kg/day) and balsalazide (110-175 mg/kg/day) are preferable treatments. Diphenoxylate hydrochloride with atropine sulfate and loperamide HCl are anti-diarrheal agents. They are contraindicated in this scenario because they would suppress the symptoms of ulcerative colitis without amelioration of the underlying cause of the condition. Elimination of lactose-containing food would have no effect because the patient is lactose-tolerant. In addition, elimination of dietary fiber would not have any effect.

Case A 2-month-old male infant is brought by his mother for his 2 month well baby assessment. He had a completely normal birth history and has been growing well. The mother nursed him for about 3 weeks and then switched to a cow milk based formula after she developed nipple soreness. He took the formula well but a week later became very fussy, crying nonstop for more than 3 hours a day, usually in the evening. He does this just about every day according to mom. The crying does not seem to be related to feedings, and there is no vomiting or diarrhea. He has not been sick or congested and has had normal stools and wet diapers. He does seem gassier during these crying episodes, and he is inconsolable despite holding and rocking. The mother also has 2 toddlers at home and is becoming very frustrated and anxious. His vitals on exam are normal, as is the rest of his physical examination. Question Ico-delete Highlights What is most appropriate management for this infant?

Correct answer: Supportive measures Explanation Colicky infants cry excessively (more than 3 hours per day, more than 3 days per week and for more than 3 weeks) without any identifiable need. It occurs in all socioeconomic, racial, and ethnic groups with no sex preference. Colic begins typically at 2 weeks of age, peaks at 6 weeks, and usually resolves completely by 16 weeks. Crying spells are episodic and unrelated to feeding with a diurnal pattern; crying is increased in the evening and night. Such infants are inconsolable for long intervals and cry with extreme intensity. They draw up their legs, have tense abdomens, arch their backs, and become 'gassy,' suggesting a GI etiology. However, aerophagia or swallowing of air will occur with excessive crying and the presentation of aerophagia and gas does not coincide with the timing of colic, thus the excessive crying accompanying colic usually leads to aerophagia and subsequent gas. It is believed to be multifactorial with behavioral, social, and neurodevelopmental components. Diagnosis is made by history along with a normal physical exam in a healthy and normally growing infant. Appropriate management should be directed toward supportive interventions, such as recommendations to simulate the fetal environment with tight swaddling, rapid swinging or juggling, non-nutritive sucking, and steady white noise. Holding for prolonged periods has been shown to decrease the intensity but not the duration of crying. Multiple formula changes or dietary changes for a breastfeeding mother have not been shown to benefit colic in otherwise healthy infants, though a subgroup of infants have colic that may be caused by allergies. If the history is suggestive for this, and emesis, diarrhea, blood in the stool, severe eczema, or urticaria are present, then a short trial of hydrolyzed formula may be appropriate. Dicyclomine, an anticholinergic, has been shown to decrease symptoms of colic; however, it has also caused sedation, apnea, seizures, and coma, causing manufacturers to contraindicate its use in infants younger than 6 months of age. Simethicone has also been suggested for gas and colic, but it has not been shown to be more effective than placebo in randomized controlled trials. Increased crying can be associated with gastroesophageal reflux (GER), however only 2% to 4% of infants with colic have been shown to have GER. Conservative measures of thickening feedings and trial of anti-reflux medication may be considered if symptoms are consistent with possible GER. Prone sleep positioning, once recommended, has the potential for increased risk of infant death syndrome. Lactose intolerance is uncommon in the first year of life and includes nonspecific symptoms that may be colic-like with bloating and gaseous distention, but also vomiting, spit-up, and nonspecific diarrhea, which are not exhibited here. There are very few indications for lactose-free formula; these include galactosemia and secondary to small intestinal injury not evident in this problem.

Case A 58-year-old man came to the emergency department 2 hours after passing large volume of fresh blood while defecating, and there was no associated pain. The patient is now complaining of lightheadedness and headache. On examination, the patient's vital data was as follows: blood pressure 90/60 mmHg, temp. 98.6° F, HR 120 beats/min. There is mild lower abdominal tenderness on palpation. The patient gave history of constipation for the last 30 years. Rectal examination only revealed blood clots around the anus. Question The patient received IV fluids and vital data was stabilized. Colonoscopy was done and diverticular disease was diagnosed. What is the best option of treatment in this case?

Correct answer: Supportive medical care Explanation 80% of patients with diverticular bleeding usually require only supportive treatment consisting of fluids or blood and close monitoring with antibiotic treatment. 20% of patients require further treatment. Diverticular disease is a disease of the large intestine, usually affecting left colon, and the affected group is usually elderly. The person who suffers from this disease usually has prolonged constipation, and their diet is usually a low-fiber diet. The complication of diverticulosis includes diverticulitis giving a very similar clinical picture to appendicitis. Diverticular bleeding is another complication usually presenting as large amount of fresh blood with stool. Diverticular bleeding usually requires blood transfusion or at least intravenous fluids to compensate for the loss. 80% of patients usually require only supportive treatment consisting of fluids or blood and close monitoring. 20% of patients require further treatment. Selective mesenteric angiography with embolization or injection of vasopressin is effective in 90% of cases to stop bleeding. In the rare occasion in which diverticular hemorrhage persists or recurs, surgery involving segmental colectomy may be required (that means excision of the part affected).

Case A 65-year-old man presents with a lump that has been bothering him for the past month and a half. He denies other symptoms or recent illness. He states that he "could eat a healthier diet," he exercises twice a week, and he is a former smoker (30 pack-years). Nursing staff reports that his temperature is 98.2°F, heart rate is 80, respirations 14, and blood pressure is 136/80. During your examination, you palpate an enlarged lymph node. Question Which of the following locations of the lymph node would be most concerning?

Correct answer: Supraclavicular Explanation Supraclavicular lymph nodes, especially on the left, are very concerning because they are associated with cancer in both children and adults. In fact, supraclavicular lymphadenopathy carries a 90% risk of malignancy in adults over 40 years old. An enlarged left supraclavicular node is called a Virchow's node and is commonly associated with stomach cancer. Supraclavicular lymphadenopathy on the right is more commonly associated with cancers of the thorax. Inguinal nodes can become inflamed with sexually transmitted diseases, lower extremity infections, or lesions of the penis, vulva, or anus. They can be palpated in many healthy adults as well. Axillary lymphadenopathy can be caused by upper extremity infections, cat-scratch disease, or breast cancer. Silicone breast implants may also cause an inflammatory reaction in the axillary nodes. Occipital nodes are mostly commonly inflamed due to skin infections of the posterior scalp, such as lice, ringworm, or cellulitis. Postauricular lymphadenopathy occurs with infection of parieto-temporal scalp or Rubella (German Measles) infection.

Case A 52-year-old woman was noted on yearly examination to have a microcytic anemia. She has a long history of constipation and describes occasional hemorrhoidal bleeding, but otherwise denies any melena. She has no prior surgical history. Her only medical problem is an elevated cholesterol level that is controlled by diet. Her pulse is 92 BPM, blood pressure is 140/78 mm Hg, respiration rate is 14/min, and temperature is 98.7°F. Upon physical examination, no cervical or supraclavicular adenopathy is present and her head, eyes, ears, neck, and throat are normal. Her lungs are clear with equal breath sounds. Her heart has a regular pulse rate and rhythm, no murmurs, rubs or gallops. Abdominal exam reveals no organomegaly or masses. Rectal exam is notable for guaiac positive stool without any masses. Neurological examination is normal. Her lab results are listed below. Test Lab Value WBC 7,400 µL Hemoglobin 10.2 g/dL Hematocrit 30.4 g/dL Platelets 245,000 Na- 138 mEq/L K+ 3.8 mEq/L Cl- 100 mEq/L HCO3 27 mEq/L Question Subsequent studies reveal a sigmoid colon carcinoma. What should be the first treatment modality for this disease?

Correct answer: Surgery Explanation Colon cancer is the second most common malignancy and the second leading cause of cancer-related deaths. The risk is increased in the United States and in Western Europe due to high consumption of animal fat, protein, and refined carbohydrates. This patient presents with a relatively asymptomatic lesion only detected by an astute physician as a microcytic anemia. Her long history of constipation and occasional hemorrhoidal bleeding may be obscuring any symptoms. Regardless, the evaluation of the anemia should proceed, and the most valuable and cost effective test would be colonoscopy. Not only will this allow for diagnosis of the carcinoma, but also for a biopsy and tissue confirmation of the diagnosis. If a barium enema were done, for instance, it would diagnose the lesion, but it would require a subsequent colonoscopy and biopsy for a tissue diagnosis. Treatment of a sigmoid colon carcinoma consists at the very least of surgical therapy, and depending on the pathologic characteristics of the lesions, the decision would be where chemotherapy would be beneficial. Neither preoperative chemotherapy nor radiation have been shown to have any advantage over surgery in the treatment of colon carcinoma. Preoperative chemotherapy and radiation may be beneficial in some patients with rectal carcinoma. As far as the patient's risk for metastatic disease (and subsequent overall survival), this would be most closely related to the nodal status of the tumor. Patients without nodal disease have a 75-100% chance of overall survival, while patients with nodal involvement have 50-60% overall survival.

Case A 48-year-old man presents with a 2-day history of left-sided groin and scrotal pain. He has had similar pain episodically for several months, but it has recently become much worse after a weekend of helping his brother move furniture. He admits that he is not in good physical shape, and he thinks he may have pulled a groin muscle. He is in a monogamous relationship with his wife of 17 years. He has never had any testicular or scrotal conditions, and he has a negative surgical history. He denies fever and urinary symptoms. He has no allergies and takes no other medications. On physical exam, the patient has normal sexual development, with no edema, warmth or erythema present in the scrotum. No skin lesions are present. On palpation, there is mild tenderness on the left scrotum. However, with Valsalva, a small bulge is palpable in the left scrotum, and the patient's reported pain level increases. When he lies supine, the bulge is no longer palpable. Question What intervention is most appropriate for this patient's suspected condition?

Correct answer: Surgical referral Explanation This patient's presentation is consistent with an inguinal hernia. For patients who are symptomatic or who have very large and/or recurrent hernias or hernias with incarceration, surgical referral is recommended. Ceftriaxone plus doxycycline is an appropriate intervention if a patient's scrotal pain is due to epididymitis. However, if the patient had epididymitis, his exam would be more likely to demonstrate edema, erythema, warmth, and tenderness not necessarily worsened by Valsalva. Manual detorsion of the testicle can be attempted in a patient with testicular torsion while awaiting surgical intervention. A patient with testicular torsion may present with unilateral scrotal pain, but he would likely also have edema, erythema, as well as even nausea and vomiting. The pain level is so intense that it is unlikely a patient would take 2 days to present. The episodic nature, which worsened with moving furniture (Valsalva), is not associated with torsion either. Rest with elevation of the scrotum can help with acute epididymitis, but it does not have a specific role in treatment of hernia. Weight training for this patient would certainly not help his hernia, and it could aggravate his condition.

Case A 74-year-old man with metastatic pancreatic cancer learns that he is eligible for a clinical trial involving an investigational chemotherapeutic agent. The patient has failed traditional treatments, and the drug being used in this study has shown promising preliminary results in reducing the size of pancreatic metastases. Question What is required for the ethical conduct of this clinical trial?

Correct answer: The patient should understand the potential risks associated with study participation. Explanation The patient should understand the potential risks associated with study participation. The conduct of clinical research involves strict adherence to basic principles of medical ethics and patient autonomy (i.e., a patient's right to be in charge of their medical decisions). In most settings, adherence to ethical research is overseen by a panel of independent parties called an Institutional Review Board. The proper conduct of clinical research requires informed consent. Components of informed consent include the expected benefits and potential risks of study participation. Participation should be voluntary and patients should have the right to withdraw from the study at any time. In all cases, patients must have decision-making capacity for consent to be valid. For studies associated with significant risks, consent should be written.

Case A 42-year-old woman presents with a 10-day history of rectal pain. Her pain develops during defecation, and it is described as sharp. There is a small amount of blood on the toilet paper when she wipes. She typically has bowel movements every 3 days, and the stool is hard in caliber. Rectal examination reveals the following image. Question What is the most appropriate medication for this patient?

Correct answer: Topical nitroglycerin Explanation Topical nitroglycerin is the correct response. The patient has an anal fissure, which is shown by a tear in the anal mucosa on physical examination. Patients should consume a high fiber diet to avoid constipation. Sitz baths may be used for relief. If patients do not respond to conservative efforts, topical nitroglycerin, topical calcium channel blockers, or botulinum injection may be used. Surgical therapy may be necessary in refractory cases. Oral loperamide is an incorrect response. Loperamide is an anti-motility agent. It is believed that anal fissures are exacerbated by constipation. This patient already has a history of constipation; therefore, she should not be given loperamide Topical mesalamine is an incorrect response. Topical mesalamine is used in the treatment of inflammatory bowel disease. Anal fissures may be associated with inflammatory bowel disease, but there is nothing in the patient history that suggests the patient has symptoms of inflammatory bowel disease. Oral ciprofloxacin is an incorrect response. The patient has no signs of infection, so prescribing an antibiotic would be inappropriate in this patient. Intravenous infliximab is an incorrect response. Infliximab is used in the treatment of conditions such as inflammatory bowel disease. There are no signs of inflammatory bowel disease in this patient.

Case A 35-year-old man presents for medical care, but he is too embarrassed to tell the nurse his chief symptom. Eventually, he admits to severe, intense itching around his anus; it has been worsening the last several weeks. He further states that he has noticed increasingly severe and tearing pain in the anal area with each bowel movement. He ranks this pain as a 10/10 on a pain scale. This intense pain makes him try to avoid having bowel movements regularly. He admits to 1 episode of a small amount of bright red blood on the toilet paper as well as on the stool itself. Reviewing documentation on this patient reveals that this is the 4th similar episode in the last 14 months. The patient denies fever and diarrhea; according to him, he has never been diagnosed with inflammatory bowel disease. Question Considering the most likely diagnosis, what pharmaceutical intervention should be initiated to treat and heal this pathology?

Correct answer: Topical nitroglycerin 0.2 - 0.4% Explanation The correct response is topical nitroglycerin 0.2 - 0.4%. The patient is likely suffering from an anal fissure. Anal fissures often affect infants as well as middle-aged individuals. The majority of fissures are considered primary, and they are caused by local trauma, such as passage of hard stool, prolonged diarrhea, vaginal delivery, or anal sex. They are described linear or rocket shaped ulcers that are usually < 5 mm in length. Common presentation of anal fissures include a tearing pain that accompany bowel movements; there is also bright red rectal bleeding that is limited to a small amount noted on the toilet paper or surface of the stool. The patient described all of these components. Patients will also note perianal pruritus or irritation, all of which the patient admits to experiencing. Because this patient has had a several incidences very similar to the one he is currently presenting with, this episode should be categorized as a chronic anal fissure. Once a fissure is branded as such, treatments considered will include topical 0.2 - 0.4% nitroglycerin, topical 2% diltiazem ointment, or even injection of botulinum toxin directly into the internal anal sphincter. All of these treatments listed result in healing a chronic fissure 50 - 80% of the time; 40% of cases can recur after treatment. Topical lidocaine 5% and lidocaine plus prilocaine 2.5% are topical anesthetics that are indicated fortemporary relief of what the patient is experiencing; however, they will not promote healing of the lesion. Topical bacitracin is not indicated for symptom relief or for healing of anal fissures. Topical hydrocortisone cream 1% may decrease inflammation of the lesion; however, it will not promote overall healing.

Case A 14-year-old boy presents with a 2-week history of 4 - 5 loose bowel movements a day with blood and mucus. This is accompanied by tenesmus and cramping abdominal pain with low-grade fever. These episodes have been occurring intermittently for the past 6 months. Physical examination shows mild pallor, temperature 99° F, pulse 88/min, and BP 100/70 mm Hg. Oral mucosa and perianal examination are normal. Abdominal tenderness is present. The rest of the examination is essentially normal. Stool examination for ova and parasites, as well as culture for pathogens, are negative. Laboratory analysis shows mild anemia and elevated ESR and CRP. Perinuclear anti-neutrophil cytoplasmic auto-antibodies (pANCA) are positive, and anti-Saccharomyces cerevisiae antibodies (ASCA) are negative. Colonoscopy shows hyperemic, edematous, friable, and ulcerated rectal and colonic mucosa; there is no normal mucosa in between. Upper intestinal endoscopy shows normal mucosa. Question What is the most likely complication that can occur in this patient at this stage?

Correct answer: Toxic megacolon Explanation Toxic megacolon (TM) is a life-threatening complication commonly occurring in severe ulcerative colitis; it is characterized by extreme distension of the colon. It occurs due to infection or inflammation and manifests with abdominal pain, distension, guarding, rebound tenderness, diminished bowel sounds, fever, and tachycardia. Complications of toxic megacolon include perforation of the colon, sepsis, and shock. The patient presents with a history of recurrent episodes of loose bowel movements with blood and mucus and crampy abdominal pain for the past 6 months. An infective cause has been excluded by stool microscopy and culture; therefore, the likely diagnosis is inflammatory bowel disease. Absence of oral mucosal and perianal lesions, along with positive perinuclear anti-neutrophil cytoplasmic auto-antibodies (pANCA) and colonoscopic findings of involvement of rectal and colonic mucosa (with no normal areas in between), is suggestive of ulcerative colitis (UC). Intestinal strictures and fistulas are more commonly seen in Crohn's disease, not in UC. Patients with small bowel disease are more likely to have an obstructive pattern characterized by fibrostenosis. Perianal fissures and fistulae are common in Crohn's disease, not in ulcerative colitis. There is a higher risk of colon carcinoma in patients with ulcerative colitis when the disease has persisted for more than 10 years. It is recommended that patients who have had ulcerative colitis for 8 - 10 years be screened by colonoscopy and biopsy every 1 - 2 years. This patient only has a 6-month history. Hemolytic uremic syndrome (HUS) is characterized by microangiopathic anemia, thrombocytopenia, and acute renal failure. The diarrheal illness that precedes HUS is caused by E. coli O157:H7 in North America and Europe, and Shigella dysenteriae in the Indian subcontinent. Infective pathology has already been excluded in the above patient.

Question Ico-delete Highlights The newborn nursery calls about a 4-hour-old term baby boy. He was born to a 31-year-old mother who was diagnosed as having some hydramnios during pregnancy; otherwise the prenatal history was normal. Apgar scores were 9 at 1 and 5 minutes and he transitioned well to the nursery until he began feedings. Shortly after nursing he began choking and coughing with mild perioral cyanosis. His nose and mouth were suctioned with excess secretions noted in his mouth. Desaturation and bradycardia occurs during these spells on pulse oximetry with some mild stiffening of his arms and legs. On exam, heart, lungs, vital signs and neurologic findings are all normal. Labs including electrolytes, arterial blood gas, and complete blood count all seem normal. Blood, urine, and CSF cultures are pending but look normal. EKG, EEG, and head ultrasound are normal. Chest x-ray shows marked air distention of the whole gastrointestinal tract. Based on these findings, what is the most likely diagnosis?

Correct answer: Tracheoesophageal fistula with esophageal atresia Explanation Tracheoesophageal fistula (TEF) occurs in 85% of cases of esophageal atresia, which occurs in 1 out of 3,000 to 4,500 live births. Esophageal atresia or TEF can occur separately, though this is less common. These anomalies are thought to arise from defective differentiation of the primitive foregut into the trachea and esophagus. The atresia is due to defective growth of endodermal cells and incomplete fusion of the lateral walls of the foregut during separation of trachea results in the TEF. Air will enter the abdomen when the fistula connects the distal esophagus to the trachea. The abdomen becomes distended, interferes with breathing and allows gastric acid to reflux into the lungs. Attempts to insert a nasogastric tube cause coiling of the tube in the blind pouch of the esophagus, which is diagnostic. This condition should be suspected when there is a history of maternal hydramnios, excessive secretions, choking, cyanosis and coughing with feedings, and failure to pass a nasogastric tube into the stomach. Management is surgical correction, consisting of ligating the fistula and anastomosing the two ends of the esophagus. Gastroesophageal reflux occurs as a result of failed normal esophageal sphincter function. The lower esophageal sphincter transiently relaxes after a food bolus has entered the stomach, allowing re-entry into the esophagus. The emesis of reflux is effortless and not associated with retching or autonomic symptoms. Esophageal atresia without TEF is less common and arises from defective growth of endodermal cells of the foregut. 30% will have associated anomalies as isolated defects or part of VATER (or VACTERL) association (vertebral, vascular defects, anal atresia, cardiac malformations, TEF, EA, radial and renal anomalies, limb defects). Diaphragmatic hernia arising from incomplete fusion of the pleuroperitoneal membrane, usually on the left, allows passage of abdominal contents into the chest thus impeding respirations. Suggestive early findings include polyhydramnios, absent or intrathoracic stomach bubble, and mediastinal and cardiac shift away from the side of the hernia. TEF without esophageal atresia or H-type is quite rare and will usually cause recurrent aspiration delaying diagnosis. This is also not associated with a history of hydramnios.

Case A 45-year-old woman presents with a 1-month history of soft and bulky stools; the stools are difficult to flush because they float on top of the toilet water. She also reports that she feels very tired and has lost 10 pounds without dieting. She is not on any medications currently, but she states that the last time she traveled to the Caribbean she developed the same symptoms. She was treated with tetracycline and did well. She has just returned from India. On examination, she is pale with no palpable lymph nodes. Her hemoglobin is 10 g/dL, and a peripheral blood film reveals megaloblastic cells. Microscopic examination of a stool sample reveals no ova, cysts, or trophozoites. Question What is the most likely diagnosis?

Correct answer: Tropical sprue Explanation Tropical sprue is an acquired disease that affects both visitors and natives of tropical areas (e.g., the Caribbean and South India). Its etiology is unknown. Patients usually present with diarrhea, weight loss, and stools that are soft and bulky (steatorrhea). They may also develop deficiencies of folate and cobalamin. Stool microscopy should be performed to look for cysts and trophozoites. Histological examination of a small bowel mucosal biopsy aids in making the diagnosis by revealing partial villous atrophy. Treatment is with tetracycline or oxytetracycline. Celiac disease is a hereditary disorder caused by gluten intolerance. Gluten is found in wheat, rye, barley, and oats. Patients may be asymptomatic or may present with diarrhea, abdominal discomfort, distention, and steatorrhea (stools are pale, malodorous, and difficult to flush because they float on the toilet water). Patients may have anemia from iron and folate deficiencies, osteomalacia and bone pains from calcium deficiency, and edema from hypoproteinemia. Diagnosis is confirmed by a small intestine mucosal biopsy that shows a flat mucosa due to villous atrophy and by subsequent improvement on a gluten-free diet. Specific treatment includes a gluten-free diet. Whipple's disease mainly affects men and is caused by the bacterium Tropheryma whippelii. It is a multisystemic disease that affects the small intestines, joints, brain, heart, and eyes. Patients can present with diarrhea, steatorrhea, abdominal pain, weight loss, and joint pains. On examination, they may be pale with lymphadenopathy. Histological examination of a small bowel mucosal biopsy reveals PAS-positive foamy macrophages. Treatment is with trimethoprim-sulfamethoxazole or chloramphenicol. Short bowel syndrome is usually the result of surgical resection of the intestines or a jejunoileal bypass. The malabsorption is a result of inadequate absorptive surface. Malabsorption of vitamin B12 results in paraesthesias; malabsorption of calcium results in bone pain and carpopedal spasms. Lactose intolerance results from deficiency of lactose. Patients complain of borborygmi, flatulence, nausea, abdominal cramps, pain, and diarrhea after ingesting lactose-containing food (e.g., milk). Laboratory investigations reveal that the diarrheal stools are acidic. When the hydrogen breath test is administered, there is elevated hydrogen content in the expired air because the colonic flora digests the unabsorbed lactose. Treatment includes a lactose-free diet.

Case A 15-year-old boy was healthy until 3 months ago. He has been having episodes of crampy abdominal pain and explosive liquid stools 2-3 times a day. He has had intermittent fevers and has noted blood in the stool. He has a documented 15-lb weight loss. Question What is the most likely diagnosis?

Correct answer: Ulcerative colitis Explanation Given his fevers, bloody stools, and 15-lb weight loss, ulcerative colitis is the most likely diagnosis. It would be unusual for bacterial diarrhea to persist for 3 months. An acquired enzyme deficiency is not associated with fevers, blood in the stool, and weight loss. Viral gastroenteritis is usually self-limited and unassociated with bloody stools. The diarrhea associated with a pheochromocytoma is a secretory diarrhea. It can be associated with weight loss and with fluid and electrolyte imbalance. It would not be bloody.

Case A 50-year-old man with a past medical history of hyperlipidemia presents with a recurrent "bitterness and sour sensation" in his mouth and a nonproductive cough; both have been occurring over the last year. There is associated progressive dysphagia and odynophagia to solid foods, but not to liquids; as a result, he has lost 15 lb over the past 5 months. He admits to drinking 1-2 beers per week, but he denies additional alcohol use. He also denies cigarette smoking and illicit drug use. He denies fever, chills, sputum production, chest pain, palpitations, dyspnea, shortness of breath, wheezing, peripheral edema, abdominal pain, nausea, vomiting, diarrhea, melena, hematochezia, or changes in bowel habits. The physical exam reveals normal vital signs. Oropharyngeal, neck, cardiopulmonary, and abdominal exams are normal. Question What is the most appropriate initial intervention for this patient at this time?

Correct answer: Upper endoscopic evaluation Explanation An important part of the medical history of dysphagia is to determine the type of food that produces the symptoms. (solids, liquids, or both). If the patient is without a history of radiation, caustic injury, esophageal cancer, or complex stricture, an upper endoscopy with or without esophageal biopsies would be the initial test to determine the cause of progressive esophageal dysphagia to solids. Mucosal inspection and the ability to biopsy is more precise via this modality. Dilation can be performed during the study if needed, so endoscopy may also potentially serve as a therapeutic tool. Alternately, a barium esophagography provides more information regarding motility disorders. A video-modified barium swallow is the initial diagnostic test of choice for oropharyngeal dysphagia, a disorder of motility. Although a trial prescription of esomeprazole may be suitable for a more straight forward case of simple GERD, the patient has multiple risk factors that puts him at a higher risk of an incidence of malignancy, warranting a more confirmatory diagnostic scenario at this time. A chest X-ray may reveal adenopathy, a widened mediastinum, metastatic lesions to the lungs or bone, or signs of tracheoesophageal fistula, such as pneumonia. It will not, however, reliably identify esophageal lesions. Ambulatory esophageal pH monitoring is the preferred study for documenting acid reflux. It is indicated in the following clinical scenarios: for patients being considered for anti-reflux surgery with a normal endoscopy in patients with a normal endoscopy but with continued reflux symptoms following therapy with a proton pump inhibitor in the evaluation of atypical reflux symptoms that continues despite anti-reflux therapy Patients with alarm symptoms, such as unexplained weight loss, odynophagia, jaundice, recurrent vomiting, blood loss, a palpable mass or lymph node, or a family history of GI malignancy, should have diagnostic testing performed promptly.

Case A 60-year-old man presents with a 2-week history of nausea and several episodes of epigastric pain. The pain is characteristically dull and lasts for several hours. His symptoms are not worsened by any particular food or activity. Question What is the most appropriate initial test?

Correct answer: Ultrasonography of the upper abdomen Explanation Ultrasonography (USG) of the upper abdomen is the first choice in the investigation of acute or subacute abdominal pain. A common cause of acute abdominal pain in the elderly is cholecystitis, which can be investigated by USG. Changes occurring in the biliary system because of aging make older patients susceptible to cholecystitis, the most common indication for surgery in this population. Differential diagnosis would include diverticulitis (the most common cause of severe abdominal pain in patients over the age of 50), bowel obstruction, pancreatitis, and peptic ulcer disease. Catastrophic conditions (e.g., abdominal aortic aneurysm rupture and mesenteric ischemia) must also be considered. This patient's symptoms are not very intense (his symptoms have been present for 2 weeks) and, therefore, do not necessitate urgent invasive investigations. If ultrasonography of the upper abdomen shows no significant findings, then a number of other tests can be performed (e.g., an upper endoscopy to rule out peptic ulcer disease, a gastrointestinal series to rule out bowel obstruction, and a CT scan or MRI of the abdomen to rule out pancreatitis or diverticulitis); any other undetermined pathology may be undertaken to exclude other possibilities.

Case A 48-year-old man presents with a 2-day history of left-sided groin and scrotal pain. He has had similar pain episodically for several months, but it has recently become much worse after a weekend helping his brother move furniture. He admits that he is not in good physical shape, and he thinks he may have pulled a groin muscle. He is in a monogamous relationship with his wife of 17 years. He has never had any testicular or scrotal conditions, and he has a negative surgical history. He denies fever and urinary symptoms. He has no allergies and takes no other medications. On physical exam, the patient has normal sexual development with no edema, warmth, or erythema present in the scrotum. No skin lesions are present. On palpation, there is mild tenderness on the left scrotum; however, with the Valsalva maneuver, a small bulge is palpable in the left scrotum and the patient's reported pain level increases. When he lies supine, the bulge is no longer palpable. Question Based on this patient's history and physical, of the following tests what would be most helpful in establishing the suspected diagnosis?

Correct answer: Ultrasound Explanation This patient's presentation is consistent with an inguinal hernia, which is often diagnosed based upon history and physical. If diagnostic testing is desired to confirm a diagnosis, ultrasound is the most useful. Plain radiographs (X-ray) involving the testicles and ovaries should generally be avoided; they would not reliably demonstrate a soft tissue hernia. A retrograde cystourethrogram is an imaging technique to evaluate urethral integrity using a contrast solution instilled through a catheter up the urethra to the bladder. It is not helpful in hernia evaluation. Transillumination of the scrotum is a technique that helps differentiate cystic versus solid scrotal contents. A hernia may not readily be differentiated from other conditions with transillumination alone. Urethral swabs for Chlamydia trachomatis and Neisseria gonorrhoeae would be helpful in a presentation of epididymitis, but not helpful for suspected hernia. If this patient had epididymitis, his exam would be more likely to demonstrate edema, erythema, warmth, and tenderness not necessarily worsened by the Valsalva maneuver. Urinalysis (UA) may be helpful in cases of epididymitis, as well as urinary tract infections. This patient's history and exam are not consistent with infection as the cause of his groin/scrotal pain, so a UA would not be helpful to establish a diagnosis of hernia.

Question A 22-year-old man presents with a right groin bulge. During physical assessment, a single sac is found protruding just lateral to the epigastric vessels. Although the clinical scenario is highly suggestive of a hernia, what initial imaging study would help support this diagnosis?

Correct answer: Ultrasound Explanation Ultrasonography is a convenient, safe, noninvasive imaging study that can be used even at bedside in this patient scenario to help confirm this is a true hernia. It is an imaging technique that utilizes sonic energy, which is also termed a nonionizing type of energy, so it is safe for use in all patients, including pregnant and pediatric patients. Images are created by echoes or reflections of the ultrasound beam as it interacts with tissues of different properties. Ultrasound waves are greatly reflected by superficial, air-soft tissue, and bone-soft tissue interfaces, making its usage somewhat limited. Because the inguinal structures are superficial, a linear transducer ultrasound is very effective at helping confirm the diagnosis. Larger body habitus does have to be taken into consideration in terms of interference of the ultrasound results. Having the patient perform the Valsalva maneuver at various stages of the ultrasound can help identify if the hernia has some degree of being transient. The characteristic movement of the herniating tissues that will be visualized during an active ultrasound study usually helps confirm the diagnosis. A radiograph would not be appropriate in this scenario. Computed tomography (CT) is another imaging study that is commonly used to help assess hernias, but it may not be the first diagnostic study chosen. CT scans produce exquisitely adequate images of all intra-abdominal organs and the abdominal wall, and they can create these images quickly. Because of its superior anatomic detail, CT may also help detect subtle signs of complication within the hernia sac, including bowel obstruction, incarceration, strangulation, and traumatic wall hernias. MRIs accurately depict groin hernias, but they may not be as readily available, and they are much more expensive than ultrasonography and computed tomography. Doppler studies are reserved for suspected issues that primarily arise in the cardiovascular system.

Case A 47-year-old Hispanic woman presents with a 2-month history of persistent abdominal pains. She becomes uncomfortable after eating, especially after consuming fats, eggs, chocolate, fried foods, fatty foods, and rich desserts. Her pains are primarily located in her right upper abdominal quadrant and often radiate to her right shoulder blade. Physical examination is essentially unremarkable at this time. Her blood pressure is 122/82 mm Hg, she has a pulse of 72 per minute, and a temperature of 98.6°F with respirations 16 per minute. Her abdominal examination reveals no tenderness, no guard, no rebound, and normally active bowel sounds. Her liver and spleen are not palpable. She has no readily palpable abdominal masses. She is not jaundiced. She has no scleral icterus. Question Ico-delete Highlights What test should be given in order to confirm the probable diagnosis?

Correct answer: Ultrasound abdomen Explanation Based on the patient's history, cholelithiasis is suspected. This condition is more common in females of Caucasian or Hispanic descent than in others. The incidence increases with age. Most (80%) of the stones in the US are the cholesterol type. Bilirubin, pigment, and calcium stones constitute the other 20%. Cholelithiasis may be asymptomatic (the presence of gallstones without symptoms), symptomatic (biliary colic), or complicated (e.g. cholecystitis, choledocholithiasis, cholangitis). Ultrasonography is a safe, reliable, and non-invasive test that can be performed at the bedside and is also safe in pregnant women. It is quite sensitive and specific for stones larger than 2 mm. Only about 10-30% gall stones are radio-opaque and hence abdominal X-ray may not be so useful. The oral cholecystogram is the X-ray taken after administering contrast medium to the patient. Although it can identify gall bladder pathology, it requires preparation and is not as convenient as the ultrasound. CT scan is not the first choice in suspected gallstones, although it may prove useful in intrahepatic stones. Biliary scintigraphy is of value in cystic duct obstruction. MRI is also an excellent choice but is expensive, requires sophisticated equipment, and is not used as the first choice investigation in most centers. Symptomatic stones are treated with cholecystectomy. Asymptomatic stones may also require surgery under special circumstances such as large (>2 cm) stones, those with spinal cord injuries affecting the abdomen, and in calcified gall bladder.

Case A 10-year-old boy presents with a 2-month history of intermittent burning pain in the epigastrium. Pain is felt more during the night and between meals and is partly relieved by eating food or by taking antacids. Pain usually lasts for about ½ to 1 hour and is accompanied by nausea and vomiting. The patient often has a feeling of bloating and burping. The child remains asymptomatic for several days in between. There is no history of taking analgesics or anti-inflammatory drugs. Physical examination shows epigastric tenderness. The rest of the examination is essentially normal. Stool examination for occult blood is positive. Question What is the investigation of choice for establishing the diagnosis?

Correct answer: Upper GI endoscopy Explanation The patient's history, clinical examination, and stool testing positive for occult blood are suggestive of peptic ulcer disease. Upper GI endoscopy is the investigation of choice in children with suspected peptic ulcer disease. Endoscopy allows direct visualization of the esophagus, stomach, and duodenum, identifies the specific lesions, and also screens for the presence of H. pylori in the biopsy specimens. H. pylori is a gram negative S-shaped rod. Biopsy should always be obtained from the antrum of the stomach regardless of endoscopic findings. Endoscopic findings may vary from grossly normal mucosa to nonspecific gastritis with prominent rugal folds to nodularity and ulcers. Stool microscopy has no role in the diagnosis of H. pylori infection. However, stool antigen testing identifies active H. pylori infection by detecting the presence of H. pylori antigen in the stool. Single contrast barium studies have overall sensitivity of 75%, but double contrast studies have sensitivity of 95% in detection of gastric ulcers. These results are comparable to endoscopy. However, barium studies have the disadvantage that the biopsy specimen cannot be obtained and ulcers smaller than 5mm may not be detected on barium studies. CT scan has no primary role in the detection of gastric ulcers. It is helpful in the detection of subphrenic collection that may occur after perforation of gastric ulcer. Abdominal ultrasound has no role in the detection of gastric ulcer but is useful in the detection of other causes of upper abdominal pain like gallstones, pancreatitis, and subphrenic collection, as well as other collections due to perforated gastric ulcer.

Case A 53-year-old man presents with increased difficulty swallowing and occasional regurgitation of his meals. His symptoms have been occurring with greater frequency and severity over the last 4 months. He also has some shortness of breath but attributes that to his weight and lack of physical activity. His past medical history is remarkable for chronic heartburn, which he treats intermittently with over the counter antacids. He takes no regular medications has no allergies. He has not had any surgeries. He is not a smoker and he denies use of alcohol and drugs. He works as a building inspector, and he lives with his wife and children. The patient is obese, but the rest of his physical exam is normal. Blood tests, electrocardiogram, and chest X-ray are done in the clinic; they are normal. He is referred for endoscopy, and esophageal biopsy shows specialized intestinal metaplastic cells (of columnar epithelium). Question What recommendation would best prevent complications from this patient's current condition?

Correct answer: Use of proton pump inhibitors Explanation This patient is presenting with Barrett's esophagus, a type of chronic esophagitis in which the normal squamous epithelium is replaced with columnar epithelium. Barrett's esophagus is a complication of chronic gastroesophageal reflux disease (GERD) and can develop into esophageal adenocarcinoma. Use of proton pump inhibitors (PPIs) reduces the risk of cancer. In order to monitor for the development of cancer, a routine endoscopy should be periodically performed in patients with known Barrett's esophagus. While the antacids are not necessarily adequate to control the GERD and damage to this patient's esophagus, avoidance of antacids will have no role in preventing subsequent Barrett's esophagus complications, such as adenocarcinoma. While the acid reflux appears to be a major mechanism for damage to the esophageal tissue, anti-reflux surgeries do not appear to prevent complications from Barrett's esophagus and subsequent development of adenocarcinoma. This patient does not need referral for fundoplication (surgical "wrapping" of the stomach around the esophagus). Regular exercise would be helpful for this patient in general terms for cardiovascular health and weight reduction, but exercise does not play a significant role in reducing risks for Barrett's esophagus complications. Obesity is a risk factor for development of Barrett's esophagus. Other risk factors include male sex, smoking, and especially chronic GERD. Once Barrett's esophagus has developed, however, weight loss does not dramatically alter the risk of complications, such as adenocarcinoma. Weight loss should be recommended for any obese patient, but the PPIs are the best approach to reduce complications from his Barrett's esophagus.

Question 6 hours after eating raw fish and other seafood at a Japanese restaurant, a couple developed nausea and vomiting, watery diarrhea, abdominal cramps, and fever. What organism is likely involved in the patients' condition?

Correct answer: Vibrio parahaemolyticus Explanation Bacillus cereus cause 2 syndromes: (1) one involves a short incubation period with nausea and vomiting and is similar to staphylococcal food poisoning; (2) the other involves a long incubation period (18 hours) with watery, non-bloody diarrhea and resembles clostridial gastroenteritis. Clostridium perfringens cause food poisoning with an 8- to 16-hour incubation period. It is characterized by watery diarrhea with cramps and little vomiting. It resolves in 24 hours. Staphylococcus aureus cause food poisoning characterized by vomiting being more prominent than diarrhea. The disease is due to ingestion of enterotoxin, which is performed in foods and hence has a short incubation period (1-8 hours). Vibrio parahaemolyticus are a marine organism transmitted by contaminated seafood, especially when raw fish is eaten. The clinical picture varies from mild to quite severe watery diarrhea, nausea and vomiting, abdominal cramps, and fever. The illness is self-limited, lasting about 3 days. Salmonella enteritidis cause enterocolitis after an incubation period of 6-48 hours. The disease begins with nausea and vomiting and then progresses to abdominal pain and diarrhea, which can vary from mild to severe and present with or without blood. Usually, the disease lasts a few days and is self-limited.

Question A 50-year-old woman with a family history of colon cancer underwent a total colonoscopy as per screening guidelines for colon cancer. 3 small (<1cm) pedunculated polyps were removed using hot-wire biopsy. Pathology reported all 3 polyps to be tubular adenomas. What type of colon polyp has the highest risk of carcinoma?

Correct answer: Villous adenoma Explanation Hyperplastic polyps are benign with no malignant potential. Hamartomatous polyps are normal tissues in an abnormal configuration with no malignant potential; juvenile polyps are the most common type of hamartomatous polyps. Tubular adenomas are the most common neoplastic polyp, and they have a 1-33% risk of carcinoma (depending upon their size). Tubulovillous adenomas have a higher risk of carcinoma (5-45%). Villous adenomas are usually sessile polyps with the highest risk of carcinoma (10-55%); this is partially because 60% of them are >2cm at discovery with a 55% malignant potential.

Case A 1-year-old boy presents with colicky abdominal pain, vomiting, and a 1-day history of irritability and lethargy. The child has vomited once, and the vomitus contained only food particles without blood or bile. Rectal bleeding and 'currant jelly' stool are also noted. The child has been healthy, and this is the 1st such episode since birth. Vitals are as follows: BP 110/70 mm Hg, P 100/min, RR 20/min, and temperature 98.8° F. On examination, a soft, slightly tender abdomen is noted. A vague vertical mass is palpable in the right upper quadrant. Other systems are normal on examination. Plain abdominal film shows evidence of obstruction, and barium enema detects a coiled-spring appearance to the bowel. Intussusception is diagnosed. Question What disease is suspected since the child is under the age of 2 years and has hypertrophy of Peyer's patches?

Correct answer: Viral infection Explanation The correct response is a viral infection. Intussusception is the 'telescoping' of bowel into the adjacent segment of bowel. It is most common in male patients 3 months to 1 year of age. The triad of colicky abdominal pain, vomiting, and rectal bleeding in this age group is a reliable hint to the diagnosis. Rare presentations (e.g., those with neurological symptoms, subacute, or chronic forms) can also occur. The most commonly involved sites are the terminal ileum and ileocecum. A mechanical lead point can be identified in children above 2 years (e.g., abdominal trauma, hemangioma, Meckels diverticulum, and foreign body); however, in younger children it is assumed to be a previous viral infection causing hypertrophy of Peyer's patches. This leads to intussusception, strangulates venous return, and results in the classic consequences of bowel swelling, ischemia, and perforation. A contrast enema is both diagnostic and therapeutic. Reduction can be achieved in most cases. There is a chance of recurrence in 10% of children; this can be treated again by contrast barium enema.

Question A 60-year-old man presents with problems seeing at night, especially when driving. You suspect he is suffering from night blindness. A deficiency in what vitamin can cause night blindness?

Correct answer: Vitamin A Explanation Vitamin A is involved in the general health of epithelial tissue. It exists in various forms that have different functions in the organism. The 11-cis-retinal form functions as a component of the vision cycle. It combines with the protein opsin to form rhodopsin, which is the photosensitive molecule in the discs of the rod cells. Upon absorption of light, the 11-cis form is converted to trans-retinal that dissociates from the opsin protein. This cascade converts light into atomic motion and then into a nerve signal. A single photon is absorbed by a rod cell and leads to hyperpolarization of the plasma membrane by closing the cation-specific channels. The trans-retinal is then isomerized back to 11-cis retinal, which can then bind opsin again and go through another cycle. A deficiency in vitamin A leads to night blindness and eventually deterioration of the outer rod segments. Vitamin A (all-trans-retinol) is converted to 11-cis-retinal in several steps. Retinyl phosphate functions in the synthesis of some glycoproteins and mucopolysaccharides as a glycosyl donor. This is especially true for certain glycoproteins necessary for normal growth regulation and mucous secretion. Retinol and all-trans-retinoic acid bind to specific receptors in the cell, the nuclear receptor superfamily. This family of receptors includes those that bind estrogen, progesterone, glucocorticoids, vitamin D3, and thyroxine, in addition to retinol and all-trans-retinoic acid. All of these receptors contain a highly conserved DNA binding domain, a hormone-binding domain, and a variable activation domain. β-Carotene functions as an antioxidant in the cell. β-Carotene is the precursor of retinal, the chromophore in visual pigments. Synthetic forms of vitamin A are also now available and used as drugs. 13-cis-retinoic acid is used in the treatment of acne. Etretinate, an aromatic derivative of all-trans- retinoic acid, is used in the treatment of psoriasis. The structures of some of these important retinoids are shown in the included image.

Question Patients with alcoholism who haveh Wernicke-Korsakoff syndrome have a deficiency of what vitamin?

Correct answer: Vitamin B1 Explanation Deficiency of vitamin B1 (thiamine) produces Wernicke-Korsakoff syndrome, a severe neurologic disorder that is characterized by dementia. It is usually associated with alcoholism. Thiamine is administered to alcoholic patients to avoid the occurrence of this syndrome.

Question Ico-delete Highlights A 44-year-old emaciated man is brought to the emergency room towards late morning by his friends. They report he was difficult to rouse. He has not eaten anything since a large meal the night before. His friends report a 25-year history of alcohol abuse. Examination shows severely restricted horizontal eye movements and ataxia of both lower limbs. What nutritional deficiency can cause this patient's symptoms?

Correct answer: Vitamin B1 Explanation This patient is a classic example of Wernicke's encephalopathy, which results due to a deficiency of vitamin B1 or thiamine. Alcohol abuse is one of the most important contributing causes. The deficiency results in ischemic damage to the brain stem and its connections. The typical triad is comprised of ocular signs, ataxia, and a confused state. Management involves the immediate administration of thiamine parenterally - 50 mg IV and 50 mg IM, followed by daily doses of 50 mg IM. This leads to prompt reversibility of signs and symptoms. Note: Intravenous glucose solutions should not be given to such patients, as they may exhaust the patient's reserve of B vitamins and cause a rapid worsening.

Question A 32-year-old woman presents with a pins and needles sensation on the sides and the bottom of her feet, as well as severe burning sensation in the feet. The burning sensation manifests primarily at night. Examination reveals a profoundly cachectic but alert patient with atrophy of muscles of the leg. Examination of the eye indicates that she also has nystagmus. Further questioning reveals that she has a history of homelessness and chronic alcoholism. What vitamin deficiency is most commonly associated with chronic alcoholism?

Correct answer: Vitamin B1 Explanation Wernicke's encephalopathy is a result of the inadequate intake or absorption of thiamine (Vitamin B1) coupled with continued carbohydrate ingestion. It is most common among chronic alcoholics who suffer from poor nutrition. It is therefore imperative that all patients with chronic alcoholism receive a dose of thiamine before they receive any glucose solution IV. It is also seen in patients on dialysis and during hyperemesis gravidarum of pregnancy. Clinical features appear acutely and include ataxia, confusion, nystagmus, partial ophthalmoplegia, apathy, confusion, severe memory loss, as well as autonomic dysfunction. Treatment of suspected or borderline cases consists of giving 5-30 mg of thiamine orally as a single dose for a period of 1 month. Parenteral thiamine of 50-100 mg is given to prevent Wernicke's encephalopathy, and it is also used in its treatment. Thiamine has to be given as IV, especially before administering fluids containing glucose. Rehydration of the patient along with supplementation of electrolytes and nutritional therapy is also required. Advanced cases require hospitalization and close supervision. Other vitamins deficiencies are not commonly associated with chronic alcoholism.

Case A 48-year-old woman underwent gastric bypass surgery. She followed the prescribed diet and lost approximately 100 pounds in 18 months. She presents with numbness in her feet, and blood tests reveal a vitamin deficiency. Question Based on these observations, this woman is most likely deficient in what vitamin?

Correct answer: Vitamin B12 Explanation The correct answer is vitamin B12. Deficiency of vitamin B12 leads to numbness and tingling of the hands and feet, weakness, fatigue, and anemia. It is one of the more common vitamin deficiencies to occur after gastric bypass surgery. Gastric bypass eliminates a substantial portion of the stomach where intrinsic factor is produced. Intrinsic factor is a carrier protein for vitamin B12 that helps it be adsorbed by the ileum. Vitamin B12 is required for 2 reactions in humans: the methylation of homocysteine to form methionine and the conversion of methylmalonyl CoA to succinyl CoA. The metabolism of vitamin B12 is tied to folic acid metabolism; therefore, a deficiency in vitamin B12 can lead to a metabolic deficiency of folic acid. This lack of usable folic acid leads to deficiencies in purine and dTMP synthesis. This, in turn, affects nucleotide synthesis needed for DNA synthesis. Rapidly dividing cells, such as blood cells, are affected, leading to pernicious anemia in cases of vitamin B12 deficiency. The absorption of biotin, thiamine, and vitamins K are usually less affected by the gastric bypass procedure. A deficiency in vitamin D can occur after gastric bypass but would not cause peripheral neuropathy. Deficiencies in vitamin D would manifest themselves in bone and calcium problems.

Question What deficiency causes Pellagra-diarrhea, dermatitis, and mental disturbances?

Correct answer: Vitamin B3 (niacin) Explanation The correct response is vitamin B3 (niacin). Vitamin Symptoms of Deficiency A (retinoids, carotenoids) rhodopsin deficiency, night blindness, growth retardation, skin disorders, and increased risk of infections B1 (thiamine) beriberi-muscle weakness (including cardiac muscle), neuritis, and paralysis B2 (riboflavin) eye disorders and skin cracking, especially at corners of the mouth B3 (niacin) pellagra-diarrhea, dermatitis, and mental disturbances Pantothenic acid neuromuscular dysfunction and fatigue

Case An 8-month-old female infant presents with a 2-day history of increasing irritability and decreased appetite. She has also had some diarrhea and low grade fever. On exam she is afebrile, very fussy, and hard to console; however, she appears alert and active. She lies with her hips and knees flexed, crying harder with any movement. A few petechiae are noted on her skin. Leg X-rays are done to look for a possible fracture that shows a pencil-thin cortex and a ground glass appearance of the bones. Further history reveals that she has been given evaporated milk since birth to save money, and she has not yet been started on solids. Question What vitamin deficiency would most likely cause this infant's symptoms?

Correct answer: Vitamin C Explanation Vitamin C is involved in cellular oxidation through reduction reactions; it is required for the normal growth and maturation of cells. It helps form the ground substance among cells of the capillary walls, collagen, and osteoid tissue. Vitamin C is abundantly present in citrus fruits, spinach, cauliflower, liver, and kidney. Scurvy is characterized by pinpoint spots on the skin (especially on the thighs and legs) called petechiae, spongy gums, poor healing of bruises, joint pain, bleeding from mucous membranes, and thin hair. It can occur in infants whose mothers have a dietary deficiency in vitamin C and also in infants fed unsupplemented evaporated milk. Infants deficient in the vitamin may present with fever, diarrhea, appetite loss, poor weight gain, and protein depletion. Irritability, generalized tenderness causing pseudoparalysis, frog position of the legs, peripheral edema, swelling of gums, scorbutic beads in the ribs, and petechial hemorrhages can also be seen. Bone involvement is typical in infantile scurvy. Radiographic changes include ground glass appearance of the bones, pencil-thin cortex, a zone of calcified cartilage at the metaphysis, and a zone of rarefaction proximal to this. Administration of 100 to 200 mg/day of ascorbic acid for 1 week will promote quick healing. The daily requirement of vitamin C for infants 7 to 12 months-old is 50 mg. Seizures, irritability, peripheral neuritis, dermatitis, and microcytic anemia can be seen with a deficiency of vitamin B6, or pyridoxine. It is a vitamin found in yeast, rice polishings, and cereals. It serves as a coenzyme in the metabolism and transfer of amino acids. Primary deficiencies are rare, but secondary deficiencies due to malabsorption, diarrhea, or certain drugs like isoniazid can occur. Vitamin A deficiency is the leading cause of preventable blindness in children, and deficiencies also increase the risk for severe infections. The skin of extensor surfaces becomes dry and scaly with follicular hyperkeratosis; dryness of the eyes is caused by keratinization of the conjunctiva and lacrimal glands. Bitot spots are small, triangular, silvery, foam-like patches that appear on the conjunctiva due to keratinization. Night blindness will occur due to a delay in resynthesis of rhodopsin. Severe inflammation may result in keratomalacia and blindness. Breastfeeding and encouraging vitamin A-rich diets, supplementation, and food fortification can prevent and treat this deficiency. Riboflavin (vitamin B2) deficiency results in cheilosis, glossitis, keratitis, photophobia, blurring of vision, seborrheic dermatitis, and anemia. It is important in hydrogen transfer reactions for the normal metabolism of tryptophan and retinal pigment. It is found in milk, cheese, liver, and green leafy vegetables. Deficiency of vitamin E causes muscle weakness, double vision, loss of position sense, hemolytic anemia, reduced reflexes, and constricted visual fields. This vitamin occurs abundantly in green plants, vegetable oils, milk, and eggs. Deficiency may occur in patients suffering from abetalipoproteinemia or fat malabsorption. Treatment includes the inclusion of unsaturated fat in the diet. Special vitamin E supplements are necessary in children who may have malabsorption.

Case A 32-year-old woman presents with a 1-month history of bleeding gums when brushing her teeth. She also reports that her wounds are taking longer than usual to heal. She is a stay-at-home mother and is breastfeeding her 6-month-old twins. On examination, you note multiple splinter hemorrhages on her nails and ecchymoses over her lower limbs. Question What is the most likely diagnosis?

Correct answer: Vitamin C deficiency Explanation In cases of vitamin C (or ascorbic acid) deficiency, patients can present with bleeding tendencies (as a result of weakened capillaries) and impaired wound healing due to impaired formation of connective tissue. On examination, the gums may be swollen and friable; the teeth may be loose. There may also be multiple splinter hemorrhages on the nails and ecchymoses, especially over the lower limbs. Causes include inadequate dietary intake and certain conditions such as pregnancy and lactation that increase vitamin C requirements. Dietary sources of vitamin C include citrus fruits, such as oranges, lemons, and tangerines, as well as tomatoes and potatoes. In cases of vitamin A deficiency, patients can present with the inability to see well in dim light or night blindness. There may also be conjunctival and corneal xerosis, as well as pericorneal and corneal opacities, and Bitot's spots. Bitot's spots are a collection of keratin appearing as triangular foamy spots on the conjunctiva. Patients may also have xeroderma, hyperkeratotic skin lesions, and increased susceptibility to infections. Causes include inadequate dietary intake and malabsorption. Dietary sources of vitamin A include fish, liver, egg yolk, butter, cream, dark green leafy vegetables, as well as yellow fruits and vegetables. In cases of pyridoxine or (vitamin B6 deficiency), patients can present with peripheral neuropathy, seborrheic dermatosis, glossitis, and cheilosis. Laboratory investigations reveal anemia with lymphopenia. Causes include malabsorption as well as medications, such as isoniazid and penicillamine. Dietary sources of vitamin B6 include liver, legumes, whole grain cereals, and meats. Niacin deficiency causes pellagra, which is characterized by: Symmetrical dermatitis, usually on parts of the body exposed to sunlight Scarlet glossitis and stomatitis Diarrhea Mental aberrations, such as memory impairment, depression, and dementia. These may appear alone or in combination. Causes include inadequate dietary intake, especially in patients with corn-based diets or alcoholism. Dietary sources include legumes, yeast, meat, and enriched cereal products. Vitamin E deficiency may cause a hemolytic anemia in premature infants. Laboratory investigations reveal low plasma tocopherol levels, a low hemoglobin level, reticulocytosis, hyperbilirubinemia, and creatinuria. Causes of vitamin E deficiency in premature infants include limited placental transfer of vitamin E and the resultant low levels at birth combined with its relative deficiency in the infant diet. Dietary sources for older children and adults include wheat germ, vegetable oils, egg yolk, and leafy vegetables.

Case A 4-month-old girl has been gaining weight well and achieving normal milestones. She was a full-term infant born via vaginal delivery without complication. Her past medical history is significant only for a mild URI the previous month, and she is current with her vaccines. She has been exclusively breastfed since birth. Her mother eats a well-rounded diet that includes meat and dairy. Question What supplementation does the infant require?

Correct answer: Vitamin D Explanation Breastfed-only infants require vitamin D supplementation to help prevent rickets, infantile tetany, and osteomalacia. The other vitamins are transferred in adequate quantity through breastmilk. Infant formula includes vitamin D. Children and adults get vitamin D through fortified milk, margarine, and fish liver oils. Vitamin D can also be obtained through exposure to sunlight or other ultraviolet sources.

Case A 23-year-old man presents with difficulty walking and frequent falls for the last 3 months. He also has a 3-month history of increased thirst and urinary frequency. There is no history of fever, weight loss, and no history of any major illness in the past. He has mild palpitations on walking. On examination, pulse 86/min and blood pressure 130/80mm of Hg are noted. Abdominal, respiratory, and cardiovascular system examinations are within normal limits. CNS examination shows nystagmus, loss of fast saccadic eye movements, absent deep tendon reflexes, and upward plantar reflexes. There is weakness in distal limb muscles and some sensory loss of fine touch and vibrations. He has ataxic gait, but no spasticity. MRI of the brain is normal. He denies alcohol consumption or any other illicit drug use. Question This condition may be associated with a deficiency of what vitamin?

Correct answer: Vitamin E Explanation There are 2 forms of Friedrich's ataxia—the classic form and the form that occurs in conjunction with vitamin E deficiency. Two types of vitamin E deficiency can occur in Friedreich's ataxia. The first is abetalipoproteinemia (Bassen-Kornzweig syndrome), which is associated with a defect in the microsomal triglyceride transfer protein. This results in the impairment of formation and secretion of VLDL in the liver and a deficiency of delivery of vitamin E to the tissues, including the CNS. The second type, called ataxia with vitamin E deficiency (AVED), is due to a defect in a tocoferol transfer protein and an impaired ability to bind the vitamin E to VLDL. In these patients, vitamin E supplementation is indicated. Vitamin A deficiency is associated with visual defects, particularly night blindness. It is not associated with Friedrich's ataxia. Vitamin B1 (thiamine) deficiency is typically seen in the context of severe malnutrition. It causes peripheral neuropathy, cardiomyopathy, and anemia. It is not associated with muscle weakness or Friedrich's ataxia Vitamin D deficiency causes osteomalacia and may cause muscle weakness but does not cause neuropathy. Vitamin D deficiency is not associated with Friedrich's ataxia. Vitamin K deficiency is associated with impaired production of coagulation factors II, VII, IX, and X. It causes a bleeding diathesis.

Case A female infant born at 28 weeks presents with jaundice and a hemoglobin level of 8 g/dL. The peripheral blood film reveals reticulocytosis; a vitamin deficiency is suspected. Question What is the most likely diagnosis?

Correct answer: Vitamin E deficiency Explanation Vitamin E deficiency may cause a hemolytic anemia in premature infants. Laboratory investigations reveal low plasma tocopherol levels, a low hemoglobin level, reticulocytosis, hyperbilirubinemia, and creatinuria. Causes of vitamin E deficiency in premature infants include the limited placental transfer of vitamin E and the resultant low levels at birth; this is combined with its relative deficiency in the infant diet. Dietary sources for older children and adults include wheat germ, vegetable oils, egg yolk, and leafy vegetables. In cases of vitamin A deficiency, patients can present with night blindness or an inability to see well in dim light. There may also be conjunctival and corneal xerosis, as well as pericorneal and corneal opacities and Bitot's spots. Bitot's spots are a collection of keratin appearing as triangular foamy spots on the conjunctiva. The patient may also have xeroderma, hyperkeratotic skin lesions, and increased susceptibility to infections. Causes include inadequate dietary intake and malabsorption. Dietary sources of vitamin A include fish, liver, egg yolk, butter, cream, dark green leafy vegetables, and yellow fruits and vegetables. In cases of pyridoxine (vitamin B6 deficiency), patients can present with peripheral neuropathy, seborrheic dermatosis, glossitis, and cheilosis. Laboratory analysis reveals anemia with lymphopenia. Causes include malabsorption and medications such as isoniazid and penicillamine. Dietary sources of vitamin B6 include liver, legumes, whole grain cereals, and meats.

Case A 20-year-old woman presents with intermittent nose bleeds for the past 2 weeks. She also reports that her menstrual periods have increased in number in the past 2 months. She recently underwent surgery for small bowel resection and eats only one meal a day. Laboratory investigations reveal prolonged prothrombin time, prolonged activated partial thromboplastin time, and a normal platelet count. Question What is the most likely diagnosis?

Correct answer: Vitamin K deficiency Explanation In cases of vitamin K deficiency, patients present with bleeding tendencies such as epistaxis, menorrhagia, and hematuria. The prothrombin time (PT) and the activated partial thromboplastin time (aPTT) are usually prolonged. The platelet count, fibrinogen level, thrombin time, and bleeding time are normal. Causes include inadequate dietary intake combined with surgery, such as small bowel resection and the use of broad-spectrum antibiotics. Dietary sources rich in vitamin K include green leafy vegetables and liver. Vitamin B1 (or thiamine) deficiency results in beriberi, which is characterized by: a bilateral symmetric peripheral neuropathy beginning in the legs Wernicke-Korsakoff syndrome, which is comprised of nystagmus, ophthalmoplegia, ataxia, memory loss, and confabulation congestive heart failure with tachycardia, peripheral edema, and cardiomegaly In cases of riboflavin (or Vitamin B2) deficiency, patients present with angular stomatitis and cheilosis. On examination, they are pale, have atrophic glossitis, and the tongue may appear magenta. In cases of vitamin D deficiency, children can present with inability to walk unsupported due to muscle weakness and lower limb skeletal deformities, such as genu varum and genu valgum. In cases of iron deficiency, patients can present with feeling weak, dizzy, and tired; they may experience syncope. On examination, they have pale conjunctivae and koilonychia.

Case A 38-year-old man presents with sudden onset of acute upper abdominal pain since the previous night associated with nausea, several episodes of vomiting, and weakness. The pain is mostly in the epigastric region with constant, severe, and steady radiation to the back. He also has a low grade fever since this morning without any chills. He denies diarrhea or dysuria. His past history is significant for hypertension, for which he takes amlodipine 10 mg daily and enalapril 5 mg daily. He has smoked half a pack of cigarettes daily for the past 12 years. He initially denies drinking alcohol except on the weekends occasionally, but on further and repeated questioning he says he drinks 3-4 beers daily and had been drinking continuously for the last 2 days with his friends while watching sports on TV. Family history is unremarkable. On examination he has a temperature of 100.6° F, pulse 106/minute, BP 150/92 mm Hg, and respiratory rate is 20/minute. There is no pallor, icterus, cyanosis, or lymphadenopathy. Mucus membranes are dry, and skin is somewhat clammy. Lungs are clear, and heart sounds normal except for sinus tachycardia. Abdominal exam reveals diffuse tenderness in the epigastric and right as well as left upper quadrants. There is some distension and mild guarding in the upper abdomen. Bowel sounds are hypoactive, but there is no ascites or hepatosplenomegaly. Rectal exam is normal. Labs reveal Hb 15g%, WBC 14,500/uL, platelets 400,000/uL, AST 42 U/L, ALT 36 U/L, AP 26 U/L, amylase 3600 U/L, lipase 546 U/L, BUN 25 mg/dL, creatinine, 1.5 mg/dL, bilirubin 1.2 mg/dL, and random blood sugar 110 mg/dL. Question Which of the following is one of the predictors of acute pancreatic necrosis if present at diagnosis along with 2 other factors?

Correct answer: WBC count more than 16,000/uL Explanation This patient is suffering from acute alcoholic pancreatitis. The 2 most common causes of acute pancreatitis are gallstones and alcohol. Other causes include hypertriglyceridemia, hypercalcemia, abdominal trauma, ERCP, and drugs like valproic acid, azathioprine, mercaptopurine, didanosine, thiazides, tetracyclines etc. Assessment of severity is done by either Ranson's criteria on admission and at 48 hours or by the Acute Physiology and Chronic Health (APACHE) II scoring system. Ranson's criteria include: (1) Age more that 55 years (2) WBC count more than 16,000/Ul (3) Blood glucose more than 200mg/dL (4) Serum LDH over 350 U/L (5) AST over 250 U/L. 3 or more criteria on admission predict a complicated course with possibility of pancreatic necrosis. At 48 hours development of any of the following indicates a worsening prognosis: (1) Hematocrit drop of more than 10% (2) BUN rise greater than 5 mg/dL (3) Arterial PO2 less than 60 mm hg (4) Serum calcium less than 8 mg/dL (5) Base deficit over 4 meq/L (6) Estimated fluid sequestration of more than 6 L. An elevated C reactive protein at 48 hours suggests the development of pancreatic necrosis. A high amylase and lipase are suggestive of acute pancreatitis. Leukocytosis is usually present. Other than the labs ordered above, imaging may also be done. Plain abdominal X-ray may show radio-opaque gallstones, the sentinel loop sign (localized ileus of a small segment of small intestine, usually in the left upper quadrant), and the colon cut-off sign (lack of air in the colon in the area of the inflamed pancreas immediately preceded by a gas filled segment of transverse colon). There may be a reactive pleural effusion with atelectasis in the lower lobes of the lungs. Ultrasound is non specific and may show gallstones. CT scan of the abdomen will show the inflamed pancreas and detect complications like necrosis or pseudocyst formation. Since Ranson's criteria and APACHE scoring systems are cumbersome and time consuming, a CT Severity Index (CTSI) has recently become popular. It uses a grading system based on unenhanced CT of the pancreas and a necrosis score based on contrast enhanced CT of the pancreas. CTSI is the unenhanced score plus the necrosis score, the maximum of which can be 10 and more than 6 indicates severe disease. MRI and MRCP are being increasingly used where available for diagnosis and management of pancreatitis. Treatment involves bowel rest, aggressive hydration, pain control, and bed rest in a hospital. Electrolyte imbalances should be corrected, especially calcium since saponification may lower the level. Broad spectrum antibiotics are not routinely recommended, but in patients with impending necrotizing pancreatitis, antibiotics have shown to decrease mortality. Nutritional support is of utmost importance. For mild to moderate cases, IV hydration and gentle advancement of oral feeding is recommended, where as in severe cases total parenteral nutrition may be needed. Alcohol abstinence will be needed to prevent further episodes. Complications include prerenal azotemia, acute tubular necrosis, shock, pancreatic necrosis, pseudocyst formation, ARDS, and pancreatic abscess. Intra-abdominal hemorrhage may cause ecchymoses around the umbilicus (Cullen's sign) or in the flanks (Grey-Turner sign), though these are rare and not specific for pancreatitis. Prognosis is good for mild cases who abstain from drinking. Recurrences are common in alcoholics. Prognosis is poor for severe necrotizing pancreatitis, especially with multi-organ involvement.

Question A 17-year-old adolescent male presents with unexplained neurological symptoms. His liver is enlarged on palpation, and he has other symptoms of hepatitis. Blood work reveals depressed ceruloplasmin levels. An ophthalmological examination reveals Kayser-Fleischer rings. What is the most likely diagnosis?

Correct answer: Wilson disease Explanation This young man is suffering from Wilson's disease, a genetic disorder of copper metabolism. It is inherited as an autosomal recessive mutation in the ATP7B genes located on chromosome 13. The protein of the ATP7B gene is a copper-transporting ATPase The frequency of the heterozygous carriers is relatively high (1/90). The incidence of homozygous recessive affected individuals is about 1/30,000. It affects all ethnic groups and both sexes equally. Neurological symptoms, hepatitis, and Kayser-Fleisher rings, greenish-brown deposits of copper in the corneal endothelial (Descemet membrane) basement membrane near the peripheral cornea where it meets the iris (limbus), are characteristic of Wilson's disease. About 40% of the dietary copper is absorbed in the gastrointestinal tract, and makes its way to the liver bound to albumin. In the liver, it is complexed with ceruloplasmin, a blood protein that carries most of the copper. Ceruloplasmin levels are abnormally low in patients with Wilson disease, although the disease is not, per se, a mutation in the ceruloplasmin gene. Ceruloplasmin is recycled in the liver by the usual lysosomal degradation pathway, and the unused copper is excreted in bile. When excessive copper is absorbed in the gut, it accumulates in the brain (producing neurological symptoms), the liver (producing hepatitis and hepatomegaly), and the cornea. Penicillamine can be used to treat this disease. It chelates copper and provides some symptomatic relief. Unfortunately, when used appropriately in pregnant women to treat potentially life-threatening Wilson disease, it is harmful to the fetus and can produce cutis laxa in the newborns of penicillamine-treated patients (see Figure H6.3)

Question A 12-year-old child has signs and symptoms of jaundice, nontender hepatomegaly and splenomegaly, and tremor. He has been healthy and is on no medications. He is afebrile. Golden-brown rings on the peripheral corneas (Kayser-Fleischer rings) are noted on slit lamp eye exam. Laboratory studies reveal low levels of serum ceruloplasmin, decreased 24-hour urine copper excretion and elevated liver copper. These findings most strongly suggest which diagnosis?

Correct answer: Wilson disease Explanation Wilson disease (Hepatolenticular degeneration) is caused by a genetic mutation on chromosome 13. It results in low levels of urine copper excretion, low levels of serum ceruloplasmin, and elevated levels of copper in the liver. Kayser-Fleischer rings are a distinctive sign. Wilson disease is treatable, but left untreated, it is fatal. Reye syndrome is much less common than it formerly was, perhaps due to decreased use of aspirin in children with influenza-like illnesses. Children with Reye syndrome have a history of a prodromal febrile illness followed by vomiting. Affected children may have hepatomegaly, but not jaundice. The presentation of autoimmune (chronic) hepatitis can vary greatly. Patients often have tender hepatomegaly. Laboratory tests will reveal deranged LFTs, prolonged prothrombin time, and normochromic normocytic anemia. Drug-induced hepatitis is not a factor for this patient, who was not on any medications. Acute hepatitis A virus (HAV) infection causes fever, malaise, nausea, and right upper quadrant abdominal discomfort. Jaundice is usually very mild. Laboratory tests will show IgM anti-HAV during acute infection.

Case A 52-year-old Caucasian woman presents with a 2-week history of rectal bleeding. Over the last 3 months, she has become progressively more tired, and could no longer play tennis on weekends. She denies diarrhea, abdominal pain or tenesmus. Her past medical history includes peptic ulcer disease 5 years ago, treated with an anti-H.pylori scheme and which has not recurred. Her vital signs are stable, and the physical examination shows no abnormalities. A CBC shows the following: Ht: 28% Hb: 8.8 mg/dl MCV: 82 fl Leukocytes: 8.100/mm3 w/ normal differential An upper endoscopy is normal except for a duodenal ulcer scar without signs of active ulceration, and a repeat colonoscopy is normal. Question What is most likely to identify the source of bleeding?

Correct answer: Wireless capsule endoscopy Explanation Wireless capsule endoscopy is performed by ingestion of a battery-powered capsule containing a camera and a transmitter. The capsule transmits images to sensors outside the patient during its transit in the gastrointestinal tract. Indications for capsule endoscopy are still evolving, but several series have shown that capsule endoscopy is more sensitive than small bowel radiography or push enteroscopy for locating the source of small-bowel bleeding. The most common etiologies of bleeding in this region are angioectasia and Crohn's disease. Several other small bowel lesions, such as tumors (leiomyoma, lymphomas, carcinoids, or carcinomas), varices, and polyps, have also been described. An important concern is the possibility of capsule impaction and small bowel obstruction. Some centers are reluctant to perform capsule endoscopy in patients with partial or intermittent obstruction of the small bowel. However, capsule impaction may reveal the site of a lesion and allow its surgical removal, obviating the need for intraoperative enteroscopy. Other relative contraindications are incapacity of cooperating with the examination (e.g. dementia), swallowing disorders, esophageal stricture, gastroparesis, and poor surgical risk. In patients with obstruction or gastroparesis, the examiner may place the capsule endoscopically. This patient has no signs of ulcer activity. Therefore, biopsy of the duodenal scar is not indicated. A 99mTc scan is the test of choice for Meckel's diverticulum, a remnant of the embryonic omphalomesenteric duct found in 1.5% of the general population. Its most common location is the antimesenteric border of the mid-to-distal small bowel. Some diverticula contain ectopic, acid-producing gastric mucosa. Therefore, they can ulcerate and bleed. Rectal bleeding caused by Meckel's diverticulum is often maroon or brick-red in color, and its incidence decreases with age. Therefore, this diagnosis is unlikely in this case. Angiography can be used if other less invasive procedures (i.e. small bowel radiography, push enteroscopy, or small bowel radiography) are non-diagnostic. Computed tomography is not indicated in the absence of other symptoms indicating a specific etiology (e.g. a palpable mass or suspicion of pancreatic disease). Of note, colonoscopy after rapid oral purging has emerged as the procedure of choice for the evaluation of acute lower GI bleeding, and it also provides a means for therapy. Scintigraphy and/or angiography also play important roles in diagnosis and embolization when colonoscopy reveals negative findings or when it is not feasible.

Case A 45-year-old man presents with fatigue and hair loss that began about 2 weeks ago. His past medical history is significant for alcoholism and type 2 diabetes mellitus. He states that over the past 2 weeks he has begun feeling more and more tired, and he has noticed that he was losing much more hair than usual. He denies pain, nausea, vomiting, fever, and chills; he states that he had been in his usual state of health until 2 weeks ago. He also denies recent illness and travel. He states that he now gets tired after very minimal activity and has poor wound healing. He admits that his diet is poor and that he does not control his blood glucose very well. Physical examination reveals a thin, frail man in no acute distress. He has an ulcerous lesion on his left heel that is draining but does not appear to be infected. Blood work including urinalysis, CBC, and WBC were within normal limits. Question What deficiency does this patient most likely have?

Correct answer: Zinc Explanation This patient most likely has a zinc deficiency. The causes of zinc deficiency include malnutrition, chronic debilitating diseases, chronic renal disease, alcoholism, drugs such as penicillamine and diuretics, and genetic disorders such as sickle cell disease. Clinical manifestations in severe cases include alopecia, diarrhea, weight loss, infections, dermatitis, hypogonadism in men, and intercurrent infections. Supplementation with zinc is the treatment of choice. Niacin (nicotinamide, nicotinic acid) deficiency is uncommon in the United States. It is often found in people who live on a diet that consists mainly of corn. Deficiency may also result from alcoholism, cirrhosis, or diarrhea. Men and women are affected equally. Symptoms of niacin deficiency include nausea, vomiting, diarrhea, rash, glossitis, stomatitis, depression, and psychosis. Niacin deficiency, also known as pellagra, manifests as the "3 Ds": diarrhea, dermatitis, and dementia. If this diagnosis is suspected, supplemental niacin should be given immediately. Some patients deficient in niacin may also be deficient in other vitamins (such as B vitamins), so they should be given as well. Folate deficiency causes megaloblastic macrocytic anemia, as folate plays a key role in nucleic acid synthesis. The early manifestation of folate deficiency, especially in its suboptimal state, predisposes to occlusive vascular disease and thrombosis. These manifestations are linked to increased homocysteine levels found in folate deficiency. Neurological and immunological disturbances may also be seen. Folate replacement is the option to prevent and to treat the deficiency. Vitamin B1 (or thiamine deficiency) causes beriberi, occurring mostly in malnourished and alcoholic patients. The deficiency manifests with acute heart failure, neurologic deficits, and epilepsy. Empiric use of thiamine and prophylactic use in high-risk population is strongly recommended even before blood reports are obtained, as the treatment is inexpensive and prevents major catastrophes. Vitamin B12 or cobalamin deficiency manifests as megaloblastic macrocytic anemia, pancytopenia, and a spectrum of neurological disorders, such as peripheral neuropathy, paresthesias, and demyelination of corticospinal tract. Nutritional deficiency, alcoholism, and malabsorption syndromes are some causes of B12 deficiency. It also is associated with homocysteinemia and atherosclerosis. Diagnosis is by serum estimation of B12. Oral supplementation is safe and effective, but intramuscular injections may also be used.

Case A 40-year-old man presents with right upper quadrant pain, generalized weakness, and weight loss. He gives a past history of hepatitis B infection. On examination, he has hepatomegaly; there are arterial bruits, ascites, jaundice, and signs of cirrhosis. Question What investigation would be most useful in the diagnosis of hepatocellular carcinoma (HCC) inthis patient?

Correct answer: α-fetoprotein Explanation The history and the findings are congruent with hepatocellular carcinoma (HCC). α-fetoprotein (AFP) is the major tumor marker for HCC, and is elevated in over 70% of patients with this disease. High levels of this enzyme carry a poor prognosis. Human chorionic gonadotropin is raised in cases of hydatidiform moles and chorionic carcinoma. Carcinoembryonic antigen (CEA) is useful in following the progression of gastrointestinal neoplasms, but it is not of diagnostic value. A rise in alkaline phosphatase indicates cholestatic liver disease, bone diseases (e.g., Paget's), osteomalacia, hyperparathyroidism, or renal failure. It may also be elevated in growing children.

Case A 50-year-old man presents with a 3-week history of a burning sensation in his feet. He also reports that he has difficulty rising from a sitting position. He is homeless and admits to drinking a lot of alcohol every day. On examination, you note a decrease in the vibratory sensation in the toes, and the ankle jerk reflex is absent. Question What is the most likely diagnosis?

hide Correct answer: Thiamine deficiency Explanation Vitamin B1 (or thiamine) deficiency results in beriberi, which is characterized by: a bilateral symmetric peripheral neuropathy beginning in the legs—patients may present with burning sensation in the feet and difficulty rising from a squatting position. On examination, there may be decreased vibratory and position sensation in the toes. Ankle jerk and knee-jerk reflexes may be absent. Wernicke-Korsakoff syndrome, which is comprised of nystagmus, ophthalmoplegia, ataxia, memory loss, and confabulation congestive heart failure with tachycardia, peripheral edema, and cardiomegaly Causes include inadequate dietary intake, especially from a diet based on polished rice and alcoholism. Dietary sources rich in vitamin B1 include yeast, legumes, liver, whole grains, and enriched cereal products. In cases of riboflavin (or Vitamin B2) deficiency, patients present with angular stomatitis and cheilosis. On examination, they are pale, have atrophic glossitis, and the tongue may appear magenta. In cases of vitamin K deficiency, patients present with bleeding tendencies, such as epistaxis, menorrhagia, and hematuria. The prothrombin time (PT) and the activated partial thromboplastin time (aPTT) are usually prolonged. In cases of vitamin D deficiency, children can present with inability to walk unsupported due to muscle weakness and lower limb skeletal deformities, such as genu varum and genu valgum. They may also have prominent costochondral junctions. In cases of iron deficiency, patients can present with feeling weak, dizzy, and tired; they may experience syncope. On examination, they have pale conjunctivae and koilonychia.

Case A 15-year-old girl presents with a 1-year history of intermittent abdominal pain with nausea and occasional bloody diarrhea. She denies fever and weight loss; there is no travel history. Past medical history is significant only for migraines. She takes a multivitamin. Her vital signs are within normal limits. She has mild diffuse abdominal tenderness to palpation and guaiac-positive stool. Her exam is otherwise normal. Her hemoglobin is 9.7, hematocrit is 28%, and her WBC is 12,000/uL. Question What is the most appropriate next step in her management?

orrect answer: Send stool studies and refer to gastroenterology for colonoscopy Explanation With the history of bloody diarrhea, stool studies should be sent to rule out a parasitic or bacterial infection. She has no travel history suggestive of infectious enteritis, so the studies will most likely be negative. In this situation, antibiotics should not be used without a diagnosis. Due to the chronic and recurring nature of the bloody diarrhea with abdominal pain, the patient should be referred to a gastroenterologist. A colonoscopy with biopsy can be done for the diagnosis of probable inflammatory bowel disease. If the patient were sicker now, the colonoscopy would probably be delayed due to risk of perforation. An abdominal ultrasound might be useful in identifying an abscess, but that is unlikely at this time considering that the patient appears fairly well and is suffering from intermittent pain. Further workup should not be delayed at this point, but the condition does not appear emergent, so there is no need to head to the emergency department.


Related study sets

BI 213 Exam: Chapter 9.2: Transcription Factors in Eukaryotes

View Set

Chapter 8: Skeletal System - Axial and Appendicular Skeleton

View Set

Chapter 50: The Child with a Musculoskeletal Alteration

View Set

Ch. 3: Ethical Decision Making Decoding the Ethics Code (Fisher)

View Set